You are on page 1of 243

QUAN L DU N PHN MM

1
QUAN L DU N PHN MM
2
Mc lc
Li ni u 6
Ch- ng 1. Nhp mn v qun l d n phn mm
Nhp mn
1.1. Nhu cu ang gia tng v phn mm
1.2. Vai tr ca vic qun l trong pht trin phn mm
1.3. Mt th d
1.4. Ginh s chp nhn cc th tc pht trin mi
1.5. Tm tt
10
10
11
13
15
17
Ch- ng 2. Nhng vn pht trin phn mm
Mt cht phng xa
2.1. Nhng vn c bn
2.1.1. Nhng vn lin quan n cc yu cu ca d n
2.1.2. Nhng thay i th- ng xuyn
2.1.3. D ton v nhng vn lin quan
2.1.4. Ngun lc bn ngoi
2.1.5. Kt thc mt d n phn mm
2.1.6. Tuyn dng nhn vin v thuyn chuyn
2.1.7. Theo di v gim st
2.2. Phn tch ri ro
2.2.1. D kin nhng vn cn gii quyt
2.2.2. Pha phn tch
2.2.3. Thc hin cc k hoch i ph bt ng
2.3. Tm tt
Bi tp
19
19
20
21
21
22
23
24
25
25
26
27
29
31
32
Ch- ng 3. Pht trin phn mm theo hp ng
Quan h khch hng - nh pht trin
3.1. Chi ph cng thm i li vi gi c nh
3.1.1. Hp ng ph cng thm
3.1.2. Hp ng gi c nh
3.2. Cc mi quan h khc gi- khch hng - nh pht trin
3.3. Yu cu i vi mt xut (RFP)
3.3.1. Mt s vn c bn
3.3.2. Chun b ca RFP
3.3.3. Pht yu cu xut RFP
3.4. xut
3.4.1. xut khng do yu cu
3.4.2. xut khi c yu cu
3.4.3. i ng chun b xut
3.4.4. Khun dng xut
3.4.5. Khng nh cng vic (SOW)
3.5. Duyt xt xut v qu trnh la chn
3.5.1. Ban tuyn chn xut
33
33
34
36
37
38
39
39
42
43
43
44
44
45
48
49
49
QUAN L DU N PHN MM
3
3.5.2. Ph- ng php nh gi xut
3.6. Mt s nhn nh b sung v xut
3.6.1. Nhng vn lin quan n khch hng
3.6.2. Nhng vn lin quan n ng- i ngh
3.7. Tm tt
Bi tp
50
52
52
53
54
55
Ch- ng 4. Chu trnh pht trin phn mm
Cc biu thi v ch thc n- c
4.1. Pha quan nim
4.1.1. Bu khng kh trong pha quan nim
4.1.2. Nhng vn trong pha quan nim
4.2. Pha yu cu phn mm
4.2.1. Bu khng kh trong qu trnh pha yu cu
4.2.2. Cc vn trong pha yu cu
4.3. Pha thit k
4.3.1. Bu khng kh trong pha thit k
4.3.2. Nhng vn trong pha thit k
4.4. Pha thc hin
4.4.1. Bu khng kh trong pha thc hin
4.4.2. Nhng vn trong pha thc hin
4.5. Pha tch hp v th nghim
4.5.1. Bu khng kh trong pha tch hp v th nghim
4.5.2. Nhng vn trong pha tch hp v th nghim
4.6. Pha bo tr
4.6.1. Bu khng kh trong pha bo tr
4.6.2. Nhng vn trong pha bo tr
4.7. Tm tt
Bi tp
57
60
61
62
62
63
63
65
66
67
68
70
70
71
73
74
75
76
77
78
79
Ch- ng 5. Nguyn tc qun l cc k s- phn mm
H c thc c g khc nhau khng ?
5.1. C cu t chc d n phn mm
5.2. C cu i ng
5.2.1. Lnh o i
5.2.2. Cc i dn ch
5.2.3. Cc i k s- tr- ng
5.2.4. Cc i chuyn gia
5.3. Cc k thut bo co c bn
5.3.1. Bo co tnh hnh
5.3.2. Cc cuc hp v tnh hnh d n
5.4. Nhng - ng li chung trong qun l cc k s- phn
mm
5.5. Tm tt
Bi tp
80
81
85
85
87
87
88
89
90
91
92
94
95
Ch- ng 6. Chia tr cc d n ln th no: phn chia v
chim lnh.
QUAN L DU N PHN MM
4
Nhu cu ln khng c ngha kh
6.1. Tinh ch tng b- c mt
6.1.1. Phn gii chc nng
6.1.2. Phn gii thit k
6.2. C cu phn tch cng vic
6.2.1. Phn gii d n
6.2.2. WBS lm cng c qun l d n
6.3. X l nhng d n ln
6.3.1. Cc h thng ph
6.3.2. - ng li phn gii chc nng
6.3.3. - ng li phn gii thit k
6.3.4. - ng li phn gii nhim v cng vic
6.4. Tm tt
Bi tp
96
96
98
99
102
103
105
106
106
108
109
110
111
112
Ch- ng 7. Cc chc nng h tr d n
H tr qun l d n
7.1. Kim tra cu hnh phn mm (SCC)
7.1.1. Thut ng kim tra cu hnh
7.1.2. Ngun lc kim tra cu hnh
7.1.3. K hoch qun l cu hnh phn mm
7.1.4. Mt s - ng li chung
7.2. Bo m cht l- ng phn mm (SQA)
7.2.1. Cung cp phn mm c cht l- ng
7.2.2. Ngun lc kim tra cht l- ng
7.2.3. K hoch bo m cht l- ng phm mm
7.2.4. o cht l- ng phn mm
7.2.5. Mt s - ng li chung
7.3. Th nghim phn mm
7.3.1. Cc loi th nghim phn mm
7.3.2. Cc th tc th nghim chnh thc
7.3.3. Mt s - ng li chung
7.4. Tm tt
Bi tp
115
116
118
119
121
123
126
126
128
130
132
133
134
135
137
138
139
140
Ch- ng 8. Tiu chun pht trin phn mm
Tiu chun pht trin: tai hi cn thit
8.1. Tng quan cc tiu chun pht trin phn mm
8.2. Tiu chun US DOD 2167
8.2.1. Tng quan tiu chun 2167
8.2.2. R sot v kim ton
8.2.3. M t hng mc d liu (DIDS)
8.2.4. Ly kch th- c tiu chun
8.2.5. Li v bt li ca tiu chun 2167
8.3. Cc tiu chun cng ngh phn mm IEEE
8.3.1. Tng quan tiu chun IEEE
8.3.2. Phn loi IEEE v cc tiu chun cng ngh phn
142
142
145
146
148
148
154
156
156
157
QUAN L DU N PHN MM
5
mm
8.3.3. Li v bt li ca tiu chun IEEE
8.3.4. So snh cc tiu chun IEEE v DOD
8.4. Cc tiu chun Ada
8.4.1. Mi tr- ng Ada
8.4.2. Tiu chun IEEE cho cc Ada PDL
8.5. Cc tiu chun pht trin phn mm khc
8.6. Tm tt
Bi tp
159
159
164
164
165
165
166
167
168
Ch- ng 9. Lp trnh d n
Lp trnh: vn
9.1. K hoch pht trin d n
9.2. Cc hot ng theo lp trnh v mc
9.2.1. Danh mc hot ng theo lp trnh
9.2.2. Cc ct mc v - ng mc ch yu
9.3. Cc biu Gantt
9.4. Cc biu PERT v con - ng ti hn
9.4.1. - ng ti hn
9.4.2. Cc khi ch- ng trnh PERT v vic tng c- ng
9.5. Nhn s lp trnh
9.5.1. Qui m i ng pht trin
9.5.2. K nng v kinh nghim
9.5.3. Thng ca con ng- i bt ngh
9.6. Lp lch cc ngun lc
9.6.1. Lp lch khng gian lm vic
9.6.2. Thit b lp trnh
9.6.3. Ch bn cc nh thu ph
9.7. Kim chng v cp nht ch- ng trnh
9.7.1. Bo co nh k
9.7.2. Cc hot ng kim chng khc
9.7.3. Cp nht ch- ng trnh
9.8. Mt s - ng li chung cho vic lp trnh v qui hoch
9.8.1. Tinh lc danh mc hot ng ban u
9.8.2. Ginh - c ph chun ch- ng trnh
9.8.3. Mi quan h gia ch- ng trnh, ti nguyn, cht
l- ng v tnh chc nng
9.9. Tm tt
Bi tp
170
171
173
174
176
177
180
182
182
183
184
187
188
189
189
190
191
192
192
193
193
194
194
195
197
198
199
Ch- ng 10. Chun b d ton: ph- ng php v k thut
D ton: vn
10.1. D ton d n
10.2. D ton tng b- c
10.2.1. Nhng thnh phn - a khi gi
10.2.2. Nhng thnh phn d- tha kinh nghim
10.2.3. Nhng thnh phn c mt phn kinh nghim
200
201
202
203
203
205
QUAN L DU N PHN MM
6
10.2.4. Pht trin mi
10.2.5. Phn tch chi tit d n mc ri ro
10.3. Uc nh pht trin mi
10.3.1. Nhng ph- ng php kiu u (nguyn mu)
10.3.2. Nhng ph- ng php thng k
10.4. M hnh chi ph xy dng (Cocomo)
10.4.1. Mc nhn s
10.4.2. Mc phc tp
10.4.3. Yu t tin cy
10.4.4. Mi tr- ng pht trin
10.4.5. Cc th h
10.4.6. Thut ton d ton ph
10.5. Chc nng phn tch im
10.5.1. Nhng b- c FPA c bn
10.5.2. ng dng ca FPA
10.6. D ton l mt lnh vc
10.7. D ton ti nguyn phn cng
10.7.1.Ti trng CPU
10.7.2. L- u tr d liu
10.7.3. Tc
10.8. Tng ph khng pht trin
10.9. Tm tt
Bi tp
206
206
207
207
209
210
210
213
215
216
218
219
221
221
224
225
229
229
233
236
238
239
241
Tham kho 243
Ti liu c thm
Li ni u
QUAN L DU N PHN MM
7
y l cun sch v qun l d n phn mm; n khng phi l mt
cun sch tip na v cng trnh phn mm. c nhiu cun sch tham
kho v cng trnh phn mm (coi danh mc tham kho cui cun sch
ny). Mc tiu ca sch ny l trnh by cng vic pht trin phn mm
theo quan im ca nh qun l ch khng phi theo quan im ca nh
pht trin.
Cun sch tp trung, trong mt cun duy nht nhiu thc tin v k
thut qun l phn mm hin i - c pht trin v tinh lc trong sut
thp k qua. Qun l d n - c trnh by nh- l mt k nng lnh hi
- c v ch khng phi nh- l ca tri cho. Chc chn, vic qun l d
n i hi ti nng qun l, nh- ng bn thn ti nng khng - c hu
hiu. Vic vn dng thit thc cc th tc pht trin phn mm hin i
i hi c cc nh qun l chuyn nghip.
V y l cun sch thc hnh (ch khng phi l mt cng trnh l
thuyt) nn nhiu ph- ng php v k thut - c m t khng c c s l
thuyt cho ring n. Tuy nhin nhng tham kho s - c cung cp sut
cun sch dnh cho nhng ai quan tm n c s l thuyt. Danh mc
km cc tham kho v ti liu c - c gi c cui cun sch
ny.
Nht thi, c gi c th thy mt s on - c nhc li trong cun
sch ny. S d c iu ny l gii quyt ci th- ng - c gi l tnh
hung nm ngn tay. iu ny xy ra khi mi nm ngn tay ca c gi
cn - c ci vo cun sch nh du trong khi c gi l- ng l gia
cc ch- ng nhm bao qut - c mt ch c bit. Cun sch ny c
gim nhu cu phi nh du bng cch lp li ci gii thch vn tt bt c
ch ch yu no - c tham chiu ngay d ch - c tho lun
chi tit u .
Sut cun sch nhng hng mc thng cng v nm cng - c s
dng thay cho cc hng mc c thng ng- i v nm ng- i. Nhng t ny
- c tho lun chi tit phn 9.5.3.
i t- ng c - c ch nh.
Qun l d n phn mm: Mt tip cn cho ng- i thc hnh - c ch
nh cho i t- ng a dng. Tr- c ht v ch yu sch - c ch nh
lm ngun tham kho cho cc nh qun l d n phn mm ang thc thi
nhim v qun l v trn c s n - c sp xp sao cho ti ch yu
bao qut mi ch- ng (tr ch- ng 1). iu ny - c tho lun thm
trong gii thch sau v vic b tr ni dung sch.
Cui cng, cun sch c th dng lm tham kho cho cc k s- phn
mm mun m rng kin thc ca minh sang nhng lnh vc qun l d
n k thut.
B tr ca cun sch
QUAN L DU N PHN MM
8
Ni chung, m- i ch- ng ca cun sch xut hin theo trnh t lgc v
cung cp cho vic i u vo lnh vc qun l d n phn mm. Tham
kho nhanh t cui mi ch- ng c dng tm tt m rng. Tm tt ny
nhm - c s dng nh- l ghi nh ln na hay nh- l mt ngun
thng tin ban u.
Ng- i c - c yu cu tm cch lm mt s bi tp cui mi
ch- ng. Cc bi tp ny s gip ng- i c hiu - c nhiu t- ng v k
thut trnh by trong ch- ng .
Ch- ng 1 cp quan nim qun l d n phn mm. Ch- ng ny
cng tham lun nhiu kh khn m cc nh qun l d n gp trong vic
ginh h tr ca b phn qun l cp trn trnh ra nhng th tc pht
trin mi.
Ch- ng 2 tm tt vn tt nhiu vn pht trin phn mm chung
nht (sau ny - c xy dng trong sut cun sch). Ch- ng ny - c
chia lm 2 phn. Phn u ginh cho c gi khng quen vi nhng vn
c bn v qun l phn mm. Phn hai ginh cho nhng ngnh ang
qun l d n c mi v c c kinh nghim. Phn ny tho lun
ph- ng php u tranh vi nhng vn tho lun tr- c y, gi l
phn tch ri ro.
Cc nh qun l d n c kinh nghim c th b qua ch- ng 1 v phn
u ca ch- ng 2.
Ch- ng 3 tho lun vic pht trin phn mm theo hp ng. Ch- ng
ny m t cc hp ng d n phn mm - c tin hnh nh- th no, cc
ngh ra sao. Mt vn bn ngh nn - c xy dng th no v nn
thit lp th no v nhng mi quan h gia khch hng v nh sn xut.
Ch- ng ny cng m t cc yu cu v vn bn ngh (RFP) v qu
trnh la chn sau khi cc ngh - c trnh.
Ch- ng 4 m t chu trnh c bn pht trin phn mm, nhn mnh n
vic tip cn theo giai on, pht trin phn mm. Nhng ph- ng php
lun khc cng - c tho lun (nh- l to mu nhanh v m hnh xon c
- Spiral). Nhng giai on c bn - c m t theo quan im ca ng- i
qun l d n nhn mnh n khng kh v nhng vn ca mi giai
on.
Ch- ng 5 trnh by mt s nhng nguyn tc c bn ca vic qun l
con ng- i. Ch- ng ny la ra mt s nhng mt c th lin quan n
vic qun l cc k s- phn mm, chng hn nh- s khc nhau ng k
v nng sut gia cc k s- phn mm v tnh kh phong ca cc nh
lp trnh ni chung.
Ch- ng 6 cp mt trong nhng vn kh khn nht ca pht trin
phn mm: lm sao qun l - c nhng d n phn mm ln. Ch- ng
ny gii thch nhng d n ln c th - c phn chia thnh nhng b
phn nh d qun l nh- th no theo ph- ng chm chia ra ch ng.
QUAN L DU N PHN MM
9
Ch- ng 7 m t ba trong nhng chc nng h tr qun l c bn:
kim tra cu hnh m bo cht l- ng v th nghim phn mm. Ch- ng
ny cng tho lun mi quan h gia nhng chc nng .
Ch- ng 8 trnh by tng quan v cc chun pht trin phn mm. c
bit hai chun - c tho lun chi tit: chun 2167 ca B Quc phng
Hoa K (DOD) v chun IEEE v pht trin phn mm. Nhng chun
khc, nh- chun pht trin phn mm ca Anh v Chu u, cng - c
nhc ti v so snh.
Ch- ng 9 tho lun vic lp lch v k hoch pht trin d n (PDP) v
k thut lp lch v lp k hoch - c m t, k c phc Gantt v
PERR c in v cu trc ph hy cng vic (WBS).
Ch- ng 10 cha mt m t tng c- ng v chi tit ca mt vi ph- ng
php v k thut chun b d ton. Ch- ng ny gm nhng ph- ng php
d tnh qui m ca d n v lch pht trin d n cng nh- d ton k
thut, chng hn nh- cc yu cu v a v v b nh. Ch- ng ny cng
gii thch kinh nghim c th - c s dng th no ci tin d ton v
m t cc d ton c th - c hon thin th no trong qu trnh pht
trin d n tin trin.
Tri n
Cc tiu chun DOD-STD 2167a v DOD-STD 2168 v cc m t hng
mc d liu lin quan ca B Quc phng Hoa K - c tham chiu v
trch dn - c php ca B Quc phng Hoa K, B ch huy cc h
thng chin tranh khng gian v trn bin.
Cc tiu chun cng ngh phn mm IEEE - c tham chiu v
nhng ti liu sau y - c trch dn - c php ca Vin tp on k
s- in v in t (IEEE).
Phn nhp mn ca F.Buckley cho bn in 1984 ca IEEE v tiu chun
cng ngh phn mm IEEE.
Phn nhp mn ca J.Horch cho bn in 1987 ca IEEE v tiu chun
cng ngh phn mm.
IEEE stel 729 - 1983 IEEE stel 1022 - 1987
IEEE stel 830 - 1984 IEEE stel 990 - 1986
Gi bn quyn mi mt @ Vin tp on k s- in v in t.
Ti xin cm n v s gip to ln ca Amir trong vic duyt v tp
hp vn bn. Ti cng bit n v nhiu gi c ch ca ng.
Ti cng xin cm n Sharon v Talya khng xo trn bn tho vn
bn.
Cui cng v quan trng nht, ti xin ht sc cm n ng vin ca
Iril, nu khng vn bn chng bao gi - c vit ra.
Nhn hiu th- ng mi
Ada l nhn hiu ng k ca Chnh ph Hoa K, Ada AJPO
UNIX l th- ng nhn ca tp on in thoi v in bo M
VMS l th- ng nhn ca tp on thit b s
QUAN L DU N PHN MM
10
MS-DOS l th- ng nhn ca tp on Microsoft
PC-DOS l th- ng nhn ca tp on my mc kinh doanh quc t
BYB l th- ng nhn ca nhm Gordon.
QUAN L DU N PHN MM
11
Ch- ng mt
Nhp mn v qun l d n phn mm
Nhp mn
Phn mm l ni trng cc gic m v thu hi c mng ... mt th gii
nhng ng- i ha si v nhng vin n bng bc Trch dn ny t Brad
cox (cox 1990) nhn mnh nhng quan tm ca cc nh qun l d n
phn mm hm nay. Lm sao c th kim tra - c con ng- i ha si ny
d- i chi tit cng ngh phn mm y ? Liu vic pht trin phn mm
c tht s l mt b phn cng trnh khng ?
Vic pht trin phn mm c th kim tra - c. C nhng ph- ng
php nhng k thut nh- ng chun v nhng cng c khi - c vn dng
ng n chng s thc y s pht trin thng li ca d n phn mm.
khng phi l nhng vin n bng bc xuyn qua tri tim ca ng- i
ha si: chng c g gi tht trong c. Nhng ph- ng php ny cung
cp mt cch tip cn c h thng ti s pht trin phn mm bt u l
nhng giai on qui hoch ban u v kt thc l vic cung cp sn phm
phn mm cui cng.
Cun sch ny cp n vic vn dng nhng ph- ng php hin i
qun l cc d n phn mm. Sch trnh by tip cn thc hnh lm
th no y hn l tip cn l thuyt mc d c nhng tham kho rng
ri cho nhng ai quan tm n vic m t l thuyt ng sau cc ph- ng
php. Mc tiu ch yu l tp trung, trong ch mt cun sch, m t bit
bao cng c v qui trnh dnh lu n nhng hot ng qun l phn mm
nh- :
- D ton chi tit d n
- Chun b cc lch trnh pht trin
- Vn dng cc tiu chun pht trin thit thc
- Chun b v nh gi cc ngh
Nh qun l d n phn mm theo th - c cung cp cc ph- ng php
v qui tnh cn thit khin cho vic pht trin phn mm c hiu qu hn
vi ba mc tiu ni ting trong tm t- ng: pht trin phn mm
1. theo ch- ng trnh
2. trong phm vi ngn sch
3. theo yu cu
1.1. Nhu cu ang gia tng v phn mm
Tht c t lnh vc cng ngh hin i li khng cha phn mm. iu
ny bao gm xe hi, hng khng v v tinh cng nh- thang my, my
fax, truyn hnh v cc c quan in t. Phn mm vn hnh h thng an
ninh x hi, h thng chi l- ng tp on, v tri ca nn kinh t ph- ng
QUAN L DU N PHN MM
12
Ty - H thng th tn dng. Phn mm - c s dng rng ri vit v
in sch.
Vic gia tng nhu cu v phn mm tr nn mt vn gay cn. N
gy ra vic gia tng nhu cu v k s- phn mm. V- t rt xa mc
cc nh chuyn nghip phn mm tt nghip cc tr- ng i hc. Do
y s pht trin phn mm - c yu cu c nng sut cao li hn, tin
cy hn v ni chung thnh cng hn.
Nhng yu cu mi c th khng - c p ng nhng ph- ng
php pht trin th thin ca nhng ngy u ca my vi tnh.
Nhng ph- ng php mi - c xut ci tin ng k con - ng
m phn mm - c pht trin. Tnh nghim trng ca vn - c
tha nhn trong khp cng ng cng ngh phn mm. Mt s cc lin
cng ti v cng xoexiom quc t - c thnh lp Hoa K, Nht Bn
v Chu u vi nhng ngn sch to ln dnh cho vic tm kim nhng
ph- ng php gim nh (nu khng phi l loi tr) vn (coi Bennatan
1987).
Cox (1990), trong mt phn tch ph- ng h- ng cng trnh phn mm
s tin hnh cho thy r mt cuc cch mng cng nghip phn mm l
th, ng tin on ngy m cc nh lp trnh s thi khng m ha
mi th khi xa v s ghp cc ng dng tr nhng catal l- u tr tt ca
cc thnh phn phn mm s dng li - c. Quan nim ny v nhng
quan nim cch mng khc nh- pht trin phn mm t ng (coi
Frankel 1985) vn cn phi mt qung - ng di tr- c khi tr thnh
nhng ph- ng tin thit thc ca pht trin phn mm.
Xu h- ng tin ti cng trnh phn mm c my tnh h tr (CASE)
to nn nhiu cng c pht trin t ng nh- ng chng may thay nhng
cng c
1
th- ng mt nhiu thi gian khng xng vi chng, nhng
lnh vc khc ca cng ngh cc h CAD/CAM
2
t ng ang - c s
dng thit k v xy dng cc thnh phn in t nh- ng pht trin
phn mm vn cn hon ton l mt c gng th cng.
Cho n lc m phn mm s dng li v pht trin phn mm ti to
v t ng bt u thay th - c cc k s- phn mm th phn mm vn
cn tip tc do con ng- i pht trin. Trong khi ch i vic gia tng theo
yu cu v nng sut v thun thc tay ngh v thng li chung ca pht
trin phn mm phi duy tr trch nhim vn phi l nh qun l d n
phn mm.
1.2. Vai tr ca vic qun l trong pht trin
phn mm.
Vic qun l d n c hiu qu i hi nhiu ti nng v k xo. Tiu
chun IEEE (IEEE 1987a) cho dch ngha sau v qun l d n phn
mm.
1
Xem Tahvanainen v Smolander An annotated CASE Bibliography (1990)
2
CAD v CAM l thit k c my tnh tr gip v ch to c my tnh tr gip.
QUAN L DU N PHN MM
13
Qun l d n phn mm l qu trnh qui hoch, t chc, nhn s iu
khin, kim tra v lnh o d n phn mm. R rng tr thnh nh
qun l d n phn mm tt th l nh pht trin phn mm tt khng cn
na. C nhng k xo qun l c bit - c yu cu- dng ngay t
nhng giai on u ca d n, chng hn nh- cc lnh vc nh- :
- Gim st v kim tra
iu ny bao gm vic qun l c hiu qu cc thnh vin i ng
pht trin v i hi thc th- ng xuyn v tnh trng thc ca cng vic
ca h v d n.
- Qui hoch
Qui hoch l mt trong nhng hot ng qun l quan trng nht v
bao gm vic chun b d ton tt, duy tr lch trnh pht trin v b tr
nhn s hiu qu.
- Quan h khch hng
Trong mt s d n, vic tip xc vi khch hng l hot ng qun l
ch yu. iu ny bao gm vit ti liu v yu cu ca khch hng,
khng ch nhng thay i do khch hng, s l vic tham gia ca khch
hng vo qu trnh pht trin, cung cp bo co v t chc xt duyt cng
trnh din sn phm.
- Vai tr lnh o k thut
Lnh o k thut tt th- ng l mt phm cht ao - c trong vic qun
l phn mm c hiu qu. iu ny th- ng i hi kh nng cung cp ch
o trong gii php ca cc vn k thut pht sinh trong qu trnh pht
trin d n. iu ny khng cn thit c ngha l d tr chung bn thn
mt gii php ch thc.
Nhng lnh vc qun l ny l vn dng - c cho mi th loi d n
Cng ngh cao. D sao vic qun l d n phn mm li kh khn hn do
s th l pht trin phn mm t c tnh xc nh hn cc lnh vc cng
ngh khc. iu ny l do nhng d n phn mm t o l- ng - c kh
d ton hn v ph thuc nhiu hn vo nhng nhn t ch quan ca con
ng- i.
Lch s pht trin phn mm y ry nhng tr- ng hp m mc
ngun lc - c yu cu - c qui hoch v d ton tt. Pht trin phn
mm t lu - c nhn nh l doanh nghip y ri ro. c nhiu
tr- ng hp cc d n phn mm v- t qu ngn sch ban u ca chng
ti hai, ba hay thm ch bn trm phn trm. Mt s phi b b sau khi
chi ht vn c bn khi thy r l d ton ngn sch ban u khng ch
no st vi chi ph pht trin thc s.
Trong nhng nm gn y, c tiu chun ho qui trnh pht
trin phn mm v to ra mi tr- ng pht trin nghim ngt trong cc
d n phn mm d d ton v kim tra hn. D sao, iu ny dn n
mt vn mi cc nh sn xut phn nn phi mt qu nhiu thi
gian vo vic thit lp t- liu v qu t cho xu h- ng pht trin hin nay.
R rng l cn tm - c a bn trung gia hai thi cc trong d n
khng c th lp trnh v d ton v d n tiu chaan qu mc, thu thp
QUAN L DU N PHN MM
14
t- liu qu mc trong chi cng sc qu ng vo tng ph v cng vic
giy t.
Khi pht trin phn mm bt u dnh dng n mt b min cng
trnh th nhng ph- ng php lun pht trin mt cch c h thng mi
cng bt u xut hin
3
. Mc ch ca nhng ph- ng php lun mi l
lm cho s pht trin phn mm thnh cng hn. Nu thng li - c do
theo mc ca ba mc tiu ni tr- c y (theo lch trnh, trong phm vi
ngn sch, theo yu cu) th s tht bi hn c ngha l trong vic hon
thnh thm ch mt trong nhng mc tiu . D sao, thnh cng v tht
bi khng phi l ci iu - c nh ngha mt cch d dng.
c nhiu nghin cu cho thy l tht bi ca d n cng l mt vn
ca nhn thc (cos Linto v mantel 1990). Mt d n c th - c nhn
nh l tht bi mt mi tr- ng trong khi mi tr- ng khc d n li
c th - c nhn nh l thng li. Ni mt cch n gin, mt khch
hng c th hi lng vi u ra ca mt d n trong khi ng- i khch khc
li khng. Theo th thnh cng hay tht bi ca mt d n khng ch lin
quan n ba mc tiu pht trin c bn m c n k vng ca khch
hng.
S khng r rng ca quan nim tht bi ca d n chc chn c th
trnh - c nu ch t ra mt ch duy nht. y l ch m khch hng
t ra ch khng phi i ng pht trin t ra. iu c ngha l:
Thnh cng hay tht bi ca mt d n - c ti hu xc nh s hi
lng ca bn yu cu pht trin (ngha l khch hng) Nhng quan nim
- c chng minh trong th d sau:
1.3. Mt th d:
Th d ny cho thy vi sai lm qun l thng th- ng m n c th
cui cng dn n tht bi ca mt d n phn mm. D n khi s vi
vi quyt nh sai lm c bn lin quan n vic khi hnh d n, n
l- t n, n li dn n nhiu quyt nh sai lm hn khi d n tin trin.
Cng ty lin kt cng ngh (TAI) l mt cng ty chuyn mn ha
trong vic pht trin v ch to thit b truyn thng. TAI l mt ban phn
mm ln chu trch nhim v s pht trin phn mm cho thit b truyn
thng. Ng- i qun l ban phn mm - c bit l qun l cng ty ang
tm kim mt cng ty phn mm bn ngoi pht trin mt h thng
bo d- ng v thi gian cho TAI.
Ban phn mm ca TAI ch ng nm bt kin, chun b mt
ngh pht trin h thng v trnh qun l cng ty. Cn c ngh ny
hai thng - c ginh tham kho kin ca ban nhn s, ban ti chnh v
tr- ng ban xc nh cc yu cu ca h thng. i ng pht trin sau
pht trin h thng trong 6 thng sau (ton b thi gian pht trin
3
Xem Shaw (1990) v mt tho lun y trn vn tin ho ca phn mm trong b mn cng
trnh hc.
QUAN L DU N PHN MM
15
k ra phi l 8 thng). Ban phn mm d tnh cn i ng 4 ng- i
cung cp cc yu cu v pht trin h thng.
kin s dng mt cng ty phn mm bn ngoi - c gc li v
ngh ca ban phn mm - c qun l cng ty chp nhn. Ngn sch pht
trin - c thng qua p ng 2 nm r- i nhn cng hay 4 ng- i trong
8 thng. Ban phn mm tin hnh lp i ng d n v chn mt ng- i
qun l d n, trong s cc ng- i qun l d n truyn thng lnh o
i ng ny.
Khi cui t hai thng ban u gn k, nh qun l d n thy r l
phi cn nhiu thi gian hn xc nh v vit t- liu v cc yu cu.
Ph- ng n chn la ca nh qun l d n l:
1. Hoc l yu cu ni rng thi gian v b sung ngn sch pht trin
2. Hoc l s dng mt phn nhng yu cu hin c.
Ng- i qun l ban mun chng minh l ban ca mnh c kh nng
pht trin c cc h thng tin v phn mm truyn thng - c lng vo.
Do ng- i qun l d n v i ng thc chn ph- ng n 2. iu
ny da trn tin l nu d n b chm v v- t qu ngn sch th d n
phi b coi l tht bi v cc h thng thng tin sau ny hn s phi - c
hp ng vi mt cng ty phn mm bn ngoi.
Tt c cc ban khc thy c nhiu vn ch yu ca h thng. Ni
tm li h thng thiu ci m cng ty cn.
Ban phn mm ngh sa cha cc sai st v yu cu ngn sch cho
vic pht trin mt version ci tin mi. D sao, s bt bnh n mc
m qun l cng ty quyt nh - a vic pht trin mt h thng han
ton mi cho mt cng ty phn mm bn ngoi. Cng ty phn mm - c
la chn pht trin thnh cng h thng. Ngc nhin l ln ny kinh ph
li t hn l ngn sch m ban phn mm TAI yeue cu sa cha cc
sai st trong h thng ban u.
Th d (c tht) ny thuyt minh mt s sai lm ch yu trong qun l
d n.
- Kinh nghim trong mt lnh vc phn mm (cc h vin thng - c
lng vao) khong cho vic pht trin thnh cng phn mm mt lnh
vc hon ton khc (h thng tin).
- Nh qun l d n nu trch nhim cam kt vo lch trnh pht trin
hay ngn sch tr- c khi d n - c xc nh . Trong phn ln tr- ng
hp cam kt ca cng ty ch c th c khi cc yu cu - c kt lun.
- Nu yu cu ca mt d n khng - c p ng th vic tham gia vo
lch trnh v ngn sch tr ln v ngha.
- Khch hng hay ng- i dng s khng phi bao gi cng cung cp
- c nhng yu cu ng (th d ch yu giao lin ban). Th- ng l trch
nhim ca ng- i sn xut phi hi nhng cu hi thch ng nhm thu
thp thng tin cn thit.
- i khi tt nht l nn pht trin mt h thng mi bng vic xa b
cn hn l tm cch cu vn mt h thng - c pht trin ti.
QUAN L DU N PHN MM
16
1.4. Ginh s chp nhn cc th tc pht trin
mi.
Mt trong nhng tr ngi m cc nh qun l d n th- ng phi khc
phc l tnh trng thiu h tr qun l cp cao hn i vi nhng
ph- ng php pht trin hin i. Vn dng cc ph- ng php lun hiu
qu khng d khi qun l cp cao hn cn tranh lun v nhu cu ca h.
iu ny dn n cc nh qun l d n n mt tnh trng tin thoi
l- ng nam, lm sao chp nhn - c ci m h tin l tt nht trong khi
vn gi - c c- ng v nh qun l d n.
R rng l nhiu ph- ng php v k thut - c m t trong cun sch
ny ch c hiu lc khi chng - c s dng. Mc tiu ca phn ny l
gip nh qun l d n ginh - c s chp nhn qun l cp cao hn
trong vic ng dng nhng ph- ng php mi.
Qun l cp cao hn (v i khi l cc k s- phn mm khc) c khi
dng nhng lp lun sau chng li vic s dng cc ph- ng php lun
pht trin phn mm hin i.
1. Nhng ph- ng php mi ny u l l thuyt trong th gii thc
s vic din ra khc.
2. Nhng nh qun l d n qu hnh thc ch ngha; h i hi mi
th bng vn bn v khng ng v mi thay i nh.
3. Chng ta khng c th gi cho mi cng vic giy t .
4. Chng ta khng th mt cng sc v s xa ph trong mi qui trnh
di dc . Chng ta vn lun pht trin - c phn mm m chng cn
nhng tng ph .
5. y l kinh doanh ch khng phi tr- ng i hc. Chng ta mt tin
v mt khch nu chng ta bt u vi vic lng ph thi gian vo mi
ph- ng php .
6. Ph- ng php l tt, nh- ng bt hnh l by gi khng phi lc thc
hin chng. Chng ta mong rng c th s dng chng mt ngy no
nh- ng khng phi ng lc ny.
7. khng c k s- no ca chng ta quen vi nhng ph- ng php mi
ny. Nh- th s mt nhiu thi gian v mt nhiu kinh ph bt u o
to li h.
Sau y l mt s tr li gi cho nhng lp lun trn.
1. H s pht trin phn mm trong th gii thc ch- a phi l qu tt.
Trn thc t, cc ph- ng php c th- ng ch dn n thm ha. C thnh
cng y nh- ng t s thnh cng so vi tht bi li qu thp.
Bt c ph- ng php thit thc no u c i cht l thuyt ng nh-
nhng ph- ng php pht trin phn mm nhng ph- ng php
- c cc cng ty t- ng t khc vn dng thnh cng v gim mnh
kinh ph pht trin phn mm v tng ng k cht l- ng phn mm.
2.Vic duy tr n np h s vit l c li cho mi ng- i cho i ng
pht trin cho khch hng v cho qun l cp cao. N m bo l nhng
QUAN L DU N PHN MM
17
giao tip ming - c hiu ng n. Nu nhng thay i hay cc h- ng
dn khc khng ghi thnh vn bn v khng - c chp nhn, th s pht
trin c th tin hnh theo h- ng sai khng ai c th chc chn l mi
thay i, c ln v nh, sau ny s - c nh li khi d n hon thnh.
Danh mc thnh vn bn cc thay i - c chp nhn gip kh nng theo
di v hch ton.
3. y c th l khiu li ng l- u ; cng vic giy t phi - c duy
tr mc ti thiu (c khi n qu mc). D sao, ng ngc nhin l cng
vic giy t c mc hin nay thc s li tit kim thi gian v khng
gy lng ph. Chng hn nhng quyt nh khng ghi thnh vn bn
th- ng khi cn - c lp i lp li v nhng c t ni ming dn n
nhng l gii gy tranh ci. Vic thiu t- liu th- ng mt nhiu thi gian
nht trong cc pha thch hp v th nghim khi bn thit k h thng ch
- c l- u trong tr nh ca ng- i.
Cng vy, mt d n khng thnh vn bn l mt c mng trong vic
duy tu. Sau khi hon thnh d n, khi cc nh sn xut phn tn i, tt
c nhng g cn li l sn phm v t- liu. Khng c t- liu sn phm
khng g hn l mt b mt.
4. Mt cu hi cn phn nh l: liu v ng vy liu pht trin phn
mm ca chng ta thc s thnh cng th no? Lp lun ny - c th
thch tt nht vi b h s t- liu - c chun b cho bit nhng vn
m cng ty c kinh nghim nhng d n tr- c. Mc tiu l chng
minh tip cn mi vi pht trin phn mm u phi l xa x m l cn
thit.
5. Nhng lp lun kh - ng u nht khi c yu t s thc trong
chng, c bit khi cng ty c nh pht trin ph- ng php lun mi
ca chnh h. Mc d nhiu cng ty c tin hnh nghin cu trong cng
trnh phn mm iu ny tht kh l cn thit mi cng ty c nhng
ph- ng php lun nhng tiu chun v nhng h- ng dn - c ghi thnh
vn bn tha ng (coi IEEE 1987b) cho chng c th - c vn dng
d dng mi cng ty m khng cn thit phi pht trin chng li.
Mt khch hng khng ch do lch trnh pht trin ko di m cn do
cht l- ng km v nhu cu k thut khng - c tha mn. Cng cn
nghim lnh hn v pha lch trnh pht trin lu hn ch khng phi v
pha sn phm phn mm tt hn.
Nh- vy, lch trnh pht trin ngn th- ng d b lc li do thi gian b
sung i hi b chnh sn phm phn mm km ci, sau khi tung
n ra ln u (coi th d phn 1.3).
6. Ti sao li ch- a lm ngay? Liu c c s thc s no cho vic ku ca
l thi gian thch hp hn s xut hin sau ny? Tri li cng thm thi
gian v cng sc u t- vo nhng ph- ng php pht trin ngho nn th
li cng kh m thay i - c. Ci cch tt nht tr li cho lp lun ny
l cung cp nhng l do kinh doanh gii thch v sao nhng ph- ng
php pht trin mi li nu - c chp nhn cng nhanh cng tt. B h
s chun b, nu trong tr li cho lp lun 4, s c ch cng vi thng
QUAN L DU N PHN MM
18
tin thu thp t cc cng ty khc. Mc tiu l by t rng nhng qui tnh
pht trin c th t s lm tng cht l- ng sn phm phn mm ca cng
ty trong khi gim chi ph pht trin.
7. Tm quan trng ca vic u t- trong o to t khi cn - c nu:
y l mt quan nim - c chp nhn rng ri. Lp lun ny kh c th
bc b khi cc ph- ng php pht trin mi - c - a ra coi nh- mt thay
i ch yu v ph- ng h- ng. Cu tr li tt nht ty thuc tnh hnh
thc t. Nu nhng ph- ng php mi thc s tiu biu cho thay i ch
yu l ph- ng h- ng th chc chn l cng ty c th nghim nhiu vn
pht trin phn mm. Nh- th cu tr li cho cc lp lun 4 v 5 l
thch ng.
Nu nhng ph- ng php mi khng thc s tiu biu thay i ch yu
v ph- ng h- ng th iu ny nu - c chng minh c s dng d liu
ca cc d n tr- c y. T- t- ng c bn l chng minh rng mc d
nhiu qui trnh pht trin hin nay l tinh vi, vic ci tin ng k c th
- c thc hin thng qua vic gii thiu mt s ph- ng php mi.
Mi lp lun chng li cc ph- ng php lun pht trin mi ch c th
bc b - c sau khi c chun b y . iu ny th- ng ngha l:
- S- u tp d liu v cc d n pht trin phn mm tr- c y trong
phm vi vng ty.
- S- u tp d liu v cc cng ty t- ng t chp nhn ph- ng php
pht trin mi.
- S- u tp cc bo co c dn t- liu, vn bn v cc chng c khc
thnh vn (cn phng qu l thuyt).
- C - c h tr ca cc chuyn gia pht trin phn mm khc hoc
trong phm vi cng ty hoc ngoi.
Mi d liu cn - c nghin cu v cc ghi ch - c chun b chng
minh nhu cu ca cc ph- ng php pht trin mi. Dng cui phi l vic
vn dng nhng qui tnh mi, thit thc pht trin phn mm l v li ch
ca cng ty.
ginh - c s ph chun cn thit ca qun l cp trn, nhng nh
qun l d n phn mm c th chuyn sang vn dng cc ph- ng php
m t trong cc ch- ng sau. B- c u trong vic tm hiu nhng vn
c bn trong vic pht trin phn mm - c tho lun ch- ng 2.
1.5. Tmtt.
Vic pht trin phn mm c th khng ch - c. C nhng ph- ng
php, nhng k thut, nhng tiu chun v cc cng c khi - c vn
dng ng th chng thc y vic pht trin thng li d n phn mm
vi ba mc tiu tr danh trong tr c pht trin phn mm.
1. Theo ng lch trnh.
2. Trong phm vi ngn sch.
3. Theo yu cu.
QUAN L DU N PHN MM
19
Thng li hay tht bi ca d n khng ch lin quan n ba mc tiu
pht trin c bn nh- ng cng c n k vng ca khch hng: mt
khch hng c th hi lng vi u ra ca mt d n trong khi khch
hng khc li c th khng. Do y, thnh cng ca d n - c xc nh
ti hu s hi lng ca khch hng.
Mt trong nhng tr ngi m cc nh qun l d n th- ng phi khc
phc l thiu h tr ca qun l cp trn i vi nhng ph- ng php pht
trin hin i. R rng l nhiu ph- ng php v k thut m t trong
cun sch ny, c hiu lc ch khi chng - c s dng.
Mi lp lun chng li cc ph- ng php lun pht trin mi ch c th
bc b sau khi c s chun b y .
Thng tin v thnh cng v tht bi ca pht trin phn mm trong qu
kh trong phm vi mt cng ty cn - c thu thp cng vi cc minh
chng h tr khc bng vn bn. Mi d liu cn - c nghin cu v cc
chi ch cn - c chun b chng minh cho nhu cu v ph- ng php
pht trin mi. Dng cui nn l vic ng dng cc chu trnh pht trin
phn mm hu hiu mi l v li ch ca cng ty.
QUAN L DU N PHN MM
20
Ch- ng hai
Nhng vn pht trin phn mm
Mt cht phng xa
Ti mt lp hc mi y v qun l d n phn mm, mt trong cc
hc vin hi:
Chng ti c mt s vn ch yu trong mt d n m ti ang
qun l. Chng ti khng c - c t- liu c h thng hay c - c mt k
hoch pht trin no v d n ang trn - ng v- t ngn sch v chm
hn lch trnh. Vy ti phi vn dng mi ph- ng php v k thut hc
y ra sao nhm lm cho d n tr li ng lch trnh ?
y khng phi l mt tnh hung khng ph bin , mt ph- ng
thuc thn k tm cho mt tnh hung gn gy thm ha. Nhng d n
qun l ti c th i n tr tr v ngn sch v- t n hai thm ch ba
trm phn trm v trong vi tr- ng hp li c th b bi b. Hu ht
nhng ph- ng php qun l d n hin i ban u bao gi cng quan
tm phng xa (ch khng phi l hiu chnh) nhng vn loi .
Vic phng nga nhng vn th d hn v t tn km hn l gii
quyt chng. Nhng bin php phng nga thit thc hn l:
- nh v sm vn v nhng vn tim n.
- Gii quyt vn tr- c khi chng tut khi tm tay.
- Lp k hoch tr- c (i ph) vi nhng vn tim n. Nhng vn
s tr nn tn km hn gii quyt khi d n tin trin n nhng giai
on pht trin cao hn. Nhng vn b lng qun cng c th lan
truyn sang cc lnh vc khc ca d n lm cho vic hiu chnh chng
tr nn kh khn hn. Do y vic thit lp nhng qui trnh nh v
sm v hiu chnh cc vn l quan trng.
Ch- ng ny gii quyt nguyn nhn ca mt s th loi rt thng
th- ng ca vn pht trin phn mm v tho lun tc ng ca chng
n qu trnh pht trin. Ch- ng ny cng tho lun vic d kin nhng
vn nhm gim thiu tc ng ca chng ti d n. Cc ch- ng sau
ra nhng ph- ng php phng nga cc vn m t y khi xy ra.
2.1. Nhng vn c bn.
C rt nhiu vn c bn m nh qun l d n hnh nh- tm thy
trong bt c d n phm mm no. Nhng vn c bn gy ra bi
nhng tnh hung sau y bao gi cng c th phng tr- c s pht sinh:
- Yu cu ban u khng y .
- Ph thuc cc ngun bn ngoi (cc ng- i bn hng, cc ch thu
ph v.v...)
- Cc kh khn trong kt thc d n.
- Thay th th- ng xuyn nhn s thc hnh pht trin (xo trn i
ng).
QUAN L DU N PHN MM
21
Cc vn c bn khc th- ng ny sinh do sai lm qun l thng
th- ng nh- :
- D ton ti.
- Theo di v gim st khng y .
- Thay i khng kim sot - c.
Con - ng tt nht nh v mt vn sm l i tm n. R rng,
u tin l tm xem vn th- ng ny sinh nhiu nht u. Chng hn
thay i th- ng xuyn v khng kim tra - c vi vic c t cc yu
cu th- ng hin nhin l ngun ch yu ca nhng vn thit k.
Nhng ch thu ph v ng- i bn hng khng gim st - c l mt trong
nhng ngun bt ng, thng th- ng nht c bit l lc h bo co cc
vn k thut v tr hon vo, chnh nhng pht cui cng i vi cc
nh qun l d n th vic bit phi kim u do quan trng nh- vic
bit phi lm g.
2.1.1. Nhng vn lin quan n cc yu cu ca d n.
c t cc yu cu ca d n s m t sn phm phi - c sn sinh ra
bi nhm pht trin (coi ch- ng 4). Nu nhng yu cu khng - c c
t y th ch cc may mn thun ty mi m bo - c l sn phm
p ng yu cu ca khch hng
4
. Sau y l mt s th d cc vn
lin quan n c t yu cu ngho nn.
- Qun cc c im.
Khch hng tin chc l mt s c im b b qun hn phi nm trong
sn phm cn c nhng tho lun khng chnh thc (th- ng vi ng- i
khng ng). Ghi chp, bnh lun v nhn xt cc cuc hp nh- ng
khng cn c c t yu cu chnh thc.
- C nhng c im khng cn thit - c - a vo.
i ng pht trin tin chc l khch hng hn ht sc vui thch v
nhng c im ngoi hng - c thm vo sn phm (th- ng khng hi
kin khch hng). Mt th d hn l thm s truy nhp h thng bng
khu ng khi khch hng li mun h thng sn sng cho bt c ai.
- c im m chng hot ng khc vi iu mong i.
Khch hng gii thch khng y v m c im cn thit v th l
i ng pht trin hiu ci yu cu theo cch hiu ca mnh. Mt th
d c th l yu cu "cp nht th- ng xuyn c s d liu". Cc nh pht
trin to ra mt h thng cp nht c s d liu mi ln mt ngy
trong khi khch hng mun ni l mt gi mi ln.
- Nhng c im cn thit m chng ai ngh n.
Khch hng khng cn thit l mt chuyn gia my tnh v do c
th khng thc - c l mt c im, c bit no phi cn n. Th
d c th l nhu cu v bade up y ; khch hng c th cho l bade up
l khng cn thit v rng nu dch v my tnh b gim on (th d do
4
T khch hng y - c s dng theo ngha rt rng bao gm khch hng chnh thc, ban tip
th, nhm ng- i dng, qun l v.v...
QUAN L DU N PHN MM
22
mt in) th vic mt mt hay hai giao dch l- u tr trong b nh s
chng thnh vn . Th nh- ng khch hng c th khng xem xt s
vic l nhng iu khin a cng c th v v mt i mi d liu ca
h.
R rng l nhng yu cu - c c t km li l vn cho nh pht
trin nhiu cng nh- cho khch hng. D sao, nh sn xut th- ng v
th tt hn bin son nhng yu cu hn l khch hng. Thng th- ng
c t yu cu tt nht l kt qu ca s c gng phi hp ca c nh pht
trin ln ca khch hng vi vn bn thc - c son thnh vn ca nh
pht trin v - c chp nhn bi khch hng.
2.1.2. Nhng thay i th- ng xuyn.
Tht cc k him khi tm - c mt d n phn mm qui hoch tt li
i n kt cc thng li vi c t yu cu - c dn nhn. Ph- ng n 1.0.
Thay i l khng trnh khi sut chu trnh pht trin phn mm. D sao,
trong phn ln tr- ng hp mt thay i - c - a ra chm th thc hin
thay i li cng tn km.
Mt s thay i tha ng hn l phi qun l - c. Khi dng cc thay
i vo nh- thc l th chng tr thnh vn . Ngay ch mt thay i
c th thnh vn nu n - c yu cu ngay trong pht trin ca d n
v nu n dn n thay i ch yu v ph- ng h- ng. Nhng thay i
qu mc to nn ci vn th- ng - c coi l hi chng mc tiu di ng.
Nh qun l d n khng nhng thay i ph- ng h- ng v i ng pht
trin tr thanh va bi ri va mt mc tiu.
Thay i c th ph v d n nu chng khng - c ghi thnh vn bn
v gim st y . Nhng thay i vi s l- ng tha ng, phi - c
qun l, vn dng c ch kim tra s thay i mt cch c h thng.
Ph- ng php trong phm vi t chc kim tra cu hnh, - c tho lun
ch- ng 7.
2.1.3. D ton v cc vn lin quan.
D ton tt l quan trng v chng to thnh nn mng ca k hoch
pht trin d n tt. K hoch , do nh qun l d n chun b - c lp
thnh trong cc giai on ban u ca d n v bao gm d ton lin
quan n.
- Ngn sch pht trin d n
- Lch trnh pht trin d n
- Ti nguyn pht trin cn n (i ng pht trin thit b pht trin
v.v...)
Nhng d ton k thut cng - d hnh thnh trong pha thit k v
bao gm:
- Cc c tnh ca phn mm (d ton v c b nh, c s d liu
v.v...).
QUAN L DU N PHN MM
23
- Cc c tnh ca phn cng v c mc tiu cn n (d ton tc
CPU, kh nng u vo, u ra; kh nng iu khin a v.v...).
D ton l c s cho nhiu quyt nh k thut v qun l d ton ti
dn n quyt nh d. D ton ti c th hiu l qu cao hoc l qu
thp v quyt nh theo hoc l to ra lng ph hoc thiu ti nguyn
pht trin. iu ny hnh thnh sai lm trong qui hoch, nh- :
- Lch trnh qu ngn hoc cao thi qu.
- Ngn sch qu thp hay qu tng gi to.
- Qu thiu hay qu tha ng- i lm v nhng sai lm trong thit k k
thut, nh- :
- Nhng my tnh trong mc tiu qu nhiu (v t hn) hn cn thit
hoc khng th h tr ng dng - c pht trin. Nhng vn ny sinh
t d ton thp th- ng gay cn hn l nhng vn ny sinh t d ton
cao. Hiu r iu ny, cc nh d ton th- ng thm vo mt s yu t bt
trc (cho rng n 30%) trong d ton ca minh, gi nh rng qu cao
cn hn qu thp. D sao d ton cao c th khng gy ra tht bi ca d
n nh- ng c th ngn cn d n chng bao gi - c khi cng. c
nhiu ph- ng php - c pht trin nhm to ra cc loi d ton khc
nhau cc giai on khc nhau ca d n (coi cc ch- ng 9 v 10). D
sao, ngay nhng d ton chun b tt c th dn n nhng vn nu
chng khng - c cp nht trn mt c s nh k v u n. R rng l
thng tin tt hn v y hn to nn d ton tt hn. Do khi d n
tin trin v c nhiu thng tin hn, d ton cn - c xem xt li v hiu
chnh. iu ny dn n vic gim nh li cc quyt nh pht trin, to
cho cc vn tim n - c cp sm tr- c khi chng tr thnh gay
cn (coi phn 2.2 v phn tch ri ro).
2.1.4. Ngun lc bn ngoi.
Cc vn pht trin d n th- ng d qun l nht khi mi nhn t
pht trin - c iu khin bi qun l d n. D sao, iu ny khng
phi bao gi cng c - c. Nhiu d n ph thuc nhiu ngun bn
ngoi nh- :
- Ch thu ph
- Ng- i bn thit b
- Nhng d n pht trin song hnh
- Nhng ng- i cung cp dch v (bo tr, hun luyn, lp t v.v...)
- Cc chc nng h tr (thng tin in thoi mng, nhng cung cp d
liu. D ph thuc vo cc ngun bn ngoi hn phi - c phn nh
trng).
K hoch pht trin d n. iu ny c ngha trong phm vi cc giao
- c v d ton nhn - c t cc ngun khc. Theo th, d ton trong k
hoch khng th tt hn d ton nhn t cc ngun khc.
Vic trng cy vo cc ngun bn ngoi c th gy ra nhng vn
sau:
QUAN L DU N PHN MM
24
- Chm tr lch trnh, do vic giao chm cc thnh phn d n.
- S km ci qu cht l- ng v thit k thit b pht trin v cc thnh
phn d n bn ngoi.
- Thnh phn bn ngoi khng t- ng hp do v s sai lch ca nh pht
trin bn ngoi hay bn hng vi c im tha thun hay cng b.
- H tr sn phm ngho nn i vi cc thnh phn bn ngoi. Do c
thc - c nhng vn tim n , nh qun l d n c th m bo
rng h - c nh v thch hp trong hp ng hay tha thun vi ngun
bn ngoi. Nhng vn c th - c ngn nga nu - a vo trong
hp ng nhng mc pht v chm tr trong giao hng hay khuyt tt
trong sn phm (coi cc ch- ng 3 v ch- ng 9). Nhng phng sm c
th - c pht hin khi th- ng xuyn xt duyt li cng vic - c pht
trin bi ch thu ph v i hi s bo co tin th- ng k.
2.1.5. Kt thc mt d n phn mm.
Nh- mi nh qun l d n u bit, cc d n kh m bt u - c
kinh nghim cho thy l chng th- ng khng t kh khn i n kt
thc. i ti on kt ca d n, lun lun cc bn quan tm khc nhau
pht sinh nhng yu cu. Nh- ph phn nhng thay i mi v nhng
hot ng pht cht khc na. iu ny l c bit c thc vi nhng d
n gi c nh pht trin cho khch hng theo hp ng (coi ch- ng 3).
Nhng vn chnh lin quan n vic kt thc d n l:
- Tranh chp gia khch hng v nh pht trin v vic l gii v cung
ng mi c im - c yu cu
- - a vo nhng thay i pht cht.
- Tht bi ca h thng v khuyt tt thit k xc nh trong qu trnh
ci t v th nghim h thng.
- Kh khn trong vic gi cho i ng pht trin hp lc li vi nhau
v nng ng. Khi tnh hnh cn thng gim i vo gn cui d n c tnh
trng st gim nhit tnh t- ng ng trong nhng thnh vin cn li ca
i pht trin.
Trch nhim ca nh qun l d n l m bo cho vic kt thc d n
c trt t v thng li. iu ny - c thc hin nh vic qui hoch chi
tit lc ban u d n v qun l d n c hiu qu xuyn sut d n. c
bit, iu ny i hi rng:
- K hoch th nghim thu phi - c chun b lp ti liu v - c
khch hng ph chun tr- c khi kt thc d n. Mc nhn s v s
phn cng cng vic phi - c lp lch trnh, c tnh n vic gim dn
trong qui m i ng pht trin vo cui d n.
-Vic - a n phm ra th tr- ng phi - c lp k hoch tt, k c ng
gi son tho t- liu, hun luyn, ci t v chuyn tip c trt t sang
pha bo tr v h tr.
S kt thc thng li ca d n l bt u ca chu trnh pht trin khc:
Nhng c t yu cu, nhng k hoch th nghim hay nhng k hoch
QUAN L DU N PHN MM
25
pht trin ngho nn, tt c u dn n nhng vn ch yu lc kt
thc d n.
2.1.6. Tuyn dng nhn vin v thuyn chuyn.
Kh khn trong vic tuyn dng cc thnh vin i ng pht trin l
mt trong nhng vn u tin m ng- i qun l d n phn mm gp
phi. Tr- c khi bt c d n no - c tung ra, i ng pht trin ban u
phi - c thit lp v cc vn ny s khng kt thc mt khi i ng
ti v. Gi - c i ng th- ng kh nh- thit lp n.
Frenkel (1985) bo co l nhu cu k s- phn mm tng theo hm s
m trong khi nng sut tng theo mc khong 5% mi nm. Cc tr- ng
i hc Hoa k v chu u ang khng cung cp k s- phn mm
b p l hng gia cung v cu. Trn thc t khng nhng l hng
khng - c lp kn m li ang rng ra mc ng lo ngi.
Khi l- ng thi gian bnh qun m mt k s- phn mm li ngh
gim khi yu cu k s- tng. iu ny khng ch gy ra bi s thuyn
chuyn ca k s- phn mm gia cc cng ty m c s s thuyn chuyn
trong phm vi cc cng ti. V cc cng ty ny ang tm cch s dng hu
hiu hn cc k s- ca mnh. S thuyn chuyn trong phm vi cc cng
ty khng ch do tnh trng thiu k s- phn mm m cn do chi ph v h
c th vn ngn tr thnh vic tuyn b sung.
5
Thuyn chuyn nhn s bn thn n l mt vn trng yu. S n
nh c ca i ng v do vo thng li ca d n.
Cc vn lin quan n tuyn nhn s v thuyn chuyn lun bao
gm:
- Vic u t- ng k - c i hi trong - ng biu din hc tp v
o to thnh vin i ng pht trin mi.
- Thuyn chuyn nhn s lun lun s gim i tinh thn i ng v tc
ng khng li n ng c i ng.
- Tuyn chn th- ng tn km v mt th gi n i hi nhiu cuc
phng vn v c ph tuyn dng.
- Thuyn chuyn nhn s lun to ra tnh trng thiu kin nh trong
pht trin d n.
Trong cc vn pht trin phn mm, nhng g lin quan n pht
trin i ng th- ng - c nhn thc l gay cn nht. Cc thnh vin i
l ngun pht trin quan trng nht c chnh h ng gp nhiu nht cho
s thnh cng hay tht bi ca d n.
5
Kinh t hc c bn ch ra rng khi c - c k s- th chi ph v h s
gim (cn c cung v cu trong phm vi th tr- ng ngh nghip. D
sao, trong nhng nm gn y, vic cung cc k s- phn mm khng bao
gi ln to ra tc ng ngoi tr nhng khong thi gian ngn
ngi ti cc a im c lp).
QUAN L DU N PHN MM
26
2.1.7. Theo di v gim st.
Theo di v gim st l nhng nhim v qun l. Khi cc vn pht
sinh nhng lnh vc ny v lin quan, chng th- ng l hu qu ca cc
qui trnh qun l d n khng thch hp v khng hiu lc. Mt trong
nhng kt qu thng th- ng nht l nh qun l d n khng c thc v
s tn ti ca nhng vn ch yu giai on m chng c th - c
kim ch v hiu chnh tt nht. Vic theo di v gim st c hiu qu
i hi s tip xc trc tip gia qun l d n v i ng pht trin (coi
ch- ng 7). Mt trong nhng nguyn nhn ch yu ca cc vn theo
di v gim st l hi chng thp ng ni tn ti k nt th- ng trc gia
qun l d n v phn cn li ca i ng pht trin. iu ny dn n:
- Lung thng tin khng chnh xc hay khng c. ng gp ng k
vo nhng quyt nh qun l ti.
- Pht trin khng phi hp: tnh hung ny th- ng - c m t khi
mt i ng pht trin ang trin khai hai d n khc nhau. iu ny xy
ra khi cc thnh vin i ng pht trin khng phi hp v khng - c
gim st li tin hnh theo nhng h- ng khc nhau.
- Tr tu lch trnh v bi chi ngn sch, iu ny gy ra do d ton ti
da vo cc thng tin khng ng.
Thng tin l thnh phn c bn ca bt c th loi qun l no. Do y,
gim st ti i i vi lung thng tin khng thch hp l ct li ca qun
l d n ti. C ba vn trn m t hu qu chung ca qun l ti. Danh
mc cc vn ko theo hn ng l bao trm hu ht cc kiu vn
pht trin d n. Cc ph- ng php to lp nhng knh thng tin c hiu
lc v nhng qui trnh bo co - c t chc tt s - c m t ch- ng
5.
2.2. Phn tch ri ro.
Nhn xa l phm cht qun l tuyt ho th- ng c th - c pht trin
theo kinh nghim. Tht vy, trong nhiu tr- ng hp, cc vn c th
on tr- c. Trong nhng tr- ng hp , nh qun l c th lp k hoch
v kh nng m vn s xy ra khi d tnh kh nng ca n, nh gi
tc ng ca n v chun b tr- c cc gii php. iu ny th- ng - c
gi l s phn tch ri ro v l mt ph- ng tin hiu qu u tranh
chng li nhng vn pht trin tim n. Tin hnh phn tch ri ro c
ngha l - c d b sn sng. y l mt hnh thc bo him, t- ng
c bn l nu mt vn c ny sinh th mt gii php c sn. Ging
nh- mi bo him, phn tch ri ro th- ng cng phi tr gi. Chi ph d
phng cho s pht sinh mt vn tr- c ht l chi ph c - c gii
php i ph sn trong tay, trong khi vn c th xy ra, c th khng.
Trong mt s tr- ng hp, chi ph c th l ti thiu: Ch l thi gian cn
tin hnh phn tch v lp t- liu cho gii php v thi gian theo
di vn . Trong cc tr- ng hp khc, chi ph c th l ln lao, chng
QUAN L DU N PHN MM
27
hn, gi ca mt b phn thay th ca thit b pht trin. Trong mi
tr- ng hp, mt vn - c phn tch v gii quyt sm hn th n
gin hn rt nhiu so vi vic gii quyt vn sau khi pht sinh bt ng.
2.2.1. D kin nhng vn cn gii quyt.
Giai on u tin ca phn tch ri ro i hi duyt xt mi k hoch
qun tr v k thut ca d n nhm minh nh cc vn tim n n bao
gm:
- K hoch pht trin d n.
- c t yu cu.
- c t thit k.
Bng 2.1. Th d v danh sch vn d liu
Vn M t
1. Chm giao my tnh
pht trin
Nu my tnh pht trin khng - c giao vo
1/6 nh- k hoch, giai on thch hp s b
chm.
2. B nh khng C ca phn l- u tr b nh ca h thng c th
v- t 8 mga bait (c b nh ti a - c vi tnh
cp d- ng).
3. Khng c chuyn gia
h thng iu hnh
H thng i hi thay i cho h thng iu
hnh chun J.Adams l chuyn gia h iu hnh
duy nht trong cng ty v ng c th bn khng
- c s dng cho h thng ny.
4. Thi gian p ng ca
h thng qu chm
Thi gian p ng ca h thng yu cu cho u
vo c th qu 5 giy so vi c t trong yu
cu.
5. Thuyn chuyn nhn
s nhiu
Lch trnh l xt xao vi thi gian trng ti
thiu. Nu c s thay th nhn s qu mc bnh
qun trong pht trin chng ta s tr- t thm lch
trnh.
6. Truyn thng qu
chm
Gc truyn thng chun qu chm. Thit k da
trn gi truyn thng nh phn mi. Gc ny
ch- a bao gi - c s dng vi h thng ny v
c th khng thch hp.
7. Chm giao v h
thng ph c s d liu
H thng ph c s d liu - c hp ng ph
vi tp on pht trin phn mm (SOI) cam kt
giao hng 15-4. SOI c th khng giao ng
thi hn nn lm chm s thch hp v pha th
nghim cui.
Mi l thuc ch yu trong k hoch pht trin d n u - c xem
xt v nh gi. Cc th d c th l s l thuc ngun bn ngoi nh-
QUAN L DU N PHN MM
28
ch thu ph, ng- i bn hng, nh cung cp v cc nh lm dch v. Cc
vn s ny sinh nu cc hp phn hay dch v bn ngoi khng - c
cung cp kp thi hay khng hot ng nh- trng i.
c t thit k d n l mt k hoch chi tit v vic lm th no
cc yu cu - c thc hin. Cc quyt nh v s thc hin - c i hi
c th cha cc vn tim nng. Chng hn, cc vn s ny sinh ra
nu phn cng - c la chn li ha ra khng thch hp, chng hn nh-
CPU qu chm, mng cc b khng tin cy, hoc b nh khng .
Sau mi vn d liu - c tp hp ln danh sch, mi vn
- c minh nh v m t v tc ng tim n nh h- ng ti d n. Bng
2.2. cho mt th d v danh sch vn d liu.
Danh sch vn d liu nn - c tp hp nh c s tham gia ca cc
thnh vin chnh ca i ng pht trin d n. Nhng ng- i khc c th
cng - c mi ng gp cho danh sch , cn c w kinh nghim cng
kin thc k thut hay qun tr ca h; C th bao gm c nhng ng- i t
cc i ng d n khc, cc nhm h tr phng php ch hay phng mua
sm (kinh doanh) ca cng ty.
Trong khi mc tiu khng phi l lit k mi vn nhn thc - c
m mi d n c th kinh qua, cn thit l minh nh nhng vn hn
- c xem mt cch hp l l c lin quan n d n. Trong mi tnh
hung, giai on phn tch sau y - c nhm cch ly ch nhng vn
no c th tc ng ln lao n d n v c th mt cch hp l - c
xem l hn s xy ra
2.2.2. Pha phn tch
Vic phn tch danh sch nhng vn d liu i hi nh gi mi
vn nhm:
1. D ton xc sut vn s xy ra.
2. D ton so ng ca vn ti d n
3. Quy cho mc nghim trng ca vn .
Xc sut v tc ng nu - c d ton bi qu mt ng- i. Mi
hng mc trong danh sch - c d ton tt nht trong mt cuc hp duy
nht nh gi vn m bo tnh t- ng i ca mc nghim
trng, gia cc vn khng b lch lc. Mc tiu l trnh nhng tnh
hung khi vic chm giao ca bn cung cp A - c mt ng- i d ton l
0.8 v vic chm giao ca bn cung cp B - c mt ng- i khc d ton
l 0.6 trong khi c 2 nh d ton hn ng l hai xc sut ny l bng
nhau. Nu hai ng- i trong cng 1 phng trong cng 1 lc th s lc lc
t- ng i gim i.
Mt cch n gin v hiu qu tnh mc nghim trng ca mi
vn - c d liu l:
1. Gn mt s k vng gia 1 v 10 cn c xc sut l vn s xy
ra, vi 10 biu xc sut cao, v 1 xc sut thp nht (th d nhn xc sut
vi 10).
QUAN L DU N PHN MM
29
2. Gn mt s gia 1 v 10 cn c tc ng ca vn vi d n vi
10 biu th tc ng cao v 1 tc ng thp.
3. Nhn tr gi c - c b- c (1) vi t gi c - c b- c (2) tnh
mc nghim trng cho vn .
Bng 2.2. gii thiu mt th d v cch tnh mc nghim trng c s
dng cc vn d liu m t bng 2.1.
Bng 2.2. Th d v cch tnh mc nghim trng.
ST
T
Vn K
vng
Tc
ng
Nghim
trng
1 Chm giao my tnh pht
trin
6 5 30
2 B nh khng 4 2 8
3 Khng c chuyn gia h iu
hnh
5 5 25
4 Thi gian p ng ca h thng
qu chm
5 3 15
5 Thuyn chuyn nhn s cao 5 8 40
6 Truyn thng qu chm 2 8 16
7 Chm giao h thng ph c s
d liu
3 9 27
Sau khi mc nghim trng - c tnh cho mi vn d liu,
danh sch - c phn loi theo d nghim trng ca vn trong vn
nghim trng nht ng u danh sch. Sau c th quyt nh xem
vn no c mc nghim trng t hn mt tr gi no (th d 10)
s khng - c xem xt. Sau y nhng vn cn li - c nh gi v
tin trnh hnh ng chi tit, gi l k hoch i ph bt ng - c la
chn cho mi vn . Ri thng tin - a vo bng i ph bt ng. Vi
mi ln - a vo bng, mt thnh vin ca i ng pht trin - c b tr
l ng- i theo di theo di vn v bo ng qun l d n khi k
hoch i ph bt ng cn - a vo hot ng giai on ny - c trnh
din bng 2.3.
Phn tch ri ro - c hon thin u tin cng sm cng tt trong d
n, nh- ng khng - c chm hn lc kt thc pha yu cu (coi ch- ng 4).
D sao, phn tch ri ro khng phi l hot ng mt ln khi d n tin
trin, nhng vn b sung c th - c d liu v nhng vn khc c
th cn - a ra khi danh sch vn . Khi c - c thng tin mi, vic
nh gi mc nghim trng hoc xc sut hn phi - c ci tin. Do
y cc bng phn tch ri ro nn - c duyt xt v cp nht nh k v
bt c khi no khi c mt s c quan trng xy ra (th d mt ch thu
ph thng bo chm tr lch trnh hay mt quyt nh thit k ch yu
thy cn phi hiu chnh).
QUAN L DU N PHN MM
30
Bng 2.3. Th d v bng ngu nhin
Vn Nghim
trng
K hoch i ph bt ng Ng- i
theo di
5 Thuyn chuyn
nhn s cao
40 Cho tin th- ng hon thnh
d n thng li.
J.Smith
1 Chm giao my
tnh pht trin
30 Yu cu lm ca m v h
thng pht trin ca d n
khc.
H.Brown
7 Chm giao h
thng ph c s
d liu
27 Thit k mt m phng h
thng ph c s d liu
dng tch hp.
W.Alda
3 Khng c chuyn
gia h iu hnh
25 B tr chuyn gia h iu
hnh ngoi cng ty v thu
lm t- vn
H.Brown
6 Truyn thng qu
chm
16 Hp ng vi cng ty
pht trin gi truyn thng
nh phn thch nghi cho gi
vi d n ny.
H.Troy
4 Thi gian p ng
ca h thng qu
chm
15 Cho vo iu khan tha
thun nng cp CPU trong
hp ng mua my tnh.
Y.Krot
2 B nh khng y

8 (khng xem xt)


2.2.3. Thc hin cc k hoch i ph bt ng.
Cc k hoch i ph bt ng - c thc hin mt trong nhng
tr- ng hp sau:
1. Vn d liu din ra hay sp xy n n ni.
2. K hoch i ph bt ng i hi - c chun b tr- c.
Ni chung, cc k hoch i ph bt ng c th - c nhn nhn theo
nh- ci k hoch, hnh ng - c xp vo ngn ko phng khi dng
n sau ny. D sao, trong vi tr- ng hp, k hoch - c thc hin tr- c
khi vn d liu xy ra nh- pht trin mt b m phng tr- ng hp vic
giao mt hp phn quyt nh b chm tr. Sau , nu hpw phn - c
giao ng hn th b m phng c th b b i.
Ly th d v qu trnh hon chnh chng ta th xem xt mt d n
truyn thng cn n mt my tnh trung tm ni bng mng din rng
vi vi v tr my tnh nh, c hai vn tim n - c minh nh.
- Hai my tnh c kin trc khc nhau c th din gii th thc truyn
thng thit k - c mt cch (th d trnh t ca cc t hai byte c th
b o ng- c - LSB MSB ch khng phi MSB LSB).
QUAN L DU N PHN MM
31
- Cng ty in thoi - c chn c th khng c kh nng lp t
- ng dy th nghim n ghn cho pha tch hp.
Bng 2.4. Danh mc vn d liu
Vn M t
1. th thc truyn thng
khng t- ng hp.
Hai my tnh c kin trc khc nhau c th
din gii th thc truyn thng thit k l
khc nhau.
2. - ng dy th
nghim c chm tch
hp.
Cng ty in thoi - c chn c th khng c
kh nng lp t - ng dy th nghim ng
hn tch hp.
Bng 2.5. o mc nghim trng
Vn K
vng
Tc
ng
nghim
trng
1. Th thc truyn thng khng
t- ng hp
5 8 40
2. - ng dy th nghim c chm
tch hp.
8 6 48
Bng 2.6. Bng i ph bt ng
Vn
nghim
trng
K hoch i ph
bt ng
Ng- i
theo di
1. - ng dy th
nghim c chm tch
hp.
48 t hng - ng dy
t 2 cng ty in
thoi ph
Will Doo
2. Th thc truyn thng
khng t- ng hp.
40 S dng gi thng tin
ASCII
I.Hope
Cc bng 2.4; 2.5 v 2.6 l cc bng phn tch ri ro cho d n truyn
thng . Nu kh nng c - c - ng dy truyn thng b chm li,
iu ny s gy t tc hi cho d n hn l vn th thc khng t- ng
hp. Vic theo di vn ny - c giao cho Wieliam Joo.Trch
nhim ca ng l m bo nhng - ng dy ny - c t hng t hai
cng ty in thoi khc (ch cho pha tch hp thi). Nu cng ty in
thoi c - u tin th sn sng ng hn th nhng n t hng t hai cng
ty khc - c hy v kh nng ph hy b phi tr.
Mt k s- khc Jndira Hepe chu trch nhim v theo di vn th
thc truyn thng khng t- ng hp. Ch ta phi m bo gi truyn thng
QUAN L DU N PHN MM
32
A.SCII n - c t cho c hai my tnh ph v gi CII s l lng ph
nu th thc nh phn - c chn s hot ng. Gii phng CII thay th
hu nh- chc chn chm hn nhiu, nh- ng n s cung cp gii php tm
thi cho n khi vn khng t- ng hp - c gii quyt.
2.3. Tmtt
Cc ph- ng php qun l d n hin i lc u quan tm n vic
cc vn pht trin d n (vic phng nga khng hiu chnh). Phng
nga cc vn th d hn v t tn km hn l gii quyt chng, nhng
bin php phng nga c hiu qu nu l:
- nh v cc vn v cc vn tim n sm.
- Gii quyt vn tr- c khi tut khi tm tay.
- Lp k hoch tr- c cho nhng vn tim n
C rt nhiu vn c bn chung cho hu ht cc d n phm mm.
Phn ln nhng vn dn xut t:
- Xc nh yu cu khng y .
- Thay i lun
- D ton ti
- Ty thuc ngun bn ngoi (ng- i bn, ch thu ph v.v...)
- Kh khn trong kt thc d n.
- Lun lun thay th nhn s pht trin (thuyn chuyn nhn s)
- Theo di v gim st khng y .
Cch tt nht nh v mt vn l sm i tm kim n. R rng,
tr- c ht l tm xem u vn hay din ra nht. Chng hn nhng
thay i c t yu cu lun lun v khng kim tra l khng thun li
- c coi nh- ngun gc ch yu ca nhng vn thit k. Nhng ch
thu ph v ng- i bn khng gim st - c l mt trong hu ht nhng
ngun gc bt ng, nhng vn k thut l- u li v chm ch vo pht
cht. Vi nh qun l d n th bit - c y phi tm l quan trng
nh- bit - c phi lm g.
Bit phi lm g bao gm c vic chun b tr- c s xut hin ca vn
. Trong nhiu tr- ng hp, cc vn c th - c d liu. Nh qun l
d n c th lp k hoch v kh nng vn s xy ra bng cch d tnh
xc sut ca n, - c l- ng tc ng ca n v chun b gii php thay
th. Ci - c gi l phn tch ri ro v l bin php c hiu qu trong
vic u tranh vi nhng vn pht trin tim n.
Tin hnh phn tch ri ro c ngha l chun b tr- c. y l mt hnh
thc bo him m t- ng c bn l nu mt vn xy ra, gii php
sn sng. Ging nh- mi bo him, phn tch ri ro th- ng phi tr gi.
Chi ph chun b cho vic mt vn xy ra tr- c ht l chi ph c gii
php gm thay th c sn trong tay. Trong mt s tr- ng hp, chi ph c
th ch l ti thiu: thi gian cn phn tch v lp ti liu cho gii php
v thi gian theo di vn . Trong cc tr- ng hp khc, chi ph c th
ln ng k: gi ca mt b phn thay th ca thit b pht trin. Trong
QUAN L DU N PHN MM
33
mi tr- ng hp vn - c phn tch gii quyt tr- c li n gin
hn l gii quyt vn xy ra bt ng.
Bi tp
1. Mt cng ty dch v truyn hnh cp ang chun b thit lp dch v
trong thi gian 8 thng. Cng ty cung cp dch v cho cc khch hng vi
ph c nh hng thng ty thuc qui m dch v m h yu cu. Cng
ty cng cho chiu nhng phim mi mi phim c th cho mt khch hng
xem theo yu cu qua in thoi vi cng ty.
Gi cng ty ang trong qu trnh t mua thit b, mua cc ph- ng tin
v k vi khch hng. Mt cng ty phn mm hp ng pht trin mt
h thng lm ha n cho cc khch hng. H thng ny s giao din vi
thit b nhn thng tin v cc bui chiu phim mi trn mn nh v s
giao din vi c s d liu ca khch hng thng tin u n v ha
n hng thng.
Bn hy chun b mt danh mc m- i vn gay cn nht m bn chi
liu trong pht trin d n lm ha n. Hy tha thun l do ca vic
chn la vn ca bn.
2. Bn hu tnh mc nghim trng cho mi mt vn tim n m
bn nh bi tp 1. Hy gii thch vic phn nh ca bn v tc
ng ca d n v cc tr gi xc sut.
Hy gi mt ph- ng php thay th phn nh mc nghim trng
cho nhng vn d liu m n cng tnh c chi ph chun b cc k
hoch i ph bt ng.
3. Bn hy gi nhng k hoch i ph bt ng cho nhng vn d
liu m bn nh bi tp 1. Hy xt hai k hoch thay th khc nhau
cho mi mt vn . Xt chi ph ca mi k hoch thay th v sau
chn k hoch tt nht da trn ph- ng php thay th phn nh mc
nghim trng m bn gi bi tp 2.
Hy chun b mt bng i ph bt ng c cha cc k hoch i ph
bt ng m bn chn.
4. Bi tp lp: Chia lp thnh cc nhm 3 hay 4 sinh vin. Giao cc
bi tp 1, 2, 3 cho mi hm. Yu cu mi nhm trnh by phn tch ri ro
ca mnh cho s nhm cn li trong lp.
Tho lun :
a) Cc danh mc vn d liu khc nhau.
b) Cc k hoch ngu nhin khc nhau.
c) Cc ph- ng php khc nhau phn nh mc nghim trng
(liu c 2 nhm no gi ph- ng php t- ng t ?)
QUAN L DU N PHN MM
34
Ch- ng ba
Pht trin phn mmtheo hp ng
Mi quan h khch hng - nh pht trin
Do nhng thay i nhanh chng trong cng ngh trong vi thp nin
gn y, cc t chc cng ngh cao ngy cng thy cn thit phi chuyn
ha trong cc lnh vc c chng, xc nh r vic chuyn ha khng ch
xc nh nhiu nhnh mi ca cng trnh hc m cn xc nh nhng
lnh vc chuyn mn trong phm vi cc b mn cng trnh hc. iu ny
c bit ng vi cng trnh phn mm.
Thng th- ng, cc t chc khng chuyn ha trong pht trin phn
mm li thu cc t chc khc pht trin phn mm cho h. Ngay c
nhng t chc c pht trin phn mm ca chnh mnh c th quyt nh
thu cc chuyn gia bn ngoi nhng lnh vc c chng. IBM thu
Mirosoft pht trin h iu hnh PC DOS, v Microft c kinh nghim
trong pht trin cc h thng vi tnh cn IBM th khng.
Ch- ng ny cp n mi quan h gia khch hng v nh pht trin
phn mm v cung cp mt s h- ng dn lm sao trnh nhng ci by c
in do nhng li ch mu thun nhau. Mc d nhiu nhng vn l
chung cho mi quan h khch hng, nh pht trin mt s vn tranh
ci l c bit cho pht trin phn mm. Vic pht trin phn mm l rt
t xc nh hn v nhiu ri ro hn cc lnh vc khc ca cng ngh. iu
ny th- ng dn n nhng hiu lm v bt ng ng ra c th trnh
- c nu - c d liu v kim ch sm.
tiu chun ha thut ng ca chng ta, t chc m ngh - c
trnh s - c coi l khch hng v t chc trnh ngh s - c coi l
ng- i ngh. Cc t khc th- ng - c s dng ni khc cho ng- i
ngh bao gm ng- i u thu, ng- i bn hng hay ch thu v cho khch
hng l ng- i yu cu hay ng- i xut yu cu. T chc - a ra ngh
- c thng thu sau khi la chn, s - c coi l nh pht trin.
3.1. Chi ph cng thmi li vi gi c nh.
Th- ng vn c mu thun v quyn li thc s hay t- ng t- ng gia
khch hng vi ng- i pht trin. Khch hng th mun chi ph t hn v
ng- i pht trin li mun thu nhp nhiu hn. Nh- chng ta s thy mi
quan h tt gia ng- i pht trin v khch hng khng cn thit phi dn
n tranh chp v quyn li nh- th.
C bn c 2 loi quan h theo hp ng gia khch hng v ng- i sn
xut.
1. Chi ph cng thm (cng gi l chi ph theo thi gian v vt liu)
QUAN L DU N PHN MM
35
2. Gi c nh
Hu ht cc quan h khc l mt hnh thc phi hp no gia hai
loi .
3.1.1. Hp ng ph cng thm
Ph cng thm l mi quan h theo hp ng theo ng- i pht trin
- c tr cho chi ph dch v lm v thm vo y - c php h- ng
mc li tha thun. iu ny thc ra ging nh- cho thu t: khch hng
tr cho s thi gian xe - c s dng (theo gi, ngy, tun v.v...) v cho
mi chi ph khc nh- bo him v xng. Theo th trong mt hp ng ph
cng thm, tng ph ca mt d n ch - c bit sau khi d n hon
thnh.
Ly th d, cng ty Alpha c th hp ng vi cng ty phn mm Bta
pht trin mt h thng. Cng ty Bta s - c cng ty Alpha tr cho
180 cho mi gi cc k s- ca minh u t- cho d n mt khon 20% b
sung c th - c thm vo b p dch v qun l th- k hay vn
phng khc. Cc chi ph ph pht sinh bi cng ty Bta v li ch ca d
n sau - c cng ty Alpha bi hon cc chi ph c th b p cc
lnh vc nh- :
- Thit b pht trin ca mc ch c tr- ng (my tnh, cc b dch,
cc mng v.v...)
- Chi ph i li pht sinh bi nhn vin cng ty Bta v li ch ca d
n.
- Thit b mc tiu do cng ty Bta cung cp cho cng ty Alpha s
dng.
- Dch v t cc ngun bn ngoi khc do cng ty Bta yu cu cho d
n.
Khch hng (Cng ty Alpha) c th yu cu ng- i pht trin (Cng ty
Bta) phi - c php tr- c v mi chi ph n l pht sinh v- t qu $250
v mi chi ph v- t qu $600 tng s hng thng. Mi vic cho php nh-
th lun phi bng vn bn. iu ny xc nh quan h hp ng c bn
v ph cng thm gia hai cng ty. Trong nhiu tr- ng hp, hp ng ph
cng thm l tt nht c vic pht trin. Tuy nhin, cng c v khi cc
vn tim tng. Mt mu thun v quyn li c th xy ra do ng- i
pht trin ng c hon thnh d n nhanh, hoc do khch hng min
c- ng cho php cc chi ph ph thm. Ph cng thm th- ng thch hp
cho nhng d n nh khng xc nh khi c kh khn trong vic nhn
bit tr- c cc yu cu ca d n. Trn thc t trong nhiu tr- ng hp pha
yu cu d n - c cho hng nh- hp ng ph cng thm v cc pha
cn li - c hp ng theo kiu gi c nh, giai on yu cu - c s
dng - a cc phn cn li ca d n n trng thi xc nh tt m
nh n c th - c hp ng theo kiu gia c nh i khi mt cng ty
QUAN L DU N PHN MM
36
- c ban cho hp ng ph cng thm cho giai on yu cu v cng ty
khc - c ban cho cc hp ng gi c nh vi giai on cn li.
Ph cng thm c th - c chung khc hng mun nm quyn kim
sot qu trnh pht trin. Trong mt s tr- ng hp, ng- i sn xut - c
coi nh- phn m rng ca t chc ca khch hng v cc hot ng pht
trin do khch hng qun l.
Hp ng ph cng thm phi bao gm cc iu sau:
- Danh sch nhng ng- i - c giao lm d n
- Xc nh cng vic
- T l phn trm giao vic cho mi ng- i
- Mc cng vic hng gi hay hng ngy cho mi ng- i
- Tng ph hnh chnh
- Chi ph - c php - c bi hon
- Th tc lm ha n
- Th tc thanh ton
- Th tc kt thc
Phn trm giao vic lin quan n s thi gian m mi ng- i s ginh
cho d n. N c th l 100% cho mt s k s- , v 50-60% cho cc
chuyn gia trong cc lnh vc c bit. T l phn trm giao vic cng c
th - c tnh hiu theo ti a hay ti thiu, c ngha, chng hn, mt k
s- bo hnh cht l- ng s ginh khng qu 20 gi mi tun cho d n v
khng t hn 10 gi mi tun cho d n.
Gi sut lp phiu c th l gi sut c nh cho mi ng- i - c giao
vic ca d n hay mc c nhn c th - c t cho tng lp ng- i.
Chng hn vi mi gi lm vic cho d n, ng- i sn xut s lp phiu
thanh ton $80, bt k ai lm vic cho gi . Hay hp ng c th qui
nh l k s- thit k - c lp phiu thanh ton $120 mt gi, ng- i lp
m $60 mt gi, ng- i vit t- liu $50 mt gi v c tip tc. Ph- ng
php gi sut lp phiu hp ng ph cng thm hu nh- kh nht l
ph- ng php lp phiu thanh ton c nhn theo Franh Jones - c lp
phiu thanh ton $90 mt gi John Shith $75 v.v... iu ny c ngha mi
khi mt ng- i - c thay th hay b sung cho d n th gi sut theo gi
li phi th- ng tho li.
i vi mt t chc pht trin phn mm, c th c nhng thun li
thit thc trong cc hp ng ph cng thm bao gm:
- Khng c ri ro ti chnh hay kinh doanh
- Thu thp bin thc v kinh nghim da vo mt t chc khc.
D sao, nh- trong phn ln tr- ng hp, nhng thun li li i i
vi mt s bt li bao gm:
- Li tc kihnh doanh thp
- C th c s bt bnh trong nhn s
- Kim tra nhn s v cng vic pht trin
- Bt ng tim n vi khch hng do thiu cc b gim i, mc ch
- c xc nh r v nhn t thc y.
- Tnh k tc ca hp ng khng bo m
QUAN L DU N PHN MM
37
Hu ht nhn vin chung c - c xc nh r rng v tn ti m h ty
thuc. Trong hp ng ph cng thm, nhn vin lm vic trong phm vi
tn ti ca khch hng nh- ng li thuc v tn ti ca ng- i pht trin v
iu ny c th gy ra bt mn.
Ni chung, theo quan im ca ng- i pht trin, hp ng ph cng
thm l mi quan h kinh doanh vng chi, li tc thp, khng ri ro.
Theo quan im ca khch hng thun li ca hp ng ph cng thm
l:
- Duy tr s khng ch pht trin
- Khng cn cam kt cho ton b hp ng d n
- Ri ro kinh doanh c th gim - c (do kh nng kt thc hp ng
bt c lc no)
Bt li c th c ca khch hng l:
- Chi ph pht trin gia tng
- Khch hng phi m nhn ri ro trong pht trin
- Tham gia nhiu hn trong pht trin
- Bt ng tim n vi ng- i do thiu mc ch xc nh r v nhn t
thc y.
Vi khch hng, kh xc nhn s hp dn ca hp ng ph cng
thm. R rng iu ny thy thuc loi d n v iu kin d n
pht trin cng nh- nhn nh kinh doanh khng k thut khc.
3.1.2. Hp ng gi c nh
Hp ng gi c nh l mt cam kt ca ng- i pht trin s cung cp
sn phm hay dch d tha thun vi ph tha thun trong phm vi lch
trnh tha thun. iu ny t- ng t vi mua tc k xe but theo cng
i xe but tha thun - a khch hng n ni nht nh trong phm vi
thi gian biu cng b vi ph tha thun. Tt nhin, du khch c th
chn thu xe ch khng mua tic k xe but v ri t mnh li n ni ca
mnh. D sao, iu ny c th tr nn tn km hn v i hi ng- i du
khch i cht k nng v kin thc tr- c nh- k nng li xe v kin
thc v hnh trnh n ni. Nu du khch (hay khch hng) phi quyt
nh gia vic t mnh to ra dch v v vic hp ng vi ai cung
cp dch v.
Hp ng gi c nh ch c th - c vn dng cho mt d n xc
nh r. C khch hng v ng- i sn xut phi c kh nng xc nh sn
phm hay dch v cui cng mong mun. Mt khi iu ny thc hin
- c, mt trong nhng yu km chnh ca hp ng c nh s - c
khc phc.
Li ch ca hp ng gi c nh cho ng- i pht trin l:
- Khng ch y qu trnh pht trin
- Li ch kinh doanh c th cao hn
- Cam kt cho d n trn vn
QUAN L DU N PHN MM
38
- Cam kt cho d n trn vn l - u vit ng k so vi hp ng ph
cng thm n c th kt thc bt c lc no ty s xt on ca
khch hng.
Tt nhin, hp ng gi c nh cng c mt s bt li cho ng- i pht
trin, bao gm:
- m nhn ri ro kinh doanh v pht trin
- Bt ng tim n vi khch hng do:
+ thay i yu cu lin tip
+ tiu chun hon thnh d n
+ gii thch yu cu
Mt t chc phn mm thng li s th- ng chung hp ng gi c
nh. th- ng l nhng d n to dng danh ting chuyn mn ca
cng ty v pht sinh li tc m bo tng tr- ng. Bt hnh l nhng d
n ny cng gy ra l v th- ng tc hi nghim trng cho cng ty cnh
tranh gay gt c hp ng quan trng i khi lm cho cng ty nhn
thu gi thp cui cng gy ra l cho ng- i pht trin.
Hu nh- khng th trnh khi trong bt k d n no ng- i pht trin
- c i hi thay i yu cu trong qu trnh pht trin. Nhng thay i
nh- th th- ng i lin vi chi ph b sung i khnh hng v bao gi
cng l nguyn nhn bt ng gia ng- i sn xut v khch hng. iu
ny th- ng l do yu cu khng r hay m h v n li dn n bt ng
v ch tiu trong vic hon thnh d n. V c bn, iu ny lm cho hp
ng tr li trng thi khng - c xc nh y .
Theo quan im ca khch hng, - u vit ca hp ng gi c nh bao
gm:
- Ngn sch c nh cho d n
- Hu ht cc ri ro pht trin - c chuyn sang ng- i pht trin
+ tham gia ti thiu trong qu trnh pht trin
Bt li cho khch hng l:
- Ri ro chm giao sn phm
- Gim s khng ch qu trnh pht trin
- Bt ng tim n vi ng- i sn xut do:
+ chi ph cao v thay i yu cu
+ ch tiu hon thnh d n khng r rng
- gii thch yu cu
Ngay d quyn li ca ng- i sn xut v khch hng c th khc nhau,
vi hp ng gi c nh vn th- ng - c c hai bn - a chung. Nu d
n l chi tit mc v r rng v nu quan h gia hai bn - c xc
nh r th cc hp ng gi c nh c th c li cho c ng- i pht trin
ln khch hng.
3.2. Cc mi quan h khc gia khch hng - nh
pht trin.
QUAN L DU N PHN MM
39
Ph cng thm v gi c nh l hai trong s mi quan h theo hp
ng c truyn gia ng- i pht trin v khch hng. C nhiu bin thc
ca hai mi quan h c bn k c cc ghp ni ph hp vi cc d n
c tr- ng. Mt s trong nhng quan h lin kt vi vai tr ca khch
hng v ng- i pht trin nhm to nhiu yu t kch thch hn cho ng- i
pht trin h tr mc tiu ca khch hng ngoi nhng trnh nhim theo
hp ng.
Nhng loi khc ca quan h khch hng - ng- i pht trin bao gm:
- Phi hp gi c nh v ph cng thm
- Lin doanh
- Tha thun bn quyn
- Cam kt lu di
O phn 3.1. chng ta xem xt mt th d v d n phi hp ph cng
thm v gi c nh trong phn cc yu cu - c pht trin theo ph
cng thm v cc phn cn li ca d n pht trin theo ga c nh.
Lin doanh l nhng tr- ng hp m ranh gii gia khch hng v
ng- i pht trin c th tr nn m o v phin nhng thun li v bt li
tho lun tr- c y c th khng vn dng - c. C nhiu tr- ng hp m
mt s hnh thc lin doanh c th l mong mun cho hai bn nh- khi
ng- i pht trin mun duy tr quyn v sn phm, hay khi ng- i pht
trin chung sc vi khch hng ti tr mt phn ca c gng pht trin.
C mt cch m khch hng c th cho ng- i pht trin tham gia va
phi vo mt kinh doanh ca d n l bng cch thay th bn quyn coi
nh- thanh ton mt phn. iu ny to nn qui m b sung cho li ch
ca ng- i pht trin trong thnh cng ca d n. Bn quyn thng th- ng
l ch tht bi ca d n c th to nn t li nhun cho ng- i pht trin
hn l mt hp ng gi c nh thng thng v thng li ca d n s
lm tng li nhun ca ng- i pht trin.
Mi quan h lu di th- ng l quan trng cho ng- i pht trin. Trong
nhiu tr- ng hp, nhng cam kt di hn cng nm trong li ch ca
khch hng. iu ny xy ra khi ng- i pht trin do gn b hp ng
ban u, ginh - c li th ch yu, qua kin thc thu l- m, i vi
nhng ng- i khc cho cng vic pht trin tip sau. R rng khi ng- i
pht trin hon thnh thng li mt d n ln v phc tp, anh ta c - c
mt li th ng k so vi cc cng ty khc v cc tng c- ng trong
t- ng lai ca d n . Cam kt lu di theo c th c li ch t- ng h
cho c hai bn trong khch hng m bo cc dch v sau ny ca
ng- i pht trin v ng- i sn xut m bo cam kt thu nhp lu di.
3.3. Yu cu i vi mt xut (RFP).
Pht trin phn mm theo hp ng bt u t vic khch hng la
chn ng- i pht trin phn mm. Yu cu v ngh hay RFT ( Anh
cng gi l mi thu) l b- c u ca qu trnh la chn. hiu xem
QUAN L DU N PHN MM
40
RFP cn - c chun b th no, tr- c ht chng ta hy duyt li cc b- c
dn ti mt quyt nh - a ra yu cu v ngh.
Trong ph- ng php tip cn theo pha pht trin phn mm, th pha
tim d n th- ng - c xem l pha thai nghn. y l giai on m
t- ng ng sau d n kt tinh v hnh thnh d n.
y cng l giai on m t chc quyt nh xem d n c th - c
pht trin ni b hay s phi hp ng vi mt cng ty khc. Cc RFP
khng ch - c pht ra cho cc d n hon chnh; chng cng c th
- c pht ra bo d- ng phn mm ca mt h thng hin c hay cho
ring mt pha n l ca d n. Mi RFP chun b k phi c cng thng
tin c bn; cc RFP khng hon chnh cho kt qu l cc ngh khng
hon chnh.
3.3.1. Mt s vn c bn.
Tr- c khi thu cc dch v pht trin ca mt t chc khc, mt s vn
c bn cn - c xem xt ti:
- Cc mc tiu ca d n l g ?
- Cc t chc no l g - c xem xt cho cng vic ?
- Loi hp ng no s - c cho (gi c nh, ph cng thm v.v...)
- Phi nhn - c cc p ng no t cc nh pht trin sao cho p
ng c th - c xem xt ?
- Khi no qu trnh la chn ng- i pht trin phi - c han tt ?
- Khi no d n phi hon thnh v khi no cc hp ng trung gian
phi sn sng ?
- Ai, trong t chc, - c y thc trch nhim la chn ng- i pht trin
?
- Mc ngn sch no - c ginh cho hp ng ?
Tt c nhng vn trn phi - c nu ra y tr- c khi sang b- c
sau: vic chun b cc RFP.
3.3.2. Chun b cc RFP.
Yu cu tt vi ngh l yu cu s li cun - c nhng p ng tt
nht ( ngh). Chun b RFP tt th- ng i hi s cng tc ca nhiu
ng- i, mi mt ng- i - c giao trch nhim v nhng phn c bit ca
RFP.
Mt RFP phi bao gm nhng phn sau:
1. Pht trin vn v cc mc tiu d n.
Phn ny cung cp thng tin nn tng chung, k c m t vn cn
- c gii quyt. Phn ny phi cung cp mi chi tit thch ng cn thit
hiu vn , k c biu , bo co v th d.
2. Cc yu cu k thut.
Phn ny m t nhng yu cu k thut c bit ca h thng nh- :
- Cc giao din vi cc h thng hin c
QUAN L DU N PHN MM
41
- Yu cu c s d liu (nh- kh nng yu cu, cc quan h d liu
v.v...)
- Truyn thng v kin trc mng
- Cc chun qun s, chun chnh quyn hay cc chun - c yu cu
khc
- Ph- ng php lun pht trin yu cu
- tin cy ca h thng
- Rng buc v thi gian
- Ngn ng lp trnh
- My tnh ch (host computer)
3. Thng tin qun s
Phn ny cung cp thng tin lin quan n vic trnh by v th cht
ngh nh- :
- Ai c th tr li cho RFP
- Yu cu lm sng t v cc thng tin b sung th no
- Ngy thng v a im hp theo lch vi cc nh ngh
- Ch tiu la chn ngh
Phn ny cng c d phng v kt qu l t chc - a ra RFP s khng
b bt buc la chn ngh chi ph thp nht hay bt c ngh no
khc.
4. Yu cu chi ph
Mi vn ti chnh - c nu phn ny. iu ny bao gm c cu
gi yu cu trong ngh cng nh- mi thng tin c tr- ng - c nu
trong ngh (nh- gii trnh chi ph tng cng hay lp gi ring cho mi
pha). Phn ny cng c th nu r loi hp ng pht trin no - c cho
(bn quyn, ph cng thm, gi c nh v.v...)
5. T- liu tham kho
Phn ny bao gm danh sch mi t- liu nu trong RFP nh- tiu
chun, t- liu h thng hin c, ti liu v sn phm khc v.v...
6. Nhng th giao - c c yu cu
Phn ny bao gm gii thch ban u v tuyn b cng vic (Sow).
Ch yu y l danh sch nhng th giao - c chnh ca d n nh- t-
liu, phn mm, o to v mi phn cng hay thit b thch ng. Phn
ny cng tho lun vic bo m theo yu cu cho h thng s - c giao.
7. nh dng ngh yu cu
nh dng tiu chun yu cu cho ngh - c m t phn ny.
iu ny bao gm ni dung yu cu v:
- ngh k thut
- ngh qun l
- ngh v gi c
- Tuyn b v cng vic
Th d v mt phc ha ngh c nu phn 3.4
Phn ny cng bao gm danh sch mi thng tin c th ph lc cho d
n, ngoi ngh c bn (theo nh dng m t trn) n c th bao gm
QUAN L DU N PHN MM
42
bo co ti chnh mi nht ca t chc ng- i ngh hay y nhim th-
k thut ca ng- i ngh.
8. trnh lch trnh v lch quyt nh
Nhng thi hn quyt nh lin quan n RFP - c m t phn ny.
N bao gm thi gian chm nht cho vic trnh mt ngh v ngy
d kin tin hnh la chn. N cng c th bao gm lch trnh mong
mun hon thnh cng vic pht trin.
Mt trong nhng mc tiu ca RFP l lm cho nhim v so snh cc
ngh - c d dng. Nhim v ny c th tr nn cc k kh nu mi
ngh - c cu to khc nhau hay nu cc ngh - c da trn
nhng gi nh rt khc nhau. Phn 3 ca phc tho ni n cuc hp vi
mi ng- i ngh. Cuc hp ny to c hi m bo l mi ng- i
ngh u c mt c s chung v hiu bit v kt qu trong cc ngh
ca h d so snh hn. N cng thnh t khi yu cu khun dng
ngh tiu chun ha, nu phn 7 ca RFP. Bng 3.1. c nu phc tho
i khi ca mt RFP.
Bng 3.1. Phc tho khi qut cho mt RFP
1. Tuyn b vn v mc tiu d n
- M t hin trng
- T- liu h tr
cc bo co
cc biu
cc th d
- M t vn
- Cc mc tiu
2. Yu cu k thut
- Giao din vi cc h thng hin c
- Truyn thng v kin trc mng
- tin cy ca h thng
- Ngn ng lp trnh
- Yu cu c s d liu
- Tiu chun qun s hay chnh quyn
- Rng buc thi gian
- My tnh ch
3. Thng tin qun tr
- Ai c th p ng cho RFP
- Ngy thng v a im cuc hp theo lch vi mi ng- i ngh
ng- i - c chn- Lm sao yu cu sng t hay thng tin b sung
- Tiu chun la chn ngh
- Cc thng tin qun tr khc
4. Yu cu chi ph
- C cu gi
QUAN L DU N PHN MM
43
cc dch v
cc sn phm
cung cp
- Bin minh cho chi ph
- Lp gi ring tng pha
- Loi hp ng pht trin - a cho
- So snh chi ph ca cc gii php thay th
5. T- liu tham kho
- Cc tiu chun
- T- liu v h thng hin c
- T- liu v sn phm
6. Yu cu chuyn giao
- T- liu
- Phn mm
- Phn cng hay thit b thch ng
- Bo hnh h thng s chuyn giao
- T- liu v sn phm chuyn giao
- Hun luyn
- Cc cng c pht trin v th nghim
7. nh dng ngh
- ngh k thut
- Tuyn b cng vic
- ngh qun l
- B sung v ph lc
+ bo co ti chnh ca t chc ngh
+ y nhim th- k thut ca ng- i ngh
+ Tm tt nhn s ch lc
- ngh lp gi
8. Lch trnh v lch quyt nh
- Hn cui cng phi trnh ngh
- Hn d kin tin hnh la chn
- Lch mong mun hon thnh cng vic pht trin
3.3.3. Pht yu cu xut RFP
C pha ph- ng php c bn trong phn pht mt RFP
- Theo mt danh mc phn pht hn ch
- Theo mt danh mc phn pht rng
- Cho mi ai yu cu
Danh mc phn pht hn ch ch c nhng t chc no - c la
chn theo b ch tiu c tr- ng. Nhiu c quan chnh ph duy tr mt
QUAN L DU N PHN MM
44
danh mc cc cng ty - c cng nhn cho mi lp RFP. Ph- ng php
ny loi tr cc t chc t c c may, - c la chn.
Danh mc phn pht rng bao gm bt c t chc no c cht t c
may - c la chn. Vi mt t chc - c b sung vo danh mc ch yu
cu iu . Cc danh mc phn pht rng c th thch hp cho nhng d
n nh, nhng d n khng i hi gim nh c bit no hay nhng d
n m t t chc thch hp nhm vo.
Khng phi khng ph bin l thng tin RFP ban u - c qung co
trong bo ch hay tp ch chuyn ngnh. Nhng qung co m t vn
tt RFP v mi cc cng ty yu cu bn sao ton b RFP. S tip cn ny
th- ng l thch hp khi phi tm kim nhng ng- i ngh tim n mi.
Bt k ph- ng php phn pht no - c chn la, t chc xut RFP
phi nh l th tc xut khng kt thc cng vi s phn pht RFP. T
chc phn pht phi sn sng cung cp mi thng tin b sung v lm
sng t nhng iu c th - c yu cu. C mt cch thc hin iu
ny l (nh- nu). nh lch hp vi mi ng- i ngh tim n nhm
lm sng t v tr li mi cu hi. Cuc hp ny cng l mt c hi cho
cc ng- i ngh i xem mt l- t cc thit b mc tiu v bit - c tn
mt nhng vn s - c d n gii quyt.
3.4. xut
Cc loi kin ngh c th phn thnh 2 phm tr c bn:
- Kin ngh do yu cu
- Kin ngh khng yu cu
Kin ngh do yu cu p ng hoc RFP chnh thc hoc li mi c
bit trnh kin ngh trong khi kin ngh khng do yu cu thng
th- ng do ng- i ngh x- ng xut. Tt nhin c nhiu t hp ca hai
biu v nh- tnh hung k d nh- ng th- ng thy khi mt kin ngh
khng do yu cu lm ny sinh vic xut RFP chnh thc.
3.4.1. xut khng do yu cu
Nhng kin ngh khng do yu cu t chnh thc hn nhiu so vi kin
ngh do yu cu v th- ng khng l g hn l mt b- c u dn n
nhng th- ng tho chnh thc hn. Kin ngh khng do yu cu phi c
nhng phn c bn sau:
1. Bin minh vic trnh kin ngh
2. M t vn phi gii quyt
3. M t gii php ngh
4. M t t chc ca ng- i kin ngh
5. Tng quan chung v kinh ph ca gii php ngh
Phn bin minh l ct t v n gii thch v sao khch hng nn c
tip. Phn ny c th nu, chng hn l ng- i kin ngh pht trin mt
cng ngh mi v thnh cng c li cho khch hng v c th thch nghi
vi nhu cu ca (t chc) khch hng. Mc tiu chnh l cung cp cu tr
QUAN L DU N PHN MM
45
li cho cu hi ca khch hng: V sao cng ty li tip cn ti v v sao
c kin ngh ny l li ch cho ti vy ?
Cc phn 2 v 3, m t cch m kin thc chuyn mn ha ca ng- i
ngh - c vn dng vo cc vn ca t chc khch hng. iu ny
i hi ng- i ngh nghin cu t chc ca khch hng nhm m bo
l kin ngh cung cp mt gii php thc cho mt vn thc.
Chi ph chnh xc ca gii php khng cn nu giai on ny. Mt
kin ngh khng do yu cu hin khi - c chp nhn ngy vng u.
Mc tiu chnh ca n l to dng mt mi quan tm. Nu kin ngh to
mi quan tm th ng- i ngh s - c mi tho lun kin ngh v
sau s trnh li cho khch hng mt bn trnh by kin ngh chi tit
hn.
3.4.2. xut khi c yu cu
Kin ngh do yu cu m khch hng x- ng xut coi nh- p li RFP
chnh thc hay mt hnh thc mi no khc trnh kin ngh. Tri vi
tnh cht khng chnh thc ca kin ngh khng do yu cu, kin ngh do
yu cu l hon chnh v chi tit v ni dung th- ng rng buc cho ng- i
ngh (kin ngh coi nh- vn bn rng buc - c tho lun thm v
minh ha li ni u ch- ng 10). Cng vi yu cu trnh kin ngh,
khch hng cng c th dn chnh xc kin ngh phi - c chun b th
bo v trnh ra sao. Mt th d v dng kin ngh chnh thc c bng
3.2.
Mt lnh vc c bn khin kin ngh do yu cu v khng do yu cu
khc nhau l nhu cu cnh tranh. Cc kin ngh do yu cu phi c kh
nng cnh tranh thng li vi cc kin ngh khc. iu ny c ngha l
vic chun b kin ngh do yu cu phi - c coi bn thn n l mt d
n mini v nh- th i hi hnh thnh mt i ng chun b kin ngh.
3.4.3. i ng chun b xut.
Vic hnh thnh ban kin ngh l c bn cho bt c t chc no d tnh
p ng thnh cng mt RFP. Ban ny ch nh ng- i c nhim v nh
v nhng RFP thch hp v trnh tho lun cn c - ng li ch o
do ban ra. - ng li ch o phi nhm vo cc RFP m:
- nm trong phm vi - ng li kinh doanh ca cng ty
- nm trong phm v gii hn kch th- c c tr- ng (d n khng qu
nh v khng qu ln).
- khng hin nhin loi tr chnh cng ty (th d yu cu gim nh
c bit hay hon thnh v an ninh).
Cn c nh gi ca ban v cc RFP ek trnh, ban kin ngh quyt
nh RFP no l p ng - c bi cng ty.
Sau ban kin ngh la chn i ng chun b cho mi kin ngh.
i ng ny c th ch c mt ng- i hay nhiu thnh vin, ty thuc
qui m ca d n kin ngh. i ng rt t kinh nghim v gim nh ca
QUAN L DU N PHN MM
46
mi nhn vin cng ty v nu cn c th s dng dch v ca cc chuyn
gia bn ngoi h tr trong vic chun b kin ngh.
Kin ngh c bn yu cu trong phm vi i ng chun b kin ngh
bao gm:
- Kin thc k thut lin quan n mi lnh vc ring r do kin ngh
l- u
- Qun l d n k c d ton v lp k hoch
- Kin thc ti chnh, k c nh ngn sch v lp k hoch ti chnh
cho ton b d n
- Quen thuc vi t chc ca khch hng
- Kinh nghim trong son tho kin ngh
Mt thnh vin ca i ng s - c ban ch nh lnh o i hay iu
phi vin. Sau khi i ng hnh thnh, hai vic y thc u tin ca n
l:
1. Duyt xt b- c u RFP
2. Chun b lch trnh cho vic hon thnh d n v giao trch nhim.
Vic chun b kin ngh tt tn tin v phi - c coi nh- l mt u t- .
Nu vic ny lm tt, n c th to ra li nhun, ngn sch kin ngh
khng thch hp s gim c may to ra kin ngh thng - c cc thnh
vin ca i ng chun b kin ngh phi - c tp trung vo nhim v v
h phi - c cung cp ngun lc thch hp.
Theo Silver (1986), cc kin ngh trong cng ngh cao v cc cng
nghip hnh khng v tr - c cp ngn sch khong 2% tr gi hp
ng, d sao, phm vi chi ph. Kin ngh l 1% n 10%. Chi ph hp
ng cng cao, phn trm dnh cho chun b kin ngh cng thp.
6
3.4.4. Khun dng xut
Kin ngh tt phi tr li - c 6 cu hi c bn: ai, ci g. ti sao, th
no, khi no, v bao nhiu. p n cho cc cu hi n gin lin quan
n:
1. Ai l t chc trnh kin ngh ?
2. Ci g - c ngh ?
3. Ti sao kin ngh - c trnh ?
4. Lm sao cng vic kin ngh - c thc hin ?
5. Khi no n s - c pht trin xong v bn giao ?
6. Bao nhiu chi ph ?
6
Silver - a ra, phm vi hp ng d n $10K n $2B vi phm vi chi
ph kin ngh t 1% n 20% Hoa K v phm vi hp ng $10K n
$1B vi phm vi chi ph kin ngh 1% n 10% Chu u. Cn c hu
ht cc d n khong $250K v $100, ng da ra phm vi chi ph kin
ngh t 1,5% n 8% Hoa k v 1% n 2 1/2% chu u.
QUAN L DU N PHN MM
47
Cu hi ti sao l quan trng cho mt kin ngh khng do yu cu v
to c s cho vic trnh n. p n cho mt RFP ch cn khng nh
Kin ngh ny - c trnh phc p yu cu - c kin ngh cu
cng ty Acme Inc s 456 ngy 5/6....
Nm cu hi khc - c nhm vo 5 hp phn chnh ca kin ngh
1. Kin ngh k thut (ci g, lm sao)
2. Kin ngh qun l (lm sao, khi no)
3. Kin ngh nh gi (bao nhiu)
4. Tuyn b cng vic (ci g, khi no)
5. Tm tt iu hnh v trong phn b sung bao gm:
- Nn tng v kinh nghim ca cng ty (ai)
- Trnh chuyn mn ca nhn s ch lc
- Cc chng vt v vn bn thch hp
Bng 3.2. Mt khi lc cho mt kin ngh k thut
1. Tng quan v vn cn gii quyt
2. Tng quan v gii php kin ngh
3. Hp phn cn mua
4. Hp phn cn pht trin
5. Thit b
- C s h tng
- Phn cng my tnh
- Truyn thng v mng
- Thit b th v kin nghim
- Thit b c bit khc
- Giao din vi cc h khc
6. Hp phn phn mm
- M t chung h phn mm
- M t chi tit tng hp phn, phn mm ch yu
- Cc giao din vi cc h khc
- Cc c s d liu
- S dng cc hp phn, phn mm hin c
- Kh nng s dng li cc hp phn, phn mm cn pht trin
7. Cng trnh nhn vn v giao din ng- i s dng
8. Nhn xt c bit
- tin
- ng thi gian
- Tch hp d liu
- Sao l- u v thu hi
QUAN L DU N PHN MM
48
Bng 3.3. Mt khi l- c cho kin ngh qun l
1. Cng c v tin ch pht trin
2. Mi tr- ng pht trin
3. Yu cu nhn s v c cu i ng pht trin
4. Ph- ng php lun pht trin
5. Cc pha pht trin
6. Xt duyt
7. Bo co
- Cc loi bo co
- Khun dng bo cao
- Chu k bo co
- Danh mc phn phi
8. Cc ch thu ph
9. Cc tiu chun
10. Th nghim:
- Cc giai on th nghim
- Th tc th nghim chnh thc
- Pht hin v hiu chnh sai
11. Kim tra cht l- ng
12. Qun l cu hnh
13. Bo d- ng
14. Lch
- Danh mc hot ng ch yu
- Cc ct mc
- Cc ph thuc
- B tr v cung cp nhn lc
15. Qun l ri ro
Bng 3.4. mu khi l- c cho kin ngh nh gi
1. Loi hp ng (ph cng thm, gi c nh v.v...)
2. Chi ph pht trin do hp ng phn d n
3. Cc chi ph ch thu ph
4. Cc chi ph mua sm
5. Tng ph
6. Bo hnh
7. Li nhun
8. Tng chi ph d n
9. Loi ti tr (khng c, c)
10. Lch thanh ton
11. Lp ha n v qun tr
QUAN L DU N PHN MM
49
Tm tt iu hnh l c bit quan trng cho nhng kin ngh ln v
phc tp v khng phi tt c thnh vin ban tuyn chn u s c kin
ngh. Tm tt phi di khong 1 n 6 trang v nn c tham chiu ti cc
lnh vc c tr- ng trong d n ton b c y chi tit hn.
Cc bng 3.2; 3.3 v 3.4 gii thiu cc th d khi l- c cho cc kin
ngh k thut, qun l v nh gi ca mt d n phn mm.
3.4.5. Khng nh cng vic (Sow)
Khng nh cng vic l c s ca hp ng gia ng- i ngh v
khch hng v th- ng - c lng trong hp ng Sow bao gm danh mc
chi tit mi cng vic m ng- i ngh phi thc hin v li ch ca
khch hng Sow bt u bng danh mc chung cc th bn giao theo yu
cu trong RFP. Bn chi tit hn ca sow - c trnh coi nh- b phn
ca kin ngh v vn ch - c coi l m t b- c u ca cng vic phi
hon thnh. Bn gii thch c tnh cht rng buc ca sow - c kt thc
trong qu trnh th- ng tho hp ng hay sau khi cc yu cu chi tit ca
d n hon tt.
Bng 3.5. Gii thiu mt th d v i th Sow cho mt d n phn
mm.
Danh sch hng mc thay i nhiu, ty thuc loi d n - c pht
trin. Chng hn khng phi mi d n u bao gm vic bn giao cc
hp phn phn cng v khng phi mi d n i hi hun lun hay lp
t.
Bng 3.5. Mt phc tho SOW mu cho mt d n phn mm
1. Vn bn tham chiu
- c t yu cu
- M t h thng hin c
- RFP ca khch hng
- ngh ca ng- i pht trin
- Ti liu k thut ca ng- i bn v ng- i pht trin
2. Cc phn bn giao - c v phn mm
- Tnh chc nng (nh- dn trong c t yu cu)
- Danh mc hp phn phn mm ch yu
3. Cc phn bn giao - c v thit b v phn cng
- Tnh chc nng (nh- dn trong c t yu cu)
- Danh mc hp phn phn cng ch yu
4. Hun luyn
- Cc lp cho ng- i s dng
- Hun luyn ng- i vn hnh
- Hun luyn lp t
5. Nghin cu th tr- ng
6. Mua sm
QUAN L DU N PHN MM
50
7. Gim st cc ch thu ph
8. T- liu
- T- liu pht trin
- T- liu ca ng- i dng
- T- liu bo d- ng
- T- liu k thut khc
9. Th nghim
- Th nghim alpha
- Th nghim beta
- Th nghim nghim thu (ATP)
10. Ll t
11. Cc dch v bo d- ng
12. Cc dch v khc v hng mc bn giao - c khc
13. Ph- ng php bn giao
- Phn mm
- T- liu
- Phn cng
H- ng dn c bn cho vic chun b Sow l bt c hot ng no, dch
v hay sn phm no m khch hng yu cu v - c ng- i sn xut tha
thun phi c trong Sow. iu ny c ngha l c th c hng mc cng
vic khng bt buc - c hiu mt cch khng chnh thc hay tha thun
ming m khng c trong Sow. Sow chnh thc phi bao gm ton b
cng vic phi hon thin v ch cc cng vic thi. iu kin ny
phng nga hiu lm v bt ng sau ny sau khi d n bt u.
3.5. Duyt xt xut v qu trnh la chn.
Sau khi kin ngh - c trnh, qu trnh duyt xt theo quan im k
thut. Khng th chi ci l, qu trnh duyt xt cn lu mi tr thnh k
thut thun ty. C t qu trnh tuyn chn kin ngh l hon ton khch
quan. Cc ban tuyn chn bao gm nhng con ng- i, tt c h u c
nhng s thch ring v khuynh h- ng ring ca mnh.
Trn thc t, c nhiu tr- ng hp m qu trnh tuyn chn kin ngh l
hon ton ch quan, iu ny bao gm nhng tr- ng hp do nh h- ng
c nhn, s quen thuc v tnh bn b gia khch hng v ch thu (nh
kin) khng c c s c li cho 1 cng ty hay thnh kin khng c c s
chng cng ty khc. Nhng tnh hung kh m khc phc - c v
th- ng kh pht hin hn. Nhng tnh hung ny c th b u tranh
khc phc vi t nhiu hiu qu thng qua vic bn hng theo tm l v
k thut marketing, ngoi phm vi cun sch ny.
3.5.1. Ban tuyn chn xut.
QUAN L DU N PHN MM
51
Ban tuyn chn kin ngh l mt nhm ng- i - c ch nh duyt xt
v nh gi cc d n v khuyn dng mt trong nhng kin ngh theo b
tiu ch xc nh tr- c. iu ny c th l b h- ng dn chung hay mt
b cc yu t v th tc nh gi cc k hnh thc.
Nhiu t chc ln hnh thc ha th tc nh gi kin ngh.
7
iu
ny thng th- ng bao gm ba knh ring bit v qun l k thut v nh
gi chi ph. Mi hp phn ca kin ngh - c phn cp theo cc th tc
nh gi c bit v s phi hp cc cp c bit to nn cp cui cng
cho kin ngh.
Cc kin ngh v cc d n nh cng nh- ln hn nn - c nh gi
theo mt th tc c tnh h thng v t- ng i khch quan. Cn c vic
nh gi kin ngh, ban tuyn chn kin ngh mi trnh li ca mnh
cng vi mt tm tt cc d liu cc tnh ton v cc gi dn n vic
tuyn chn . Th- ng khi c 2 hay 3 li tin c - c trnh v vic ti
din ln hai - c bt u vi nhng cng ty - c chn nhm chn ra
- c kin ngh thng cuc.
Danh sch cc cng ty tuyn chn li ln hai th- ng - c gi l danh
mc vn v vng hai ca cc tho lun v kin ngh - c gi l vng
chn ngh tt v cui cng.
3.5.2. Ph- ng php nh gi xut.
Bng 3.6. Trang nh mc mu trong vic nh gi kin ngh.
Cp Loi M t
9-10 Xut sc Mc kin thc hay ti nng chuyn mn cao;
xng ng - c chp nhn, d hiu, iu ch trch
rt t.
7-8 Tt Trnh chuyn mn hay kin thc kh quan;
chp nhn - c, d hiu, ni chung khng i hi
b sung thm thng tin hay lm sng t.
5-6 - c Trnh chuyn mn hay kin thc chp nhn
mc ti thiu; i hi b sung thm thng tin hay
lm sng t
3-4 Ngho nn Trnh d- i mc ti thiu chp nhn - c, i
hi ci tin ng k, cho thy trnh chuyn mn
hay kin thc thp.
0-2 Bc b Thiu kh nng hon thin cng vic; d liu
khng th nh gi hay khng c.
7
Chng hn, US DOD pht ra Directives 5000.1 Major Systems Acquisition, v 4105-62
Proposal Evaluation and Source Selection, v DOD FAR Supplement 15.6 Source Election, NASA
pht Directive 5103.6a Source Evaluation Board Manual, v FAR Supplement 18-16.608-70 Proposal
Evaluation.
QUAN L DU N PHN MM
52
Hu ht cc ph- ng php nh ga kin ngh da trn mt s bin th
ca k thut gn trng s, trong mt s yu t tnh im - c gn
trng s v phi hp vi nhau to nn mt cp bc tng th cho kin
ngh Silver (1986) gi mt k thut xp loi cho mi yu t l t- ng t
nh- bng 3.6. Ch l vic m t cc loi thp hn c tnh n rng
mt c hi c th c cho ng- i ngh dng hiu chnh hng mc
- c chm im.
Sau , k thut xp loi m t bng 3.6 - c vn dng cho mi hp
phn ch yu ca kin ngh v trng s trung - c tnh. Ly th d, bn
hp phn ch yu ca kin ngh c th - c gn trng s nh- sau:
K thut 35%
Qun l 25%
Chi ph 30%
Nn tng hc vn v tr thc cng ty 10%
iu ny c ngha l hp phn k thut ca kin ngh c tm quan
trng hn bt k hp phn no khc.
Bng 3.7. Cc hng mc ngh v qun l - mc nh gi
1 Lch trnh 23%
Ngy tt 12%
Kh nng ng 5%
Ct mc (pht trin) 4%
Mc chi tit ha 2%
2 Nhn s 16%
Nhn s ch lc 12%
C cu i ng pht trin 5%
3 Khng ch pht trin 17%
Kim tra cht l- ng 7%
Th nghim 7%
Qun l cu hnh 3%
4 Ch thu ph 12%
5 Bo d- ng 10%
6 Qun l ri ro 8%
7 Ph- ng php lun pht trin 5%
8 Mi tr- ng pht trin 5%
9 Cng c v tin ch pht trin 4%
Trng s trung bnh 100%
C th - c tip tc hon thin, tiu ch nghim thu b sung c th
tuyn b rng bt c kin ngh no m mc nh gi d- i 5 phn 10
QUAN L DU N PHN MM
53
trong bt c 1 hp phn ch yu no u b kh- c t bt k s im cui
cng tnh - c ca n. iu ny c ngha, chng hn mc d cc kin
ngh k thut, qun l, chi ph tt nh- th no chng na, nu c s hc
vn v kinh nghim ca cng ty yu, th kin ngh tng th vn b bc.
Bng 3.7. cho mt th d v cc hng mc - c gn trng s trong
phm vi hp phn qun l ca kin ngh. Cc bng t- ng t cn - c
chun b cho cc hng mc to thnh mt trong nhng hp phn ch yu
ca kin ngh. C hai danh sch hng mc - c tnh im cng nh-
cc trng s ca c th cn - c khu ni cho mi RFP nhng vic khu
ni ny phi - c tin hnh tr- c qu trnh nh gi v tuyn chn. Mt
bn dn t ch yu khi qut bn u bng chung ca cc bng nh gi
nn - c duy tr lm c s cho mi d n mi.
3.6. Mt s nhn nh b xung v xut.
Vic pht trin phn mm t c tnh cht xc nh hn cc lnh vc
khc ca cng ngh cao. Thng th- ng d tnh cc yu t ca d n pht
trin phn cng hay in t l d hn nhng yu t ca d n phn mm
kinh nghim cho thy l tnh trng v- t tri ca d n phn mm. Bn
cht th- ng xuyn hn v tn km hn nhng lnh vc khc ca
cng ngh. iu ny c ngha l cc kin ngh phn mm i hi c s
quan tm c bit nhng lnh vc c tr- ng nh- lp lch trnh, phn tch
ri ro, qun l nhn s v tnh kinh ph. Nhng lnh vc ny l quan trng
bt c lnh vc pht trin cng ngh no nh- ng li cng quan trng hn
nhiu trong pht trin phn mm.
C khch hng v ng- i ngh phi c thc v nhng c im ny
v nhng c im ring bit khc ca pht trin phn mm. Khch hng
phi ginh quan tm c bit n nhng lnh vc ny khi nh gi kin
ngh ng nh- ng- i ngh phi quan tm c bit n chng khi chun
b kin ngh.
3.6.1. Nhng vn lin quan n khch hng.
Khi chun b mt RFP, mt trong nhng song thng th- ng nht
ca khch hng chnh l phi cung cp chi tit n th no y. Qu
nhiu chi tit c th lm nhng ng- i ngh nn lng khi - a cho
nhng gii php ca chnh h trong khi qu t chi tit c th pht sinh
nhng gii ph khng thch hp do thiu thng tin. Khch hng thng
th- ng c i cht t- ng l vn nn - c gii quyt ra sao. Nu
t- ng ca khch hng l kin quyt th nhng th- ng cn - c chi
tit ha trong RFP. D sao, nu nhng ng- i ngh - c t do ngh
gii php ca chnh h th mt trong nhng gii php c th tt hn rt
nhiu gii php m khch hng nhn thy r.
Khch hng c nhiu quan tm khc, chng hn nh- ci g s xy ra
khi thay i yu cy. O m- c no RFP s kha khch hng trong
gii php kn mt n mc no kh thay i - c. Hay ci g xy ra
QUAN L DU N PHN MM
54
cho mt d n khn ch n thun khng c thi gian hon thnh
- c th tc qui trnh nh gi kin ngh tt.
iu sau y tm tt nhng vn ny n hn nn l- u tm n tr- c
khi - a ra RFP.
1. Mc chi tit.
Nhng vn c bit thng th- ng i hi nhng gii php c hiu.
Nu vn l hp v - c xc nh r hay nu khch hng c gii php
c hiu trong c th phi cung cp chi tit ng k trong RFP.
2. nh gi cc kin ngh.
Vic nh gi khch quan cc kin ngh khng phi d thnh cng, t
ra hai ng- i phi nh gi v tnh im mi hng mc trong cc kin
ngh v phi tnh im trung bnh. Nu cc im - c tnh khc nhau
nhiu, th ng- i th ba (v ngay c ng- i th t- ) phi nu tnh im hng
mc v c hai im - c tnh (hay ba) st nhau nht nn - c ly
trung bnh.
3. Thay i yu cu
Tha thun chnh thc v d n gia ng- i sn xut v khch hng
phi cho php c mt s tha ng cc thay i (sm trong d n). Yu
cu ny c th - c - a vo l b phn ca RFP. Vic vn dng t tha
ng phi xem xt mt mt nhu cu, c c s ca khch hng hiu
chnh sai lm trong cc yu cu d n, mt khc nhu cu ca ng- i pht
trin trnh s v trong tnh ton chi ph v lch trnh.
4. Tit kim thi gian.
Qu trnh xut RFP, nh gi cc kin ngh v hon thnh th- ng
tho c th mt th gi. Trong mt s tr- ng hp, d n c th l khn vi
cng vic pht trin l thuc khong thi gian cng thng. Mt trong
nhng cch tit kim thi gian i vi khch hng l cho php cng vic
pht trin bt u ngay khi la chn kin ngh - c thng. iu ny
th- ng lm khi gi th- nh (LOL) cho ng- i sn xut - c chn. Ri
ro ca khch hng trong 1 LOL c th gim khi gii hn ng- i sn xut
ngn sch c bit cho n khi hp ng cui cng - c k.
5. Cc kin ngh chung nhau.
Mt trong nhng vn chnh vi kin ngh chung nhau l thiu xc
nh trch nhim r rng. Cc kin ngh chung nhau bao gi cng phi
giao trch nhim chung cho mt bn. Cc bn tham gia khc trong kin
ngh, ng vai tr ch thu ph, cng phi - c xc nh.
3.6.2. Nhng vn lin quan n ng- i ngh.
Khng c ph- ng php chc chn no vit kin ngh chc thng.
Nhng cu hi lin quan n cc vn kin trc v k thut, bao gm
cc ph- ng n chn la hay trnh gii php nhiu mi ty thuc loi
kin ngh v hon cnh trnh. Sau y l mt s h- ng dn chung cho
vic chun b mt kin ngh.
1. V ngoi ca kin ngh.
QUAN L DU N PHN MM
55
V ngoi ca kin ngh hu nh- cng quan trng nh- ni dung ca n.
t c kin ngh ln xn vng v, cu th no - c tuyn chn l kin
ngh, chc thng. Ng- i ngh quan tm rt nhiu n m hc ca vn
bn, ha, n buc, phim n chiu v bt c dn gii ming no i
km.
2. Cc yu cu ty chn.
Mt RFP cng c th bao gm c cc yu cu ty chn, C nhng c
im hay kh nng ph khng nht thit cn cho vic hon thnh d n
nh- ng li l ci m th- ng - c coi l c th p. Kin ngh c c im
ty chn c th ginh - c im ban duyt xt kin ngh.
Nhng c im ty chn - c ch nht trong phn b sung ca
kin ngh v nn - c nh gi, lp lch trnh ring r.
3. Mi yu cu.
Mt kin ngh khng nht thit p ng mi yu cu ca RFP. D sao,
phi c tham chiu mi yu cu nhm chng minh l chng khng b b
qua. Nhng chch h- ng vi RFP phi - c bin minh v gii thch.
4. Gii php thay th.
Mt kin ngh c th c hn mt gii php ngh. D sao nhng gii
php thay th phi - c trnh ring r sao chng c th - c l- u v
nh gi ring r. Cc gii php thay th khng cn thit phi nhc li
trn vn cc phn trong gii php ngh th nht, chng c th tham
chiu nhng phn (th d "t chc i ng pht trin cho gii php ca
x l phn tn vn s l nh- vi gii php H thng tp tm.)
5. Cc cng c chuyn nghip.
Cc tin ch v cng c t ng nn - c dng h tr trong vic
chun b mt kin ngh, ngoi cc b x l vn bn v kim tra chnh t
tt yu, nhng cng c nh- b trn gi lp trnh, b sinh sn thit k v
yu cu , b trn gi th v.v... C th tit kim khi l- ng ng k
cng sc v thi gian trong chun b kin ngh chuyn nghip. Mt danh
mc cc cng c v tin ch c trong th- mc m Tahvanainen cng cng
s cng b (1990).
3.7. Tmtt.
Ch- ng ny cp n quan h gia khch hng v nh sn xut phn
mm v n vic chun b, trnh v nh ga kin ngh. N cng cung
cp mt s h- ng dn v vic lm sao trnh - c nhng by c in do
quyn li mu thun nhau ca khch hng v nh sn xut. Mc d nhiu
nhng vn ny l thng th- ng cho mi quan h khch hng, nh pht
trin mt s vn l c tr- ng cho s pht trin phn mm.
V c bn c hai th loi h hp ng gia khch hng v nh pht
trin.
1. Ph cng thm (cng gi l theo thi gian v vt liu).
2. Gi c nh.
QUAN L DU N PHN MM
56
Hu ht cc quan h khc l mt hnh thc phi hp ca hai quan h
ny.
Ph cng thm l quan h hp ng m nh pht trin - c tr tin cho
chi ph dch v cung cp v thm vo y - c php mt mc li tc tha
thun, ph cng th- ng l thch hp cho cc d n nh, khng xc nh
khi kh minh nh cc yu cu ca d n. Ph cng thm c th - c
chung khch hng mun gi quyn kim tra qu trnh pht trin.
Hp ng gi c nh l cam kt ca nh pht trin trong vic cung cp
mt sn phm hay dch v tho thun vi ph tha thun trong lch trnh
tho thun. Nu d n chi tit v r v nu quan h gia hai bn - c
xc nh r th gi c nh c li cho c 2 bn.
C nhiu bin th ca 2 quan h c bn k c nhiu bin th - c
"ct may" thch hp vi d n c tr- ng.
Pht trin phn mm theo hp ng bt u bng vic khch hng
tuyn chn nh pht trin phn mm. iu ny thng th- ng - c thc
hin khi pht ra mt yu cu cho kin ngh (RFP).
C 2 phm tr kin ngh c bn: kin ngh di yu cu v kin ngh
khng do yu cu. Kin ngh di yu cu p ng c RFP chnh thc cng
nh- li mi c bit yu cu trnh kin ngh trong khi kin ngh khng
do yu cu thng th- ng do ng- i ngh khi x- ng.
Sau khi trnh kin ngh cho khch hng, mi kin ngh - c ban
tuyn chn duyt xt. Cc kin ngh cho d n nh cng nh- ln phi
- c nh gi theo 1 th tc c h thng v t- ng i khch quan.
Th- ng hai hay ba li khuyn - c trnh v vic lp li ln hai - c
bt u vi cc cng ty - c tuyn chn tm ra kin ngh thng v
to dng hp ng pht trin tha thun.
Bi tp.
1. Cng ty xe ti Acme c 1 i 500 xe ti vi 30 kho cha xe ti khp
n- c. Acme - a cc xe ti trn tuyn giao nhn. Mi cuc hnh trnh bt
u kho gn u tuyn nht v sn c xe v kt thc kho gn
nht cui tuyn. Acme mun pht trin mt h thng - a xe i trn vi
tnh ti - u ha vic s dng xe ca h.
Tho lun li v bt li cho Acme v cho nh pht trin d n, m
nhim cc loi hp ng pht trin khc nhau.
2. Chun b RFP cho d n lch trnh xe ti ca Acme. Bao gm phn
yu cu k thut, phn bn giao - c theo yu cu v phn thng tin
qun tr v cho mi phn RFP ch yu khc.
3. Chun b mt kin ngh k trnh Acme v d n lch xe ti bao gm
kin ngh k thut, tuyn b cng vic v tm tt iu hnh cng phc
tho cho mi phn ch yu khc (k c b sung).
4. Chun b ph- ng php v tiu ch duyt xt v tuyn chn cho ban
tuyn chn kin ngh ca Acme. Gn trng s cho 4 hp phn ch yu
ca kin ngh v cc hng mc c trong sc trong mi hp phn ch yu.
QUAN L DU N PHN MM
57
Tho lun mi tiu ch duyt xt v tuyn chn c bit nh- mc ti
thiu cho mt s hng mc hay hp phn ch yu.
5. Bi tp lp: la chn mt trong nhng RFP chun b bi tp 2 l
RFP cho d n lp lch xe ti cho Acme. Ch nh ba n nm i ng
chun b kin ngh chun b kin ngh p ng RFP ch nh ban tuyn
xt kin ngh duyt xt cc kin ngh v tuyn chn kin ngh thng.
Tho lun th tc duyt xt v tuyn chn.
QUAN L DU N PHN MM
58
Ch- ng bn
Chu trnh pht trin phn mm
Cc bin thi v ch thc n- c
Pht trin phn mm ng nh- hu ht cc hot ng khc c khi
u, trung on v kt thc. Kt thc ca mt hot ng pht trin i
khi - c nhn nhn l - c tip ni vi khi u ca mt hot ng pht
trin mi v bi vy to ra mt chu trnh khi u - trung on - kt thc,
tip ni, khi u, trung on - kt thc , tip ni v c th mi cch nhn
v pht trin phn mm - c coi l chu k cuc i (vng i) ca
pht trin phn mm.
C nhiu bin thc ca chu k cuc i ca pht trin phn mm. Hnh
4.1 cho thy mt vng i n gin thng th- ng trong vi thp k u
ca pht trin phn mm . Vo nhng ngy u ca pht trin phn
mm, nh lp trnh hn sng to ra nhng ch- ng trnh bng cch lp
li t m n phm tr ri tr li m v ri li phm tr na n khi c
cht g chp nhn - c (mong nh- vy) sn sinh ra khi u chu trnh
th- ng khng c quan nim r rng v ci g - c yu cu v qui trnh
pht trin c bn l mt hnh thc tip cn '' hy xem chng ta c th
lm - c g''.
Ph- ng php pht trin phn mm biu th bng chu trnh pht trin
hnh 4.1 th- ng - c coi l ph- ng php m v phm tr (v nhng l do
hin nhin). Nhng ph- ng php lun pht trin phn mm i mt
chng - ng di t nhng ngy code and fix mc d rng tht ngc
nhin ch c bit bao nhiu phn mm vn cn - c pht trin theo
cch . Vic qun l thng li bt c d n no, c bit cc d n phn
mm, i hi phi lp k hoch v khng th lp k hoch vi ''m v
phm tr, n hon ton khng th tin liu - c g. Qun l pht trin
phn mm trong b mn cng trnh li da trn mt b pha pht trin
ngn np hn. Nhng pha khng do ch ring cc nh lp trnh thc
hin, chng i hi c cc k s- phn mm. Trn thc t, lp trnh tr
thnh mt b phn t- ng i nh ca chu trnh pht trin phn mm hin
i nh- ta thy bng 4.1.
Hnh 4.1
Ph- ng php Code v Fix.
Quan nim
M Fix
Duy tu
QUAN L DU N PHN MM
59
Nhng con s bng 4.1 ly t s chung c nhn mnh n qui
hoch phn mm (yu cu v thit k) v th nghim. Nhng h thng x
l d liu th- ng mi, tr mt s ngoi l, vn cn phi tiu ph mt khi
l- ng ng k thi gian pht trin trong pha lp trnh v th nghim n
v. Cc h thng thi gian thc th- ng phc tp hn nhiu v c th bao
gm s tch hp bao qut phn cng/phn mm. iu ny thng th- ng
i hi qui hoch nhiu hn v tch hp cng th nghim nhiu hn.
Bng 4.1 phn trm d tch v thi gian hao ph trong pha pht
trin phn mm ch yu.
Qui hoch m v th
nghim n v
nht th v th
nghim
X l d liu
th- ng mi
25% 40% 35%
Cc h thng thi
gian thc
35% 25% 40%
Cc h thng
qun s
40% 20% 40%
Hnh 4.2
M hnh pha ca vng i pht trin phn mm
Thit k
mc cao
Tm bt
yu cu
Hnh
thnh
nim
Th
nghim
Bo tr
Thc thi
Tch
hp
Thit k
chi tit
QUAN L DU N PHN MM
60
Cc h thng qun s i hi tin cy cao v th- ng - c khch
hng gim st cht ch dn n gia tng ng k thi gian cn qui
hoch.
Tt nhin cc d liu trong bng 4.1 biu th mt s khi qut ho; cc
h thng x l d liu th- ng mi c th ng l phc tp nh- h thng
thi gian thc ( coi phn 4.4 tho lun thm v chuyn mc ny)
H .4.2 cho thy m hnh cc pha ca mt vng i pht trin phn
mm. M hnh gi l m hnh thc n- c c tn nh- vy do cch m
mi pha rn rp thc sang giai on sau ( do s gi u) nh- minh ho
hnh 4.3.
Mt s l gii v m hnh thc n- c nh- l gii sau y, phi hp cc
pha thit k mc nh v thit k chi tit thnh mt pha thit k duy nht
v cc pha tch hp v th nghim vo mt pha duy nht. Trn thc t c
nhiu bin th ca m hnh thc n- c c in nh- ng tt c u da trn
s chuyn tip c h thng t mt pha pht trin ny sang pha sau cho tn
n khi d n - c hon thnh.
Hnh thnh nim
Tm bt yu cu
Thit k mc cao
Thit k chi tit
Thc thi
Tch hp
Th nghim
Bo tr
T
Hnh 4.3
M hnh thc n- c ca vng i pht trin phn mm
C nhng ph- ng php lun pht trin khc khng chuyn t pha ny
sang pha sau nh- m hnh thc n- c to nguyn mu nhanh, chng hn
lp li trong pha pht trin mini n khi nguyn mu ca h thng - c
pht trin (coi hnh 4.4) sau khi hon thnh nguyn mu th cch tip
cn thc n- c c th - c thc hin hon thnh ton b h thng. To
nguyn mu nhanh c bit c ch trong nhng d n m yu cu kh c
t. Nguyn mu c th - c dng lm cng c phn tch v xc nh phi
c yu cu g.
QUAN L DU N PHN MM
61
M hnh xon c - c Boehm m t (1988) l ph- ng php pht trin
khc lp li gia cc pha yu cu thit k v thc hin. D sao, m hnh
xon c tip tc lp li n khi h thng cui cng hon thnh. Trong
mi ln lp li, m hnh xon c theo cch tip cn chia pha t- ng t m
hnh thc n- c.
H.4.4
To nguyn mu nhanh tip theo l ph- ng php chia pha
Hu ht cc h bin ho hin i pht trin phn mm bao gm mt s
hng ca m hnh chia pha c bn. Do y quan trng l phi hiu - c
cc pha khc nhau v bit chng lin quan vi nhau nh- th no. Ch- ng
ny m t mt s vn qun l lin kt vi m hnh chia pha k c
khng kh v nhng vn c tr- ng mi pha.
4.1 Pha quan nim
M hnh thc n- c bt u bng pha quan nim khi thu, trong pha
nhu cu v h thng phm mm. Pha ny cng cn - c IEEE gi l
pha thm d quan nim
8
n to c s cho :
- Chun b yu cu cho kin ngh (RFP); RFP l c ch khi cc d n
- c hp ng vi cc nh pht trin khc (coi ch- ng 3) bn ngoi
- Xc nh cc yu cu phn mm (pha).
- Qui hoch ban u v chun b d ton; iu ny th- ng - c dng
lm bn u ca k hoch pht trin ca d n.
- Pha quan nim to nn hai loi vn bn d n :
- M t sn phm tr- c ht l t- liu tip th v - c dng nh- l mt
bn cng b sau ny cho sn phm hay nh- l mt tng quan chung ca
sn phm. T- liu quan nim l t- liu k thut v to thnh c s cho
nhng hot ng k thut chnh ca pha ny (RFP k hoch ban u ) T-
8
T vng chun cho thut ng cng trnh phn mm ca IEEE (Std729-1983) (1987b) mc d tha
nhn thiu s tho thun chung v cc pha yu cu to ra chu k sng ca phnf mm vn cho mt th
d gm c khai thc quan nim yu cu, thit k, thc hin, th nghim, lp t v kim th, vn hnh
v bo tr v mt giai on l th gi l rt lui .
Vng to mu nhanh
Hnh thnh
nim
To nguyn mu Yu cu
Thit k
Thc thi Tch hp Th nghim
QUAN L DU N PHN MM
62
liu quan nim cng l mt trong nhng ngun tham chin chnh cho
vic to lp cc yu cu phn mm trong pha sau ca d n.
Pha quan nim khng phi l pha pht trin chnh thc u thc. N
th- ng - c tin hnh khng chnh thc tr- c khi c bt c cam kt no
cho s pht trin sau ny ca d n
4.1.1 Bu khng kh trong pha quan nim.
Bu khng kh ca pha quan nim bin i trn c s tng ln, gim
xung ca s thiu qu quyt v s do d. Theo th pha u ny ca d
n th- ng - c c tr- ng :
- Ao - c ca i ng k thut mun nm - c s vn ng ca d n
- Thiu cam kt y v pha qun l : Ch c nhng ngn sch ban
u th- ng - c phn vo giai on ny.
- S tht vng trong qun l do tnh trng bt lc ca i ng k thut
khng gip cho qun l - c g ngoi d ton th thin.
- Tht vng ca i pht trin : do bt lc ca khch hng ( k c qun
l tip th v nhng ng- i s dng) Khng cho - c nhng nh ngha
chnh xc ca h thng m h yu cu.
Tt c nhng iu l sn phm ca vic thiu cam kt ca cc bn
lin quan khch hng v qun l vn cn ch- a c quyt nh chc chn
v vic liu h c mun tin b- c tip vi d n na khng . Thm vo
y, h li ch- a hon ton - c thuyt phc v tm quan trng ca pha
quan nim khi m rt t kt qu nhn thy - c . Cc bn c xu h- ng
dnh lu ch khng hon ton cam kt
9
vi d n v do ch trng i
kt qu m chng chu cung cp ngun lc.
Vic b tr ngn sch lun l mt du hiu cam kt chc chn nht vo
d n v ngn sch , khi thu cng ln th s cam kt cng chc chn.
Vic kim ra ngn sch ban u ca d n th- ng l nhim v ch yu
u tin ca ng- i qun l d n. iu ny c th - c thc hin coi nh-
kt qu ca :
- Vic chun b mt t- liu quan nim tt. Mt t- liu quan nim tt l
kt qu ca vic phn tch th tr- ng v kho st ng- i dng d hiu. V
n bao gm tng quan son tho tt v cc yu cu chc nng ca h
thng t- liu ny cng phi cp n tnh kh thi ca d n.
- Hnh thnh nhu cu r rng cho h thng cn - c pht trin mt
trong nhng cch tt nht c - c s ph duyt cho d n pht trin l
minh nh v m t r rng vn m d n ny - a ra mt gii php cho
vn .
- Cung cp nhng d ton ban u y thuyt phc cho s pht trin
ca d n. Khng c t chc no c th cam kt cho mt ph tn khng
9
Cc quan nim dnh lu v cam kt' th- ng ln ln v - c s dng thay th cho nhau. C
mt giai thoi ni ting nhn mnh s khc nhau gia hai quan nim: Trong gim bng v trng, g
th dnh lu nh- ng ln th cam kt.
QUAN L DU N PHN MM
63
bit. Nu d ton ban u
10
khng th chun b - c th vo pht cui
cng phi trnh - c d ton cho giai on yu cu
4.1.2 Nhng vn trong pha quan nim.
C nhiu vn trong pha quan nim l do kh khn trong vic y d
n tin ln. R rng iu ny l kt qu ca khng kh bin ng m t
trong phn tr- c. iu ny dn n kh khn trong vic nhn - c cam
kt rng buc t khch hng v t qun l hng u.
Nhng vn sau l thng th- ng trong pha quan nim.
- C nhiu vn lin quan n vic thnh lp i ng pht trin d n
ban u. t ng ng- i vo i ng d n him khi l mt vic d dng.
Tht vy, b tr ng- i qun l d n thch hp khng phi bao gi cng l
mt vic d dng.
- Vn thng th- ng hoc l thiu hoc l tha lnh o d n. Trong
nhiu tr- ng hp , pha quan nim do nhiu ng- i ch o, ng- i ny c
th chu trch nhim v phn tch th tr- ng, ng- i kia v d ton v li
ng- i khc na v son tho cc t- liu quan nim. iu ny c th gy
ra tnh trng thiu phi hp gia nhng ng- i c trch nhim v cc hot
ng khc nhau, to nn nhiu mu thun v nhng quyt nh ban u
ti.
- Khi i ng ban u yn v v k hoch pht trin s b
- c son tho, khng phi khng ph bin c chuyn nhiu kin khc
nhau v vic sn phm nn phi thc s nh- th no to ra t- liu quan
nim - c tho thun th- ng c th tr nn mt vn ch yu . Vn
ny th- ng - c gii quyt thng qua vn dng cc h thng trnh din
hay nguyn mu.
Vic to nguyn mu nhanh c th gip ch khi tho thun chung v
quan nim kh t - c. Cc quan nim th- ng d thng nht hn khi c
mt ci g c th - c trnh ra v duyt li bi cc bn lin quan.
4.2 Pha yu cu phn mm.
Giai on yu cu phn mm, cng gi l pha xc nh, l giai on
chnh thc u thc u tin ca pht trin phn mm. Giai on ny cung
cp m t chi tit v h thng phn mm - c pht trin. iu ny l cn
c c t yu cu sao cho sn phm phn mm - c th nghim vo
cui d n nhm chng minh rng sn phm - c yu cu thc tnh
- c sn ra. c t yu cu tr li ci g trong khi tm cch trnh cu
hi th no.
Pha yu cu to thnh c s cho :
- - ng li u tin ch yu ca h thng
11
- Thit k h thng ( giai on tip)
10
Vic chun b d ton - c bn n chi tit ch- ng 10
11
tho lun ch- ng 9
QUAN L DU N PHN MM
64
- Th tc th nghim nghim thu (ATP)
Pha yu cu son tho mt t- liu sn phm chnh :
- T- liu c t yu cu phn mm
v hai t- liu qui hoch d n
- K hoch pht trin d n
- K hoch th nghim d n
Pha yu cu chnh thc kt thc vi vic duyt xt ch yu u tin
cho d n : Duyt xt yu cu phn mm (SRR) chnh vic duyt xt ny
kt thc vic c t yu cu v chnh thc tuyn b t- liu yu cu nh- l
- ng li c bn d n u tin - c ph duyt.
4.2.1 Bu khng kh trong qu trnh pha yu cu.
Pha yu cu th- ng - c nhn thc l giai on quan trng nht ca
chu trnh pht trin phn mm. Chc chn rng n l mt trong nhng
phn kh khn nht, mt phn do kh khn trong vic lp t- liu mt m
t - c chp thun cho yu cu phn mm . Bao gi cng c mu thun
c bn v quyn li gia khch hng
12
v ng- i pht trin. Khch hng
min c- ng kt thc yu cu v nhn bit rng mt khi lm iu ny,
mi thay i sau ny c th phi tr gi. Mt khc, ng- i pht trin cn
kt thc yu cu cng sm cng tt v tin trin s chm i chng no sn
phm cn ch- a hon ton - c xc nh v cng chnh iu ny l tn
km. Nn bu khng kh ca pha yu cu c th - c c tr- ng nh- l
mt hnh thc ca tr ko co gia khch hng v ng- i pht trin.
Hp ng ph cng thm theo ng- i pht trin - c tr theo gi hay
theo ngy (coi ch- ng 3) c xu h- ng c t tranh chp thuc loi ny. D
sao hp ng ph cng thm trch nhim chnh cho vic kt thc sm
cc yu cu vi khch hng.
Do y giai on yu cu - c c tr- ng :
- Mu thun gia ng- i pht trin v khch hng trong vic kt thc
c t yu cu.
- Bt ng v cc d ton sa i v rng buc v cng :
- Bi ri do trch nhim chuyn giao v b tr lc l- ng khng chuyn
mn.
Yu cu kt thc vic khng phi bao gi cng d dng v i hi kinh
nghim kin nhn v kin quyt. y l vic ca ng- i qun l d n mt
mt c- ng quyt v dt khot trong vic yu cu cc bn kt thc yu cu
v d n. iu ny - c thc hin tt nht khi gii thch cho khch hng
l khng kt thc gy ra chm tr lch trnh v chm tr l tn km.
4.2.2 Cc vn trong pha yu cu
12
T khch hng s dng y theo ngha rng nht ca n v lin quan n t chc m d n yu
cu nh- khch hng bn ngoi, phng tip th hay ng- i dng.
QUAN L DU N PHN MM
65
i khi xy ra cc tranh chp, bt ng v ln ln biu th khng kh
ca giai on yu cu chnh l ngun gi ch yu ca cc vn . Danh
sch th di nh- ng nhng vn thng th- ng nht l :
- Thay i yu cu lun : Trong pha ny, th- ng c lung thay i c
v v tn lm cho vic thu thp c im yu cu thm kh khn.
- Chp thun yu cu v k: pha ny khng th hon tt nu khng c
t- liu yu cu - c ph duyt chnh thc.
- Tnh kh thi ca yu cu : i khi iu ny kh xc nh tr- c giai
on thc hin.
- B tr nhn s : Vic sp xp thnh vin i ng pht trin thch hp
c th l mt nhim v kh. Vic b tr cc thnh vin i ng phi - c
hon tt trong pha yu cu.
- Cung cp thit b : Nhng vn ngn sch v sn sng s dng - c
c th lm t on k hoch b tr thit b.
- Nhng d n bt buc
Nu nh- nhng vic ny ch- a - c th chng phi - c hon
tt tr- c khi kt thc pha ny.
Nh- chng ta thy, c - c s ph duyt chnh thc v c t yu
cu l nhim v kh khn nht ca giai on ny. Th- ng cc yu cu
tin trin dn dn vi nhiu ph- ng n d tho ca c t tp trung vo t-
liu - c ph duyt cui cng. D sao nu nhng thay i ch yu - c
- a ra lin tc th s kh ln k hoch. Trong nhng tnh hnh nh- th,
vic to nguyn mu nhanh c th c ch nht trong vic gip khch hng
xc nh yu cu.
Tnh kh thi ca yu cu phi - c xc nh nu c th tr- c khi hon
tt cc yu cu. Bao gi cng tn km khi pht hin nhng yu cu bt
kh thi trong qu trnh nhng pha sau ny ca d n.
Tnh kh thi ca cc yu cu c th tr nn tt hn nh.
- S dng nguyn mu th nghim yu cu
- S dng nht thi cc chuyn gia vo th tc phn tch tnh kh thi
- Yu cu ph duyt cc yu cu thay th nu yu cu chnh khng kh
thi.
- To ra nhng yu cu m h tu theo mt s kt qu. (th d b nh
, hay kh nng x l CPU c sn). Bt k ch no c th - c nhng
yu cu m h hay kh nghi phi - c loi b. Lp k hoch d n tin
trin song song vi pht trin cc yu cu. Nhng v lin quan vi vic
b tr ti nguyn con ng- i v k thut c th lm d n tr tr. Tm - c
nhng thnh vin tt cho i ng bo gi cng l mt vn cho ng- i
qun l d n. D sao, mt khi h - c b tr v u thc, thit b pht
trin tho ng phi sn sng.Do nhng vn lin kt vi vic cung
cp thit b cng tr nn nghim trng hn khi i ng pht trin ln ln.
Nhng vn nhn s th- ng c th gii quyt bng cch:
- S dng nhn lc tm thi
- Tng dn i ng sau ny
- Hun luyn nhn lc hin c.
QUAN L DU N PHN MM
66
Cc vn thit b cng th, c th - c gii quyt bng cch:
- S dng thit b tm thi ( vay hay thu).
- S dng phn mm m phng thay th cho thit b t tin.
- C - c ngn sch ban u ti tr thit b pht trin c bn .
Nhng vn thit b v nhn s th- ng tc ng n d ton v lch
trnh trong k hoch pht trin d n. Nu thit b chm hay nu b tr
nhn s chm th d ton s cn - c xem li v lch cn - c iu
chnh. Nhng vn ny - c bn chi tit ch- ng 9.
4.3 pha thit k.
Trong pha thit k, cc yu cu - c phn tch v ph- ng php thc
hin - c xc nh. Pha yu cu tr- c y t cu hi ci g?, pha thit
k t cu hi th no? Vic tr li cho cu hi ny - c c trong t-
liu c t thit k phn mm.
Pha thit k th- ng - c chia lm hai pha ring bit: thit k mc nh
v thit k chi tit. - ng phn chia gia hai pha ny - c t c mt ci
g khng nht thit v cn c mc phn h thng thnh cc thnh
phn (coi ch- ng 6) hnh 6.5 cho mt th d ca s phn chia thit k
mt h thng thnh thit k mc im v thit k chi tit.
Mt trong nhng li th trong vic s dng hai pha thit k so vi thit
k mt pha l ch thit k mc nh th nht cung cp mt mc b
sung mc c th nh gi v ph duyt cch tip cn thit k . iu
ny l c bit quan trng cc d n va v ln (ngha l hn 18 nm
cng) m do nhng sai lm thit k ch yu phi - c xc nh cng
sm cng tt. Khi tm - c sai lm ch yu cui pha thit k mc nh
th s d dng sa n hn l nu tm thy n sau khi ton b thit k
hon thanhf . Hy xt th d sau :
Mt h thng phn mm ang - c thit k cho mt my tnh c th
c cu hnh c bn 8 megabai b nh v 256 mgabai a cng. B nh
c th tng thm mt mgabai v a cng c th tng thm 256 mgabai.
Thit k ban u ca h thng - c pht trin khng khp vo cu
hnh b nh c bn v do h thng - c thit k s dng ovlray.
Vic s dng overlay l mt quyt nh thit k ch yu v n tc dng
n vic nh gi, tnh phc tp s dng b nh v a. Nhiu quyt nh
thit k tip theo chu nh h- ng ca quyt nh nay v vic s dng
overlay. Ho ra l s dng overlay l mt quyt nh thit k ti v s
dng a cng cao. iu c ngha phi cn n b c a 256
mgabai th hai v r rng tn km hn 2 megabai ph ca b nh cn
c khc phc nhu cu v ove rlay.
Nu quyt nh v ove rlay ch - c pht hin l sai vo cui giai on
thit k th vic vit li ton b thit k s li l cn thit. Cu hnh my
tnh c th cng phi thay i v c ghi a ph 256 megabai s phi
tr li (cho rng ng- i bn s chp nhn n). D sao, nu quyt nh ny
- c pht hin sm ngay trong thit k mc nh th khi l- ng vit li
QUAN L DU N PHN MM
67
s l ti thiu v nh- th s c c may tt hn l thay i cu hnh my
tnh tr- c khi giao hng.
Pha thit k to c s cho :
- - ng mc, h thng ch yu th hai ca h thng
- Thc hin h thng ( giai on sau)
- Ch- ng trnh pht trin - c cp nht.
Pha thit k to nn nhng vn bn sau :
- c t thit k (vi nhng d n ln : c t thit k mc nh v c
t thit k chi tit)
- K hoch tch hp
- c t tr- ng hp th nghim, m t chi tit mi th nghim mc
thp cho tng c th.
Vn bn c t thit k to lp - ng mc ch yu th hai ca d n.
Tr- ng hp hai pha thit k th c t thit k chi tit - c coi l - ng
mc ch y ca thit k v c t thit k mc nh - c coi l - ng
mc th yu (Cc - ng mc s - c tho lun ch- ng 9).
Vo cui pha ny, nhiu ci ch- a bit ca d n li tr nn bit, do
to nn ci tin ng k trong d ton k hoch pht trin cc thng s
pht trin d n khc nhau nh- ti nguyn v lch tch hp v cc tr- ng
hp th nghim thc cho pha th nghim, gi y c th - c lp k
hoch - c k hoch pht trin d n - c cp nht do y c th coi
rng giai on nay k hoch l ng tin cy hn nhiu.
Theo th, song song vi thit k ca h thng, cc hot ng sau y
cng tin trin :
- Nn tng, tch hp v pht trin - c thit lp bao gm mi thit b
yu cu cho pht trin v tch hp h thng .
- D ton - c ci tin ng k.
- phn tch ri ro ca d n - c duyt xt li v cp nht.
- Lch pht trin ca h thng - c cp nht.
Mi thng tin trn - c - a vo trong vic sa i mi ch yu ca k
hoch pht trin d n.
Pha thit k kt thc vi vic k kt t- liu c t thit k. iu ny
thng th- ng xy ra trong vic duyt xt thit k chinh thc, - c gi l
duyt xt thit k c ph phn (CDR). Nu mt c t thit k mc nh
trung gian - c chun b, th t- liu ny - c dng vic duyt xt thit
k s b (PDR) ban u.
4.3.1 Bu khng kh trong pha thit k.
Pha thit k th- ng l mt phn t- ng i lc quan v tin t- ng ca d
n pht trin. Giai on ny c tr- ng :
- S nhit tnh : d n c , ngn sch - c duyt v gi y
ang - c chi tin v mt i ng pht trin mi yn v .
- S chm tr : nhiu thay i trong yu cu v thit k - c - a vo
do
QUAN L DU N PHN MM
68
+- kin chm
+- tnh bt kh thi ca yu cu
+- Thng tin mi b sung
+- S bi ri : i ng pht trin nhanh, tn ti d n v trch nhim
cn ch- a r.
Pha thit k, i vi ng- i qun l d n, l thi k, t chc trong thi
gian c cu i ng d n - c hon tt v vic phn trch nhim
xong. Nhng nhim v phi - c ng- i qun l d n hon thanmhf
tr- c khi kt thc giai on thit k v s ln ln c th c trong hai pha
u ca d n khng th - c chuyn sang giai on thc hin.
4.3.2 Nhng vn trong pha thit k
Nhng vn chnh ca giai on thit k lin quan n :
- Nhng tranh ci v thit k k thut
- Kh khn v nhn s
- Cung cp ngun lc pht trin
- Cc quan h khch hng
Nhng vn thit k k thut lin quan c n nhng g do cc yu
cu bt kh thi cng nh- nhng gid do nhng quyt nh thit k v thc
hin phc tp. Chnh trong pha thit k tt c nhng vn phi - c
gii quyt v vic gii quytd chng s tr nn tn km hn khi d n
tin trin.
i khi, nhng vn nhn s - c chuyn t giai on yu cu tr- c
y. Vic tuyn ng- i cho mt s nhim v l kh hay chm do y quan
trng cho ng- i qun l d n l phi tm cch b tr nhn vin vo mi
v tr d n cng sm cng tt thm ch tr- c nhng ng- i hin nay - c
yu cu. Trn thc t, y l mt thi quen tt trong vic bt u b tr
nhn vin vo nhng v tr tr- c ngy m nhn vin thc s tham gia
d n . Nhiu vn lin quan n vic cung c ngun c pht trin
- c tin hnh t giai on tr- c song pha ny ca d n nhng vn
ny thwng - c phi hp hoc vi nhng ng- i bn hng bn ngoi v
nhng ng- i cung cp hoc thit b cng ty hin c m n vn cn rng
buc vi cc d n khc. Nh- nu tr- c y, nhiu nhng vn
th- ng c th - c gii quyt bng cho vay hay thu thit b. D sao tc
ng khng li ca nhng gii php tm thi nh- th li tng ln khi d
n chuyn sang nhng giai on pht trin tin tin hn. Trn thc t,
nhng gii php , nu khng - c x l thn trng, c th dn n
nhng vn mi :
- Thu c th tn km v c th hao ph nhng s tin ng k ngn
sch pht trin.
- Vay n (hay n vay nh- th- ng - c gi) c th b thu li, lm cho
thnh vin ca i ng pht trin khng c ph- ng tin pht trin cn
thit.
QUAN L DU N PHN MM
69
- Nhng gii php thay th tm thi c th dn n cht l- ng st gim
v thi gian pht trin ko di. Rt cc iu ny li dn n tnh trng
lch trnh b tr- t
Mt s vn lin quan khch hng, ng- i sn xut vn cn c th
xy ra trong pha thit k. Vi nhng yu cu gi y - c chp thun
.Phn ln cng thng gim i nh- ng khi pha thit k kt thc vi vic
tng duyt chnh thc, khch hng chia x trch nhim v thit k
.iu ny tng thm dnh lu ca khch hng trong pha thit k v c
th dn n nhng bt ng v h thng phi - c thc hin th no.
Nhng loi vn - c gii quyt tt nht bng cch ch nh mt
thnh vin i ng (c th l ng- i qun l d n) hnh ng nh- mi
lin lc vi khch hng. iu ny gim nhng im tip xc gia i ng
v khch hng trong khi vn m bo rng khch hng c mt a ch
duy nht cho mi yu cu v nhn xt.
4.4 Pha thc hin.
Trong pha thc hin, cc m un phn mm - c m ho v cc th
nghim n v ban u - c tin hnh. Th nghim n v - c ng- i
lp trnh tin hnh trn mi m un c th ngay sau khi m un - c
m ho. Sau cc m un - c b phn kim tra cht l- ng phn mm
ph chun v - c trnh b phn kim tra cu hnh (coi ch- ng 7 v
tho lun cc chc nng kim tra ). Sau b phn kim tra cu hnh
chuyn cc m un hp nht.
c t thit k chi tit v cu trc tt dn n vic lp m t- ng i
trn chu v thng bng
13
. Theo th vic lp m (hay lp trnh) thot k
thu - c nhn thc l ng ngha vi pht trin phn mm , nay tr
thnh mt pha ring r trong chu trnh pht trin phn mm . Trn thc t
pha thc hin li cng khng l pha di nht. Mt kinh nghim thng
th- ng d tnh cc pha ca chu trnh pht trin phn mm s dng
cch phn chia theo t l 40-20-40 cng sc v thi gian (coi bng 4.1).
iu ny c ngha khong 40% thi gian - c ginh cho c t (yu cu
v thit k ) 20% cho thc hin (lp trnh v th nghim n v) v 40&
cho tch hp v th nghim xu h- ng chung l tm cch gim 20% ginh
cho thc hin trong khi tng 40% gianh cho c t ? . Mt kha canh ca
lp lun ng sau tip cn l cc giai on pht trin sm hn t tn
km hn cc giai on sau. Cc pha c t th- ng c nhn lc thp v t
thit b pht trin hn pha thc hin. Cng vy nh- th- ng chng
minh khi cng sc - c ginh nhiu hn cho yu cu v thit k, pha tch
hp d dng hn v hiu qu hn.
Pha thc hin l cu ni cc pha thit k v tch hp h thng v
th- ng gi u r rt vi mi mt trong hai pha (coi hnh 4.5). tnh
trng gi u s th- ng xy ra khi nhiu b phn ca thit k h thng
13
iu ny - c thuyt minh qua nhiu c gng pht trin cc my sinh m t ng to lp m
t c t thit k, coi Balyre 1985
QUAN L DU N PHN MM
70
- c thc hin t- ng i nhanh li mt s vn thit k b ng mt
thi gian kha kh. Trong nhng tr- ng hp gi u c th rt ngn
ng k lch pht trin.
Tnh trng gi u ca cc pha thit k v thc hin i hi phi rt
thn trong trong vic m bo ch nhng m un thit k hon chnh
- c ph chun thc hin sm. C ri ro l mi thay i chm sau ny
v thit k ca nhng m un c th i hi phi lp m li. Theo th
lng ph ti nguyn . Cng c ri ro l thit k thay i v m khng thay
i. D sao nhng ri ro th- ng vn nm bt - c tt. Vi qui hoch
v khng ch kim tra cu hnh tt nhng vn c th - c khc
phc
T
Hnh 4.5
S gi u ca cc pha
Mt khc ca pha thc hin, tnh trng gi u ca lp m v ch hp
th- ng t ri ro v nu - c qui hoch ng n c th l mt s tit
kim thi gian tuyt ho. Trnh t thc hin ca cc m un phi - c
qui hoch tt m bo chng - c - a ra theo trnh t yu cu tch
hp . iu ny cng c ngha nhng sai lm trong thc hin c th nh
v trong qu trnh tch hp v - c hi tip li pha thc hin .
Pha thc hin bao gm nhng hot ng chnh sau y :
- Pht trin m phn mm
- Chun b cho s tch hp v th nghim h thng (giai on sao)
- pht trin k hoch bo tr
Ngoi m hin ti - c vit (v mong - c bnh tt) mt s t- liu
khc - c pht trin trong pha ny bao gm :
- S ghi ca ng- i lp trnh dn ghi li cc quyt nh lp m nhng
th nghim n v v gii php cho nhng vn thc hin .
- K hoch bo tr v thu thp t- liu, k c t- liu ct yu cn thit
cho vic bo tr h thng.
Pha thit k
Pha thc thi
Pha tch hp
QUAN L DU N PHN MM
71
- Cc bn ban u v t- liu cho ng- i dng, k c s tay tham kho
v h- ng dn cho ng- i iu hnh.
14
Trong pha thc hin, d n b- c vo thi k hot ng khn tr- ng.
Nhiu hot ng khc din ra bao gm :
- Pha thit k - c hon tt sm trong qu trnh pha thc hin .
Nn tch hp - c nh v v vic tch hp bt u.
Nn ca th nghim (mi tr- ng v thit b th nghim) - c nh v
chun b cho vic th nghim h thng
- Cc tnh hung ci ro c th - c tr thnh hin thc v khi cc
k hoch d phng bt trc - c - a vo hot ng.
- K hoch d n - c duyt li v cp nht.
4.4.1 Bu khng kh trong pha thc hin
Nh- chng ta thy, vic t chc nhn s t n nh cao trong cc
pha thc hin v hp nht. Trong lc thc hin bu khng khi chu nh
h- ng bi nhiu hot ng khc tin hnh song song. Bu khng kh ny
c tr- ng .
- p lc - c tip tc v - a ra ci g . iu ny th- ng l kt qu
ca vic gia tng ng k v mc tng chi ph nh- ng vn cn t ch ra
- c chi ph pht trin.
- Tranh chp vi cc t chc bo him cht l- ng phn mm (SQA) v
kim tra cu hnh. Nhng tranh chp ny l do s dnh lu gia tng ca
cc t chc gim st v vai tr ca h trong vic thc p cc tiu
chun v cc th thc pht trin c trnh t
- Hoath ng ca i ng pht trin gia tng ch cn th gi cn li
trong ngy giao sn phm ti gn v tr nn thc tin hn.
Ni chung, pha thc hin l thi k chuyn tip t c t sang xy
dng. Bu khng kh tu thuc rt nhiu thnh cng ca cc pha c t
tr- c y v thnh cng trng i cc pha tch hp v th nghim.
Nhng nhn t ng gp bao gm :
- Yu cu c th cn ch- a - c xc nh y .
- Ti nguyn khng - c b tr
- Thi gian pht trin khng .
- Mt s vn k thut c th cn ch- a - c gii quyt
- H tr qun l c th thiu
- Quan h khch hng c th cng .
4.4.2 Nhng vn trong pha thc hin
Nu pha thit k - c thc hin tt, hu ht nhng vn k thut
phi - c gii quyt vo cui dt thit k. Nu khng - c nh- th c
th dn n ln ln do s cn thit phi lp trnh v gii quyt ng thi
14
Nhiu tiu chun m t ni dung ca t- liu c - c trong d n phn mm. USMilstd- 2167a
(1988a) c bit d hiu v chi tit. Coi ch- ng 8 v tho lun chung cc tiu chun pht trin phn
mm
QUAN L DU N PHN MM
72
nhng vn k thut. nhng tnh hung nh- th th- ng ri vo ph- ng
php m v nh v (coi hnh 4.1)
Cc vn khc ca giai on thc hin bao gm :
- Cc thay i pht cht : y l mt vn ch c th - c gii quyt
bng cch m bo th tc khng ch cc thay i mt cch nghim ngt
v c trnh t
- Giao tip v phi hp gia cc thnh vin i ng, vn ny c
bit nghim trng trong cc d n ln
- Kim tra v iu khin cc ch thu ph v ng- i ph v ng- i bn
hng .
Lung thay i l tai ha ca cng trnh ni chung v ca pht trin
phn mm ni ring. R rng thay i phi - c php trong mt chng
mc nh- ng lung thay i khng kim sot - c c th lm cho d n
sp . (qu).
Cng c nguy c l cc i ng ln, mt s thnh vin c th khng
nhn thc - c mt thay i c bit.
Truyn thng chnh thc khi - c x nh gia cc i nh.
Trong mt s tr- ng hp cc h con c th - c u thc cho nhng
ch thu ph. D sao vic khng ch nhng ch thu ph v nhng ng- i
bn c th tr thnh mt cng vic trn thi gian. Nhng bo co ming
v vit t khi l , nhng cuc thm ving th- ng xuyn ti ch pht trn
th- ng l cn thit m bo tnh sn sng ca thng tin chnh xc.
Trong nhng tr- ng hp , ng- i sn xut tr thnh khch hng trong
h vi ng- i bn hay ch tu ph.
4.5 Pha tch hp v th nghim.
Trong cc pha tch hp v th nghim, cc m un phn nn - c phi
hp thnh mt h duy nht v tnh chc nng ca h - c th nghim v
mt p ng vi yu cu. Tch hp bt u bng dng ban u ca h
thng m h thng ny tng dn v chc nng cho n khi mt th thng
y - c lp gp li . Sau mi giai on th nghim nh gio s
hon thin ca h thng nhm minh nh cc vn phi chnh l tr- c
khi h thng c th - c - a ra.
Giai on tch hp v th nghim to c s cho :
- Vic xy dng h thng phn mm t cc thnh phn phn mm
khc nhau.
- Tch hp thit b phn mm v phn cng
- Xc nh xem h thng c - c xy dng theo nhng c t yu cu
hay khng
-To ra cht l- ng ca h thng.
Pha ny th- ng - c phn lm hai pha ring bit v kh song song :
Pha tch hp v pha th nghim. trong nhng d n ln, s phn chia ny
th- ng l cn thit c bit khi vic th nghim - c mt nhm ring
bit c lp tin hnh (i ng th nghm c lp) lm vic st cnh vi
QUAN L DU N PHN MM
73
nhng ng- i sn xut. trong nhng d n nh, iu hp l ng sau s
phi hp ca hai hot ng vo mt pha l s tch hp khng th thnh
cng nu khng c th nghim rng ri - c tin hnh song song. do y
nu hai hot ng - c cng mt i ng thc hin, th- ng tt nht l
phi hp chung trong mt pha duy nht (coi ch- ng 7 c tho lun chi
tit hn v i ng th nghim phn mm).
C nhiu k thut v ph- ng php tch hp
- T trn xung
- T d- i ln
v mt cch tip cn l th - c gi l
- T trong ra
Tip cn trfn xung i hi ct li ca h thng (th- ng l cc m
un uu hnh trung tm) phi - c thc hin tr- c. Ri chng - c
phi hp mt h thng cc nh s dng nhng th tcv rng thay cho
cc m un ch- a - c thc hin. Nhng v rng ca m tr li nhng
gi tr c nh v chng c tnh lgic g c, th- ng - c gi l cung. Sau
cc cung - c thay dn bng cc m un thc, theo mt cch xy
dng tng tin c k hoch tt ca h thng sao mi ln th thng - a ra
mi, li tng thm nhiu chc nng hn.
Tip cn d- i ln bt u t cc m un c th cp thp nht (th d :
- b iu khin u vo u ra, b nh khun,dy thao tc,d liu, b
i thoi ng- i dng,my v.v..) v dn dn gp chng thnh nhng nhm
mi lc mi ln hn cho n khi ton b h thng - c ghp li.
- Cch tip cn d- i ln t khi - c khuyn dng lm chin tch hp d
hiu, trong hu ht tr- ng hp cch tip cn trn xung hn v t
nhin hn. D sao trn thc t, hu ht cc chin l- c tch hp h thng
thnh cng l mt phi hp ca cch tip cn trn xung v lc c i
ch theo cch tip cn t d- i ln.
Tch hp t trong ra (indide out integration) l thng th- ng trong s
pht trin ca cc h c s d liu ln khi cc cu trc file ni b - c
xy dng tr- c sau l logic x l d liu v cui cng giao din vi
con ng- i. tip cn ny l tt nht khi h thng v mt logic l cc tng
chc nng k tip nh- ng im yu chnh ca n l giao din vi con
ng- i li th- ng l - c tch hp cui cng. iu ny c th i hi vit
m th nghim tm thi u ra c th - c duyt xt li. Do y th
nghim th- ng chm hn v kh hn so vi tip cn t trong ra.Vic th
nghim bt u bng vic tch hp v tip tc ti khi cui cng h thng
- c bn gio cho khch hng. Cc loi th nghim bao gm :
- Th nghim tch hp : Thc hin bng cc my tch hp ca h
thng.
- Th nghim c lp : Thc hin bi nhm th nghim bn ngoi
m bo th nghim khch quan khng thin lch.
- Th nghim lp t . i u ny bao gm th nghim hon thin
chung. i hi khi m mt phin bn ca h thng u tin - c lp t
QUAN L DU N PHN MM
74
trong mt mi tr- ng iu hnh. Nhiu h thng bao gm mt b th
nghim lp t m bo h thng - c lp t thnh cng .
- Th nghim alpha v bta.
cc th nghim nytin hnh trong h thng trong mt mi tr- ng thc
(khng phi th nghim trong phng th nghim test). Th nghim alpha
th nghim h thng khng c d li sng. Th nghim beta th nghim
h thng c s dng d liu sng c gim st th- ng xuyn chnh l
mi vn c th pht sinh.
- Th nghim nghim thu. y l ct mc cui cng ca d n v s
hon tt thng li ca n c ngha l s nghim thu ca khch hng v
sn phm - c pht trin.
Vo lc kt thc pha tch hpv th nghim mi t- liu ph v
sn sng giao bao gm :
- T- liu bo tr
- T- liu ca ng- i dng cui cng
- Mi t- l;iu pht trin cp nht
- T- liu th nghim v bo co th nghim
Song song vi tch hp v th nghim, cc hot ng qun l v khng
pht trin sau y din ra :
- Lp ngn sch cui cng ca d n, ph tn cho cc thay i (pht
minh) - c xc nh hot dng d phng ri ro - c nh gi v ngn
sch - c cp nht .
- Vic hun luyn - c tin hnh cho ng- i dng, ng- i vn hnh,
khch hng, ng- i lp t, k s- bo tr v s s- tip th.
- a im lp t - c chun b v h tng c s cho phn cng c
thit b c bit - c lp k hoch v lp t.
- Qui m i ng pht trin - c gim i.
4.5.1 Bu khng kh trong pha tch hp v th nghim
Cho n im ny trong vng di ca d n vn bn - c son
tho, m - c pht trin v thit b pht trin - c lp t khng k cc
nguyn mu th chela c chc nng no l - c pht trin xong pha
tch hp v th nghim bt u lc chi ph nhn s v ngn sch t nh
cao. Do y hp nht v th nghim th- ng - c biu th bng :
- p lc c - c ci g lm vic . iu ny bt u trong pha thc
hin tr- c y v gi ang gia tng. Qun l t nhn thy mt cht
thc cht bin minh cho u t- ca h. Mi chm tr lch trnh im
ny c th nghim trng quan trng i vi ng- i qun l d n,l phi
to - c phin bn ban u ca h thng cng sm cng tt.
- p lc hon thnh d n. p lc ny xut hin vo lc cui d n v
tr thnh mnh lit hn nu lch trnh bt u tr- t.
- Qu thi hn. Thi gian ca lch th- ng hay nguy cp hn l thi gi
pht trin (th d cc thng cng). Gii php u tin cho vic tr- t lch
trnh th- ng l lm thm gi.
QUAN L DU N PHN MM
75
- Tranh chp vi khch hng. Cui cng khch hng c th nhn thy
phin bn ban u ca ci m sn phm s c dng v th no. V iu
ny, c nhng l gii yu cu khc nhau ny sinh ra v th- ng cn - c
gii quyt cp qun l cao hn.
- Tht vng. Nhng gii php cho cc vn tch hp v cc li thc
hin th- ng khs nm bt v c th i hi tr li pha thit k.
Pha tch hp v thit k vn l kh lp k hoch nht. cng li l
quan trng nht trong lp k hoch v hin c q trnh - c tt nht
khi m bo.
- Thit k tt
- K hoch lp m mun hiu qu
- i ng pht trin c t chc tt
- Mt k hoch tch hp tt -
Nn cho tch hp v th nghim l tch hp
4.5.2 Nhng vn trong pha tch hp v th nghim
Chnh l trong pha tch hp v th nghim hu ht nhng vn pht
trin xut hin. Nhng s c m t tr- c y trong d n nh- l cc ri ro
th by gi thnh hin thc v nhng vn bt ng khc ny sinh
khng th trnh - c . Nhng vn gm :
- Ph sn vo pht cht : sai lm thit k v nhng vn thc hin
ny sinh. l nhng loi vn kh khm ph nu khng c phin bn
vn hnh ca h thng. Dp y chng khng - c pht hin sm trong d
n.
- Nhng vn v bn th ba k c giao chm v pha ng- i bn v
ch thu ph v khuyt tt trong h thng ph v thnh phn ca h
- Thay i pht cht. Vn ny tn ti mi pha nh- ng cng tr nn
nghim trng hn khi d n cng tin trin. Gi y thay i tr nn tn
km hn.
- V- t qu ngn sch (thm ch, bi chi) vn ny l do nhng thay
i v sai lm thit k cng nhu sai lm trong qui hoch d n.
- Nhng vn ng vin nhn s. iu ny th- ng xy ra vo cui d
n trong cc giai on th nghim cui cng.
- Nhng vn nghim thu d n. Vn ny c bit ph bin l
nhng d n gi c nh (coi ch- ng 3) khi cc tranh chp ny sinh lin
quan n vic hon thnh d n
Nhiu nhng vn c th trnh - c hay t ra tnh nghm trng
ca chng c th gim i khi chun b n chng sm trong d n chnh
v qun l ri ro bao gm c chun b k hoch d phng trong tr- ng
hp s c ri ro din ra th tnh nghim trng ca bt c vn no
th- ng c th - c gim i khi c k hoch cho n.
Nh- tho lun tr- c y, thay i l khng trnh khi v - c x l
tt nht nh c ch khng ch cc thay i c trt t.
QUAN L DU N PHN MM
76
iu ny c ngha khng c thay i no khng ngoi l c th - c
chp nhn tr phi - c trnh thng qua cc knh khng ch thay i.
Qui hoch cho nhng tht bi pht cht v sai lm thit k li cng
kh khn hn. Gii php fthng th- ng cho nhng loi vn ny l
chp nhn rng tht bi v sai lm l khng trnh - c . iu ny c
ngha chng phi - c tnh n trong lch trnh d n ngay d chng
khng th - c minh nh tr- c. Cch c truyn trong vic lp lch cc
vn ch- a bit l thm mt phn trm c nh vo lch trnh pht trin.
Kinh nghim l cng thm 30% cho nhng chm tr khng l- ng - c
v nhng vn khng d kin - c mc d mt t chc pht trin s
cui cng tch lu thng tin cn thit xc nh nhng d phng lch trnh
ca ring mnh, bng 4.2 l mt khi im tt. Ch- ng 10 tho lun v
nhng bin php phc tp hn nhm ci tin d ton da trn loi d n
- c pht trin.
15
Loi phn mm B sung vo lch trnh
H thng x l d liu th- ng mi nh 10%
H thng x l d liu th- ng mi loi va 15%
H thng x l d liu th- ng mi loi ln 20%
H thng truyn thng 33%
Cc h thng khoa hc, b bin dch v.v. 25%
Cc h thngiu hnh 25%
Cc h thng thi gian thc 33%
Cc giao din ca ng- i dng 15%
Pht trin phn cng,phn mm 35%
Bng 4.2
Nhng nhn t d phng lch trnh ca d n phn mm
4.6 Pha bo tr
Pha bo tr hon tt chu k pht trin phn mm v dn n vic cung
cp cho th tr- ng sn phm phn mm hon chnh vi vic pht trin sn
phm mi. T bo tr phn mm l c th gy tranh ci v n bao hm
nhu cu sa cha b h- . Trong cc h thng c hc hay in t, vic
bo tr c th i hi sa cha hay thay th nhng thnh phn b h- hi
v vic bo tr d phng c th i hi vic dch v vi cc thnh phn
phng nga h- hng. D sao phn mm khng h- . B my mang phn
15
Hiu qu ca cc nhn t lp lch c th - c ci tin bng cch phn d n ra nhiu thnh phn
(th d x l d liu truyn thng , thi gian thc v.v.) v vn dng cc nhn t thch hp vo mi loi
thnh phn khc nhau.
QUAN L DU N PHN MM
77
mm c th h- nh- ng bn thn phn mm x khng thay i
16
nu khng
c can thip ca con ng- i.
nh ngha ca IEEE v bo tr phn mm (IE E 19876) bao gm c
vic hiu chnh li c trong phn mm tr- c khi - c giao hng. Cng
nh- nhng thay i nhm ci tin cho hon m hay thch ng sn phm
vi mt mi tr- ng b i.
Hai hot ng khc l mt phn ca bo d- ng l
- Duy tr thng tin cp nht
-Cp nht nhng lp hun luyn ng- i dng
Khng ging nh- bo tr phn cng, khng c mt hot ng no trong
li lm cho phn mm tr li trng tho ban u ca n.
iu tri li mi hon ton ng : mc tiu y l sa i phn mm
sa i phn mm bao gm mi c im ca pht trin phn mm.
Nhng iu sa i cn - c m t, thit k, thc hinm ho nhp v th
nghim chnh thc, v tt nhin,nhng sa i cn - c ti tr . Do y
l thc hnh cng trnh vng chc thc hin giai on bo tr coi
nh- mt lot nhng d n phn mm nh.
Kim tra cu hnh l c bit quan trng trong thi gian bo tr qun
l nhng thay i vi phn mm v kim tra nhng t pht hnh v
phin bn ca h thng. iu ny m bo nhng t pht hnh nh k
c trt t ca phn mm v trnh - c h tr phiu l- u v c nh ti ch
c th tr thnh cn c mng cng trnh.
Qun l d n khng phi bao gi cng - c tin hnh t pht trin
sn phm phn mm sang giai on bo tr bo tr i h mt i nh
hn nhiu v qun l kiu khc. Trn thc t, mt nhm bo tr duy nht
c th - c thit lp bo tr nhiu sn phm vi qun l chung, khng
ch cu hnh k s- lp t v hin tr- ng v bo tr t- liu.
T- liu cn - c cp nht trong giai on ny bao gm :
- T- liu pht hnh phin bn
- Cc bo co vn
- Mi t- liu pht trin
- Mi t- liu ca ng- i dng
- nht k bo tr v bo co dch v khch hng
Bo tr - c tip c chng no sn phm phm mm - c lp t v
hot ng. Khi h thng gi i, cc k hoch - c chun b cho mt d
n pht trin mi thay th cho h thng lo ho. Vo lc , cc c gng
bo tr - c gim ti mc ti thiu do k vng l mi sa i yu cu v
hiu chnh sai k thut s - c c trong h thng mi.
4.6.1 Bu khng kh trong pha bo tr
16
Theo mt ngha no , phn mm c th - c coi l ang thay i nu do sai st v con rp hay
thit k n t sa i (gi l b tn cng). iu ny c th phng - c khi l- u gi phn mm trong
ROM (b nh ch c)
QUAN L DU N PHN MM
78
Trong nhiu tr- ng hp, bo tr khng to ra thch pht trin mi.
iu ny c th dn n i ng bo tr khng n nh vi s k s-
th- ng xuyn nhp v ri b i ng. D sao bo tr c th to ra nhiu
thch chng hn nhn bit nhng vn gay cn, to ra gii php v
gi cc ci tin. Ni chung, giai on bo tr c tr- ng .
- Thiu nhit tnh, trong nhiu tr- ng hp, do thiu thch k thut
xc nh c th.
- p lc to ra nh v nhanh chng. Trong mut mi tr- ng bo tr c
trt t, hiu chnh vn th- ng him khi nhanh - c .
- V mng do thiu ngn sch y . Cc hot ng bo tr khng
phi bao gi cng - c cng nhn l quan trng v do th- ng khng
ngn sch chi
4.6.2 Nhng vn trong pha bo tr
Nh- chng ta thy, bo tr phn mm bao gm mi pha ca ton b
d n pht trin. Do nhiu vn pht trin th- ng thy trong cc pha
pht trin c bn li cng l thng th- ng trong pha bo tr.
Cc vn khc c tr- ng cho pha ny bao gm :
-Khng k s- bo tr c kin thc do kh khn trong vic tuyn
k s- t nguyn nhn cng vic bo tr
- Nhng vn ngn sch lin kt vi nhng pht hnh mi ca h
thng.
- Chp v chng p ca h thng hin c. Sau mt thi gian, ngay c
th tc bo tr c trt t cng c th to ra mt h thng phn mm chp
v. iu nyth- ng nhc nh s pht trin h thng mi.
- Thiu thit b h tr v nn th nghim. Vn ny li cng th- ng
l kt qu ca ngn sch khng cho cc hot ng bo tr.
Vi ng- i qun l bo tr, tuyn i ng bo tr ch yu l vn to
lp s b nhim l th v thch thc. iu ny th- ng c th thc hin
bng cch giao trch nhim v mt d n n gin hay mt b phn xc
nh r ca mt d n ln cho mt k s- phn mm. vic u nhim ny
bao gm c vic minh nh vn , cung cp gii php v gi ci tin
trong phm vi trch nhim ca k s- .
Cch tip cn ny thc y s tn tu ca k s- vi nhim v v s
ng nht ca anh (hay ch) vi s thnh cng ca c ging bo tr.
Khng ging nh- ng c nhn lc i ng, nhng vn ngn sch v
thit b khng ph thuc tr- c ht ng- i qun l bo tr ngn sch phi
- c m bo t qun l cp cao v th- ng - c ti tr bi cc ng- i
dng h thng cn - c bo tr. L ph hng thng hay hng nm l cch
thng th- ng cho vic to ngn sch bo tr.
Vic tin trin tun t ca h thng chp v khng c gii php thc
no khc hn l ri cng phi pht trin h thng mi. D sao nhiu kh
khn lin kt vi h thng tin trin lin tc c th - c gim ng k
bng cch thc hin ch- ng trnh khng ch cu hnh mt cch trt t.
QUAN L DU N PHN MM
79
Nh- chng ta thy, bo tr m khng khng ch cu hnh c th tr
nn cc k kh khn cho qun l
4.7 Tmtt .
Vng i pht trin phn mm bao gm mi pha pht trin
T quan nim ban u ti bn giao cui cng ca h thng. C nhiu
cch tip cn khc nhau ti chu k pht trin phn mm k c ph- ng
php m v c nh, nhng ph- ng php lp (nh- to nguyen mu nhanh)
v cch tip cn theo pha
Cch tip cn theo pha ti pht trin phn mm phn chia vng i
pht trin thnh
- Pha quan nim
- pha yu cu
- Pha thit k
- Pha thc hin
- Pha tch hp v th nghim
- Pha bo tr
Tip cn theo pha (cng gi l m hnh thc n- c) - c bao gm trong
hu ht cc ph- ng php lun pht trin phn mm khc. Trn thc t c
nhiu bin th ca m hnh thc n- c c in, nh- ng tt c u cn c
mt qu c h thng t mt pha pht trin ny sang pha sau cho n
khi d n hon thnh.
M hnh thc n- c bt u bng pha quan nim ban u, trong qu
trnh nhu cu v h thng phn mm - c xc nh v quan nim c
bn ca h thng phn mm tin trin. Tip theo l pha yu cu n cung
cp m t chi tit h thng phn mm cn - c pht trin..Pha yu cu
quy nh - ng mc ch yu u tin ca d n. Sau xc nh ca yu
cu pha thit k s phn tch cc yu cu v xc nh ph- ng php thc
hin, pha thit k th- ng - c phn lm hai pha ring bit: Thit k mc
nh v thit k chi tit. - ng phn chia gia hai pha ny - c t ra
i phn tu , cn c mc phn gii ca h thng. Thit k ca h
thng quy nh - ng mc ch yu th hai ca d n trong pha sau (pha
thc hin) cc m un phn mm - c m ho v cc th nghim n v
ban u - c tin hnh. Sau , cc m un - c kim tra cht l- ng
phn mm chp thun v - c trnh ln kim tra cu hnh. Ri kim
tra cu hnh pht hnh nhng m un tch hp
Trong pha tch hp v th nghim cc m un phn mm - c tp hp
li v h thng dn d - c hnh thnh. Sau h thng - c th nghim
xem c p ng c im yu cu.
Cui cng pha bo tr hon tt chu k pht trin phn mm v ni tip
vic - a ra sn phm phn mm trn vn vi vic pht trin sn phm
mi. Bo tr phn mm gm c vic hiu chnh li c trong phn mm
tr- c khi giao nhn cng nh- nhng thay i nhm ci tin cho hon
thin v thch ng sn phm vi mi tr- ng thay i.
QUAN L DU N PHN MM
80
Bi tp
1. Mt hng sn xut my bay ln quyt nh cho cc k s- lm
vic ti nh mnh. Cc k s- - c cung cp my u cui v m un v
h s lin lc t nh vi my vi tnh ca cng ty. Hng sn xut my bay
cn b ch- ng trnh phn mm truyn thng v gim st qun l cng
vic ca k s- . B ch- ng trnh phi c - c c tr- ng an ton tinh vi
phng nga ng- i khng - c php nhp vo h thng ca cng ty v s
lm khi ng cc h thng bo ng nu c c mt truy nhp khng
- c php nh- th. H thng cng phi gim st gi gic k s- lm vic
ti nh v phi cung cp ngun lc chia x h s d liu chung v cc
tin ch phn mm. Cui cng b ch- ng trnh phi c cha c trc tuyn
v mode theo b bo co cc c im cung cp thng tin v vic
dng h thng bi cc k s- t nh h.
Hy phn tch b ch- ng trnh phn mm v kho st cc ph- ng php
thc hin khc nha. c bit, hy so snh cch tip cn to nhanh
nguyn mu vi tip cn thc n- c. Gii thch li v bt li ca mi cch
tip cn i vi d n .
2. Xem xt s chng cho cc pha khi vn dng ph- ng php thc
n- c vi h thng m t trong bi tp 1. c bit xem xt s chng cho
ca pha thc hin vi pha thit k. Chun b k hoch lp m cc m un
sao cho pha thc hin c th bt u cng sm cng tt. Gii thch nhng
m un no c th - c tch hp an ton tr- c v bnh lun ri ro c th
c.
3. Nh- bi tp 2 hy xem xt ri ro chng cho pha thc hin vi pha
hp nht.
Chun b mt k hoch tch hp cc m un phn mm sao cho tch
hp c th bt u cng sm cng tt. Hy gii thch nhng m un no
c th - c tch hp an ton tr- c v bnh lun ri ro c th c .
So snh vic s dng k hoch tch hp t trn xung vi k hoch t
d- i ln v k hoch t trong ra ngoi. M t tip cn no bn c th
khuyn nn dng cho d n ny.
4. Hy tho lun nhng vn chnh m bn c th i ch trong vic
pht trin h thng m t bi tp 1. c bit hy xem xt nhng vn
lin kt vi vic xc nh yu cu. Ai l khch hng ?
Cng xem xt nhng vn tch hp v th nghim trong d n ny v
tho lun nhng thay i yu cu c th xy ra. Cc vn s - c s
l tt nht th no ?
5. Hy xem xt t ra 6 c im b sung v ci tin cho h thng m t
bi tp 1. Chun b k hoch bo tr di hn bao gm vic pht hnh cc
h thng mi c hiu chnh sai st k thut v mt s c im mi
trong mi ln - a ra pht hnh.
Chi ph in thoi gi n cho cc k s- b b qua v khng th
- c h thng bi hon. Hy ngh gii php cho vn ny v - a n
vo lch trnh ca mt trong cc t pht hnh mi ca h thng.
QUAN L DU N PHN MM
81
Ch- ng nm
Nguyn tc qun l cc
k s- phn mm
Liu h c thc s c g khc ?
Theo nhiu nghin cu, qun l k s- phn mm kh khn hn qun l
k s- hu ht cc lnh vc cng ngh khc. K s- phn mm in hnh
(nu c con ng- i nh- th) th- ng - c biu th l va ti t va lgic
cng nh- va phc tng va tht th- ng. Nhng nt c th thy bt
c nhm ng- i no nh- ng c v tri hn trong cc k s- phn mm.
Cc k s- phn mm cng li rt a dng trong mc hiu sut ca
h. Xa x- a, nm 1968, Sackman v cng s c t- liu v s chnh lch
to ln v hiu sut gia cc nh lp trnh phn mm. Sackman bo co
v t l hiu sut ti 25:1 v lp trnhv 28:1 v g ri (ch- ng trnh).Tt
c nhng nh lp trnh tham gia vo th nghim u quen thuc vi cc
lnh vc ng dng ca ch- ng trnh, khin Sackman l gii kt qu coi
nh- phm vi tinh thng ca h
Hnh 5.1
Bn ct mc qun l
Cc kt qu ca Sackman c th c i cht cc oan. D sao trong
mt d n trung bnh khng phi khng ph bin c t l hiu sut 1:5
mt trong nhng mc tiu ca cng ngh phn mm thi hin i l
gim phm vi hiu sut kinh khng gia cc nh pht trin phn mm.
iu ny lm - c thng qua vic pht trin c t chc v c h thng
hn vn khng - c s hi lng ca nhng ng- i ng u (cha tinh
thn) u cao ca phm tr hiu sut. Nhng ph- ng php lun pht
trin phn mm cng gim i tc ng ca nhng nt nhn bn
khc ti qu trnh pht trin. iu ny - c thc hin qua vic vn dng
t- liu, bo co, tiu ch pht trin v cc cuc hp qui ch rng ri, bao
qut. D sao nhng th tc ch l s t ph- ng tin qun l con ng- i.
Con ng- i - c qun l thng qua c cu t chc. C cu tn ti cn c
3 Thm quyn Authority 4 Trch nhim Responsibility
2 Gim st Supervision
1 U thc Delegation
Qun l d n
QUAN L DU N PHN MM
82
trn bn ct mc qun l : u quyn, thm quyn, trch nhim v gim
st. (coi hnh 5.1). U quyn trao thm quyn, v thm quyn li to ra
(v i hi) trch nhim. C thm quyn v trch nhim i hi gim st
v gim st c hiu qu li i hi c cu t hc thch ng.
Hu ht cc d n - c t chc thnh i ng vi mi i ng - c
giao ph nhng chc nng c th trong phm vi d n. Cc loi d n
khc nhau i hi nhng loi c cu i ng khc nhau nh- chng hn
i ng cc nh lp trnh trung cp i hi lnh o i ng k thut
trong khi i ng chuyn gia c th ch i hi lnh o i ng hnh
chnh. l trch nhim ca ng- i qun l d n trong vic chn la c
cu thch ng nht cho d n.
Ch- ng ny cp n nhng vn v qun l con ng- i. Xuyn
sut ch- ng nhn mnh ch yu nhng vn lin quan n d n phn
mm. V tho lun khi qut hn cc ph- ng php qun l v ng vin
con ng- i, bn c vui lng tham chiu cun l th do vn phng thc
hnh kinh doanh s- u tp (1981).
5.1 C cu t chc d n phn mm
C nhiu cch t chc d n phn mm. D n cng ln th c cu t
chc cng tr nn gay cn hn. Nhng d n t chc ti gieo rc ln xn
v ln xn dn n d n tht bi.
H.5.2
Biu t chc d n ln phn mm/ phn cng
H.5.2 m t c cu c bn ca d n trong bn d- i ng- i qun l
d n ng l c 2 chc nng tng qut pht trin v h tr.
C cu d n phn mm rt c bn ny khng phi khng ph bin
nhng nm 50 v 60. vn cn l c cu d n c gi tr cho nhng d
n rt nh (c n nm nh sn xut) mc d c khi ngy nay vn cn
thy nhng d n ln.
H.5.3 M t biu t chc chi tit k c nhng chc nng h tr ch
yu. C cu t chc ny thch ng vi nhng d n ln (vi nhn s trn
20). Nhng d n nh c th khng i hi ph qun l d n hay cc
nhm kim tra cu hnh v bo him cht l- ng ring bit (coi ch- ng 7
tho lun chi tit v cc chuyn ny). Nhng d n rt ln (nhn s
Ng- i qun l d n
i pht trin
d n
Cc chc nng
tr gip d n
QUAN L DU N PHN MM
83
v- t qua 40) th- ng c th - c qun l d dng hn khi phn d n
thnh nhng d n ph. Hnh 5.4 cho thy biu t chc mt d n ln.
Biu ny bao gm c i ng pht trin phn cng v phn mm v
nhm hp nht trong phm v mi nhm.
Hnh 5.3
Biu t chc d n phn mm
Hnh 5.4
Biu t chc d n phn mm/phn cng ln
Ly th d, ta hy xt t chc ca 1 d n v tinh ln. Ngui qun l d
n trn thc t chu trch nhim v mt s d n. Trm kim tra mt t,
bn thn trm v tinh v trm ho tin. Phn mm cho mi d n ph
- c qun l trong phm vi vn phng d n duy nht ri mi d n ph
li do mt ng- i qun l d n ph, ph trch. Biu t chc t- ng t
nh- m t hnh 5.4 c th - c p dng cho d n v tinh, biu kt
qu - c m t hnh 5.5.
Chnh qun l d n
Ph qun l d n
K s-
h thng
Nhm th
nghim c lp
Bo him cht l- ng Khng ch cu hnh
Th- k
i pht trin 1 i pht trin 2 i pht trin n
i1 i1 i 2 i 1
i 2
i3
i2
i3
Chnh qun l d n
Ph qun l d n
K s-
h thng
Nhm th
nghim c lp
Bo him cht l- ng Khng ch cu hnh
Th- k
i pht trin 1 i pht trin 2 i pht trin n
QUAN L DU N PHN MM
84
Hnh 5.5
Biu t chc d n v tinh
R rng c cu t chc ca d n tu thuc loi d n - c pht
trin. Mt s vn phi xem xt l :
- Qui m d n : d n cng ln, t chc cng quan trng. Nhng d n
ln c tng ph thng tin v phi hp ng k v con ng- i do i hi
nhiu chc nng h tr hn.
- D n pht trin phn mm/phn cng. Vic pht trin lin tip phn
cng v phn mm khng d dng. vic lp k hoch, hp nht v th
nghim li cng phc tp hn v i hi nhng nhm h tr tn tu
Cc gi/c d n
Cc gi/c chc nng
Hnh 5.6
Biu t chc
Chnh qun l d n
Ph qun l d n
K s-
h thng
Nhm th
nghim c lp
Bo him cht l- ng Khng ch cu hnh
Th- k
i pht trin 1 i pht trin 2 i pht trin n
i1 i1 i 2 i 1 i 2 i3 i2 i3
i1
G/.
D n B
G/.
D n A
G/. th nghim
i th nghim
i bo him
ch/l.
i th nghim
i bo him
ch/l.
G/. cng trnh
f/m.
G/. bo him c/l
nghim
i3 i2 i1 i3 i2
QUAN L DU N PHN MM
85
- Cc h thng tin cy cao. Bt c h thng no nhy cm vi cc vn
tin (nh- h thng qun s hay cu mng) i hi c gng ch yu
v bo him cht l- ng. Cht l- ng cng l mt nhn nh quan trong
trong nhiu sn phm phn mm - a ra th trung (th d b ch- ng trnh
thng tin). Nhng loi d n i hi mt t chc bo him cht l- ng
ring.
- C cu tp on. T chc d n ph thuc rt nhiu vo c cu tng
th ca cng ty trong d n - c pht trin. Nhiu nhng chc nng
h tr d n c th - c cc nhm tp trung ho trong cng ty cung cp.
Trn thc t, nhng dch v c bn nh- dch v ti chnh vn phng php
l th- ng do t chc b con hay ca tp on cung cp.
C cu tp on th- ng nh ra mt trong hai loi t chc d n c bn
: ma trn hay khi thp. Hnh 5.6 m t c cu ca mt t chc ma trn
(so snh vi c cu khi thp trong hnh 5.4).Trong mt t chc ma trn
tp on, ng- i qun l d n qun l nhng hot ng k thut ca i
ng d n trong khi dnh lu ca anh hay ch y tong nhng vn nhn
s phi k thut (th d duyt l- ng, bt, o to) th rt t.
Li ch ca t chc ma trn l :
- Tinh thng hn: Mt t chc ma trn c th gi chuyn gia trong
nhng lnh vc c th (thng tin, c s d liu, ho.v.v) ri nhng
ng- i - c u nhim vo cc d n khc nhau. D n n c khng
bao gi c th t cho c - c iu xa x trong vic gi chuyn gia mi
lnh vc.
- C ng : d chuyn ng- i hn t d n ny n d n khc. iu
ny dn n s dng - c tt hn s tinh thng c - c.
- Ch trng v qun l d n: Ng- i qun l d n - c rnh rang v
nhiu nhng nhim v qun l i ng, ginh - c nhiu thi gian hn
tp trung vo nhng kha cnh k thut ca d n. D sao t chc ma trn
cng c nhng bt li ng k.
- t bin php qun l hn. Mt trong nhng cng c u tin trong
vic hnh thnh ng c, bt l tut ra khi tay ng- i qun l d n.
Ngui qun l t c nh h- ng hn n l- ng ca ng- i sn xut v vai
tr chuyn mn trong t chc.
- Tnh trung thnh ca i ng thp hn. Mi nhn vin u mun bit
chnh xc ai l cp trn ca h. Trong mt t chc ma trn, nhn vin c
hn mt cp trn. iu ny to ra s phn ho trung thnh v mi lin kt
yu hn gia nhn vin v ng- i qun l.
Nhng bt li th- ng ln t nhng li im ca t chc ma trn tp
on. ng c l yu t ch o cho s thnh cng ca d n v mi iu
g xi mn ng c th- ng i ng- c li li ch tt nht ca d n. Bt
hnh l li ch tt nht ca d n li khnglun lun trng hp vi nhng
li ch tt nht ca cng ty.
Nhng t chc khi thp to nn mt trt t r rng xc nh c th
trong mi c nhn bit - c c- ng v ca chnh mnh v c- ng v ca
QUAN L DU N PHN MM
86
nhng ng- i trn v d- i h. Khi bt v qui ch ng vai tr ch yu
trong hnh thnh ng c : (v h th- ng lm th) th t chc khi
thp l c hiu qu nht. Nhiu nhng yu t khc hnh thnh ng c
thc hon thnh khen ngi v qu trng v y mi ch l mt vi (coi
phn 5.4 tho lun thm v chuyn ny). Mc d bt v qui ch
khng phi lun lun ; ng c hiu qu nht, ng- i qun l d n rt
him khi t b bt c cng c t chc khi thp th- ng l tt nht.
5.2 C cu i ng.
Tr vi nhng d n rt nh (s ng- i sn xut t hn 5) th nhng d
n phn mm - c t chc tt nht l thnh nhng i ng pht trin
nh. Qui m l t- ng ca mt i ng pht trin l khong bn v su
ng- i sn xut. Nhng i ng ln hn ch kh nng ca lnh o i
- c hot ng l ng- i sn xut v do tng tng ph qun l v hn
ch dnh lu k thut ca lnh o i vo d n
Cc i ng em li cho ng- i qun l d n nhiu li im, bao gm :
- Qun l d dng v tt hn : C cu i ng h tr vic u nhim
thm quyn.
- Trao i thng tin v kin hiu qu hn do lm quen - c rng hn
trong i ng vi nhim v ca mi thnh vin.
i ng gim kh nng k s- khng thay th - c khi chia x kin
thc duy nht trong phm vi i khng c ng- i n c no tr thnh
ngun thng tin gay cn c.
Trong cc d n nh, c s ng nht mnh hn vi d n. nhng d
n ln nhng ng- i sn xut c xu h- ng cm thy h chnh l mt trong
s rt nhiu v ng gp ca h cho d n s tin hnh khng ai hay.
Nh hn, gn b hn, i ng c th hnh thnh tn tu hn.
5.2.1 Lnh o i.
Lnh o i ng - c coi l knh thng tin chnh gia ng- i qun l
d n vi cc thnh vin i. iu ny khng c ngha l khng c
thng tin trc tip gia ng- i qun l d n vi cc thnh vin i. D
sao nu mi thng tin l trc tip th nh- th iu ny hn lm cho nhng
mc ch chnh ca c cu i b tht bi: u nhim thm quyn v trch
nhim - c hiu qu (coi hnh 5.1).
Vai tr ca lnh o i l :
- i din qun l d n thng qua u nhim thm quyn
- i din i tr- c ng- i qun l d n
- i din i tr- c cc i d n khc v cc chc nng t chc.
Ng- i lnh o i cng c th c nhng trch nhim khc, tu theo
th loi c cu i Hnh 5.7 cho nhng th d hai c cu i khc nhau
(nhng c cu i ny - c bn sau). Mi lnh o i lnh o hnh
chnh khng phi tt c lnh o i lnh o k thut i ng k s-
tr- ng i hi kh nng lnh o k thut theo i hi thnh vin k
QUAN L DU N PHN MM
87
thut. C i ng dn ch v i ng k s- tr- ng i hi kh nng lnh
o i v hnh chnh.
Hnh 5.7.a
T chc i pht trin theo kiu dn ch
Hnh 5.7.b
T chc i pht trin theo kiu k s- tr- ng
i
vin
i
tr- ng
i
vin
i
vin
i
vin
K s-
Tr- ng
i
vin
i
vin
i
vin
i
vin
QUAN L DU N PHN MM
88
y l trch nhim ca ng- i qun l d n trong vic tuyn chn cc
i ng v u nhim nhng vai tr ca lnh o i. iu ny c th (v
th- ng phi) - c tin hnh qua tham vn cc lnh o i c kinh
nghim v cc thnh vin d n thm nin khc. C cu t chc v u
nhim nhng nhim v ch cht l mt phn ca k hoch pht trin d
n v phi - c hon thnh cng sm cng tt trong chu k pht trin d
n (bn chi tit v chu k pht trin coi ch- ng 4).
Cc lnh o i tr- c ht phi - c tuyn chn trn c s kh nng
lnh o c bn ca h. Nu kh nng ny khng phi l c hu trong
tnh tnh ca c nhn th vic o to t khi l . Sai lm chung l bt
mt k s- tt tr thnh lanh o i ti
17
.
Lnh o i ng, tr- c ht v trn ht l mt chc nng qun l v
nh- vy i hi - c o to. Mi lnh o i phi - c o to chnh
thc v k nng qun l c bn cn thit c cho vic qun l i v cho
vic i din vi qun l d n. Lnh o i c kh nng - c o to
tt tr thnh vic trin khai thng li ng- i qun l d n.
5.2.2 Cc i dn ch.
Nghim tc m ni, cc i dn ch khng c lnh o, chc nng ca
vai tr lnh o i l iu phi vin nhiu hn. cc i dn ch, cc
lnh o i ginh mt phn nh thi gian ca h cho :
- vic i din i trong thng tin vi qun l d n v cc i khc.
- Phi hp hot ng trong i
- S l cc nhim v hnh chnhkhc nh- bo co, lp trnh v gim st
hot ng.
Mi quyt nh k thut trong mt i dn ch - c c i thc hin
lnh o i triu tp cc cuc hp trong nhng vn gay cn v cp
thit - c - a ra tho lun. Nh- ng mi thnh vin i tham gia trong qu
trnh ra quyt nh v chu trch nhim v u ra.
Cc i dn ch th- ng thch ng cho cc nhm nh sn xut thm
nin c kinh nghim. Theo vai tr ca lnh o i gim tng ph
hnh chnh bng cch giao nhim v hnh chnh ca i cho mt thnh
vin duy nht. C cu i dn ch c bit khng thch hp cho cc
nhm hn hp, hay nhm bao gm ch yu nhng ng- i sn xut thanh
nin. Trong c hai tr- ng hp , vai tr lnh o r rng l cn thit.
5.2.3 Cc i k s- tr- ng.
i k s- tr- ng (cng gi l i cc tr- ng lp trnh) tin hnh lnh
o i pht trin. Vai tr ca lnh o i va l iu phi vin (nh-
trong tr- ng hp i dn ch) va l h- ng dn trong nhng d n phc
tp, lnh o i c th - c yu cu ginh n 50% thi gian vo nhng
hot ng k thut v hnh chnh.
17
Coi cun nguyn tc Peter , Peter v Hull (1970) tho lun l th v hin t- ng ny
QUAN L DU N PHN MM
89
Hot ng chnh ca lnh o i k s- tr- ng l:
- Giao nhim v v trch nhim cho cc thnh vin i
-Gim nh cng vic ca cc thnh vin i.
- Gp v h- ng dn cc thnh vin i.
- Hot ng hnh chnh v phi hp (t- ng t nh- lnh oi dn
ch). Cc i k s- tr- ng l thch ng cho i hn hp v i ch yu
c nhng ng- i sn xut than nin hay khng c kinh nghim. chc nng
ca lnh o i vi t- cch ng- i qun l hng u v do phi - c
p to thch hp v k thut qun l c bn. Cc i k s- trung cng
c th thnh cng trong nhng i ng- i sn xut n anh v c kinh
nghim nh- ng vai tr th- ng khng cn thit khi c cu ny - c vn
dng cho mt i cc k s- c kinh nghim th vic ng- i lnh o i
c nhng k nng qun l c bn li quan trng gp hai nu khng ffng
chm c th pht trin gia thnh vin i v ng- i lnh o i. V mt
ny, lnh o i ng ng- i chuyn mn c kinh nghim li kh hn l
lnh o i ng ng- i mi vo ngh.
5.2.4 Cc i chuyn gia.
i ng chuyn gia l nhng i ng nh - c thnh lp gii quyt
nhng vn c th trong mt d n. Mt i ng chuyn gia c th
- c thnh lp trong qu trnh pht trin d n khi c vn phc tp
ny sinh v sau i i c th - c gii tn khi vn - c
giiquyt. Trong mt s tr- ng hp, cc i chuyn gia c th h tr d
n sut chu k pht trin. Mc tiu ca i chuyn gia l tp trung gim
nh trong mt lnh vc ring ca d n.
Ly th d, ta hy xt mt h thng th qu ngn hng t ng c hai
h thng ph ch yu : conputer trung tm ca ngn hng v th qu t
ng t xa. K hoch pht trin cho hai h thng ph giao vic
pht trin ca mi h ph cho mt i ring nh- ng qui m ca d n
khng th m bo c mt i hp nht ring. C hai i s dng b
m phng thc hin th nghim v hp nht h thng ph ban u. D
sao khi hai h thng ph - c hp nht vi nhau, thng tin gia hai h
thng khng c.
Trong nhng tnh hung nh- th bao gi cng c nguy c l mt trong
hai i tm kim vn trong cng vic ca i kia. Nay d hai i hp
tc vi nhau tt, khc bit trong thc hin (hay trong thit k) c th lm
cho vn kh - c gii quyt. V v kch trnh bt u tr- t. iu ny
tr thnh mi quan tm ch yu cho ng- i qun l d n.
Trong nhng tr- ng hp nh- th, ng- i qun l d n c th quyt
nh thnh lp i chuyn gia gii quyt vn thng tin. i d n)
v hai k s- d n, mi mt ca hai i. Sau i chuyn gia c th tp
trung c gng ca mnh vo vic gii quyt vn cng nhanh cng tt.
Trong khi hai i kia tip tc cc hot ng pht trin khc. Sau khi vn
thng tin gia hai h thng ph - c gii quyt, i chuyn gia c
QUAN L DU N PHN MM
90
th gii tn v mot s mt, cc i th nghim c lp v i bo him
cht l- ng c th - c coi l i chuyn gia ( coi ch- ng 7 bn v cc
chuyn ). Cc i th nghim c lp lc u hot ng trong cc
giai on hp nht v th nghim ca chu k pht trin. i bo him
cht l- ng l mt th d v i chuyn gia hot ng xuyn sut chu k
pht trin ca d n.
i chuyn gia th- ng c nhng k s- c kinh nghim cao trong
nhng tr- ng hp nh- th iu chc chn nht l i - c hnh tanh nh-
i dn ch. Trong th d tr- c, i c th l i dn ch hay c th do
mt chuyn gia thng tin n anh lnh o.
5.3 cc K thut bo co c bn.
Vi ng- i qun l d n, iu ch yu l - c th- ng xuyn thng bo
v iu kin (tnh hnh) thc ca d n. iu ny - c thc hin khi m
bo - c lung thn tin chnh xc u n ca cc i pht trin. Nhiu
ph- ng php thu thp thng tin khng khch quan v da vo tnh chnh
xc ca cc bo co do chnh nhng ng- i sn xut d n cung cp. Cc
bo co gm :
- Bo co vit nh k v tnh hnh
- Bo co ming.
- Hp v tnh hnh
- Chng minh sn phm (denoi)
cc chng minh sn phm c bit ch quan v ch chng minh ci g
m ng- i sn xut mong mun - c ph ra. Ng- i qun l d n cn
thng tin khch quan. thng tin nh th th- ng c th - c thu thp t
nhng bo co ca cc nhm h tr nh- :
- bo co bo him cht l- ng.
- Bo co th nghim c lp
Mc d bo co v hp ng l nhng ngun thng tin c ch, khng
c g c th thay th tip xc trc tip gia ng- i qun l d n v nhn
lc pht trin. Nh- ng cuc ni chuyn th- ng xuyn khng chnh thc
vi nhng ng- i sn xut l nhng ngun thng tin tuyt ho c bit khi
- c tin hnh trong bu khng kh khng chnh thc (v khng phi
trong vn phng ca ng- i qun l d n).
Ng- i qun l d n phi duy tr cnh gic th- ng xuyn vi sia lm
th- ng - c gi l '' hi chng 90/50 '' khng nh l ''phi mt 50%thi
gian hon thnh 90% cng vic v 50% thi gian ph thm hon
thnh s 10% cng vic cn li'' . iu ny c ngha l nhng ng- i sn
xut d n s bt u khoe khoang sm l h ''hu nh- xong'' nhim
v ca h, bt hnh l c s khc bit ln gia ''hu nh- xong'' vi
''xong''.
Xong nhim v - vit ti liu v thanh ton mt vi vn cui cng -
th- ng li lu hn d kin ca nhng ng- i pht trin. C iu ny do
nhng hot ng to ra mt s kt qa nhn tin v nhng ng- i pht
QUAN L DU N PHN MM
91
trin c xu h- ng (sai lm) kt hp cng vic vi kt qu. Do y, nhng
ng- i qun l c th c - c nhiu thng tin hn nhng ng- i sn xut
khi hi h d tnh bao lu na th xong cng vic ch khng hi h
hon thnh - c bao nhiu cng vic.
5.3.1 Bo co tnh hnh
Mi thnh vin ca i ng pht trin khng tr ai u - c yu cu
lm bo co tnh hnh, cc bo co phi - c trnh nh k, th- ng l
hng tun hay hai tun mt ln v phi c t ra ba phn sau ( coi hnh
5.8).
1 Hot ng trong thi k bo co.
Mi phn ph trng... phn ny, m t hot ng ch yu trong thi k
bo cao m t mi hot ng phi ko di hai n ba dng. Hot ng
phi h- ng v danh mc nhim v d n hay c cu phn tch cng vic
(wbs) ( coi ch- ng 9 v m t wBS).
2. Hot ng qui hoch cho thi k bo co ti.
Mi phn ph, trong phn ny, m t hot ng ch yu d kin cho
thi k bo co ti. M t mi hot ng phi mt n hai dng.
3. Vn .
Mi phn ph, trong phn ny, m t vn ch yu hoc xy ra
trong thi k bo co hoc - c bo co , tr- c y v vn ch- a - c
gii quyt. iu ny c ngha l vn s - c tip tc bo co cho n
khi - c gii quyt. Ni ting, phn ny phi gii thch ti sao phn 1
ny ca bo co khng t- ng ng vi phn 2 ca bo co tr- c.
Mi bo co cng phi c :
1. Ngy bo co
2. Thi k bo co (th d 3-7 n 10-7-1992)
3. Tn bo co ( th d bo co tnh hnh i thng tin).
4. Tn ng- i trnh bo co.
Vic chun b bo co tnh hnh nh k phi mt khong 20 pht
nh- ng khng - c qu 30 pht. Nhng ng- i sn xut phi trnh bo
co trnh hnh ca mnh cho lnh o i mnh, sau lnh o i phi
hp cc bo co ca i thnh mt bo co tnh hnh duy nht trong khi
vn gi cng c cu bo co . Hot ng ny chim khong 30 pht
ca ng- i lnh o i nh- ng khng - c qu 45 pht (iu ny lm
- c d dng khi bo co - c chun b v trnh bng th- in t)
Mi lnh o i trnh bo co tnh hnh ca i cho ng- i qun l
d n.Cc bo l v tnh hnh khng cn trnh nhng bo co - c
d- a vo h s v ch trnh cho ng- i qun l d n khi c yu cu.
Ng- i qun l d n cng nhn - c cc bo co tnh hnh ca nhn
s h tr d n khc nh- k s- h thng d n hay ph qun l d n. Sau
ng- i qun l d n chun b bo co tnh hnh d n bng cch kt
hp cc bo co l nhn - c thnh bo co 3 phn duy nht. Ri bo
co tnh hnh d n - c trnh cho qun l u ngnh.
QUAN L DU N PHN MM
92
Cc bo co tnh hnh d n khng cn thit phi trnh cng tn
nh- bo co tnh hnh d n ni b. Cc bo co d n c th - c
trnh hai tun hay mi thng mt ln.
18
Hnh 5.8 th d v bo co tnh hnh hng tun.
ca Fohn Doe lnh o i
Gi Fnank Smith, qun l d n.
Ngy 15-6-1993
i giao din ng- i dng : bo co tnh hnh hng tun
Thi k 5-12 thng 6-1993
1. Hot ng trong thi k bo co
1.1 Thit k mn hnh gip ng- i dng (hot ng 3.12.6) - c hon
thnh ng lch . c im thit k - c trnh kim tra cu hnh.
1.2 Lp m khu ng ch huy bng m un (nhm hot ng 5.12)
tip tc v th- ng chm hn lch khong 1 tun
2 Hot ng d kin cho tun ti
2.1 Lp m khu ng ch huy bng m un (nhm hot ng 5.12) s
- c hon thnh v cc th nghim n v s bt u.
2.2 Hai thnh vin ca i (Rd v Foan) s d lp hc 2 ngy v giao
din ca ng- i lp trnh cho b ch- ng trnh giao din mi ca ng- i
dng. y l mt hot ng khng theo lch - c ph chun trong
cuc hp d n mi y. iu ny khng lm chm lch trnh, do hon
thnh sm khu ng ch huy bng m un (coi phn 1.2 trn)
3 Vn
3.1 B ch- ng trnh giao din cho ng- i dng chng ti qui hoch
lc u, xt thy khng thch hp cho d n. Hai thnh vin s nghin
cu b ch- ng trnh mi ngh (coi phn 2.2 trn) . Nu b ch- ng
trnh mi li xt khng thch hp, iu ny s nh h- ng nghim trng
lch trnh pht trin ca chng ta.
3.2 Mt thnh vin ca chng ti (pack Brown) s dng im cui
c VT100 ch khng phi trm cng tc trong 2 tun qua, do thiu cp
thi trm cng tc... y l l do v sao nhim v 5.12 ca pack Brown
khng hon thnh tun ny theo lch.
5.3.2 Cc cuc hp tnh hnh d n
Cc cuc hp tnh hnh d n phi - c tin hnh nh k, thng
th- ng mt tun mt ln. Thi gian tt cho cuc hp tnh hnh hoc l
vo cui ngy cui tun hay u ngy u tun. Cc cuc hp tnh hnh
cng ng gp vo khng kh trt t v kim tra trong phm vi d n v
phi - c tin hnh u dn vo gi gic nht nh. Nhng thnh vin
18
Ch- ng 9 bn thm cc bo co tnh hnh - c s dng th no cp nht lch trnh pht trin
ca d n
QUAN L DU N PHN MM
93
khng th tham d cuc hp tnh hnh d n c th vi s ng ca
ng- i qun l d n, u nhim thnh vin khc ca i mnh tham d.
Ng- i qun l d n chun b cho cuc hp tnh hnh bng cch duyt
li cc bo co tnh hnh ca cc thnh vin ch cht ca d n trnh
(c bit xoy vo phn vn ). Do y bo co tnh hnh phi - c
trnh t ra hai n ba gi tr- c cuc hp tnh hnh.
Cc thnh vin ch cht ca d n tham d cc cuc hp tnh hnh d
n. Cuc hp bt u bng bo co hot ng ca d n v nhng vn
tng qut do ng- i qun l d n trnh by. Sau mi thnh vin - c
ginh 5 n 10 pht bo co v hot ng ca i mnh hay lnh vc
trch nhim ca mnh. Vic tho lun vn khng gii hn ng- i bo
co vn v ng- i qun l d n. Mi vn c th - c mi thnh
vin nu ra vi h tr c th c gia cc lnh o i theo th lm cho
kinh nghim ca h c - c xuyn sut d n. Vai tr ca ng- i qun l
d n khng phi l cung cp gii php cho cc vn m l h- ng dn
cc thnh vin i h- ng v gii php.
Gii php phi c ging c - c trong cuc hp tnh hnh. Bt c vn
no khng - c gii quyt trong phm vi nm pht phi - c hon li
cc bn lin quan tho lun sau cuc hp tnh hnh. Bin bn ca cc
cuc hp tnh hnh d n phi - c l- u h s nhng bn cp nguyn vn
khng c yu cu mc d nhng vn sau phi c trong bin bn;
1 ngy hp
2. tn cuc hp
3. C mt (danh sch ng- i d)
4. vng mt (danh sch ng- i - c mi vng mt
5. Cc mc hnh ng (tn , hnh ng, ngy hon thnh)
6.quyt nh ch yu cc vn tho lun.
Bin bn hp tnh hnh d n phi - c nh my v phn pht sm
khng chm qu cui ngy . iu ny l c bit quan trng khi c
nhng mc hnh ng phi hon thnh trong cng ngy. Khi d n kh
ln cn c th- k th bin bn s - c lp v nh my bi th- k.
Trong nhng d n nh hn, ng- i qun l d n c th quay vng nhim
v ny mi tun gia cc thnh vin.
5.4 Nhng - ng li chung trong qun l cc k
s- phn mm.
Nhiu - ng li trong phn ny cng vn dng - c cho mi loi k
s- . D sao do nhng c im a dng ca cc k s- phn mm tho lun
u ch- ng ny, nhng k thut qun l tt li quan trng trong vic
qun l phn mm hn hu ht cc lnh vc cng nghip khc. Do y,
- ng li c bn cho cc nh qun l phn mm l vic gio dc trong
lnh vc cc ph- ng php qun l hin i, c bit trong vic qun l
con ng- i, l thit yu cho s thnh cng. iu ny l ng vi cc lnh
o i v c cho cc nh qun l d n.
QUAN L DU N PHN MM
94
Qun l theo tho thun tt hn qun l theo ch th. Bao gi cng tt
nht l c - c k s- chp nhn nhim v - c giao. iu ny to ra ci
vn th- ng - c coi l s hu nhim v v lm tng ng k ng c ca
k s- . Nhm thc hin iu ny, cc k s- phi - c giao nhim m h
mun bt k khi no c th - c. Nu iu ny khng th, ng- i qun l
d n phi m t trung thc nhng ph- ng n c - c v gii thch cho
k s- v sao nhng nhim v c th li khng c.
Trch nhim lun phi i lin vai k vai, vi thm quyn. Mi thnh
vin iu hnh d n bt k l thanh nin, phi - c giao thm quyn
hnh ng trong phm vi trch nhim ca h. Cc k s- bo him cht
l- ng phi c - c thm quyn ph duyt hay bc b nhng thnh phn
sn phm v cc k s- pht trin phi c - c thm quyn c nhng
quyt nh trong thit k lin quan n thnh phn m h ang pht trin.
D sao, khng c thm quyn no l tuyt i, v nhn s cp cao hn
trong d n phi th- ng xuyn xt duyt li cc quyt nh v nhy
vo khi h cm thy c sai lm xy ra.
Qui tc chung l ch sai lm r rng phi - c hiu chnh. iu ny
dn n mt trong nhng qui tc c bn ca pht trin d n ; cch tt
hn lm - c iu g khng phi l c s thay i . Ni cch
khc nu ng- i qun l cm thy l anh hay ch c th c quyt nh
tt hn, nh- ng quyt nh hin nay chp nhn - c th quyt nh phi
khng - c thay i.
Quyn s hu khng ch lin quan n nhim v m cn n lch v
ngun i i vi nhim v. Mi lch trnh phi - c s tho thun ca
ng- i thc hin trn thc t cc d ton lch trnh phi - c chun b
cng vi nhng ng- i thc hin. iu ny m bo cam kt ca ng- i
thc hin hay ng- i sn xut tham gia vo lch trnh.
Cui cng, ng c l yu t duy nht quan trng hn c trong vic
qun l thnh cng ca i ng pht trin. ng c c th - c khuyn
khch bng nhiu cch.
- L- ng v th- ng
- bt
- iu kin v mi tr- ng lm vic
-Quyn li trong cng vic - c giao
- Qi trng
- Tn gi tr
- Thc y thc hon thnh
Nhiu ng- i qun l c sai lm khi cho l ng c hng u l l- ng.
iu ny khng phi th. ng c hiu qa nht l thc hon thnh v
tn gi tr (coi Giegold 1982). iu ny c ngha quan trng l phi gii
thch cho cc thnh vin i ng pht trin chnh cng vic ca h quan
trng th no cho d n v cng ty (v c th c cho t n- c hay nhn
loi nh- trong tr- ng hp pht trin h thng h tr y t). Con ng- i
- c kch ng cao khi h tin l h ang lm iu g quan trng.
QUAN L DU N PHN MM
95
Tn gi tr cng l mt ngun ng c cao. Vi ng- i qun l d n
th vic gi ng- i no n vn phng ca mnh v ni ''cm n anh
lm mt cng vic v i'' c th to nn k tch. Nhng cch khc tn
ga tr l gi th- nh gi c bn sao gi qun l cp cao hn v cho ban
nhn s hoc cho nhn vin c c gng c bit - c i ngh cui
tun. Mi iu s tng thm ng c, khng ch cho c nhn nhn
vin - c cm n m cn cho nhng ng- i khc xuyn sut d n.
5.5 Tmtt.
Qun l cc k s- phn mm kh hn qun l k s- hu ht cc lnh
vc cng ngh khc. Cc k s- phn mm kh a dng v mc hiu
sut. Trong mt d n trung bnh, khng phi khng ph bin l t l
hiu sut 1.5. Mt trong nhng mc tiu ca cng ngh phn mm thi
hin i l gim phm vi hiu sut trong cc nh sn xut phn
mm thng qua nhng ph- ng php pht trin phn mm c h thng v
c t chc hn.
C nhiu cch t chc d n phn mm. C cu t chc d n thch
hp nht tu thuc loi d n - c pht trin. Nhng d n ln v phc
tp i hi c cu t chc ln. Bt k qui m d n, cc d n phn mm
phi - c t chc thnh nhng i pht trin nh. V mt l tung, mt
i phn mm s c bn n su nh sn xut v s do lnh o i ph
trch.
C ba loi i phn mm :
- Nhng i dn ch, do lnh o i lnh o v mt hnh chnh
nhng quyt nh k thut do mi thnh vin i ra
- Cc i ng k s- tr- ng (hay i ng cc lp trnh tr- ng) do k s-
n anh c kinh nghim lnh o v trch nhim l lnh o c v mt
hnh chnh v k thut.
- Cc i chuyn gia - c lp gii quyt cc vn c bit trong
phm vi d n. Cc i ny c th - c gii th khi vn dc bit - c
gii quyt
Vi ng- i qun l d n, iu cn bn l - c thng bo. Th- ng
xuyn v tnh trng thc ca d n. C nhiu ph- ng php thu l- m
thng tin phn ln da trc tip vo vic cung cp thng tin ca ng- i
sn xut. D sao, nhng cuc trao i th- ng xuyn khng chnh thc
vi nhng ng- i sn xut cng l nhng ngun thng tin tuyt vi, c
bit khi - c tin hnh trong bu khng kh khng chnh thc (v khng
phi trong vn phng ca ng- i qun l d n).
ng c l tyu t duy nht quan trng trong vic qun l thng li ca
i pht trin. nhiu ng- i qun l c sai lm trong vic cho ng c
hng u l l- ng. iu ny khng phi th. ng c hiu qa nht l
thc hon thnh v t tn. Con ng- i - c c ng c cao khi h tin rng
h lm - c iu g quan trng v khi h cm thy c gng ca h
- c cng nhn.
QUAN L DU N PHN MM
96
Bi tp
1. Bn l ng- i qun l cho mt d n thng tin tp hp mi ca hng
vo mt dy chuyn ln cc ca hng. Mi ca hng thng th- ng c
my vi tnh ring mi mng vi ang k kim tra. Qun l dy chuyn
mun thit lp mt my vi tnh trung tm vn phng chnh v ni mng
mi my vi tnh ca cc ca hng vo vi tnh trung tm thng qua mng
khu vc rng ln. My vi tnh trung tm s nhn - c thng tin, thi gian
thc v giao dch cc ca hngt v mun cp nht c s d liu kim k
trung tm.
Bn hy chun b biu t chc khi thp cho d n thng tin ca
ca hng. Gii thch c cu m bn chn v v sao c cu li thch
hp hn cc c cu t chc khc c th c.
2. Lp k hoch lung thng tin trong phm vi t chc bn ngh
bi tp1. Cn nhng bo co no v phi - c trnh th- ng xuyn
th no? c nhng cuc hp tnh hnh no l cn v phi - c triu tp
th- ng xuyn th no? Ai d cc cuc hp ?
Gi tn bin ng v bo co tnh hnh v cc cuc hp nm tnh
hnh, cn c theo cc giai on pht trin khc nhau.
3. Chun b biu t chc m trn cho d n bi tp 1 . So snh n
vi c cu khi thp bn ngh bi tp 1 : c cu no l thch hp hn
cho d n ? c nhng im li v bt li no ca mi c cu t chc cho
d n ny
4. Bn hy nghi c cu ca cc i pht trin v h tr cho d n
m t bi tp 1. Gii thch c cu m bn chn. Bn c d kin nhu
cu cho cc i chuyn gia trong s pht trin ca d n ?
5. D n ca lp hc phn lp thnh 3 hay 4 nhm sinh vin . giao cc
bi tp 1,2 v 4 cho mi nhm cho mi nhm trnh by gii php ca
mnh tr- c lp.
So snh cc gii php m mi nhm ngh. Tho lun khc nhau gia
cc d n.
QUAN L DU N PHN MM
97
Ch- ng su
Chia tr cc d n ln th no :
phn chia v chimlnh
Nhu cu ln khng c ngha l kh
Nhiu i t- ng phc tp c th - c xem l mt b v vn i t- ng
dn gin hn. Mt th d thch hp l mt ho cht hnh thnh t nhiu
phn t khc nhau mi phn t - c to thnh khi phi hp nhiu nguyn
t khc nhau . Cc nguyn t mc dn bn thn phn chia - c c th
- c coi l phn t nh nht ca mt ho cht.
Theo cch t- ng t d n phc tp c th - c phn thnh nhng
thnh phn n gin hn. Trong khi ton b d n c th kh qun l th
mi thnh phn s d s l hn nhng d n phn mm c th - c phn
thnh nhng thnh phn nh hn nhm d tnh tt hn v khi l- ng
cng vic hoc nhmiu khin cc hot ng ca cc i pht trin khc
nhau.
Vic phn gii d n phn mm l mt trong nhng nhim v u tin
ca ng- i qun l d n phn mm . D sao ph- ng php phn gii c
th khc nhau, tu theo mc tiu thc s ca ng- i qun l d n.Vic
phn tch d n theo chc nng c th khng nh- l phn tch thit k.
Phn tch chc nng chia d n thnh nhng thnh phn c bn ca n
theo cch nhn ca ng- i dng trong khi phn tch thit k chia d n
thnh thnh phn hay m un lp trnh c bn.
Ch- ng ny bn n nhng ph- ng php qun l hiu qu nhng d
n phn mm phc tp v gii thiu nhng ph- ng php phn tch cc d
n thnh nhng thnh phn qun l - c. Cc loi phn gii d n cng
- c bn n v mc tiu ca mi loi phn gii - c gii thch.
6.1 tinh ch tng b- c mt
Theo trc gic c v khng hp l khu tm cch minh nh mi thnh
phn d n trong mt b- c duy nht. R rng mt qui trnh lp c th dn
dn cung cp nhiu chi tit hn, hn l d s dng hn. Nhng ph- ng
php lp thuc loi ny - c gi l tinh ch tng b- c mt v s phn
gii - c tip tc tinh ch mi b- c k tc
Hnh 6.1 trnh by ninh ho tng qut v tinh ch tng b- c mt . H
thng ban u - c phn thnh ba thnh phn cp cao v ri mi thnh
phn cp cao li - c phn tip thnh cc thnh phn cp thp hn, v c
th cho n khi t - c cp thnh phn thp nht.
Trong phn gii d n tng b- c, mi thnh phn phn gii thnh
nhng thnh phn trc tip d- i n sao cho mi b- c phn gii u m
QUAN L DU N PHN MM
98
t ton b h thng nh- ng mc chi tit khc trong hnh 6.1 cc thnh
phn 1,2 v 3 to nn h thng hon chnh. - c chi tit hn, chng ta
ly b- c phn gii tip v thy cc thnh phn 1.1,1.2,2.1,2.2,2.3,3.1 v
3.1 biu th tan b h thng.
Hnh 6.1
Phn gii phn mm bng tinh lc tng b- c
S tinh ch tng b- c mt t- ng t nh- biu mt h thng tn ti
trt t. D sao iu quan trng l phi hiu rng tinh ch tng b- c l c
bn khc nhau v cc khi cu thnh ca biu khc nhau. S h
thng tn ti m t mi quan h tn ti gia cc thnh phn khin cho mi
thnh phn trong s t- ng ng thc s vi mt thnh phn thc trong
h thng. D sao s tinh lc tng b- c, thnh phn cp cao hn ch l
tn t theo qui - c cho mt nhm cc thnh phn thc xut hin ngay
d- i n.
Trong h.6.2(a) phn mm ca h thng truy nhp - c c kim tra c
nm thnh phn , phn mm cp thp, nhn bit truy nhp phi php v
kch hot bo ng. Mi mt trong nm m un c th t- ng ng vi
mt m un phn mm thc ti.
19
Hai thnh phn cp cao kim tra truy
nhp v h thng bo ng khng tn ti l m un phn mm thc ti v
ch xut hin nh- l tn t cho hai nhm thnh phn cp thp hn.
Hnh 6.2 (b) m t cng phn mm ca h thng truy nhp - c c
kim tra nh- ng ln ny - c biu th nh- biu tn ti . y mi-
thnh phn trong biu biu th mt thnh phn phn mm thc.Thnh
phn khung chnh iu hnh ca h thng gi ra ba thnh phn khc nhn
bit khch thm , kim tra kho ca v kch hot bo ng thnh phn
nhn bit khch.
19
Tt nhin , y l s n gin ho h thng thc.Trn thc t nm thnh phn cp thp c th
- c phn gii tip thnh nhng- thnh phn cp thp hn
Hp phn 3
H thng
Hp phn 1 Hp phn 2
1.1 1.2 2.1 2.2 3.1 3.2 2.3
2.1.1 2.1.2 2.1.3 2.2.2 2.2.1 2.3.2 2.3.1
QUAN L DU N PHN MM
99
Hnh 6.2.a
Phn giI thnh phn mc trn ra cc thnh phn mc d- i
Hnh 6.2.b
S cu trc tn ti trt t
thm gi ra hai hnh phn : Nhn bit truy nhp nh- ng php v qun l
h s truy nhp.
6.1.1 Phn gii chc nng
Vic phn gii theo chc nng mt d n phn mm l s phn chia h
thng thnh nhng thnh phn theo hot ng ca n nh- ng- i dng
nhn nh. Vic phn gii theo chc nng l mt b phn ca pha yu cu
ca d n. Mc tiu ca pha ny l xc nh mi c im ca h thng
theo cch nhn ca ng- i dng.
Ta hy th xem xt mt h thng th qi ngn hng t ng kh nng
thng tin trc tuyn gia cc th qu t ng xa xi v my vi tnh trung
tm ca ngn hng nhm cung cp thng tin ti khon - c cp nht
l c im chc nng ca h thng. iu ny thng th- ng s - c xc
Phn mm h thng
truy cp - c /kh.
Kch hot
bo ng
Nhn ra truy cp
bt hp php
H thng
bo ng
Qun l tp truy
cp
Khng ch ng
h ca
Nhn ra
ng- i thm
Khng ch
Truy cp
Vng lp chnh
tha hnh h thng.
Bo ng
Qun l tp truy
cp
Nhn ra vic
truy cp bt hp php
Nhn ra
khch thm
iu khin
kho ca
QUAN L DU N PHN MM
100
nh trong pha yu cu ca chu k pht trin. D sao, ph- ng php
chuyn giao gia th qu t ng v my vi tnh trung tm khng phi l
c im chc nng ca h thng v y l bi trong thit k v thc hin
ca h thng ch khng hin nhin cho ng- i dng. Ph- ng php
chuyn gio k c nghi thc lin lc th- ng s - c xc nh trong giai
on thit k ca s pht trin ca h thng.
20
Hnh 6.3
H thng th qu t ng -- biu phn gii chc nng
D sao, ph- ng php chuyn giao gia th qu t ng v my vi tnh
trung tm khng phi l c im chc nng ca h thng v y l bn
trong thit k v thc hin ca h thng ch khng hin nhin cho ng- i
dng. Ph- ng php chuyn giao k c nghi thc lin lc th- ng s - c
xc nh trong giai on thit k ca s pht trin ca h thng.
O cc quyt nh thc hin c th tng lc - c - a ra trong giai on
yu cu v - c coi l yu cu thc hin. iu ny c th bao gm c
im nh- l loi my tnh i t- ng, ngn ng lp trnh - c s dng
hay ph- ng php truyn thng - c dng tt hn ht l nn hon, cc
quyt nh thc hin cng chm cng hay cho n pha thit k .
Hnh 6.3 nu mt th d phn tch theo chc nng ca mt h thng
th qu ngn hng t ng thnh nhng thnh phn chc nng mc thp
hn.
Trong hnh 6.3 chng ta xc nh l s c mt c s d liu khch
hng c th - c xem l quyt nh thit k. iu ny l khng trnh
khi. Phn tch chc nng him khi hon ton trnh - c mi nhn nh
thit k. Nh- chng ta bit phn gii chc nng th- ng khi l khi
im cho thit k ban u ca h thng.
6.1.2 Phn gii thit k.
20
Cc quyt nh thc hin c th i khi - c - a ra trong giai on yu cu v - c coi l
yu cu thc hin. iu ny c th bao gm nhng c im nh- l loi my vi tnh i
t- ng, ngn ng lp trnh - c dng hay ph- ng php thng tin s dng. Th- ng tt hn
nn chm quyt nh thc hin n giai on thit k bt c lc no c th - c.
H thng th qu
ngn hng t ng.
Dch v
ng- i vn
hnh
B sinh
bo co
Dch v my tnh
trung tm
Chc nng giao
din my th qu
Cp nht v hi
my tnh trung
tm
Dch v
ng- i dng
Dch v
th qu t ng
B sinh
bo co
QUAN L DU N PHN MM
101
Phn gii thit k ca mt h thng phn mm l phn chia h thng
thnh nhng thnh phn cp thp khp vi thnh phn phn mm thc s
ca h thng. Trong phn gii thit k hon ton ca mt h thng phn
mm, cc thnh phn thp nht t- ng ng vi cc m un lp trnh
(thng th- ng l cc th tc, ch- ng trnh con hay chc nng ch- ng
trnh)
ng nh- pha yu cu i tr- c giai on thit k vic phn gii chc
nng ca mt h thng phn mm thng th- ng i tr- c vic phn gii
thit k. Phn gii chc nng th- ng cung cp nhiu thng tin cn thit
cho vic phn chia tip h thng thnh nhng thnh phn thc hin.
Trn thc t, phn gii chc nng th- ng l ni tt khi s thit k
mt h thng phn mm v thnh phn chc nng ch yu ca mt h
thng th- ng t- ng ng vi phn chia ban u h thng thnh nhng
h thng con hay thnh phn mc cao.
Hai trong nhng chc nng chnh ca mt h thng ch qu ngn hng
t ng c th - c coi l my vi tnh trung tm ni l- u tr v bo tr
ton b thng tin ti khon v cc my th qu t ng giao din vi
khch hng. Mng thng tin ni my vi tnh trung tm vi cc a im
th qu c th - c xc nh l thnh phn b sung cp cao. Ri th
mng ny tt nhin l phn chia u tin ca h thng t vin cnh thit
k: (1) h thng ph th qu t ng
21
(2) h thng ph vi tnh trung tm
v (3) thit b mng thng tin.
Hnh 6.4
H thng th qu t ng -- biu phn gii thit k
21
nhng h thng hn cc h con th- ng tiu biu cho phn gii mc th nht ca h
thng. iu ny - c bn thm phn 6.3.
H thng th qu
ngn hng t ng.
Logic giao
din th qu
Logic th
qu
My tnh
trung tm
Giao din phn
cng
Giao din
truyn thng
my tnh th
qu
Giao din truyn
thng th qu
Giao din
mng din rng
Qun l c
s d liu
khch
hng
Th qu t ng
Driver
pht ting
bp
Driver
my in
Driver
mn hnh
Driver bn
phm
QUAN L DU N PHN MM
102
Hnh 6.4. nu mt th d phn gii thit k h thng th qu ngn hng
t ng thnh nhng cp thnh phn thit k thp hn. cp th ba thnh
phn th qu t ng phn gii thnh cc giao din phn cng v logic
th qu. Ri cp sau c th phn gii giao din phn cng thnh b kch
thch bn phm, b kch thch mn hnh, b kch thch my in v my
pht ting bp, cp ny, cc b kch thch c th i din m un
phn mm hin nay.
Mt im quan trng cn nh l trong phn gii thit k, ch c cc
thnh phn cp thp hn hin nay - c thc hin. Cc thnh phn cp cao
hn i din cho mt nhm cc thnh phn cp thp hn. iu ny - c
minh ha trong hnh 6.2(a) theo cc thnh phn kim tra truy nhp v
h thng bo ng i din cho hai nhm thnh phn cp thp hn (nhn
bit khch thm, kim tra kha ca v.v...).
V c bn phn gii thit k to ra hai loi thnh phn h thng thnh
phn cp cao v m un cp thp hn. Cc tiu chun pht trin phn
mm khc nhau s dng thut ng khc nhau nhn bit cc cp phn
gii khc nhau.
Chng hn, tiu chun 2167A US DOD (DOD 1988a) s dng mt
ph- ng php phn gii nhiu cp i cht nng n (coi h. 8.4). Tiu
chun s dng t CSU (n v phn mm vi tnh) nhn bit thnh phn
phn gii cp thp nht. Nhng thnh phn cp trung - c gi l CDC
(thnh phn phn mm vi tnh) v nhng thnh phn ny c th i din
mt nhm CSU v CSC. Cc CSC cp cao - c nhm thnh CSCZ (khon
cu hnh phn mm vi tnh) v nhng CSCZ - c qun l trong phm vi
d n coi nh- nhng n vi pht trin bn c lp. CSCZ l nhng thnh
phn c th - c thit k, chng minh bng t- liu v ph chun nh-
nhng thc th ring bit trong phm vi ton b d n phn mm. Tiu
chun DOD 2167a cho pht trin phn mm - c bn thm ch- ng 9.
Hnh 6.5.a
Phn gii phn mm mc nh ban u
Hp phn 3
H thng
Hp phn 1 Hp phn 2
QUAN L DU N PHN MM
103
Hnh 6.5.b
Phn gii phn mm mc trung gian -- thit k mc nh
Hnh 6.5.c
Phn gii phn mm mc chi tit
Mt h thng phn gii hon ton vi mi thnh phn cp thp ca n,
khng phi bao gi cng d nm bt, iu ny c bit ng trong s
trnh by h thng lc duyt d n khi h thng cn - c nhng ng- i
khng dnh li vo vic thit k n nhanh chng hiu - c. Trong nhng
tr- ng hp nh- th, k thut tinh lc tng b- c l ph- ng php thun li
trnh by tun t chi tit tng dn bng cch lc u choi thy cp
phn gii u tin v ri chm chm pht hin nhng cp tip theo. iu
ny - c chng minh trong hnh 6.5. Trong b- c phn gii trung thch
hp, chng ta c th phn thit k lm hai: cp cao hn v cp thp hn.
iu ny c bit - c dng khi giai on thit k - c thc hin thnh
2 giai on ring bit: thit k cp cao v thit k chi tit (coi hnh 6.5).
6.2. C cu phn tch cng vic.
Nh- vy, chng ta bn n vic phn h thng phn mm thnh
nhng thnh phn hoc chc nng hoc thit k. Chng ta cng s xem
xt vic phn d n phn mm thnh nhng thnh phn cng vic c bn.
Hp phn 3
H thng
Hp phn 1 Hp phn 2
1.1 1.2 2.1 2.2 3.1 3.2 2.3
Hp phn 3
H thng
Hp phn 1 Hp phn 2
1.1 1.2 2.1 2.2 3.1 3.2 2.3
2.1.1 2.1.2 2.1.3 2.2.2 2.2.1 2.3.2 2.3.1
QUAN L DU N PHN MM
104
Tng s ca nhng thnh phn cng vic bao gm mi nhim v cn
thit - c thc hin nhm hon thnh thng li d n.
6.2.1. Phn gii d n.
ng nh- mi nhim v phc tp ln khc, vic pht trin d n phn
mm d qun l hn nhiu bng cch tip cn chia v tr (phn chia v
chim lnh) vic tinh lc tng b- c, khi p dng cho mt d n phn
mm, to nn mi nhim v cng vic cp thp. iu ny bao gm nhim
v pht trin, nhim v qun l, nhim v h tr v nhim v hnh chnh.
Vic phn gii mt d n phn mm thnh nhng nhim v - c coi l
c cu phn tch cng vic hay WBS.
Hnh 6.6
Cu trc lit k cng vic (a) phc tho cng vic
Hnh 6.6. cho mt v d v biu c cu phn tch cng vic v bng
6.1. danh mc nhim v c cu phn tch cng vic t - c. Danh mc
nhim v WBS c mi nhim v cng vic d n v c th - c dng
lm cng c kim chng kim tra tnh hnh cc nhim v cng vic
- c giao. Danh mc nhim v WBS ban u th- ng t lch trnh d n
m ra lch c danh mc hot ng ca d n. Danh mc hot ng ca
lch t- ng t nh- WBS mc d mc ch ca n c khc nhau v th- ng
t chi tit hn. Danh mc hot ng ca lch - c m t ch- ng 9.
Hot ng bo ng
Qun l file
Hot ng
bo ng
Hot
ng bo

Hot ng bo ng
Qun l fil
Hot ng bo
ng
Hot ng
bo ng
Qun l
Hot ng bo
ng
Qun l file
Hot ng bo ng
Qu
Hot
ng bo
ng
Hot ng bo ng
Hot
ng bo
Hot
ng bo
ng
Hot
ng bo

Hot
ng bo
n
Hot ng
bo ng
Qu
QUAN L DU N PHN MM
105
Hnh 6.6
Cu trc lit k cng vic (a) phc tho cng vic
Bng 6.1. Danh mc nhim v c cu phn tch cng vic.
Nhim v M t Tnh hnh Giao cho Nhn xt
1. Qun l v hnh chnh
2. Pht trin phn mm
2.1. Phn tch yu cu phn
mm
2.2. Thit k phn mm
2.2.1. Logc kim tra
2.2.2. Giao din ch huy
2.2.3. Tin ch mn hnh
2.2.3.1. B khun mn hnh Trn vn J.Smith
2.2.3.1. B kch thch mn hnh Khi
cng
F.Brown
2.2.3.2. My pht khun Khi
cng
A.Black
2.2.4 Thng tin
2.3. M ha phn mm
2.4. Ha nhp phn mm
3. Cung cp v h tr pht
trin
Hot ng bo ng
Qun l file
Hot ng
bo ng
Hot
ng bo

Hot ng bo ng
Qun l fil
Hot ng bo
ng
Hot ng
bo ng
Qun l
Hot ng bo
ng
Qun l file
Hot ng bo ng
Qu
Hot
ng bo
ng
Hot ng bo ng
Hot
ng bo
Hot
ng bo
ng
Hot
ng bo

Hot
ng bo
n
Hot ng
bo ng
Qu
QUAN L DU N PHN MM
106
D n WBS
i khi li c ch khi gp cc nhm nhim v cp cao hn trong danh
mc nhim v WBS (th d nhim v 1, 2, 3, 2.1, 2.2, v.v... trong bng
6.1). iu ny gip truy nguyn c li cho mi nhim v cp thp nhm
nhn bit cc nhm nhime v t suy ra. D sao, vic - a cc nhm
nhim v cp cao trong danh mc nhim v WBS cng c th gy ra hn
n v nhng khon ny khng i din cho nhng nhim v hin nay
- c giao v nhn thuc tnh tnh hnh v y nhim cho khng p dng
- c cho chng.
Danh mc WBS cc nhim v d n - c suy t bn thng k cng
vic ca d n (SON) xc inh quy m ca d n. SON th- ng - c
chun b tr- c khi trong d n chnh thc (coi ch- ng 3) v th- ng l b
phn ca hp ng d n gia khch hng v nh sn xut. Vi nhng d
n ni b, khi mt t chc ti tr cng vic pht trin ca chnh mnh,
SON tr nn v danh vi vic xc nh chi tit d n hay mt t- liu
t- ng t xc nh qui m cng vic cho ng- i qun l d n phn mm.
6.2.2. WBS l mt cng c qun l d n.
C cu phn tch cng vic xc nh mi nhim v phi thc hin
trong qu trnh pht trin ca d n. iu ny s bao gm c nhng
nhim v t cc phm tr d n nh- :
- Pht trin phn mm
- Bo tr
- o to
- Dn chng t- liu
- Lp t
- Qun l
- Cung cp
Nhng nhim v ny - c m t chi tit hn bng 6.2.
Mi lc, bt c cng vic no - c mt thnh vin ca i d n
phn mm thc hin phi l mt b phn ca nhim v WBS khng mt
thnh vin no ca i - c thc hin bt c nhim v g khng c trong
danh mc WBS cc nhim v.
WBS l mt cng c c ch vi iu kin phi lun lun cp nht. N
phi - c cp nht nh k cng vi k hoch pht trin d n v lch d
n. iu hp l l nn d kin danh mc WBS c - c nhng nhim v
b sung, sa i hay thm ch rt ra trong khi tin pht trin ca d
n WBS ch yu l mt cng c qun l cung cp kh nng y thc
nhim v xc nh r cho cc thnh vin ca i pht trin. Chnh l qua
WBS m tin - c gim st khi cc nhim v hon thnh v nhng
vn tim tng - c pht hin. Nhng nhim v mi tng b b qua
ny - c nhn bit v cc d ton - c xem xt li da trn nhng ngun
hin nay - c s dng cho nhng nhim v hon thnh.
QUAN L DU N PHN MM
107
WBS cng l mt cng c ngn sch cung cp ph- ng tin tnh gi
mi hot ng pht trin vo mi on thch hp trong ngn sch d n.
y l mt trong nhng ph- ng php c bn lp k hoch v gim st
chi ph d n.
C nhiu tin ch vi tnh ha c - c h tr bo tr WBS. Nhng
tin ch ny hot ng c trn my vi tnh nh loi PC v nhng phn
chnh ln. Nhng tin ch WBS i khi c - c nh- l b phn ca tine
ch qui hoch chung ca ng- i qun l v cung cp cc c im lch
trnh v gim st khc nh- phn tch PERT v hnh thnh bo co.
C nhng ph- ng php khc - c pht trin qun l nhng nhim
v cng vic cp thp bao gm mt d n ln. C v vn bin thc v ci
tin k thut c cu phn tch cng vic
22
, mt s s dng nhng cng c
truy tm tinh vi, s khc s dng nhng k hiu v k thut c bit
phn tch v gim st danh mc nhim v cng vic.
6.3. X l cc d n ln.
R rng, nhng nhim v nh d s l hn nhng nhim v ln. Nh-
chng ta thy, y l lp lun bo v vic phn cc d n ln thnh
nhng thnh phn nh hn.
Trong mt tranh lun v kin trc ca h thng phn mm tu con thoi
v tr, Carlow (1984) nhn xt l "... ch cng c v k thut thi
khng th phng nga hay khc phc nhng vn s tn ti nu kin
trc h thng cu trc tt khng - c thit lp trn u mt tin ca
qui trnh pht trin." Trong tham lun ca ng, Carlow m t lp lun
bo v c cu v kin trc ca phn mm phc tp cho mt trong nhng
h thng m phn ban h thng lin bang IBM pht trin cho tu con
thoi v tr NASA. im phi lm l kin trc phn mm vn - c xc
nh sm trong chu k pht trin ng vai tr ch o trong vic xy
dng cht l- ng ca sn phm phn mm cui cng. Hay, tm li, nhng
sai lm ch yu th- ng bt u sm.
Con - ng m mt h thng - c phn gii, ng gp ng k vo
kin trc phn mm. Thng th- ng khng c iu g ta nh- kin trc
duy nht ng hay phn gii duy nht ng c. H.6.7 gii thiu hai phn
gii cp cao c th - c ca mt h truy nhp - c kim tra. C hai c v
l nhng on hp l ca mt h thng nh- ng kin trc trong h.6.7.(b) c
th i hi t giao din hn gia cc thnh phn phn mm chnh.
Gi chng ta s xem xt mt s h- ng dn phi theo khi la chn mt
c cu phn gii ring.
6.3.1. Cc h thng ph.
22
Cc ph- ng n k c m hnh Wilson v Sifer (1990) thay th WBS vi k thut qui hoch c
cu da trn trt t biu dng sng cng vic (WFD), mt hnh thc mng nhim v bao gm mi
u vo v u ra ca nhim v. Ph- ng php Wilson v Sifer l ph- ng php chnh thc ha cao v
- c coi l phc tp hn WBS c in.
QUAN L DU N PHN MM
108
Cc h thng i khi gm nhng thnh phn ch yu bn c lp bn
thn chng c th - c coi l h thng. Mt h thng th qu ngn hng
t ng tt nhin c th - c phn lm hai h ph hn: h ph vi tnh
chnh v h ph vi tnh th qu t ng. Theo th mt h ph cha hu
ht c im ca mt h, tr khi h ph khng c nh hot ng trn
chnh n.
Hnh 6.7.a
Phn gii h thng mc cao. Cch (a)
Hnh 6.7.b
Phn gii h thng mc cao. Cch (b)
Khi cc h thng phn mm ln c th chia thnh nhng h thng ph
chng loi b mt s tnh phc tp, c tnh cht rng ln, t- ng bo
v cho tip cn cn c vic qun l pht trin mi h thng ph coi
nh- h ring bit trong phm vi c th - c.
Vi mi h thng ph, ng- i lnh o i hay ph qun l d n - c
y thc phn ln trch nhim cho pht trin trong khi cc trch nhim
chng cho mi h thng ph - c qun l d n trong tm s l.
Hot ng bo ng
Hot ng bo ng
Qun l file truy nhp
Hot ng
bo ng
Hot ng bo
ng
Qun l file t
Hot ng bo ng
Hot ng
bo ng
Qun l f
Hot ng
bo ng
Q
Hot ng bo
ng
Qun l file
Hot ng bo ng
Qun l file truy
Hot
ng bo
ng
Hot ng bo ng
Qun l file truy nhp
Hot ng
bo ng
Hot ng bo
ng
Qun l file
Hot ng bo ng
Hot ng
bo ng
Qu
Hot ng
bo ng
Hot ng bo
ng
Q
Hot ng bo ng
Q
QUAN L DU N PHN MM
109
O mt d n qui m nh, nh- h thng th qu ngn hng t ng,
tho lun tr- c y, hai lnh o i c th - c giao trch nhim cho
hai thnh ph chnh ca h thng. Ng- i qun l d n s gim st hot
ng mi i v s phi hp hot ng giao din k thut v hnh
chnh gia cc i. Vic y nhim ca lnh o i phn mm - c bn
thm ch- ng 5.
O mt s d n qui m ln nh- pht trin phn mm cho tu con thoi
v tr NASA, d n qu khng l n mc trn thc t - c pht
trin nh- v vn d n ring bit. Vn phng Trung - ng chu trch
nhim v s phi hp v ho nhp mi h thng ph (c phn mm v
phn cng). Tham lun ca Dadden v Rone (1984) cp su n qui
m ln lao ca d n phc tp ny.
6.3.2. - ng li phn gii chc nng.
Nh- chng ta thy phn chia ban u ca mt h thng phn mm
l phn gii theo chc nng t- ng ng vi c cu ca phn mm nh-
- c ng- i dng nhn thc. Vic phn chia ny gip chng ta xc nh
nhng yu cu cho h thng v cung cp ph- ng php nhn bit cc chc
nng cp thp v gn chng vo nhng yu cu cho h thng ch yu.
Nhiu vn bn ngh l phn gii chc nng phi theo s phn tch
yu cu ca h thng. Trong nhiu tr- ng hp, phc tho chung cc yu
cu s to nn biu chc nng cp cao. Trn tinh thn , vic tnh lc
phn gii chc nng khng kht vi vic phn tch h thng trong biu
- c lun lun xem li v tinh lc khi li c thm - c thng tin.
Phn tch chc nng cp cao c bn ca h thng phn mm th- ng
cn c nhng t- ng tim nghim din tin trong giai on quan
nim. Ri nhng t- ng c th nh ra s phn chia c th ca h
thng, t din tin s phn gii chc nng.
S phn chia ban u ny ca h thng khng phi bao gi cng l
lgc nht v thch hp nht theo vin cnh ca ng- i sn xut. H thng
kim k rng ln ban u c th nhn thc l bao gm:
- Giao din con ng- i
- B pht bo co
- C s d liu
- Lgc cp nht.
Nh- ng giao din ca con ng- i c th li ko mt s cc my vi tnh
khc v mt phn nhng my c th ha nhp mnh vi b pht bo
co. Nhng c im ch yu, nh- tin v d tr c th b b qua.
Do y, s phn chia h thng tt hn c th khng xc nh giao din
ca con ng- i l chc nng cp cao m lng cc chc nng cp thp ca
giao din con ng- i ring r trong nhng chc nng ch yu khac. Cng
vy, d tr c th - c lng trong chc nng bo tr cp cao cng li bao
gm c cc chc nng nn v tinh luyn c s d liu.
QUAN L DU N PHN MM
110
- ng li chung l khng c phn gii chc nng duy nht no - c
la chn ch v - c quan nim u tin c.
Nh- chng ta thy, phn gii chc nng tt l quan trng v r rng
c th xc nh thit k v kin trc ca h thng - c pht trin, Chin
l- c tin cy trong vic xc nh phn gii chc nng phn mm - c tt
l triu tp cuc hp ca nhng ng- i trng yu ca d n bn mt s
nhng phn chia khc nhau ca h thng. Sau phn gii chc nng
phi - c la chn trn c s.
- L do (th d cc my vi tnh khc nhau th- ng h tr cc chc nng
khc nhau).
- D thc hin (th d phn gii chc nng tt s th- ng dn n thit
k tt).
- D hiu ( p ng ht cc chc nng ch- a ?)
Vic d thc hin ty thuc tiu ch thit k m ta s bn sau.
6.3.3. - ng li phn gii thit k.
Chng ta bit l phn gii chc nng ca h thng phn mm c th
c bn khc vi phn gii thit k ca h thng. D sao phn gii chc
nng tt s ch n thit k nh- giai on pht trin sau v s th- ng
l khi im tt cho vic phn chia h thng thnh nhng thnh phn
thit k cp cao.
Vic phn gii thit k ca h thng ch l mt phn ca ton b thit
k phn mm (c nhiu ph- ng php thit k, chng hn xin coi Fairly
(1985) hay Irossman (1987). D sao theo vin cnh ca ng- i qun l d
n, y l giai on quyt nh v vic phn gii thit k ca mt h
thng xc nh c cu ca phn mm nh- n s - c hnh thnh. Sau y
l tng quan v nhn nh thit k c bn nh h- ng n ph- ng php
phn gii h thng.
Mt trong nhng quan im c bn sm ca cng ngh phn mm i
hi tip cn v c cu vi vic thit k v lp trnh phn cng. Bn cht
c cu ca phn mm - c xc nh t nhng giai on phn gii u
tin. Vo cui nhng nm 1960 v u 1970. Difkstra i u trong
tip cn ny (1972). Mc tiu ch yu l chuyn sang pht trin phn
mm (hay n gin l lp trnh nh- sau ny - c gi th) t khi ch- a
thnh thc ti tr thnh mt ngnh cng ngh han ton pht trin. T
n nay nhiu k thut thit k c cu khc - c Yourdon (1978),
Jackson (1976) De Marco (1979) Warrier v Orr (1977) pht trin, y l
mi ch k mt s, nh- ng ch- a c tiu chun duy nht no ni chung
- c chp nhn.
Khi thu l- m - c nhiu kinh nghim hn trong pht trin cc d
n phn mm phc tp, tr nn hin nhin l phn gii thit k tt nht
phi tin ti to nn - c nhng thnh phn phn mm c lp hay
mun. Nhng giao din phc tp gia cc mun khng - c
khuyn kch nhiu v nhng thut ng nh- ghp ni chm, t kt cao v
QUAN L DU N PHN MM
111
che du thng tin tr thnh nhng khi cng trnh c bn cho vic thit
k mun phn mm - c tt.
Ni mt cch n gin, phn gii thit k tt to nn nhng mun
nh, n gin, c lp. Tt nhin, trong mi h thng, khng c hai
mun thc s c lp, nn t ng c lp y phi hiu theo ngha
cng c lp cng tt trong phm vi cc rng buc ca h thng - c
pht trin.
cp phn gii thp nht, mc c lp ca cc mun - c coi l
mc ghp ni tn ti trong thit k. Nhng bin php ghp ni cc c
im ph thuc ln nhau nh- d liu ni dung kim tra v mun (c
ngha chng cho cc bin mun).
Chc chn mt trong nhng nguyn tc c bn nht trong phn gii
thit k phn mm - c nh tm chung quanh quan nim hp en.
Nguyn tc , cng - c coi l che du thng tin tin ti to ra nhng
mun che du thit k ca chng. Cc hp en ch - c nhn bit u
vo v u ra, ch khng ph- ng php s dng pht sinh u ra t
u vo m c. Vic che du thng tin to ra nhng mun che du lung
lgc ca chng v cc c cu d liu ca chng vi nhau. Mc d t hp
en i tr- c tin trin ca cng ngh phn mm l mt ngnh, quan nin
che du thng tin trong thit k phn mm - c Iarnas - a ra u tin
(1972).
23
6.3.4. - ng li phn gii nhim v cng vic.
Chng ta thy cng vic yu cu hon thnh d n c th - c
phn thnh nhm nhim v n gin hn - c xc nh r biu th bng
c cu phn tch cng vic hay WBS. WBS khng phi l s phn gii
phn mm to ra bi d n. l s phn gii bn thn d n v bao gm
nhng hot ng nh- qun l, cung cp, lp t v tt nhin pht trin
phn mm.
C cu thit k ca h thng to nn nhng nhim v cng vic pht
trin cp thp. Mi mun cp thp - c y thc ba nhim v cng vic
c bn: thit k mun, lp m v th nghim n v. Nhng nhim v
pht trin b sung nh- ly nguyn mu th nghim v hi nhp l t cc
giai on pht trin khc m ra.
Bng 6.2. c mt danh mc tiu biu cc nhim v WBS cp cao - c
- a vo danh mc nhim v WBS chnh thc (coi bng 6.1). y khng
phi l mt danh mc tn dng mi nhim v pht trin d n v khng
phi mi d n s i hi mi nhim v m t. D sao, bng ny s c
ch lm danh mc kim tra h tr xc nh v tr cc nhim v c th b
b qua.
Nhng hot ng khng pht trin nh- nhim v WBS qun l cp cao
l qui chun n mt mc rng v mi khc nhau - c xc nh hoc
23
Tho lun chi tit hn v cc quan nin thit k c th thy trong cc vn bn ca Fairly
(1985) v Fressman (1987).
QUAN L DU N PHN MM
112
do qui m ln lao ca d n hoc - a vo mt m hnh qun l mi. Th
d danh mc cc nhim v qun l quan trng nht i hi cho hu ht
d n pht trin phn mm. Nhng nhim v no khng bt buc cho mi
nhim v - c nh du nh- th trong danh mc.
Nn ghi nhn l phn tch ngn sch v qun l ngn sch khng phi
l nhim v qun l bt buc n gin v khng phi tt c d n qun l
ngn sch ca chnh chng. Mt s t chc c quan chc ti chnh chu
trch nhim qun l cc ngn sch ca d n. Giao din khch hng cng
phi l nhim v qun l bt buc v khng phi mi d n u c khch
hng. Trong tr- ng hp cc d n ni b cng ty, qun l cp cao, cng
vi nhng ng- i s dng h thng - c ch nh, ng vai tr khch
hng. Th- ng l h nu r nhng yu cu d n ban u v ng- i qun l
d n phi n h - c ph chun cui cng v - c nghim thu h
thng ln cui.
6.4. Tmtt.
Nhng d n phn mm phc tp c th phn thnh nhng thnh phn
n gin hn v mc d ton b d n c th kh qun l, mi thnh
phn li s d s l hn. Vic phi gii cc h thng phn mm thnh
nhng thnh phn nh l c ch trong vic gim st nhng hot ng
- c y thc cho cc i pht trin khc nhau. Ph- ng php phn gii c
th khc nhau, ty thuc mc tiu ca ng- i qun l d n.
Vic tinh lc tng b- c l mt ph- ng php lp li phn gii mt d
n thnh nhng thnh phn qun l - c. Vic tinh lc tng b- c l cng
c c ch cho vic xc nh, cc phn gii cng vic, thit k v chc
nng ca mt d n phn mm. Trong phn gii tng b- c mt d n,
mi thanhphfn phn gii thnh nhng thnh phn trc tip d- i n. Mi
b- c phn gii m t ton b h thng nh- ng mc chi tit khc
nhau.
Phn gii chc nng ca mt h thng phn mm l phn chia h thng
thnh nhng thnh phn vn hnh: l nhng c im no m ng- i
dng nhn thy. Phn gii thit k mt h thng phn mm l phn chia
h thng thnh nhng thnh phn cp thp hn trng vi nhng thnh
phn lp trnh ca h thng. C cu phn tch cng vic (WBS) l phn
gii d n phn mm thnh nhng nhim v cng vic cp thp.
R rng, nhng nhim v nh d s l hn nhng nhim v ln, v y
l lp lun bo v vic phn cc d n ln thnh nhng thnh phn nh.
Nhng h thng ln th- ng bao gm nhng thnh phn ch yu bn c
lp, gi l h thng ph, bn thn chng c th coi l h thng, khi cc h
phn mm ln c th chia thnh h ph, nhng h thng loi b mt
s tnh phc tp do rng ln m c. t- ng bo v tip cn cn c
qun l pht trin ca mt h thng ph nh- l mt h thng ring bit
mc c th - c.
QUAN L DU N PHN MM
113
Phn gii chc nng cp cao c bn ca mt h thng phn mm
th- ng cn c nhng t- ng tim nghim din tin trong giai on
quan nin. Nhng t- ng c th nh ra s phn chia c th ca h
thng t din tin ra phn gii chc nng. D sao, phn chia ban u
cu h thng khng phi bao gi cng l lgc nht v thch hp nht
tr- c vin cnh ca ng- i sn xut. V - ng li chung, khng c phn
gii chc nng duy nht no li - c la chn ch v - c quan nin u
tin.
Phn gii chc nng ca mt h thng phn mm c th cn bn khc
vi phn gii thit k ca h thng. D sao phn gii chc nng tt s c
tnh n thit k l giai on pht trin sau v s lun l khi im tt
cho vic phn chia h thng thnh nhng thnh phn thit k cp cao. Sau
, cc thnh phn thit k cp cao li - c phn gii thnh nhng cp
thp hn lin tip cui cng to nn nhng mun lp trnh. Thit k
mun tt to nn nhng mun c lp, n gin, nh.
C cu phn tch cng vic khng phi l phn gii ca phn mm do
d n to nn m l phn gii ca chnh d n v bao gm nhng hot
ng nh- qun l, cung cp, lp t v tt nhin, pht trin phn mm.
Nhiu nhim v pht trin WBS l do ph- ng php pht trin - c s
dng v do thit k v kin trc ca h thng m ra.
Bi tp.
1. Cng ty h thng phn mm (SSI) ang pht trin mt vi tnh c
mc ch c bit da trn b vi x l chung. SSI c b bin dch cho
cho b vi x l hot ng trn khung chnh ca cng ty. Cng ty quyt
nh pht trin mt h thng vn hnh s hu khim tn cho my vi tnh
mi.
Hy xt mt h thng n gin ch mt ng- i dng. Chun b mt
ngh v vic phn gii chc nng ca h thng vn hnh s dng tinh lc
tng b- c. M t biu phn gii chc nng cho ba cp u.
2. Vi h thng vn hnh m t bi tp 1 v cn c phn gii chc
nng, chun b phn gii thit k. M t biu din phn gii thit k cho
ba cp u. Chn thnh phn cp cao duy nht v m t ton b phn gii
ti tn cp mun phn mm.
Gii thch bn tnh n nhng h- ng dn cho cc mun c lp th
no. Gii thch bn thc hin h- ng dn che du thng tin th no.
3. Vi h thng vn hnh bi tp 1, chun b biu c cu phn tch
cng vic cho ba cp u v chun b danh mc nhim v WBS. Gii
thch v sao mt s nhim v bng 6.2 khng p dng - c cho d n
ny.
4. Xt phn mm cho 1 d n v tnh k c vic phng v theo di v
tinh v vn hnh ca v tinh trong v sau khi phng.
Nhn bit nhng h ph chnh trong d n v gii thch li im t
- c khi xc nh chng l h ph.
QUAN L DU N PHN MM
114
By gi xt h thng danh sch nhn vin k c nhn lc bo tr h
s,pht phiu kim tra danh sch v.v... liu c li im g trong vic
xc nh cc h ph cho d n ? Gii thch.
Bng 6.2. Nhim v c cu phn tch cng vic cp cao.
Pht trin phn mm
Phn tch yu cu
Pht trin nguyn mu
c im nguyn mu
Thit k nguyn mu
Thc hin nguyn mu
Th nghim
Th nghim Alpha
Th nghim bta
Nghim thu
Lp t
o to
Cung cp
Thu thp cng c pht
trin
Thu thp cc thnh phn
h thng
Thit k
Thit k cp cao
Thit k chi tit
Bo tr
Hiu chnh sai lm
Tng c- ng phn
mm
Chn la thitbi
Chn la ng- i bn
Th tc t hng
Kim tra kim k
Thc hin
Lp m
Th nghim n v
Qun l
Qui hoch
Tuyn nhn s
Qun tr v dch v
khc
T- liu
Bi vit k thut
Hot ng xut bn d n
T- liu pht trin khng
giao - c.
Hi nhp
Hi nhp phn mm
Hi nhp phn cng/
phn mm.
Qun tr ngn sch
Qun l nhn s
Bo him cht l- ng
Qun l cu hnh
T- liu pht trin giao
- c
T- liu v bo tr
T- liu ng- i dng
Bng 6.3. Nhim v qun l v hnh chnh
y
nhim
Nhim v qun l
v
v
v
v
v
v
v
v
v
v
1. Quy hoch
2. Chun b d n
3. Phn tch ri ro v qun l ri ro
4. Lch trnh
5. Tuyn nhn s
6. Phn tch ngn sch v qun tr ngn sch
7. Qun l nhn s
8. y thc nhim v
9. Giao thm quyn
10. y thc ngun pht trin
11. Gim st bo tr thit b pht trin
12. Gim st v kim tra pht trin
13. T chc duyt xt v cc gii thiu chnh thc
14. Lp tiu chun v ph- ng php
QUAN L DU N PHN MM
115
v
v
v
v
15. Bo him v kim tra cht l- ng
16. Qun l v kim tra cu hnh
17. Gim st cc ch thu ph v ng- i bn
18. Giao din v phi hp qun l cao
19. Giao din v phi hp khch hng
20. Bo co
21. Qun tr v dch v
QUAN L DU N PHN MM
116
Ch- ng by
Cc chc nng h tr d n
H tr qun l d n.
Qun l d n phn mm l qu trnh qui hoch, t chc, tuyn nhn
s, gim st, kim tra v lnh o d n phn mm
24
. Him khi mi nhim
v li c th do qun l d n thc hin m trn thc t v mt l
t- ng h khng th. Nhiu hot ng kim tra v gim st c th - c y
thc cho cc nhm h tr d n. Nhng nhm h tr ny khng ch gim
nh nng cho qun l d n v k s- pht trin bng nhng nhim v h
tr: h cng thc hin nhng nhim v tt hn bng cch tp trung
mi c gng ca h vo nhng chc nng h tr c tr- ng.
C nhiu loi chc nng h tr d n. Dch v th- k, h tr hnh
chnh xut bn v cung cp ti liu l nhng th d v chc nng h tr
khng k thut; th nghim, kim tra cu hnh, cng ngh h thng qun
l hi nhp v bo him cht l- ng l nhng th d v chc nng h tr
k thut.
D n cng ln v cng phc tp li s i hi chc nng h tr nhiu
hn. Chng hn, mt d n ln th- ng c t chc kim tra cht l- ng ca
n trong khi mt d n nh c th chia x chc nng vi cc d n
khc. T- ng t, nhiu t chc duy tr nhm th nghim c lp m vai
tr l th nghim mt sn phm phn mm tr- c khi - a ra. nhng d
n ln, nhm th nghim c lp l mt b phn ca i d n v tham
gia trong th nghim v qui hoch th nghim xuyn sut chu trnh pht
trin.
Ch- ng ny m t ba chc nng h tr d n k thut ch yu:
- Kim tra cu hnh
- Bo him cht l- ng phn mm
- Th nghim.
Ba chc nng c bn - c yu cu trong mi d n pht trin phn
mm. Kim tra cu hnh qun l thay i cho sn phm phn mm - c
pht trin, bo him cht l- ng gim st v kim tra cht l- ng sn phm
cn th nghim th li p ng vi cc yu cu ca sn phm. Trch
nhim ca ng- i qun l d n l t chc cc nhm h tr d n v cung
cp t- liu cc hot ng quy hoch ca cc nhm cho k hoch pht
trin d n (coi ch- ng 9). Nu nhng nhm c trong t chc, th
h tr ca cc nhm cn - c phi hp v ln lch cho d n. Nu cc
24
nh- xc nh trong IEEE (1987a)
QUAN L DU N PHN MM
117
nhm khng tn ti th chng phi - c thit lp trong i pht trin d
n.
Qui m ca mt nhm h tr r rng ty thuc qui m d n, chng
hn, mt d n ln c th yu cu mt nhm hai hay ba k s kim tra
cu hnh v mt d n nh c th giao nhim v mt phn thi gian cho
k s- pht trin.
Cc quyt nh ny phi do qun l d n - a ra trong nhng giai on
ban u ca d n. Cc chc nng h tr d n - c qui hoch tt lc
khi u d n s ng gp vo vic qun l hiu qu d n sut d n.
7.1. Kimtra cu hnh phn mm(SCC).
Kim tra cu hnh l chc nng h tr qun l h tr nhiu hot ng
khc nhau lin quan n thay i cho sn phm phn mm. iu ny bao
gm nhng thay i m ca ch- ng trnh, yu cu v thay i thit k
cng thay i trong vic - a ra phin bn. Kim tra cu hnh th- ng - c
nhng nh sn xut coi l tr ngi hn l li ch v n hn ch s t do
ca i pht trin v t ra nhng gii hn v ci g c th v khng th
lm. Kim tra cu hnh d sao cng to ra mi tr- ng trong phn mm
c th - c pht trin mt cch trt t.
T cu hnh - c s dng y m t s phi hp cc thnh phn
phn mm to thnh h hp cht. Khi kt hp vi t kim tra, t - c s
dng m t mt ph- ng php trt t v hiu qu theo s phi hp
ca cc thnh phn c th - c thc hin, chng hn, xy dng cc h
phn mm t nhng thnh phn cp thp khng phi l nhim v n
gin. iu ny - c minh ha tt nht giai on sau.
Mt cng ty ngn hng ln kt hp lm dch v thng tin ti chnh
quc t. Dch v ny c th cung cp cho ngn hng truy nhp trc tuyn
vi c s d liu trung tm cha thng tin th- ng xuyn cp nht v
nhng th tr- ng ti chnh th gii. Trong th gii hin i thng tin in
t nhanh, y l mt dch v ch yu cho mi t chc ngn hng hin
i. D sao, my tnh ca ngn hng khng c - c kh nng giao
din vi dch v ti chnh.
Ngn hng y thc ng- i qun l d n pht trin phn mm cn thit
c nhu cu cho giao din. Sau khi giai on hi nhp bt u, mt trong
nhng nh sn xut bo co l ct mc ch yu hon thnh lin tc vi
dch v thng tin - c thit lp thnh cng. Ng- i qun l d n bo
co iu ny cho cc cp trn ca anh ta v thng bo vi h l pht trin
- c tin hnh theo lch.
Mt tun sau, cc thnh vin qun l hng u n thm d n v yu
cu chng minh lin lc vi dch v thng tin. Tuy nhin, ng- i qun l
d n khng th cung cp chng minh. Nh sn xut bo co ct
mc khng th lp li thnh cng tr- c y. y l do nhng c im
ph - c b sung cho phn mm lin lc v nhng lin lc ny li
QUAN L DU N PHN MM
118
ch- a hot ng. Ngay c nhng c im tr- c y cng khng cn hot
ng na.
Bt u
C sa
khng?
Mun yu cu kim
tra cu hnh
Sa mun
Tin trnh kim tra
cu hnh
Th nghim
n v
Mun pht trin
Kt thc
Mun do kim tra
cu hnh pht ra
C nghim
thu khng?
QUAN L DU N PHN MM
119
Hnh 7.1
Dng iu khin cu hnh mun phn mm
R rng, hn nh sn xut gi mt bn sao ca phn mm lin lc
khng c nhng c im mi. Trong mt d n t chc tt, nhim v
phng v, mt phin bn phn mm tr- c y hn - c ng- i qun l
cu hnh thc hin.
Mt cch l th xem n kim tra cu hnh l coi nh- mt ph- ng
php m bo l d n tin trin (hay t ra khng tht li). Trong tr- ng
hp xt trn hin nay d n tr- t li.
Kim tra cu hnh l ch yu cho mi hng mc pht trin k c m, t-
liu v hp nht thnh phn hnh 7, m t dng kim tra cu hnh pht
trin mun phn mm. Mt dng t- ng t hn vn dng - c cho t-
liu m d n pht sinh kim tra cu hnh cng quan trng trong giai
on bo tr m bo khi vic pht hnh mi mt h thng - c gi
li do nhng khuyt tt nghim trng c th b thay th trong t pht
hnh tr- c.
7.1.1. Thut ng kim tra cu hnh.
C nhiu t - c dng lin quan n kim tra cu hnh. Chng may,
khng c mt s dng no hay mt ngha no ca chng - c chun
ho: rt nhiu t khc nhau - c s dng m t cng chc nng;
cng t- ng. Mt trong nhng c gng tt nht nhm tiu chun ha c
trong t vng IEEE thut ng v cng ngh phn mm (IEEE 1987b). D
sao, ngay t vng cng phn nh s thiu st trong l gii v s dng
thng nht cc t khi m nhiu t kim tra cu hnh xut hin vi v
vn nh ngha. Sau y l gii thch hn l nh ngha chnh xc ca mt
s nhng thut ng c bn.
Hng mc kim tra cu hnh phn mm (SCCI) l hng mc d n
phn mm - c coi l mt n v cho nhng mc ch ca kim tra cu
hnh. iu ny c th bao gm nhng iu nh- m un phn mm (1)
phin bn h thng phn mm hay t- nliu.
Kim tra thay i l qu trnh kim tra cc thay i. iu ny bao gm
ngh thay i, nh gi thay i, chp nhn hay bc b, ln lch v
theo di thay i. Kim tra phin bn nh- vn dng cho pht trin phn
mm, l qu trnh kim tra pht hnh cc phin bn phn mm.
25
iu ny
bao gm l- u gi v phng nga mi t pht hnh v chng minh bng
t- liu cc khc bit gia cc t pht hnh.
Kim tra cu hnh l qu trnh nh gi, chp nhn hay bc b, v
qun l thay i cho nhng hng mc cu hnh. Kim tra cu hnh cng
bao gm cc chc nng kim tra phin bn.
25
Nhiu gi thch t hng mc cu hnh khng bao gi hng mc cp thp nh- m un phn mm.
Tiu chun (DOD 1988a) pht trin phn mm US- DOD std.2167 vn dng thut ng cho nhng thnh
phn cp cao nh- xc nh trong tiu chun Std.480a kim tra cu hnh DOD. Gii thch ny lin quan
n nhng hng mc c th - c pht trin c lp hay sa cha v bo tr c lp.
QUAN L DU N PHN MM
120
Qun l cu hnh l ng dng k thut v hnh chnh kim tra cu
hnh. iu ny cng bao gm vic duy tr t chc kim tra cu hnh, thay
i v cc tiu chun kim tra phin bn v cc ph- ng tin kim tra cu
hnh.
Cc t khc l ch nh nhn bit cu hnh - c s dng nhn bit
cc hng mc cu hnh, ban kim tra cu hnh chp nhn hay bc b
nhng thay i cng ngh v kim ton cu hnh kim tra thch ng vi
cc tiu chun kim tra cu hnh.
Mc tiu ca qun l cu hnh - c xc nh tt nht khng phi nh-
nhng nh ngha chnh thc t vng IEEE nhng thc ra nh gii thch
c tnh cht m t hn nh- nu trong li ni n tiu chun IEEE 828
(1987b) cho nhng k hoch qun l cu hnh phn mm, khng nh:
Cung cp cu hnh phn mm (SCM) l ngnh cng ngh chnh thc
cung cp ph- ng php v cng c cho cc nh sn xut v s dng phn
mm bit phn mm - c pht trin, lp ra nhng - ng mc, thay i
kim tra cho nhng - ng gc , l- u gi v theo di tnh hnh v kim
ton sn phm.
SCM l ph- ng tin thng qua s hp nht v lin tc ca sn
phm phn mm - c l- u gi, thng tin v kim tra.
Mt s nhng nhim v qun l cu hnh chng cho vi nhim v ca
hot ng h tr khc, kim tra cht l- ng phn mm (bn n sau).
Trong cc d n phn mm m kim tra cht l- ng v kim tra cu hnh,
do cc nhm ring r thc hin, vic nh ngha r trong phn chia trch
nhim l cn thit.
7.1.2. Ngun lc kim tra cu hnh.
Hnh 7.2
iu khin cu hnh mng
Kim tra cu hnh l mt trong nhng lnh vc u tin ca cng ngh
phn mm - c cng nhn l d tuyn t ng ha, nhiu hot ng
kim tra cu hnh nh- kim tra phin bn v kim tra thay i - c t
C c d liu
kim tra cu
hnh
Cc my tnh u cui pht trin d n
My tnh kim tra cu hnh
QUAN L DU N PHN MM
121
ng ha vo u nhng nm 1970 vi nhng cng c nh- ng mch v
SCCS (coi Rochkind 1975)
26
. Mt s nhng cng c dch chuyn t
cc h vn hnh UNIX ni chung - c s dng u tin n cc mi
tr- ng khc nh- MS-DOS v VMS ca Digital.
Nhiu hot ng kim tra cu hnh chnh l nhng d tuyn t nhin
cho cc cng c t ng ha CASE (Cng ngh phn mm c h tr bng
my tnh) nh- - c xc nh r, c i cht lp li v sn sng hi nhp
vo qu trnh pht trin. Nhng cng c c th - c d dng giao din
vi cc cng c m phn mm (th d cc bin tp vin v s- u tp) v cc
b x l t sn sinh t- liu. Kim tra cu hnh t ng l tt nht khi
- c s dng trong mi tr- ng pht trin nhiu ng- i s dng nh- LAN
vi cch mi nhn t - c kim tra l- u gi trong c s d liu trung
tm v vic truy nhp ca mi nh sn xut - c qun l t h thng
kim tra cu hnh trung tm (coi Hnh.7.2).
Vic kim tra cu hnh c hiu qu i hi s t chc hiu qu v
- c xc nh r. Mi ph- ng php kim tra cu hnh phi cn c bn
quan nim sau:
1. Phi thnh lp cp thm quyn qun l cu hnh - c xc nh r.
2. Phi sn xut v phn phi cc tiu chun kim tra cu hnh, cc th
tc v h- ng dn kim tra cu hnh cc nh sn xut.
3. Kim tra cu hnh khng th c hiu qu nu khng c cng c v
ph- ng tin cn thit.
4. Phi pht trin k hoch qun l cu hnh ngay khi u d n.
y l trch nhim ca ng- i qun l d n trong vic giao quyn
qun l cu hnh. iu ny c th tin hnh t i kim tra cu hnh
trong cc d n ln n k s- kim tra cu hnh mt phn thi gian trong
cc d n nh. Trong c hai tr- ng hp, c quyn hn v trch nhim
phi - c xc nh r. K s- kim tra cu hnh phi - c tham gia trong
mi hot ng pht trin v phi c quyn hn c th trong vic chp
nhn hay bc b hng mc cu hnh.
Cc thnh vin i pht trin phi lm quen vi cc tiu chun kim tra
cu hnh, cc th tc v h- ng dn phi d hiu v r rng. iu ny c
th gim bt s phn bc trong kim tra cu hnh do khng thch ng vi
cc tiu chun khng quen thuc.
Mi tr- ng qun l cu hnh bao gm nhng cn thit cho vic thc
hin k hoch kim tra cu hnh. iu ny bao gm:
- Cng c kim tra cu hnh, k c:
+ Kim tra phin bn t ng v cng c kim tra thay i.
+ Gim st, kim ton v ng k cc tin ch h tr.
26
S tin trin ca qun l cu hnh - c m t trong tho lun ca Ambriola v cng s (1990)
cng bn n t ng ha cc hot ng kim tra cu hnh khc nhau.
QUAN L DU N PHN MM
122
- Ph- ng tin l- u gi: kho cha an ton cho mi hng mc cu hnh
- c ph chun, k c:
- L- u gi ti ch cho qu trnh pht trin hng ngy.
- L- u gi ngoi phm vi phc hi tai ha.
7.1.3. K hoch qun l cu hnh phn mm.
K hoch qun l cu hnh phn mm (SCMP) l mt b phn ca k
hoch pht trin phn mm c d n. SCMP c th xut hin nh- mt t-
liu ring bit hay mt phn trong phm vi k hoch pht trin d n.
SCMP cung cp d liu cho cc ngun c nh- cu, chng - c s dng
nh- th no, v c nhng tiu chun th tc g s - c vn dng trong d
n. Theo th SCMP tr thnh - c y nhim cho nhm kim tra cu hnh
trong pht trin d n. Vic - a ra k hoch l trch nhim ca ng- i
qun l d n, mc d trong nhng d n ln n c th - c ph thc cho
ng- i qun l kim tra cu hnh.
Bng 7.1. C mt danh mc cc ti chnh nm trong SCMP. Nu c
mt ti no trong nhng ti nm ch khc (th d trong k
hoch bo him cht l- ng phn mm) th ti c th - c b qua
SCMP v - c thay bng con tr. Trong t- liu m n ng phi .
Mc d phn ln cc ti trong bng 7.1. l t m t, sau y l mt s
h- ng dn:
- Bo co tnh hnh cu hnh m t cch thng tin tnh hnh din trin:
- T ng- i sn xut n t chc qun l cu hnh (kim ton v
duyt).
- T t chc qun l cu hnh ti qun l d n (th tc bo co
tnh hnh).
- Nhn bit cu hnh m t ph- ng php ch nh cc hng mc pht
trin nh- SCCI. y l mt b phn ca phn gii h thng cp cao
thnh nhng thnh phn pht trin ch yu (coi ch- ng 6). Phn v
cc ph- ng php nhn bit m t cch m mi thnh phn pht sinh,
t d n - c nh du nhn bit duy nht.
- An ton, truy nhp hn ch v phn loi nhm pht trin an ton cc
sn phm nhy cm (nh- t- liu phn mm, bng pht minh, thng tin
qun s xp hng v.v...). Th- ng thun li l y thc nhiu nhng
nhim v cho kim tra cu hnh do nh- cu phi tham gia vo vic
duyt v phn loi t- liu v cc hot ng khc lin quan gn vi an
ton.
- Cc ch thu ph, ng- i bn hng v ng- i cung cp c th hay
khng th thc hin k hoch qun l cu hnh ca chnh mnh. y l
trch nhim ca ng- i qun l d n phi m bo l cc ch thu ph
hay cc nh sn xut bn ngoi phi trnh duyt CMP hay ng- i qun
l cu hnh ca d n chu trch nhim v cng vic ca h.
QUAN L DU N PHN MM
123
SCMP cng c th bao gm nhng s v biu dng chy m
t th tc trnh cc yu cu thay i hay bo co vn
27
.
Hnh 7.3. gii thiu mt th d v biu dng chy tng qut kim
tra cu hnh m tiu chun US DOD 2167A (DOD) 1988a gi (so snh
vi dng kim tra cu hnh m un phn mm trong h.7.1).
Bng 7.1. Th d v ni dung k hoch qun l cu hnh phn
mm.
1. T chc v ngun qun l cu hnh phn mm
- C cu t chc
- Kh nng v trnh k nng nhn s
- Ngun
2. Tiu chun, th tc, chnh sch v h- ng dn.
3. Nhn bit cu hnh
- Ph- ng php xc nh SCCI
- M t SCCI cho d n ny
4. Ph- ng php nhn bit (nh danh v nh du t- liu, cu kin
phn mm, duyt pht hnh v.v...).
5. trnh hng mc cu hnh th tc chp nhn / bc b.
6. Kim tra thay i.
- Th tc kim tra thay i (ph- ng php trnh duyt, chp
nhn v bc b).
- Bo co t- liu (yu cu thay i, bo co vn )
- Th tc duyt thay i v ban xt duyt
7. Kim tra phin bn.
- Chun b phin bn phn mm v t- liu
- Th tc chp nhn pht hnh.
8. L- u tr, x l v cung cp cc knh thng tin d n
- Yu cu l- u tr.
- Sao chp
9. Kim tra cu hnh ca cc ch thu ph, ng- i bn v ng- i cung
cp
10. Kim tra b sung
- Th tc kim tra linh tinh
- Kim tra c bit d n (an ton v.v...)
11. Bo co tnh hnh cu hnh
- Kim ton v duyt cu hnh
- Th tc bo co tnh hnh cu hnh
12. Ct mc ch yu qun l cu hnh
13. Cng c, k thut v ph- ng php lun
27
v m t SCMP chi tit hn, coi IEEE (1987)
QUAN L DU N PHN MM
124
7.1.4. Mt s - ng li chung
Hnh 7.3
Th d v l- c mt dng iu khin cu hnh.
Danh mc hot ng m kim tra cu hnh tin hnh cn ch- a t tiu
chun l chc nng h tr qun l d n, phm vi kim tra cu hnh bao
gm nhiu hot ng ty chn hay tuyn chn. Do , vi t- cch mt
h- ng dn c bn, n hon ton c th chp nhn - c gn cho qun
l cu hnh trong phm vi d n chuyn , bt k hot ng no lin
quan nh- phn b ngun pht trin hay b tr v t chc cc trnh din
Khng
Nng cp
phn mm
Thay i phn
mm
Cc vn
Phn tch v
- c nh
nh h- ng
Chun b xut thay i k
thut
Sp nhp s
thay i
Ban r sot thit k
phn mm
Ban iu khin cu hnh
phn mm
C
nh gi
xut thay i
k thut
Cung cp lin h
ng- c cho ng- i
khi u
Kim th thay
i
Kt thc
Chp thun
hay khng?
L- u tr thay
i
Phn tch v truy
cp nh h- ng
QUAN L DU N PHN MM
125
cho khch hng. C hai th d s dng thng tin c - c cho ng- i
qun l cu hnh: cu kin sn phm no - c sn xut, khi no v do ai.
i hi
thay i phn mm
S : Trang:
Tn ng- i khi u: in thoi: Ngy:
D n: H thng/Sn phm: Version:
L do thay i:
M t thay i:
Ng- i r sot: Ch k: Ngy:
c l- ng s cng: Lch thi gian: Lp lch nh h- ng:
Ng- i chp thun d
ton:
Ch k: Ngy:
C chp thun khng
(C/KHNG):
Tn ng- i quyt nh: Ch k: Ngy:
Hnh 7.4
Th d biu yu cu thay i phn mm
Vic l- u gi h s l quan trng trong mi hot ng hnh chnh ca
d n nh- ng c bit quan trng trong kim tra cu hnh. Tai ha ca
QUAN L DU N PHN MM
126
qun l d n vn l tranh chp vnh cu v cc tha thun ming l gii
sai hay hiu sai...
Kim tra thay i yu l ph bin trong vic gy nn tranh chp gia
khch hng v ng- i sn xut. T- ng t nh- vy vic kim tra phin bn
yu c th l tai ha, c bit khi khng c h s v d bit gia cc
phin bn.
Hnh 7.4. cho 1 th d v mu yu cu thay i. Mu ny l- u gi thay
i phn mm t ln trnh ban u, qua chp nhn hay bc b v cui
cng n thc hin v kim nghim (khi - c chp nhn). Cn ghi nh
nhu cu ch k; nhng mu ny khng th l- u gi bng in t - bn gc
c ch k phi - c l- u li.
Sau y l mt s h- ng dn b sung qun l cu hnh hiu qu.
Mt s nhng h- ng dn ny ng thi p dng - c cho cc chc nng
h tr qun l khc.
- Qun l cu hnh i hi quyn hn c hiu lc. Quyn hn ny
phi - c qun l d n giao c th cho cc k s- c trch nhim. Bt c
k hoch qun l cu hnh no s tr nn v ngha nu k hoch khng
th - c tn trng.
Tt nht nn trnh, bt c khi no c th, vic c- ng bc tn trng bt
c k hoch, chnh sch hay tiu chun no. Mt trong nhng c tnh
ca ng- i qun l tt l kh nng vn dng chnh sch vi c- ng bc ti
thiu. Bt c khi no cc chnh sch v tiu chun sn sng - c ng- i
sn xut chp nhn th chng cng - c t nguyn tun th v rt t c
tr- ng hp bc b vt liu trnh. iu ny dn n qu trnh pht trin
hiu qu hn.
- Qun l cu hnh phi - c thc hin t lc khi u d n phn
mm. Nhiu t- liu chnh thc xut trong giai on quan nim ban u
l trng yu cho cc giai on yu cu v thit k v phi - c t trong
s kim tra cu hnh.
Vic vn dng sm qun l cu hnh l c bit quan trng trong cc
m en xoy c, ly nguyn mu nhanh hay cc ph- ng php lun pht
trin lp li. Nhng tip cn pht trin ny khi u to ra v vn phin
bn ca mi sn phm. Nhiu phin bn khc nhau c th tr thnh c
mng cng ngh nu khng c kim tra cu hnh c trt t.
- i khi mt s hot ng kim tra cu hnh phn mm c th chng
cho vi cc hot ng bo him cht l- ng phn mm. Trong nhng d
n nh, c hai chc nng c th giao cho mt k s- h tr duy ht.
Ngay trong nhng d n ln, i khi chc nng c th do mt nhm
h tr duy nht thc hin.
V h- ng dn chung cui cng, phi ghi nhn l qun l cu hnh c
th - c cao thi qu. Cc hot ng kim tra cu hnh bn thn
chng khng phi l i t- ng. Chng l ph- ng tin. Mt th d in
hnh v vic vn dng sai qun l cu hnh (v kim tra cht l- ng lch
h- ng) l yu cu sa li phn mm - c dng li p ng cc tiu
chun v th tc thng dng. Phn mm ti s dng l phn mm pht
QUAN L DU N PHN MM
127
trin tr- c y trong d n khc v - c thy thch hp ha nhp vo
d n thng th- ng. Trong nhng tr- ng hp , him khi li c ngha
trong vic sa i mt sn phm hon chnh v hot ng nhm cho n
p ng vi cc tiu chun hnh chnh d kin lm n tr thnh mt
sn phm hon chnh v hot ng - c.
7.2. Bo mcht l- ng phn mm(SQA).
Cht l- ng kh xc nh - c c bit khi vn dng cho mt hp ng
pht trin sn phm. Mc d khng phi mi phn mm - c pht trin
theo hp ng, cht l- ng vn l mi quan tm u tin ca khch hng
(v mi d n cui cng u c khch hng). C quan tiu chun Anh
(1986) khng nh "cht l- ng nm trong con mt ca ng- i xem, mt
ti phn x ca khch hng".
Nu c kh khn trong vic tm kim - c nhng nh ngha chp
nhn rng ri v thut ng kim tra cu hnh th v thut ng bo m
cht l- ng phn mm li kh gp i t vng tiu chun IEEE v thut
ng cng ngh phn mm (IEEE 1987b) c khng t hn bn nh ngha
ring bit v cht l- ng phn mm
28
.
1. Ton b kha cnh v c im ca mt sn phm phn mm c kh
nng p ng nhu cu ra: chng hn thch hp vi cc c im k
thut.
2. Mc m phn mm c - c s phi hp mong mun ca cc
thuc tnh.
3. Mc m khch hng hay ng- i dng nhn thc l phn mm p
ng - c cc k vng hp th ca h.
4. Cc c im hp th ca phn mm xc nh mc m phn mm
s dng s p ng - c cc k vng ca khch hng. US DOD (1988b)
xc nh cht l- ng phn mm li n gin l:
5. Kh nng ca sn phm phn mm p ng nhng yu cu c
dng ca n.
Cht l- ng phi - c o theo ngha k vng ca khch hng. D sao,
vin cnh ca khch hng l ch quan. Phn ny xt n kim tra cht
l- ng theo vin cnh ca ng- i qun l d n phi coi vic thc hin
kim tra cht l- ng l mt qu trnh khch quan, mang tnh h thng.
7.2.1. To ra phn mm cht l- ng.
Nh- chng ta thy, mt trong nhng vn chnh trong vic to ra
phn mm cht l- ng l s kh khn trong vic xc nh mc cht
l- ng trong phn mm. V khng c nh ngha duy nht no chp nhn
- c rng ri v cht l- ng v v nhng ng- i khc nhau nhn thc cht
28
tiu chun pht trin phn mm, k c kim tra cht l- ng phn mm - c bn n chi tit
ch- ng 8
QUAN L DU N PHN MM
128
l- ng theo nhng cch khc nhau nn c ng- i sn xut ln khch hng
phi t - c tha thun c o m v cht l- ng (iu ny - c bn n
chi tit hn sau ny). Ph- ng php o l- ng cht l- ng c th khc nhau
i vi nhng d n khc nhau.Vn ny - c bn n trong tham lun
ca Wesselius v Ververs (1990) trong cc ng kt lun khng th t
- c tnh khch quan trn vn trong vic gim nh cht l- ng. H nhn
bit - c ba thnh phn khc bit ca cht l- ng:
- Thnh phn gim nh - c 1 cch khch quan
- Thnh phn gim nh - c ch quan
- Thnh phn khng gim nh - c.
Cht l- ng mt sn phm nh gi - c khch quan khi cc c im
ca sn phm, nh- nu trong cc chi tit yu cu, c th nhn bit - c.
Cht l- ng mt sn phm nh gi - c ch quan khi cc c im
ca sn phm p ng - c cc k vng ca khch hng.
Cht l- ng mt sn phm khng nh gi - c khi n tc ng theo
cc k vng ca chng ta trong nhng tnh hung khng d kin.
Wesselius v Ververs gi l, cht l- ng ca mt sn phm phn
mm nh gi - c, cng nhiu c im cng tt - c chuyn t cc
thnh phn ch quan v khng nh gi - c thnh phn nh gi - c.
C bn, iu c ngha l chi tit yu cu phi m t cng nhiu c
im o m - c ca sn phm cng tt.
Th nghim h tr kt lun ca Wesselius v Ververs. Nhng yu cu
xc nh ti bao gi cng l ngun tranh chp gia nh sn xut v khch
hng. Nhng yu cu xc nh r, chi tit v o - c gim thiu cc
tranh chp v bt ha khi pht trin ca sn phm l trn vn.
D sao, nhiu ph- ng php pht trin c khong cch ko di gia chi
tit yu cu v giao sn phm (coi ch- ng 4 bn v chu k pht trin ca
phn mm). Vic xc nh cht l- ng phi khng - c tr hon n khi
pht trin trn vn. Vic kim tra cht l- ng phn mm c hiu qu i
hi c nhng nh gi th- ng xuyn sut chu k pht trin. Theo th,
kim tra cht l- ng c hiu qu i i vi chi tit yu cu tt r rng s
tng cht l- ng ca sn phm cui cng.
Vic thit lp kim tra cht l- ng c hiu qu th- ng vp phi nhiu
quan nim sai lch v huyn thuyt, ph bin nht l ci g lin quan n
tnh hiu qu vn u t- trong kim tra cht l- ng. Cobb v Mills (1990)
lit k nhiu nhng huyn thuyt v gi nhng ph- ng php ph
chng. Hai trong s nhiu huyn thuyt ni tri m Cobb v Mills xc
nh - c m t nh- sau.
Huyn thuyt: cht l- ng xng vi ng tin. y l mt trong
nhng huyn thuyt ph bin nht (khng ch trong pht trin phn
mm). Trn thc t, cht l- ng trong phn mm th- ng tit kim tin.
QUAN L DU N PHN MM
129
Cht l- ng ti gieo rc tht bi. C mt t- ng quan tch cc gia tht bi
v chi ph ch li tn km hn trong vic loi tr tht bi thi cng trong
phn mm hn l thit k phn mm loi tr tht bi thi cng.
Huyn thuyt: tht bi phn mm l khng trnh khi. y l mt
trong nhng huyn thuyt ti t nht v iu khng nh c phn no thc
v do th- ng - c s dng bin h cho phn mm cht l- ng ti. Yu
sch bao gi cng c con rp khc khng bao gi - c l thng s
trong thit k hay thc hin phn mm c.
Khi nhng huyn thuyt mt ch ng trong nhng tip cn hin
i vi pht trin phn mm th yu cu v cc tiu chun v th tc thch
hp v kim tra cht l- ng li gia tng. IEEE - a ra tiu chun u
tin ca h cho nhng k hoch bo him cht l- ng phn mm nm
1984 (IEEE 1984) tip theo l h- ng dn chi tit h tr tiu chun, cng
b nm 1986. B Quc phng Hoa K - a ra tiu chun ring bit
2168 cho cc ch- ng tnh cht l- ng phn mm ca cc h thng Quc
phng (DOD 1988b) to thnh ng hnh cho tiu chun ni ting US
DOD 2167A (DOD 1988a) cho vic pht trin phn mm ca cc h
thng quc phng. Tiu chun Chu u IS0 9000-3 ca nm 1990 (IS0
1990) - a ra mt ngha rng hn cho t bo him cht l- ng v bao
gm c kim tra cu hnh na.
7.2.2. Ngun lc kim tra cht l- ng.
Khi y thc SQA bao gm cc hot ng kim tra cu hnh, cc ngun
yu cu cng bao gm c cc ngun yu cu cho kim tra cu hnh. Vic
ha ng SQA v kim tra cu hnh khng phi khng ph bin v c th
loi tr mt s y nhim v hot ng phi nhn ln. C hai biu t
chc xen k - c m t hnh 7.5. Nn nh l vi nhng d n nh, vic
ha ng hai nhm n gin c th c ngha l ph thc c hai nhim v
cho cng mt ng- i.
Mc d nhiu cng c l ph bin c cho kim tra cht l- ng v kim
tra cu hnh, ch c t cng c - c c bit ch nh cho kim tra cht
l- ng. Sau y l mt s cng c h tr chung c th c ch trong vic h
tr cc hot ng ca SQA:
- Tin ch t- liu
- Cng c thit k phn mm
- H tr tm sai (Ch- ng trnh g ri)
- Cc b x l tr- c - c cu thnh
- B so snh h s
- Nhng b phn tch c cu
- Nhng b kim ton tiu chun
- Nhng b m phng
- Nhng b phn tch thi cng
QUAN L DU N PHN MM
130
- Nhng b gim st thc hin
- Nhng cng c hp cht CASE
- Nhng b kch thch th nghim
- Nhng b pht tr- ng hp th nghim.
Nhng cng c h tr kim tra cht l- ng trong mi giai on pht
trin phn mm. Cc b h tr t- liu c th cung cp phn no cc b
vit t- liu t ng, cc b kim tra chnh t v cc s- u tp chuyn ngnh
(Thesaurus) v.v... cc b x l tr- c - c cu thnh (nh- tin ch UNIX
lint) c ch c trong vic tiu chun ha lit k m ln cung cp cnh bo
b sung thi gian bin dch m cc b bin dch th- ng b qua. Nhng
cnh bo sm lin quan n cc vn thi gian thc hin c th - c
cc m phng, cc b phn tch thi gian thc hin v cc b gim st
thc hin cung cp. Th nghim thu o h thng phn mm th- ng c
th - c t ng thc hin nh cc b pht h th nghim v cc b thc
hin th nghim t ng.
Hnh 7.5
Kim tra cu hnh v kim tra cht l- ng phn mm
Mi cng c SQA s dng trong pht trin phn mm phi - c nhn
bit ngun bn m cht l- ng yu cu v chi tit l cc ngun - c
vn dng nh- th no. Theo th, vo lc khi u d n, cc ngun SQA
(a)
Gim c d n
Khng ch cht l- ng phn mm
Gim c v cc k s- qun l
cht l- ng
Khng ch cu hnh
Gim c cu hnh
Cc k s- cu hnh
(b)
Gim c d n
Bo m cht l- ng phn mm.
Gim c
Khng ch cu hnh
Cc k s- khng ch cu
hnh
Khng ch cht l- ng phn
mm
Cc k s- khng ch cu hnh
QUAN L DU N PHN MM
131
c th - c ti tr v cung cp nh- l mt phn ca ngun pht trin d
n yu cu (coi ch- ng 9).
7.2.3. K hoch bo m cht l- ng phn mm.
K hoch bo him cht l- ng phn mm (SQAP), cng nh- k hoch
cu hnh phn mm, cng l mt b phn ca k hoch pht trin tng
th d n phn mm. SQAP cung cp t- liu m cc ngun cn, chng
phi - c s dng th no v c nhng tiu chun v th tc no s - c
vn dng trong d n. Sau SQAP tr thnh y thc cho nhm bo m
cht l- ng trong pht trin d n. Vic xut k hoch ny l trch
nhim ca ng- i qun l d n mc d trong nhng d n ln. Vic ny
th- ng - c giao cho ng- i qun l bo m cht l- ng. SQAP c th
xut hin nh- l t- liu ring bit hay b phn trong k hoch pht trin
d n v c th bao gm k hoch qun l cu hnh) nu vic ny
khng - c cung cp t- liu ring bit.
Bng 7.2. nu danh mc cc ch chnh c trong SQAP
29
. Khi mt
trong nhng ch - c t u khc nh- trong k hoch qun l cu
hnh phn mm (SCMP) th n c th b qua SQAP v - c thay bng
con tr v t- liu. Ni n - c t. D sao, SCMP v SQAP - c quan
tm nhiu kha cnh khc nhau ca hng mc - c kim tra. SCMP lc
u - c quan tm kch th- c ca hng mc - c kim tra trong khi
SQAP li tham gia nhiu hn vo ni dung ca cc hng mc - c kim
tra.
SQAP phi bao gm cc ch thu ph, ng- i bn hng v ng- i cung
cp, bt k cc thc th bn ngoi c hay khng c t chc bo m
cht l- ng ca chnh chng. Trong bt c d n no, cht l- ng ca cc
thnh phn bn ngoi rt cc l quan tm ca ng- i qun l d n v t
chc SQA ca anh ta. Khi mt h thng tht bi thng th- ng t c khc
bit l tht bi do thnh phn pht trin bn ngoi hay thnh phn pht
trin trong ni b. Cc k hoch gim nh cc nhm bn ngoi phi
thch nghi vi loi thnh phn bn ngoi - c cung cp (ngoi bng hay
pht trin mi) v loi t chc (liu chng c - c t chc bo him cht
l- ng ca chnh chng).
SQAP, coi nh- b phn ca k hoch pht trin d n, phi - c duyt
li v cp nht nh k v bt c khi no c nhng yu cu th tc pht
trin d n, ph- ng php lun hay cc hot ng khc lin quan no - c
thay i. H- ng dn SQAP ca IEEE khuyn nn nh gi nh k hai
kha cnh ca k hoch (1) ni dung ca k hoch v (2) thc hin k
hoch.
Ni dung ca k hoch phi - c nh gi c cn c tiu chun c
tr- ng SQAP s dng m bo k hoch thch ng tip tc vi tiu chun
ngay d nhng c im ca d n phn mm c thay i.
29
bng 7.2 l h- ng dn ch khngphi nh ngha tiu chun. Cc tiu chun pht trin phn mm
- c bn ch- ng 8
QUAN L DU N PHN MM
132
Bng 7.2 Th d v ni dung ca mt k hoch bo m
cht l- ng phn mm (BCLPM).
1. T chc v cc ngun lc BCLPM.
C cu t chc
Hc vn v mc tho ngh ca nhn s
Cc ngun lc
2. Cc chun, th tc, chnh sch, - ng li BCLPM.
3. Cc yu cu t- liu BCLPM.
Danh sch cc ch t- liu khng ch cht l- ng.
M t ph- ng php nh gi v chp thun.
4. Cc yu cu phn mm BCLPM.
nh gi v chp thun phn mm.
M t ph- ng php nh gi.
nh gi tin trnh pht trin phn mm.
nh gi phn mm ti s dng.
nh gi phn mm khng phn pht - c.
5. nh gi vic l- u tr, vic x l, v phn pht.
T- liu d n.
Phn mm.
Cc file d liu.
6. R sot v kim ton.
7. Qun l cu hnh phn mm (khi khng nhm ti mt t- liu
chuyn bit no).
8. Bo co cc vn kh khn v hnh ng chnh n.
9. nh gi cc th tc th nghim.
10. Cng c, k thut v ph- ng php.
11. Khng ch cht l- ng ca cc ch thu ph, ng- i bn hng v
cung cp.
12. Khng ch ph thm
Cc th tc khng ch linh tinh.
Khng ch c th d n.
13. Bo co, ghi nhn v t- liu BCLPM.
Th tc bo co tnh trng.
Bo tr.
L- u tr v an ninh.
Chu k s hu.
Vic thc hin k hoch phi - c nh gi theo gii hn phm vi thay
i ca d n, k c nhng nhim v v trch nhim tham chiu trong k
hoch v cc c im mi khc hay thay i ca d n. Khi cp nht
SQAP phi xem xt cc hot ng v hin t- ng sau y ca d n:
QUAN L DU N PHN MM
133
- Yu cu mi hay thay i ca hp ng
- Tiu chun v chnh sch b sung
- Ti liu d n b sung
- Thay i trong c cu t chc ca d n
- Cng c v tin ch mi
- Ch thu ph v ng- i bn b sung.
7.2.4. o cht l- ng phn mm.
c nhiu quan tm n cc cu hi lin quan n vic o l- ng cht
l- ng. Lm sao chng ta c th xc nh mc m sn phn phn mm
c - c thuc tnh m h gi l cht l- ng? Khi no cht l- ng ca
sn phm phn mm l cao v khi no l thp?
Mt trong nhng pht trin mi y nht trong vic bo him cht
l- ng (khng ch i vi phn mm) l vic thy rng cht l- ng khng
phi l mt thuc tnh nh phn tn ti hay khng tn ti. Kaposi v
Meyer (1990), trong mt tham lun v bo m cht l- ng trn c s o
m, ni ln nim tin ca h l bo m cht l- ng sn phm v qui
trnh cng ngh phn mm phi cn c o m. Vic o m cht
l- ng cng sm bt u, cc vn cng sm xc nh. Cohen v cng s
(1986) khi ni n kinh ph khc phc sai lm trong cc giai on u
ca pht trin phn mm x- ng s tn ti ca lut ly tha ni
ting
30
.
Cht l- ng ca hai sn phm c th so snh - c v thc hon ton
chp nhn - c khi ni l cht l- ng mt sn phm ln hn cht l- ng
ca sn phm kia. Cng chp nhn - c trong vic o m v suy ra mc
khuyt tt d kin cn c trn kt qu o - c.
B tr s o - c lin quan n cht l- ng sn phm - c coi l o
cht l- ng ca sn phm, o cht l- ng phn mm c th - c dng
xc nh mc m sn phm phn mm p ng yu cu. Vic s dng
o cht l- ng gia tng tnh khch quan trong vic nh gi cht l- ng
sn phm. Vic nh gi cht l- ng ca con ng- i l ch quan v do
l ngun gc bt ng c th c, c bit gia khch hng v ng- i sn
xut.
Mt s ph- ng php thit lp o cht l- ng phn mm th- ng - c
pht trin mc d ch- a c tiu chun no ni chung chp nhn - c,
chng hn, d tho ban u ca IEEE Std 1061 (1990) bao gm chi tit
h cht l- ng phn mm ni chung k c ph- ng php lun gi v
o vn dng v nhiu th d cng h- ng dn. Mt khi - c xc nh,
o cht l- ng thc vy gia tng tnh khch quan nh- ng bn thn nh
ngha li khng cn thit khch quan v ty thuc ln nhu cu ca t
chc - a ra nh ngha.
Cch tip cn c bn vn dng o cht l- ng phn mm i hi:
30
chi ph khc phc sai lm tng theo ly tha khi pht pht trin phn mm din tin dc chu k
pht trin.
QUAN L DU N PHN MM
134
- Nhn bit mi thuc tnh cht l- ng phn mm yu cu. iu ny
th- ng xut pht t c im yu cu phn mm.
- Xc nh cc tr s o - c gn vi mi thuc tnh cht l- ng
- M t ph- ng php m mi tr s o - c s - c o m.
- Th tc cung cp t- liu v kt qu o cht l- ng ca sn phm phn
mm.
Mt b nhiu tr s c th - c s dng xc nh cht l- ng tng
th ca sn phm phn mm. D sao, c th to ra ln o duy nht biu
th cht l- ng tng th ca sn phm phn mm. iu ny i hi:
- Ph- ng php c cn nhc trong vic kt hp cc thuc tnh cht
l- ng o - c thnh ln o duy nht cht l- ng cho sn phm.
Mt s th d ca o phn mm l:
tin cy t l thi gian h thng - c vn hnh c kt qu (th d 23
trn 24 gi to ra 100 x (23/24)%).
phc hi - c l- ng thi gian m h cn c phc hi sau tht
bi (th d 1 gi np li t d tr v 30 pht ti lp ban u h
thng).
Thn hu ca ng- i dng l- ng thi gian hun luyn cn cho ng- i
dng mi.
Vic o cht l- ng phn mm khng - c thc hin ch vo cui d
n. Mc cht l- ng phi - c thc hin ch vo cui d n. Mc
cht l- ng phi - c o vi nhng khong cch u n trong pht trin.
Do , mi rt gim trong vic o cht l- ng tng th phi - c coi nh-
cnh bo cho mi ng- i qun l d n khin phi c hnh ng hiu
chnh. Cht l- ng cao vo cui d n - c thc hin khi m bo cht
l- ng cao xuyn sut pht trin ca d n.
7.2.4. Mt s - ng li chung
Cc hot ng bo m cht l- ng phn mm c bn bao gm vic
duyt v ph chun ph- ng php lun pht trin, phn mm v t- liu
cng gim nh v ph chun th nghim.
Cc hot ng SQA khc nh- vic gim nh duyt li, chn la v ph
chun cng c pht trin hay qun tr kim tra cu hnh ty thuc cch
m SQA - c thch ng vi d n c tr- ng. Qui m ca d n th- ng l
yu t quyt nh. Nhng h- ng dn sau bn n mt s thng s cn
xem xt cho cc loi d n khc nhau khi lp k hoch SQA.
- Trong nhng d n nh, c nhiu hot ng SQA c th do ng- i
qun l d n thc hin. iu ny bao gm t chc v gim nh duyt
li v kim ton, nh gi v la chn cc cng c pht trin, la chn v
vn dng cc tiu chun.
- Cc th tc th nghim v th nghim lun l tt nht khi - c mt
i ring bit c lp tin hnh (bn n sau). Quyt nh l gim nh
QUAN L DU N PHN MM
135
cc hot ng th nghim no c th - c y thc cho SQA ty thuc
nhiu yu t, k c s c lp ca i SQA, qui m ca d n v tnh
phc tp ca d n (coi ch- ng 6 bn v tnh phc tp ca cc d n).
Khi vic th nghim - c i th nghim c lp tin hnh th d dnh
lu ca SQA s l ti thiu. Trong hu ht cc tr- ng hp khc, trch
nhim ca i SQA s l lp k hoch v gim nh th nghim ca h
thng.
- V h- ng dn chung, th- ng khng nn mt thnh vin ca i
pht trin thc hin SQA. D sao, cc d n nh th- ng khi khng th
xc minh chi ph ca mt k s- - c b tr SQA. Vn ny c th - c
gii quyt khi c mt k s- SQA duy nht chu trch nhim cho hai hay
ba d n nh (vi mi d n ti tr phn ng gp ca mnh v dch v
SQA).
Mt h- ng dn b sung - c cn c trn cc kt lun ca Wesselius v
Ververs (1990) v vic vn dng kim tra cht l- ng c hiu qu.
- Kh nng kim tra cht l- ng phn mm hiu qu trc tip vi cht
l- ng ca c im yu cu ca phn mm. Kim tra cht l- ng i hi
chi tit khng m h ca cng nhiu c im yu cu ca cht l- ng
phn mm cng tt.
7.3. Th nghimphn mm.
T th nghim c nhiu ngha nh- ng trong s dng thng th- ng nht
ca n t - c vn dng vo vic xem xt v nh gi ci g nhm xc
nh s tn ti ca mt s c tnh. Trong th nghim phn mm, cc c
tnh gn vi cc yu cu. Xc nh tr- c ca phn mm. Th nghim
phn mm l qu trnh xc nh mc theo phn mm tha mn
nhng yu cu c th. Do , th nghim phn mm i hi c chi tit
yu cu phn mm tr- c khi th nghim, c th - c tin hnh.
Ni mt cch n gin, chng ta khng th th nghim phn mm vi
y ngha nu chng ta khng bit phn mm - c d kin lm g.
Th nghim c th do ng- i lp trnh, k s- hp nht hay mt nhm
th nghim c lp thc hin. Trong hu ht tr- ng hp, phn mm
khng bao gi do ng- i lp trc tip chu trch nhim vit m - c th
nghim li th nghim c. Ng- i lp trnh him khi li khch quan - c
vi chnh m ca mnh v th nghim ca anh ta s khng hiu lc. C
nhiu cch tng tnh khch quan ca th nghim. Cch tt nht l s
dng nhm th nghim c lp. Nhng ng- i th nghim c lp l
nhng k s- m y thc chnh ca h l pht trin cc k hoch th
nghim v cc tr- ng hp th nghim v th nghim h thng mt cch
khch quan v nghim ngt p ng c im yu cu.
7.3.1. Cc loi th nghim phn mm.
Th nghim l hot ng pht trin cui cng ca d n phn mm. D
sao, iu ny khng c ngha th nghim ch - c tin hnh vo cui d
QUAN L DU N PHN MM
136
n. Phn mm phi - c th nghim. mi giai on ca pht trin. Cc
th nghim phn mm bao gm:
- Th nghim n v
- Th nghim hp nht
- Th nghim h thng ph
- Th nghim h thng
- Th nghim suy thoi
- Th nghim alpha
- Th nghim bta
- Th nghim nghim thu
Thng th- ng tt nht l th nghim m un phn mm ngay sau khi
- c m ha. y l mt trong s t tr- ng hp m cc nh lp trnh th
nghim m ca h. Nhng th nghim m un ban u, gi l th
nghim n v, - c ng- i lp trnh tin hnh nhm xc nh xem chng
c ph hp vi mt b yu cu ti thiu. Chng bao gm nhng th
nghim nh- :
- Nhp v xut (khng c vng vnh cu) cho mt b nh c bn cc
d liu u vo.
- Vi u vo cho, u ra l hp l
- Nhng ch- ng trnh ph - c gi ra theo ng trnh t. iu ny
- c th nghim khi s dng cc ngch (v h thng ph rng).
Sau th nghim n v thnh cng, cc m un - c trnh kim tra
cu hnh. Sau , chng - c - a vo hp nht ni chng - c tp hp
v th nghim coi nh- b phn tun t cc m un vo trong h thng.
Mi tnh chc nng b sung - c th nghim, giai on ny, tnh chc
nng th nghim tr- c y li phi - c th nghim m bo khng c
m un mi no ph hy h thng. iu ny - c coi l th nghim suy
thoi.
nhng h thng phn mm ln, cc m un tr- c ht c th - c tp
hp v hp nht vi phn cng thnh nhng h thng ph. Sau khi cc h
thng ph - c th nghim ring r, chng - c phi hp thnh mt
h thng hon chnh.
Sau khi hp nht h thng hon thnh, cc th nghim cui cng
- c tin hnh. iu ny bao gm th nghim alpha, bta v th nghim
nghim thu. Th nghim alpha - c tin hnh cng vi h thng hon
chnh, nh- ng khng c d liu sng th nghim bta s dng d liu
sng nh- ng i hi c s gim nh th- ng xuyn ca thnh vin i
pht trin. Mt khi h thng n nh, cc th nghim nghim thu - c
tin hnh v nu thnh cng, h thng - c - a ra pht hnh (nhng giai
on ny - c bn thm ch- ng 4).
QUAN L DU N PHN MM
137
Cc giai on cui cng ca th nghim h thng - c biu th trong
th d sau mt h thng th qu ngn hng t ng - c pht trin cho
mt ngn hng ln. Khi h thng ph th qu hon chnh, n tin
hnh th nghim h thng ph m bo l cc chi tit th qu t ng
hot ng tt. Sau h thng ph my tnh trung tm - c th nghim
v d liu th nghim m bo h tin hnh d liu ng. Cui cng cc
h thng ph thng tin lin lc - c th nghim vi th qu m phng
mt u v vi my tnh trung tm m phng u kia.
Ri ba h thng ph - c hp nht v vic th nghim ton b h
thng - c tin hnh m bo th qu t ng v my tnh trung tm lin
lc v hot ng tt.
Sau khi th nghim h thng, h thng th qu t ng hp nht hon
ton - c th nghim trong mi tr- ng th nghim alpha. H thng - c
th nghim vi d liu th nghim thc tin v hot ng ca mi chi tit
- c xem xt v so snh vi cc yu cu ca h thng nhim v ny - c
giao cho nhm th nghim c lp.
Sau khi h thng th qu t ng qua th nghim alpha, c v
tin cy c th tin hnh d liu thc. y l th nghim bta. D sao,
ch mt trm th qu t ng duy nht - c u vi my tnh ca ngn
hng v vn hnh ca h thng th- ng xuyn - c cc thnh vin ca i
pht trin gim st. Mi vn - c nh v v hiu chnh ngay v th
nghim bta tip tc n khi h thng th qu hot ng khng sai khng
hng trong mt thi gian xc nh tr- c (th d mt thng). Vic gim
nh ca giai on bta cng - c y thc cho mt nhm th nghim c
lp.
Sau khi thc hin thnh cng th nghim bta, h thng th qu b- c
vo th nghim nghim thu cui cng. Cc i din ca b my k thut
ca ngn hng gim nh b th nghim - c xc nh tr- c v xc nh
hay bc b thnh cng ca th nghim nghim thu.
Theo th, h thng th qu ngn hng t ng qua mi giai on
th nghim ch yu. Khng phi tt c nhng giai on vn dng
- c cho mi d n. Chng hn, th nghim alpha v bta c th vn
dng cho cc h thng th- ng mi hay h- ng v ng- i dng nh- ng
khng cho s pht trin phn mm cho mt v tinh thng tin lin lc hay
ng dng qun s. Mt th d khc l th nghim h thng ph r rng
ch p dng - c khi h thng hin nay - c phn thnh cc h thng
ph.
Trong mi tr- ng hp, th nghim c lp - c ht sc khuyn dng
cho cc giai on th nghim tin tin hn (t th nghim h thng tr
i). Th nghim c lp khch quan s th- ng nh v. Vn sm hn
th nghim c lp m cc nh sn xut tin hnh nhiu. y l mt li
ch ch yu cho d n v vn - c nh v cng sm th li t tn km
hn trong vic hiu chnh.
7.3.2. Th tc th nghim chnh thc.
QUAN L DU N PHN MM
138
C nhiu tip cn v th tc khc nhau cho mi giai on th nghim
phn mm. iu quan trng l chn la tip cn thch hp cho mi giai
on. Cc tiu chun th nghim phn mm chnh thc
31
bao gm c
h- ng dn tiu chun - c vn dng cho mi giai on th no.
Tiu chun 2167 US (DOD 1988a) lin quan n cc giai on c bn
sau:
1. Th nghim n v phn mm ca my tnh (CSU)
2. Th nghim cu kin phn mm ca my tnh (CSC). Lin quan n
vic th nghim ca nhm CSC lin quan (coi hnh 8.4).
3. Th nghim hng mc cu hnh phn mm my tnh (CSCI). y l
giai on tin tin ca th nghim hp nht nhng h thng nh, iu
ny t- ng ng vi th nghim h thng, v nhng h thng ln c
nhiu CSCI, iu ny t- ng ng vi th nghim h thng ph.
4. Hp nht v th nghim h thng. iu ny t- ng ng vi th
nghim h thng tin tin.
Mi giai on th nghim phn mm DOD c bng tiu ch nh gi
km theo, m t nhng yu cu nh gi th nghim. iu ny bao gm
cc tiu ch nh- tnh nht qun ni b, tnh thng hiu, tnh theo di - c
qua du vt v p ng yu cu. Sau nhng tiu ch ny - c ra cho
nhng hng mc phi nh gi nh- m ngun (v sau ny l m ngun
cp nht), cc th tc v kt qu th nghim.
Th tc th nghim hin nay - c cung cp t- liu nh- sau:
- K hoch th nghim cung cp t- liu chnh sch th nghim tng
qut cho d n, ph- ng php lun th nghim - c s dng v cc
ngun yu cu.
- T th nghim bao gm chi tit v:
- Th tc th nghim c th, m t ci g cn th nghim phi - c
tin hnh nh- th no, u vo s l g v u ra d kin phi l g. iu
ny cng m t tiu ch thng qua khng t cho mi th nghim.
- Cc bo co th nghim thng tin t- liu v tin hnh th nghim v
kt qu nhn - c.
Ni chung, mi th tc th nghim phn mm chnh thc - c cn c
4 thnh phn: qui hoch th nghim, m t th nghim, tin hnh th
nghim v bo co th nghim. Hu ht cc tiu chun th nghim h tr
cc yu t v khc v mc chi tit v thng tin t- liu yu cu.
Tiu chun IEEE 829 (1987b) v thng tin t- liu th nghim phn
mm c m t chi tit hn nhiu v tip cn nn lm trong th nghim
phn mm, k c v vn th d v h- ng dn s dng. Tip cn IEEE
31
Cc tiu chun pht trin phn mm - c bn n ch- ng 8 v cc ph- ng php lun pht trin
phn mm ch- ng 4.
QUAN L DU N PHN MM
139
cng bao gm ba cu kin c bn: k hoch th nghim, chi tit th tc
th nghim (t- ng t vi m t th nghim DOD) v bo co tnh hnh
th nghim (t- ng t bo co th nghm DOD). Coi cc hnh 8.5 v 8.6
v tng th mi hot ng th nghim v t- liu chnh thc theo tiu
chun pht trin phn mm IEEE. Cc th tc th nghim khng hon
ton - c cc tiu chun vn dng xc nh. Ph- ng php v k thut th
nghim hin nay phi thch nghi vi loi d n - c pht trin v mi
tr- ng d n cng cng c c - c. Cng thm vi lp k hoch v bo
co th nghim, trch nhim ca k s- th nghim (hay nhm th
nghim) l la chn k thut thch hp - c s dng trong th nghim h
thng phn mm. Th nghim c tr- ng v k thut hp nht - c bn
n ch- ng 4.
7.3.3. Mt s - ng li chung
Trong bt c d n phn mm no u c gii hn v ci g c th v
phi th nghim. Nhiu th tc th nghim l phc tp v tn km mt
cch khng hp l. iu quan trng l sao ph hp c- ng th nghim
vi chi ph khuyt tt h thng khng pht hin. H thng kim k khng
cn phi th nghim mc nh- phn mm i vi h thng h tr
cng tc y t.
Th nghim qu mc cng c th sai lch. Nhiu th tc th nghim
i hi thay i h thng - c th nghim (th d cc gim st vin trc
tuyn, nhng tin ch du vt v.v...) hay s dng thit b th nghim c
bit. iu ny c th c ngha l h thng - c th nghim khng phi l
h thng - c bn giao.
Mt cch nhn th v v vn ny - c Laplante trnh by (1990)
trong bi bo lin h nguyn l bt nh (th- ng p dng cho vt l) ca
Heisenberg vo vic th nghim phn mm
32
. Thuyt ca Laplante khng
nh l h thng phn mm cng - c xem xt cht ch bao nhiu th qu
trnh xem xt cng tc ng n h thng - c th nghim. L thuyt
ny c th gi khng trung thc nh- ng lm sng t thng ip c
bn:
Th nghim qu mc c th c hi cho h thng - c th ba trong
nhng yu cu c bn th nghim tt l:
- Yu cu - c vit r
- Th tc th nghim tt
- Nhng ng- i th nghim c hiu qu
Nhng yu cu tt - c tho lun cc ch- ng 4 v 8 nh- ng mt
trong nhng iu kin c bn c - c yu cu tt l phi th nghim
32
Laplante gi cng thc sau Az As = H trong As biu th s bt trc ca m phn mm. As s
bt trc ca cc chi tit th nghim v H l hng s no .
QUAN L DU N PHN MM
140
- c. Do y nhng yu cu chnh thc lun phi - c vit vi ngh
th nghim trong tr. c im c yu cu khng th th nghim cng
khng th t ra tn ti - c v do phi khng c ch ng trong chi
tit yu cu chnh thc cho d n.
Nhng th tc chng phi - c da trn tip cn gi l th nghim
tiu cc. iu ny c ngha mc tiu ca nhm th nghim l chng
minh nhng ng- i sn xut khng lm tt cng vic. Nu th nghim
- c tin hnh ng n th khi nhng ng- i th nghim tht bi, d n
thnh cng
33
. Nhng ng- i th nghim phi khng - c d tnh (nguyn
vn lch s) v cui cng nhng ng- i dng cng c d tnh u.
Nh- chng ta thy, th nghim - c tin hnh tt nht khi c mt
nhm th nghim c lp, khch quan, d sao ch c nhng d n ln c
th h tr mt nhm th nghim c lp. Nhng d n ln c th h tr
mt nhm th nghim c lp. Nhng d n nh c th chia x dch v
ca cc nhm th nghim c lp v thm ch c th thu cc k s- pht
trin nht l th nghim. Theo th, k s- pht trin trong mt d n c
th l mt ng- i th nghim khch quan kim nhim trong mt d n
khc.
Cui cng, th nghim phi - c dn t- liu y . Mt trong nhng
loi th nghim ti t nht l th nghim tht bi chng th lp li. Nu
tht bi khng th sn sng lp li th n th- ng khng - c hiu chnh.
Do t- liu th nghim phi ng- i sn xut c th lp li phn
hin t- ng dn ti th nghim tht bi.
7.4. Tmtt.
Cc nhm h tr d n khng ch gim nh cng vic h tr cho ng- i
qun l d n v cc k s- pht trin, m cn thc hin nhng cngvic
tt hn khi tp trung c gng ca h vo chc nng h tr c tr- ng.
C nhiu loi chc nng h tr d n. Dch v th- k, h tr hnh chnh,
xut bn v cung cp t- liu l nhng th d h tr khng k thut, th
nghim kim tra cu hnh, cng ngh h thng, qun l hp nht v bo
m cht l- ng l nhng th t v chc nng h tr k thut. Ba trong
nhng chc nng c bn cn c bt c d n pht trin phn mm no
l:
- Kim tra cu hnh: qun l thay i cho sn phm phn mm - c
pht trin.
- Bo m cht l- ng: gim st v kim tra cht l- ng sn phm - c
pht trin.
- Th nghim: th p ng vi chi tit yu cu chnh thc ca sn
phm.
K hoch qun l cu hnh phn mm (SCMP) v k hoch bo m
cht l- ng phn mm (SQAP) cung cp t- liu cc ngun cn cho mi
33
iu ny nhn mnh im qu - n gin, r rng, c nhiu yu t khc cng ng gp vo s
thnh cng ca d n phn mm
QUAN L DU N PHN MM
141
nhm h tr, s dng cc ngun th no v c nhng tiu chun v th
tc no s - c vn dng trong d n. SCMP v SQAP tr thnh y
nhim cho hai nhm trong pht trin d n. Th nghim phn mm l qu
trnh xc nh mc m phn mm tha mn - c nhng yu cu c
th. Nhng th tc th nghim - c chun b nhm chng minh cc yu
cu - c hon tt (hay khng hon tt). Do , chng cn - c da trn
tip cn gi l th nghim tiu cc. iu ny c ngha mc tiu ca nhm
th nghim l chng t rng nhng ng- i sn xut khng lm tt cng
vic. Nu th nghim - c tin hnh ng n, th khi nhng ng- i th
nghim tht bi, d n l thnh cng.
Nhng nhm th nghim c lp l nhng ng- i th nghim h thng
phn mm - c chung hn. Vic th nghim c lp khch quan s
th- ng nh v - c vn sm hn cc th nghim ch quan do nhng
ng- i sn xut thc hin. y l li ch ch yu cho d n v vn cng
sm - c nh v bao nhiu th cng t tn km hn trong vic hiu
chnh.
y l trch nhim ca ng- i qun l d n trong vic t chc cc
nhm h tr d n v dn t- liu cc hot ng theo k hoch ca h
trong ch- ng trnh pht trin d n (bao gm SCMP v SQAP). Cc chc
nng h tr d n - c lp k hoch tt lc khi u d n s ng gp
cho vic qun l d n c hiu qu xuyn sut d n.
Bi tp.
1. Bn - c ch nh lm qun l d n pht trin h thng kim k
cho mt cng ty sn xut ln. H thng kim k cn pht trin - c cn
c ph- ng php t hng sS (khi mc kim k cho mi hng mc tt
xung d- i s th l- ng - c t s nng mc ln S: mi hng mc c tr
s sS ca n.
Gi bn phin bn trung gian ca h thng - c pht hnh ni b
cho vic th nghim h thng. Th nghim alpha, th nghim bta v
pht hnh cui cng. Hi chnh lch chc nng gia mi phin bn s l
g? M t th tc kim tra phin bn - c dng v gi mu m t phin
bn.
2. L ng- i qun l d n, bn hy xc nh cc t chc qun l cu
hnh v bo m cht l- ng cho h thng kim k m t bi tp 1, c
bao nhiu ng- i - c yu cu lm nhng nhim v ny v trch nhim
ca mi ng- i ra sao? Gii thch quyt nh ca bn.
3. L ng- i qun l m bo cht l- ng phn mm, hy vit ch- ng
trnh duyt xt v kim ton ca SQAP v s pht trin h thng kim k
m t trong bi tp 1. C nhng duyt xt v kim ton no - c tin
hnh v khi no? M t yu cu mi duyt xt v kim ton v mi hng
mc yu cu - c ph chun th no. M t th tc hiu chnh sau mi
duyt xt v kim ton.
QUAN L DU N PHN MM
142
4. Lp k hoch cc giai on th nghim cho vic pht hnh h thng
kim k m t bi tp 1. C nhng giai on th nghim no m bn
gi sau giai on hp nht? Vit cc ch- ng trong k hoch th nghim
m t mi mt giai on th nghim . C yu cu nhng ngun no v
d liu th nghim no? y thc nhng ng- i th nghim cho mi giai
on.
5. Hy xc nh nm thuc tnh cht l- ng ch yu ca h thng kim
k m t trong bi tp 1 v xc nh o cht l- ng cho nhng thuc
tnh . Gii thch l do v o m bn xc nh.
6. S dng mu yu cu thay i m t trong hnh 7.4, in mi
mc vo mu cho cc thay i sau (1) mt c im gim st n t
- c b sung cho hng mc d- i mc 1 (2) mc sS cho mi hng mc
- c b sung vo bo co kim k (3) yu cu tr li 6 giy cho mt vn
kim k - c gin ra ti 20 giy v (4) cng sut yu cu ca c s
d liu - c tng t 2000 ti 5000 hng mc. Gii thch cc vn v
nhn nh lin quan ti mi mu.
7. Bi hc lp: chia lp thnh 4 nhm. Giao bi tp 6 cho mi nhm.
Tho lun chnh lch gia cch mi nhm in mu yu cu thay i (th
d tho lun khc bit v d ton).
Tho lun nhng vn khc nhau m mi nhm nhn bit v nhng
nhn xt - c tnh n.
QUAN L DU N PHN MM
143
Ch- ng tm
Cc tiu chun pht trin phn mm
Cc tiu chun pht trin: tai hi cn thit
Vi nhiu k s- phn mm, c mu thun gia pht trin phn mm v
tiu chun. Cc tiu chun gii hn phn ln t do ca ng- i sn xut
phn mm, v tt nhin, nhng tiu chun - c lm vi mc ch nh-
th. Cc tiu chun i km theo chu k pht trin t u n cui c
nhng tiu chun thit k, tiu chun t- liu, tiu chun lp m, tiu
chun th nghim, v thm ch tiu chun trnh v nh gi ngh
(coi ch- ng 3).
Mc d cc tiu chun c th - c coi l ni bun cn thit, vic vn
dng cc tiu chun t - c kt qu xng ng; iu lm cho vic
pht trin phn mm d qun l - c hn. iu ny khng c ngha l
ch c ng- i qun l c li trong vic s dng tiu chun. Cc tiu chun
thc y mc ngn np v thch nghi, gip ng- i sn xut hiu - c
cng vic ng- i khc lm v khuyn khch h to ra cng vic m ng- i
khc hiu - c.
Mt trong nhng thch ch yu cho ng- i qun l d n l vic la
chn tiu chun ng. Trong nhiu tr- ng hp cc chun pht trin - c
c t nh- l yu cu, nh- vn th- ng xy ra vi cc d n chnh ph. D
sao, ngay khi tiu chun c th - c yu cu, n vn phi - c may
o ph hp vi yu cu ca d n - c pht trin. Mt s phn ca tiu
chun c th b "loi" nu chng khng vn dng - c v nh- phn tch
thi gian theo thit k ca mt h thng khng nghim ngt v thi gian
hay vic - a ra nhng tiu chun lp m cho m ch- ng trnh - c ti s
dng.
Ch- ng ny cung cp cho ng- i qun l d n thng tin c bn v la
chn mt b tiu chun pht trin phn mm thch hp. Ch- ng bn n
cc loi tiu chun phn mm v m t uy tn ca hai tiu chun ph bin
nht: tiu chun cng ngh phn mm IEEE v tiu chun 2167A ca B
Quc phng Hoa K.
8.1 Tng quan v cc tiu chun pht trin phn
mm.
T tiu chun - c ng dng rng ri cho nhiu loi - ng li. Mt
s tiu chun c th hot ng lm h- ng dn, k thut gi hay gp
pht trin hay qui m t- liu. Nhng tiu chun khc c th hot ng
nh- l mt b nhng nh lut nghim ngt ch o mi kha cnh hot
ng pht trin. Hnh 8.1 m t cc tiu chun pht trin phn mm chnh
v t- ng quan gia cc tiu chun. Mt trong nhng tiu chun pht trin
T- liu nim c bn
QUAN L DU N PHN MM
144
Hnh 8. 1
Cc chun pht trin phn mm ch yu.
phn mm d hiu nht l tiu chun US DOD 2167 (DOD 1988a). Vic
ra tiu chun tuyn b lp ra nhng yu cu thng nht cho pht trin
phn mm vn dng - c xuyn sut chu k cuc i ca h thng v
tiu chun khng nhm nhn mnh hay khuyn bc vic s dng bt c
i hi xut
xut
K hoch bo m
cht l- ng
K hoch qun l cu
hnh
c t yu cu phn
mm
K hoch pht trin
d n
c t thit k mc
nh
K hoch th nghim Cc chun lp m
Cc th tc th
nghim chp thun
K hoch tch hp
c t bo tr
T- liu ng- i dng
c t thit k chi tit
QUAN L DU N PHN MM
145
ph- ng php pht trin phn mm c bit no. D sao, tiu chun 2167
nng nghing v ph- ng php lun pht trin theo pha nh- h bin ha
thc n- c. Overmyer (1990) trong mt phn tch l th v tiu chun, bo
co l nhiu nh nghin cu pht trin thnh cng nhng m hnh ph
hp vi tiu chun trong khi - a ra nhng quan nim mi v yu cu v
thit k lp li, mc d ng kt lun l 2167 khng - c pht trin vi
thit k lp li trong t.
Vi nhng ai tm kim tiu chun t i hi hn tiu chun 2167, cc
tiu chun pht trin phn mm IEEE - a ra tip cn t- ng t mm do
hn nhiu. Nhiu nhng tiu chun IEEE ta nh- nhng h- ng dn hn
l tiu chun cng nhc.
Cc tiu chun d xc nh v vn dng cho cc hot ng pht trin
c bn (nh- dn t- liu v lp m) hn l cho cc qui trnh pht trin (nh-
thit k, th nghim v hp nht). Buckley trong nhp mn ca mnh v
mt trong nhng phin bn sm sa
34
ca cc tiu chun cng ngh phn
mm IEEE gii thch nh ca IEEE i t tiu chun sn phm (th d
mt t- liu) n tiu chun qu trnh. Trn thc t, nh ngha cc tiu
chun cho qu trnh th nghim phn mm l mt trong nhng qu
trnh, u tin - c cc tiu chun IEEE cp. iu ny phn nh s
bnh tr- ng ca phm vi tiu chun.
Buckley trng i nhng tiu chun t- ng lai p ng - c nhng lnh
vc mi nh- cc h mt nng sut phn mm. Nh- ng c mt s iu d
tiu chun ha hn nhng iu khc v ti h mt nng sut l mt
trong nhng ti kh hn. Zimmer (1991) khng nh vi nh ngha
"nng sut" khng c tiu chun, th rt kh m bit - c phi o ci g,
li cng t c kh nng o - c n.
Th nghim phn mm l mt lnh vc trong c nhiu tiu chun
- c to ra. iu ny cng ng ca thit k phn mm mc d c
nhn mnh nhiu n cu trc ca dn t- liu thit k. Vic hp nht
phn mm kt thc t thun li hn tr- c ht v c kh khn trong vic
tiu chun ha nh- hot ng loi b trc gic.
Th nghim khng ch l lnh vc - c tiu chun ha rng. Tht
ngc nhin khi thy vic to lp tiu chun cho mt trong nhng hot
ng pht trin phn mm t quyt nh hn li phn thnh trong nhng
nm gn y, c th cho vic kim tra cht l- ng phn mm. Kh khn
trong vic nh ngha cht l- ng phn mm - c minh ha trong t vng
tiu chun IEEE v thut ng cng ngh phn mm (IEEE 1987b) c
khng d- i 4 nh ngha ring bit:
1. Ton b cc nt v c im ca sn phm phn mm c trong kh
nng ca n p ng cc yu cu ra, chng hn ph hp vi cc c t.
2. Mc m phn mm c - c s phi hp mong mun v cc
thuc tnh.
34
Nhp mn ca Buclkey xut hin trong bn in 1984 cc tiu chun IEEE.
QUAN L DU N PHN MM
146
3. Mc m khch hng hay ng- i dng nhn thc - c l phn
mm p ng - c cc k vng phc hp.
4. c im phc hp ca phn mm xc nh mc m phn mm
s dng p ng nhng k vng ca khch hng.
Cht l- ng phn mm - c US DOD
35
nh ngha li n gin hn l:
5. Kh nng ca mt sn phm phn mm p ng - c cc yu cu
c th ca n.
Trong khi cc nh ngha 1 v 2 c v hi ti ngha th cc nh ngha
3; 4 v 5 l ch quan v c v gi cht l- ng, ging nh- ci p, l
con mt ca ng- i quan st.
Cht l- ng phn mm khng ch l t c nh ngha khng phi l
chung nht, iu ny cng ng v nhng t c bn nh- yu cu, thit k
v bo tr. Cch m nhng t ny - c s dng trong cc tiu chun khc
nhau li cng thay i. iu ny th- ng gy ra ln ln khi cng nhng t
c ngha cc iu khc nhau cho nhng ng- i khc nhau.
C nhiu l do v sao tiu chun li khng tiu chun v tnh ch quan
li chnh l 1 trong nhng l do . Cc d n phn mm thay i d, nhu
cu ca khch hng thay i nhiu v cc t chc pht trin phn mm
cng thay i ln. Do y khng ngc nhin l B Quc phng Hoa K
khi cng nhn d n v nhu cu thay i, cho php may o ng k
tiu chun 2167 pht trin phn mm ca cc d n.
Nhiu tiu chun pht trin phn mm - c to lp p ng cc
hot ng nh- nh gi ngh (coi ch- ng 3), r sot li, k thut v
kim tra cu hnh. Gi y cc tiu chun tn ti mi hot ng pht trin
d n ch yu.
Nhiu tiu chun phn mm - a tri l tha mn nu - c vn dng
ng. Cc tiu chun i hi k lut vn khng phi lun lun d dng
buc trong cng ngh phn mm.
8. 2 Tiu chun US DOD 2167.
Tiu chun 2167 - c B Quc phng Hoa K ta lp cho s pht
trin ca mi h thng phn mm quc phng s mnh khn tr- ng. Sau
khi cng b nhiu d tho v phin bn ban u tiu chun - c xut
nm 1985 thay th tiu chun. Hi qun 1679A vn s dng rng ri v
tiu chun 1644B MIL. nh l nm 2167 tr thnh tiu chun duy
nht chnh thc cho vic pht trin phn mm ca h thng quc phng
cho qun s Hoa K.
Phin bn u ca tiu chun 2167 thu ht nhiu bnh lun ca cc
nh thu phn mm DOD v ngay trong ni b B Quc phng. iu ny
dn n vic cng b tiu chun DOD 2167A sa i nm 1988 vi cc
35
nh ngha US DOD v cht l- ng phn mm xut hin trong tiu chun DOD 2168. Ch- ng
trnh cht l- ng phn mm ca h thng quc phng (M).
QUAN L DU N PHN MM
147
tiu chun mi km theo 499 cho vic qun l cng ngh v 2167 cho
vic bo m cht l- ng phn mm.
8. 2. 1. Tng quan tiu chun 2167.
Mc tiu - c n nh ca tiu chun DOD 2167 (DOD 1988a) l thit
lp nhng yu cu thng nht cho vic pht trin phn mm p dng - c
sut c chu k i ca d n. Tiu chun bao gm vic vn dng cc tiu
chun km theo sau:
DOD-STD-480 Kim tra cu hnh - Thay i v t b cng ngh
MIL-STD-490 Thc hnh c t
MIL-STD-499 Qun l cng ngh
MIL-STD-1521 R sot li v kim ton k thut cho cc h thng,
thit b v phn mm my tnh.
DOD-STD-2168 Ch- ng trnh cht l- ng phn mm h thng quc
phng.
MIL-STD-881 Cu trc phn tch cng vic cho cc hng mc vt
liu quc phng.
DID M t hng mc d liu (DID - c pht m hip vn
vi kid)
Tiu chun 480 kim tra cu hnh m rng sang ch th v h- ng dn
qun l cu hnh ni chung xut hin ngay trong thn ca t- liu 2167.
Cc tiu chun 490 v 499 bao gm cc ch th cng ngh chung cho
vic c t v qun l khng c tr- ng cho pht trin phn mm.
Tiu chun 1521 m t r sot li v kim ton chnh thc - c tin
hnh trong chu trnh pht trin.
Tiu chun 881 m t yu cu DOD cho vic sn xut v s dng kt
cu phn tch cng vic (WBS). Trong ln pht 2167A ca tiu chun,
881 khng cn - c c t na.
Tiu chun 2168 (DOD 1988b) cha cc yu cu cho s pht trin, dn
t- liu v thc hin ch- ng trnh phn mm bao gm vic lp k hoch v
tin hnh.
- nh gi cht l- ng phn mm.
- Dn t- liu phi hp v cc hot ng lin quan.
- Hot ng tip theo cn thit m bo gii quyt vn kp thi v
c hiu qu.
Cc DID l mt b t- liu d hiu bao gm mi pha ca pht trin, bo
tr phn mm v xut bn cc cun tham kho cho ng- i dng. Cc DID
bao gm mt b phn gi l h- ng dn chun b to nn mc t do
rng ri bng cch cho php ly kch th- c t- liu v s dng cc ph- ng
n v th thc trnh by. B DID y - c m t trong bng 8.1.
Bng 8. 1. M t hng mc d liu DOD (cc DID)
QUAN L DU N PHN MM
148
T- liu pht trin
1. K hoch pht trin phn mm
Dn t- liu h thng
2. c t on/h thng
3. T- liu thit k on/h thng.
Thit k v yu cu phn mm
4. c t yu cu phn mm
5. T- liu thit k phn mm.
Thit k v yu cu giao din
6. c t yu cu giao din
7. T- liu thit k giao din.
M t phin bn
8. T- liu m t phi bn.
T- liu th nghim
9. K hoch th nghim phn mm
10. M t th nghim phn mm
11. Bo co th nghim
Pht hnh cc s tay
12. S tay ng- i vn hnh h thng my tnh
13. S tay ng- i dng phn mm
14. S tay ng- i lp trnh phn mm
15. S tay h tr phn sn (ch- ng trnh c s).
T- liu bo tr v m ngun.
16. T- liu h tr hp nht ngun my tnh
17. c t sn phm phn mm
Tiu chun 2167 khng nh khng c nh chi tit ha hay phn bc
vic s dng bt c ph- ng php pht trin phn mm c bit no (DOD
1988a). D sao, nh- nu tr- c y, tiu chun nng nghing v cc
ph- ng php lun pht trin theo pha, nh- h bin ha thc n- c. Tip
cn theo pha l c hu trong cc pha pht trin c yu cu v cc r sot
li, i hi vic thit k h thng - c tip ni nh nhng yu cu phn
mm, thit k, thc hin v th nghim phn mm. Tiu chun c m t
tip cn , - c minh ha hnh 8. 2.
Tiu chun cng nu cc ph- ng php lun thit k khc c th - c
dng cng vi quan nim chung 2167, nh- ly nguyn mu nhanh vy.
Nh- ng, nhiu ph- ng php lun khc nhau v c bn nh- m hnh xoy
c
36
khng d p dng cho tiu chun. D sao, Overmyer (1990) trong
bnh lun v 2167, kt lun l ni vic ly kch th- c y , 2167 c th
- c vn dng cho cc ph- ng php pht trin lp li. Vic ly kch
th- c 2167 - c bn n sau trong phn ny.
36
M hnh xon c m Boehm (1988) m t l m thc hi t lp li r rng kh thch nghi vi tiu
chun 2167. y l m thc c ch cho cc d n lc u khng - c xc nh .
QUAN L DU N PHN MM
149
8. 2. 2. R sot li v kim ton
R sot li v kim ton l mt trong nhng cng c kim tra ch yu
xy dng thnh tiu chun 2167. R sot li thnh cng th- ng l tiu
cho vic tin hnh sang pha pht trin sau. Theo th r sot li chnh thc
xc nhn s ph duyt ca khch hng v cng vic pht trin tr- c. R
sot li chnh thc cng th- ng xuyn km theo ct mc thanh ton ch
yu. iu ny tr thnh s kin gay cn c cho khch v ch thu.
Cc r sot li d n phn mm chnh v quan h gia cc r sot li
- c nu hnh 8. 3.
Bin bn r sot li chnh thc - c m t tiu chun MIL 1521 bao
gm:
- T- liu v hng mc phi duyt li
- Ch nh ch ta vic r sot li
- Hng mc phi trnh by v tho lun
- Th tc ph chun / khng ph chun
- Th tc tin hnh hiu chnh
R sot li d n c ni no quyt nh pht trin d n ch yu - c
thng qua ln cui. Nhng quyt nh ti hn - c dn trong t- liu
c t pht trin v - c coi l - ng gc
37
. Sau - ng gc tr thnh
nhng ngun tham kho s khi cho pht trin sau ny ca sn phm
phn mm c ba - ng gc ch yu.
- - ng gc chc nng - c t lc r sot li thit k h thng
thng qua ln cui cc yu cu chc nng ca h thng (ngha l quan
im ca ng- i dng v h thng).
- - ng gc phn b - c t lc r sot li c t phn mm thng
qua ln cui cc yu cu phn mm.
- - ng gc sn phm, - ng gc ny - c t lc kt thc cho k
pht trin v thng qua ln cui pht trin ca sn phm phn mm.
- ng gc trung gian ph c th - c t lc r sot li thit k ti
hn (CDR) thng qua ln cui thit k ca sn phm phn mm. Cc
- ng gc trung gian khc c th - c b sung xc nh kt thc cc
hot ng pht trin quan trng.
8. 2. 3 M t hnh mc d liu (cc DID).
Cc m t hng mc d liu xc nh tiu chun t- liu chnh thc m
t hnh mc d liu yu cu pht sinh trong lc pht trin phn mm theo
37
- ng gc - c bn thm ch- ng 9
QUAN L DU N PHN MM
150
Hnh 8. 2
Ph- ng php pht trin phn mm DOD 2167A
QUAN L DU N PHN MM
151
Hnh 8. 2
Ph- ng php pht trin phn mm DOD 2167A
QUAN L DU N PHN MM
152
Hnh 8. 3.
Th d r sot li v kim ton DOD (t DOD Std 2167A).
tiu chun 2167.Cc DID vn dng cho s pht trin ca mt hay nhiu
hng mc cu hnh h thng my tnh (CSCI), l thut ng - c dng
trong sut tiu chun 2167 nh r cc thnh phn phn gii mc cao
ca h thng my tnh. Mt CSCI, nh- - c p dng cho phn mm,l
thnh t ca h thng c th - c kim tra, cu hnh, th nghim v dn
QUAN L DU N PHN MM
153
t- liu ring bit cc CSCI th- ng xuyn - c duyt li v ph chun coi
nh hng mc pht trin ring r v mc d vic duyt li hay kim ton
n thun c th xem xt hn mt CSCI, mi mt li th- ng - c cp
ring r trong qu trnh duyt li. Khng c h- ng dn no tht s r
rng trong vic phn chia h thng phn mm thnh cc CSCI v s phn
chia ch yu l mt trong cc thun li nh- ng ni chung cc ph- ng
php s dng t- ng t nh- nhng k thut phn tch cp cao m t
ch- ng 6. Hnh 8. 4 gii thiu th d phn tch h thng thnh cc CSCI
v cc CSU cp thp, theo tiu chun 2167A.
Bng 8. 2 c danh mc cc DID tham chiu theo tiu chun 2167A,
cht l- ng phn mm DID - c tham chiu ring r trong tiu chun
2168 ch- ng trnh cht l- ng phn mm DOD.
Bng 8. 2. Tiu chun m t hng mc d liu DOD
Tn gi yu cu d liu B danh
1. T- liu thit k on h thng SSDD
2. K hoch pht trin phn mm SOP
3. c t yu cu phn mm SRS
4. c t yu cu giao din IRS
5. T- liu thit k giao din IDD
6. T- liu thit k phn mm SDD
7. c t sn phm phn mm SPS
8. T- liu m t phin bn VDD
9. K hoch th nghim phn mm STP
10. M t th nghim phn mm STD
11. Bo co th nghim phn mm STR
12. S tay vn hnh h thng my tnh CSOM
13. S tay ng- i dng phn mm SUM
14. S tay lp trnh phn mm SPM
15. S tay h tr phn sn FSM
16. T- liu h tr nht ngun my tnh CRISD
17. ngh thay i cng ngh ECP
18. Ghi ch thay i c t SCN
H thng
1 phn 1 phn
HWCI
HWCI
QUAN L DU N PHN MM
154
-
Hnh 8. 4
Mt th d v phn gii h thng DOD 2167 (t DOD- Std- 2167A)
Mi kch th- c t- liu DID theo cng khun mu t- ng t. Rt nhiu
phn cng chung nht nh- hu ht, nu khng phi tt c t- liu, v nh- :
- Kch th- c trang ba
- Bng mc lc
- Phm vi (k c nhn bit tng quan tham chiu v. v. . . )
- Cc t- liu khc vn dng - c
- Ghi ch v ph lc.
CSCI IRS HWCI
CSCI
HWCI
IRS HWCI HWCI HWCI
CSCI IRS
CSC CSC CSC
CSC CSC CSC CSC CSC CSC CSC
CSU
CSU CSU
CSU
CSU CSU
CSU
CSC CSC
CSU CSU CSC CSC
CSU CSU CSU
CSU
CSC CSC
CSU CSU
CSU
**
** Cng CSU do cc CSC khc nhau s dng
*
* Phn mm khng pht trin
**
*
QUAN L DU N PHN MM
155
Cng vy, kch th- c trang, s nh s trang, s nh s phn v
nhiu h- ng dn chun b khc l chung nht. iu ny r rng gi
vic s dng cng c t ng h tr trong vic chun b t- liu, mt thi
gian rt - c khuyn khch trong tiu chun 2167. Nhiu cng c nh- th
- c pht trin h tr 2167 v Polack, trong mt tham lun phn
tch vic s dng cc cng c CASE cho cc d n DOD (Polack 1990),
kt lun l nhng cng c ny thc t tit kim thi gian v dn n h
thng phn mm cht l- ng cao
38
.
Mi DID m t nhng yu cu trong vic chun b t- liu c th
nh- ng cn nhn mnh chnh n ni dung yu cu ch khng phi kch
th- c yu cu. iu ny c bit - c cp n trong vic chun b cc
h- ng dn km theo mi DID khng nh l cc kiu trnh by khc, k
c biu , bng hay ma trn, l chp nhn - c (th d Hartley v
Pirbhai 1988 - hay Ward v Mellor - 1986). Cng c s mm mng c
thc trong cc yu cu xt v ni dung t- liu. Tiu chun to cho vic
ly kch th- c nhiu, thch ng tiu chun vi loi d n - c pht trin.
8. 2. 4. Ly kch th- c tiu chun.
Ly kch th- c tiu chun 2167 khng ch l khuyn khch m cn l
yu cu. Li ni u cho 2167 khng nh tiu chun phi - c ng- i
qun l ch- ng trnh ly kch th- c thch hp m bo ch nhng yu cu
c hiu qu v kinh ph - c nu trong cc yu cu v hp ng quc
phng.
DOD xy dng h- ng dn ly kch th- c c th - c dng lm
ngun tham kho thch nghi cc tiu chun vi loi d n - c pht
trin
39
C hai nguyn tc c bn vn dng cho vic ly kch th- c:
- Qu tnh ly kch th- c l xa i nhng yu cu khng vn dng
- c.
- Ly kch th- c tiu chun phi do cng ty c hp ng tin hnh.
Nguyn tc u tin c ngha nhng sa i ch c th bao gm.
Vic xa b i cc yu cu khi tiu chun (v khng phi l nhng thay
i yu cu trong tiu chun). Nguyn tc th hai c ngha l ch thu
(ngha l ng- i sn xut) khng th ly kch th- c tiu chun m khng
- c php ca cng ty c hp ng (ngha l DOD).
Vic ly kch th- c tiu chun 2167 phi - c hon tt cng sm cng
tt. iu ny - c thc hin tt nht hoc trong cc h- ng thuyt hp
ng hoc l mt trong nhng sng kin ban u ngay khi d n bt u.
38
Teamwork - cng vic ng i- ca Cadre l mt th d v cng c CASE h tr tiu chun 2167
39
h- ng dn ly kch th- c c th tm thy DOD- HDBK 248. H- ng dn vn dng v ly kch
th- c yu cu trong vic thu thp vt liu quc phng
QUAN L DU N PHN MM
156
Sau y l m t th tc c bn trong vic ly kch th- c tiu chun
2167.
1. R sot li cc yu cu tiu chun 2167 bao gm:
- R sot li v kim ton
- Dn t- liu
- Hot ng th nghim
- Hot ng bo m cht l- ng
- Hot ng kim tra cu hnh
- Cc hot ng pht trin khc c yu cu.
2. Nhn bit cc yu cu khng vn dng, xc minh hay l gii - c
cho d n - c pht trin. Chng hn, s tay h tr phn sn, s khng
- c yu cu nu khng c phn sn - c pht trin hay hai r sot li
thit k (PDR v CDR) c th l khng cn thit cho mt d n nh.
3. Chun b danh mc yu cu xa b tiu chun. iu ny c th
bao gm:
- Loi tr t- liu
- Loi tr hot ng
- Loi tr cc on trong t- liu
- Loi tr cc phn hot ng.
4. Chun b m t vit cc chnh trang cho mi hng mc c yu cu
ly kch th- c.
5. xut yu cu ly kch th- c, cng vi chnh trang, cng sm
cng tt (nn tr- c khi d n bt u).
Nhm c kh nng phn ha gia cc hng mc b qun v cc hng
mc ly kch th- c, nn cc hng mc ly kch th- c phi - c c tham
chiu r rng. Khi trnh danh sch t- liu r sot li chnh thc hay
lm - ng mc, mi t- liu ly kch th- c phi - c ln danh sch
cng vi xc nhn hiu qa ca n. Trong phm vi mt t- liu, khi on
- c ly kch th- c ri, xc nhn v hiu qu s trc tip xut hin
sau s ca on. Nu mt on v mi on ph ca n - c ly kch
th- c, ch c s ca on cp cao nht cn - c - a vo.
Danh mc cc DID cng vi danh mc cc ph chun ly kch th- c l
mt b phn hp nht ca nhng th giao - c ca d n. Cho n khi
c - c tip cn ly kch th- c, ng- i sn xut buc phi cung cp danh
sch y cc DID cng vi mi tp cht ca chng. y l l do v sao
vic ly kch th- c phi - c kt thc cng sm cng tt.
8.2.5. Li v bt li ca tiu chun 2167.
Mt trong nhng khiu ni ph bin nht v 2167 l n to ra nhng
d n m sn phm chnh l giy ch khng phi phn mm. Ni cch
khc, mt khi l- ng khng l v thi gian v cng sc cn - c u t-
QUAN L DU N PHN MM
157
cho vic sn sinh t- liu. Sau , thi gian ph li cn u t- gi cho
cc t- liu th- ng xuyn - c cp nht.
Nhng li ph phn tiu chun (coi Polack 1990) bao gm khiu ni l
tiu chun ngn vic s dng thc hnh pht trin phn mm hin i nh-
ly nguyn mu nhanh v ti s dng phn mm. Nh- chng ta thy,
nhng ph- ng php ny c th - c vn dng cho 2167 mc d chng c
th thch hp mt cch t nhin vo tip cn chung ca tiu chun (coi
Oierwyer 1990).
Tiu chun khng d dng, vn dng - c cho cc d n qu nh v n
i hi ly kch th- c c bn nhm gim cng vic hnh chnh mc
tha ng. Mt khc, nhng d n rt ln li c th c li rt ln tiu
chun v n lm cho d n d qun l hn v cc hot ng pht trin r
rng hn.
Tiu chun 2167 cung cp cho khch hng tm nhn, ng k trong
mi pha pht trin ch yu. iu ny c th tng thm xc sut tha mn
ca khch hng v sn phm cui cng. D sao tt c mi iu c gi
ca n. Vic kim tra, dn t- liu v bo co i hi phi c nhng
ngun ch yu v nhng ngun ny lm tng kinh ph c gng pht trin.
Mt trong nhng li ch chnh ca 2167 l cng tc tiu chun ha
tuyt ho. N l mt trong nhng b tiu chun d hiu nht tn ti cho
vic pht trin phn mm v cung cp yu cu c tr- ng r rng kim
tra - c hu ht cc hot ng pht trin phn mm.
y khng phi l s la chn ca ng- i sn xut, v c t ng- i sn
xut sn sng chn 2167 u. y l s la chn ca khch hng v n
h tr kim tra qu trnh pht trin theo vin cnh ca khch hng. Theo
vin cnh ca ng- i sn xut, 2167 to ra mt b yu cu r rng loi b
- c nhiu iu m h v ti ngha dn n cc tranh chp gia ng- i
sn xut vi khch hng.
8.3. Cc tiu chun cng ngh phn mmIEEE.
Nm 1984, vin cc k s- in v in t (IEEE) cng b b tiu
chun cng ngh phn mm u tin ca mnh (IEEE 1988). B ny bao
gm bn tiu chun pht trin bao gm cc yu cu bo m cht l- ng,
th nghim v qun l cu hnh v t vng tiu chun th nm bao mong
i v cc t cng ngh phn mm.
B tiu chun u tin, tuy cn xa mi hon ho, nh- ng chnh thc
ha - c xu h- ng ti mm mng, cc tiu chun ging nh- h- ng
dn nhiu hn v c nhiu th d v gi . Nhng k xut bn sau ca cc
tiu chun bao gm cc lnh vc khc ca cng ngh phn mm nh- thit
k, kim tra v hp thc ha (V v V) v tiu chun c th cho pht
trin phn mm Ada. V s l- ng tiu chun bt u tng, IEEE cng
li - a ra tiu chun v cc tiu chun phn mm - c s dng lm
ph- ng php lp k hoch pht trin v nh ga cc tiu chun.
QUAN L DU N PHN MM
158
8.3.1. Tng quan cc tiu chun IEEE.
Nm 1987, IEEE xut bn mt b cc tiu chun pht trin phn mm
bao gm:
1. ANSI/ IEEE Std 729- 1983
T vng tiu chun IEEE v thut ng cng ngh phn mm
2. ANSI / IEEE Std 730- 1984
Tiu chun IEEE v cc k hoch bo m cht l- ng phn mm
3. ANSI / IEEE Std 828 - 1983
Tiu chun IEEE v k hoch qun l cu hnh phn mm
4. ANSI / IEEE Std 829 - 1983
Tiu chun IEEE v t- liu th nghim phn mm
5. ANSI / IEEE Std 830 - 1984
H- ng dn IEEE v cc c im yu cu phn mm
6. ANSI / IEEE Std 983- 1986
H- ng dn IEEE v qui hoch bo m cht l- ng phn mm
7. IEEE Std 990 - 1986
Thc hnh IEEE v Ada coi nh- ngn ng thit k ch- ng trnh
8. IEEE Std 1002 - 1987
Nhn loi hc tiu chun IEEE v cc tiu chun cng ngh phn mm
9. ANSI / IEEE Std 1088 - 1987
Tiu chun IEEE v th nghim n v phn mm
10. ANSI / IEEE. . . 1912 - 1986
Tiu chun IEEE v cc k hoch kim tra v hiu lc phn mm
11. IEEE Std 1016 - 1987
Thc hnh IEEE v m t thit k phn mm.
Vo lc cng b 11 tiu chun , IEEE c 12 d n khc tin trin
bao gm cc tiu chun v r sot li v kim ton phn mm, cc k
hoch qun l d n phn mm v cc k hoch bo tr phn mm cng
nh- cc h- ng dn cng ngh phn mm
40
.
r rt ca cc tiu chun IEEE l chuyn ph- ng php lun pht
trin cho ng- i sn xut. Cc tiu chun d khp vi tip cn thc n- c
tng pha mc d cng c th - c thch nghi vi cc ph- ng php khc
nh- ly nguyn mu nhanh v m thc xon c. Tiu chun 930 ca cc
yu cu phn mm l mt th d tuyt vi v t- liu tin trin tt v- t qu
chc nng c bn ca tiu chun. iu ni ln h- ng dn ng- i
dng v vic pht trin "c tr- ng yu cu phn mm tt". Chng hn
tiu chun lit k cc c im c bn ca mt SSS tt nh- :
- Khng m h
- Sa - c
- y
- Theo di - c
40
Jolm Horch trong m u bn in 1987 v cc tiu chun IEEE (IEEE 1987b) bo co l IEEE
ang trong tin trnh chuyn t- liu chun v thc hnh h- ng dn.
QUAN L DU N PHN MM
159
- Th - c
- Nht qun
- S dng - c trong pha vn hnh v bo d- ng.
Sau , danh mc ny c th - c dng lm danh mc kim tra
41

nh gi nhng c im yu cu theo cch sau:


1. Liu mi yu cu c r rng v c cng cch l gii cho mi ng- i
c n ? (khng m h).
2. Liu mi yu cu c t- liu y , liu chng ta m bo khng
tn ti mi vic hiu "ming" (thuyt khu) ? (y )
3. Liu chng ta c th chng minh mt cch hp l vi chi ph hp l
l mi yu cu - c p ng (th - c) ?
4. Liu c yu cu no mu thun vi yu cu khc ? (nht qun)
5. Liu c im yu cu - c dn t- liu theo cch lm cho d
dng hiu chnh hay thay i sau ny - c khng ? (sa - c).
6. Liu nhng nguyn nhn ca mi yu cu c r rng (theo di
ng- c li) v t- liu thit k v th nghim sau ny c th theo di cc
yu cu? (theo di tip din).
7. Liu c im yu cu - c vit sao cho khng ch t chc vit
ra n hiu - c m c t chc bo tr phn mm na (s dng - c).
Cc tiu chun khc ca IEEE cng bao gm nhiu h- ng dn v th
d. Tiu chun thit k phn mm - c m t nh- l mt b cc thc
hnh nn lm trong vic m t thit k. Tiu chun bao gm mt phc
mu cho t- liu c im thit k cng nh- nhng gp v ni dung mi
mc.
Vic th nghim c - c s ch ng k trong cc tiu chun
IEEE. Tiu chun 829 bao gm vic chun b t- liu th nghim tiu
chun 1008 bao gm vic th nghim n v phn mm v tiu chun
1002 bao gm kim tra v hiu lc sut chu k pht trin. Tiu chun
1012 m t tuyt vi bao gm cc hot ng th nghim pht trin d n.
iu ny - c m t trong cc hnh 8. 5 v 8. 6.
Tiu chun 990 l b- c khi u khng bnh th- ng t tnh khi qut
v cp n ngn ng lp trnh c th, Ada. Vi nhng ai quen thuc
vi Ada, l do hu thun cho iu ny s r rng v Ada hn l mt ngn
ng lp trnh. Hin t- ng ngn ng Ada - c bn n phn 8. 4.
Phn loi hc cc tiu chun cng ngh phn mm cng l mt hin
t- ng khng bnh th- ng v l th, xng - c quan tm c bit.
41
Mt c im thc ra r rng hn v thc hin - c hm l phi c th thc hin - c mi yu
cu trong phm vi rng buc ca d n. Nhng yu cu khng thc hin - c c th do nhng mc
khng th t ca tin cy v hon thin hay giao din bt kh gia cc thnh t khng so snh
- c.
QUAN L DU N PHN MM
160
8.3.2. Phn loi hc IEEE cho cc tiu chun cng ngh
phn mm.
Tiu chun 1002, phn loi hc cc tiu chun cng ngh phn mm l
mt hin t- ng l th ch nhm cung cp tiu chun la chn, phn
loi v so snh cc tiu chun phn mm khc. Mc ch ca n cng l
gup nhn bit nhu cu cho cc tiu chun u khng c hay u
khng c ci thch hp. Tiu chun cng c danh sch cc nh ngha bao
gm nhiu loi tiu chun pht trin phn mm v thut ng - c s
dng nhn bit chng
42
.
Phn loi hc l tham kho tuyt vi hiu r - c cc tiu chun
pht trin phn mm v quan h gia cc tiu chun. Phn loi hc to
nn s phn chia cc tiu chun phn mm theo loi v s phn chia cng
ngh phn mm theo chc nng v chu k i. hon chnh phn loi
hc, khung m t quan h gia hai phn chia.
Cc hnh 8.7a v 8.7b m t s phn chia cc tiu chun theo loi v s
phn chia cng ngh phn mm theo chc nng v chu k i. Hnh 8.8
l th d v li bng s dng xc nh khun kh phn loi c bn.
Tiu chun 1002 c th c ch trong nhng t chc xem xt chuyn
sang pht trin phn mm chnh thc hay khi kh nng thch ng ca cc
thc hnh chnh thc hin c uc nh gi li. Tiu chun khng ph
thuc cc tiu chun IEEE khc v c th - c p dng cho bt c tiu
chun pht trin phn mm chnh thc no khc.
8.3.3 Li v bt li ca cc tiu chun IEEE.
Cc tiu chun cng ngh phn mm IEEE l mt b tiu chun,
h- ng dn v gp mm do c th p dng cho nhiu ph- ng php
lun khc nhau. Cc tiu chun t- ng i d s dng v c nhiu th
d chng minh cch m cc tiu chun c nh - c s dng.
Mc d mi tiu chun cng ngh phn mm u so snh - c cc
tiu chun - c pht trin sao mi tiu chun c th - c s dng
ring bit. iu ny c nghi mi tiu chun IEEE c th - c s
dng m khng cn phi s dng tiu chun.
42
Tiu chun IEEE 1002 bao gm nh ngha t phn loi hc l "s phn chia khung kin thc
v nh ngha quan h gia cc phn. S - c s dng phn loi v hiu thm kin thc".
QUAN L DU N PHN MM
161
Hnh 8. 5
Cc hot ng th nghim
QUAN L DU N PHN MM
162
Hnh 8. 6
Cc hot ng th nghim
QUAN L DU N PHN MM
163
a) Phn chia tiu chun.
Cc chun tin trnh
- Ph- ng php
- K thut
- o l- ng
b) Phn chia cng ngh phn
mm
Chc nng ngh nghip
Cc tiu chun sn phm
- Yu c
- Thit k
- Thnh t
- M t
- K hoch
- Bo co
Chc nng cng ngh sn phm
Phn tch yu cu
Thit k
Lp m
Hp nht
Chuyn i
G ri
H tr sn phm
Bo tr phn mm
Cc tiu chun chuyn mn
Chc danh ngh nghip
M o c
Bng cp
L lch
Kim tra v hiu lc
R sot li v kim ton
Phn tch sn phm
Th nghim
Cc tiu chun ghi ch
Danh mc
i din
Ngn ng
Chc nng qun l k thut
Qun l qu trnh
Qun l sn phm
Qun l ngun
Vng i
Pha quan nim
Pha yu cu
Pha thit k
Pha thc hin
Pha th nghim
Pha nh tnh
Pha sn xut
Pha lp t v kim tra
Pha vn hnh v bo tr
Pha ngh h- u.
Hnh 8. 7
(a) Phn hoch cc chun theo kiu
(b) Phn hoch cng trnh phn mm theo chc nng v vng
i
Theo IEEE Std 1002-1987
Kiu
tiu chun
QUAN L DU N PHN MM
164
Chun
tin
trnh
Chun
sn
phm
Chun
chuyn
nghip
Chun
quc
gia
*
Kim
nghim
v
Thm
nh
R sot
v
kim ton
Phn tch
sn phm
Th nghim
Qun
l k
thut
Qun l
tin trnh
Qun l
sn phm
Qun l
ngun lc
**
nim
Yu cu
Thit k
Thc thi
Th nghim
Ch to
Vn hnh v bo tr
Ngh vic
(*) Chc nng cng vic
(**) Vng i phn mm
Hnh 8. 8
Bng khung phn loi c bn (trch IEEE Std. 1002. 1987 IEEE
Standard taxonomy for sopftware Engineering standards ca Vin k
s- in v in t vi s ng ca IEEE)
D sao nhiu li im ca tiu chun cng c th - c nhn thc
l bt li, chng cho ng- i thc hin - c t do rt nhiu khin
cho, v li ch ca tiu chun ho, mi t chc phi xc nh ci cch
m h phi thc hin.
Chc hn mt trong nhng bt li chnh ca tiu chun l ch
chng khng thc s cung cp mt b tng qut duy nht cc tiu
chun pht trin phn mm. Chng i hi mt hnh thc ph- ng
h- ng hay tiu chun bc thy phi hp cc tiu chun ring l vo
mt ton th. D sao, iu ny khng kh thc hin do tnh t- ng hp
QUAN L DU N PHN MM
165
gia cc tiu chun v kch th- c t- ng i chung nht - c s dng
xc nh chng.
8. 3. 4. So snh cc tiu chun IEEE v DOD.
Cc tiu chun IEEE v DOD - c cn c cch l gii t- ng t v
cng ngh phn mm ch thut ng t- ng t v tip cn chung
t- ng t. Tuy nhin mc tiu li khng t- ng t. Tiu chun DOD
2107 - c xy dng h tr li ch ca khch hng (DOD) trong
khi cc tiu chun IEEE - c xy dng vi li ch c ca khch
hng v ng- i sn xut cng nh- nhau trong tm tr. Sau y l tm
tt ca nhng khc nhau chnh gia hai tiu chun:
1. Tiu chun DOD hu ht l mt b ph- ng h- ng trong khi nhiu
nhng tiu chun IEEE trn thc t l nhng h- ng dn v gp .
2. Tiu chun DOD cung cp mt b tng qut cc ph- ng h- ng
trong khi IEEE l mt b cc tiu chun ring bit.
3. Cc tiu chun IEEE mm mng hn v to ra mc t do rng
ln hn nhng tiu chun DOD.
4. C tiu chun IEEE u c th ly kch th- c theo yu cu ca d
n c th. Nh- ng cc tiu chun IEEE li i xa hn khi vic chp
nhn tiu chun ring bit khng i hi chp nhn tiu chun no
khc.
5. C tiu chun IEEE v DOD u nghing v tip cn theo pha vi
cng ngh phn mm. Mc d c hai u c th h tr cc ph- ng
php pht trin khc. IEEE d thch nghi vi cc tip cn lp li
hn nhiu.
8. 4. cc Tiu chun ADA
Ada l ngn ng lp trnh nh- ng l ngn ng lp trnh duy nht.
N c mt trong nhng nh tiu th ln nht cc dch v pht trin
phn mm ln nht trn th gii. B quc phng Hoa k. ADA l tiu
chun qun s, k hiu l ANSI/MIL- STD- 1815a. Ada cng n vi
h- ng v c bit chu u v trn thc t, n - c chp nhn vo u
nhng nm 80 l ngn ng thc hin chung ca cng ng kinh t
chu u cng ng chu u cng tham gia tch cc vo vic thit k
ngn ng v r sot li Ada v nhiu d n ln chu u cng sau
- c xy dng bng Ada.
Ngy nay Ada - c u nhim cho mi phn mm xy dng cho US
DOD cho n khi b t b. Thnh cng ca Ada vn cn ang tranh
ci v s ng nhng kh- c t - c ginh cho DOD l mt biu
hin nhng kh khn m ngn ng ang th nghim trong vic - c
- c nghim thu. Ging nh- tiu chun 2167a, Ada l s la chn
ca khch hng ch khng phi ca ng- i sn xut. C nhiu ng- i
sn xut hn ca ngi Ada nh- ng cng li c nhiu ng- i hn
QUAN L DU N PHN MM
166
nguyn ra Ada ngn ng lc u - c s dng cho nhng ng dng
thi gian khng thc v c nhng vn chuyn nhim v chm vi
nhng b bin son Ada u tin. Nhng vn ny bin i khi c
- c nhng b bin son Ada tinh xo hn v Ada gi y - c dng
x l d liu pht trin h thng v ng dng thi gian thc.
8. 4. 1. Mi tr- ng Ada
Ada cng - c dng lm cng c thit k lc khi u Ada - c
coi l mt ngn ng mi nhiu hn. Cu trc ca ngn ng lm cho
n thch hp dng lm ngn ng thit k ch- ng trnh (PDL). c
im ca Ada khng phi l ngu nhin khi DOD t ch- ng trnh
ca cc mi tr- ng h tr Ada l mc tiu song hnh vi s pht trin
ca bn thn ngn ng. Cc yu cu v ngn ng Ada - c xc nh
trong vn bn ca Stulm v mt vn bn th hai, vn bn Steelman
43
xc nh cc yu cu cho cc mi tr- ng h tr.
Nhiu b bin son Ada ngy nay - c cung cp nh- mt b phn
nguyn th ca mi tr- ng pht trin Ada ba thn b bin son, iu
ny bao gm cc cng c CASE sn sinh t- liu, thit k phm mm,
g ri v ho nhp v th nghim
44
.
8. 4. 2. Tiu chun IEEE cho cc Ada PDL.
IEEE chn Ada l cng c thit k phn mm xng - c quan
tm c bit. IEEE cng b tiu chun 990 nm 1987 xc nh mt
b qui trnh k thut nu theo trong vic s dng Ada lm PDL (ngn
ng thit k ch- ng trnh).
Tiu chun 990 lin quan n c im PDL ca Ada ch khng
phi s dng Ada lm PDL. iu ny l do c gng ca tiu chun
trong vic khi qut ho bng cch cp bt c PDL no da trn c
php v ng ngha Ada (theo th tiu chun sng to ra t Ada PDL).
Tiu chun IEEE 990 chnh thc ho qui trnh k thut tt nhin
pht trin vi nhng ng- i dng Ada. Tiu chun m t c im yu
cu ca Ada PDL hu ht c th sn sng p dng cho mi PDL tt.
Nhng c im yu cu bao gm vic h tr nhiu ph- ng php
lun thit k v mt s qui trnh thit k tt nh- kh nng iu bin
nh- n du thng tin v lin kt thng tin. Tiu chun c th - c
dng lm phiu kim tra nh gi cc PDL v c bit hn n c th
c ch trong vic hiu Ada c th - c s dng tt nht lm PDL nh-
th no.
43
DOD cng b t- liu Stoneman nm 1976 xc nh yu cu cho cc mi tr- ng h tr lp
trnh Ada v t- liu Steelman nm 1978 xc nh yu cu cho cc ngn ng lp trnh my tnh
lnh cao dn n nn mng cho ngn ng Ada (- c gi theo tn Augusta Ada Byron - c coi
l nh lp trnh my tnh u tin v con gi ca thi ho Anh, b t- c Byron).
44
Tahvanainen v Smolander (1990) l ngun thng tin c ch v cc cng c thit k, m
phng, lp m v cc lnh vc khc ca pht trin phn mm.
QUAN L DU N PHN MM
167
Tiu chun IEEE v Ada PDL cng dn t- liu v mt hin t- ng
tt nhin do nhng ng- i dng Ada pht trin. L ngn ng lp trnh.
Ada l PDL d - c vn dng cho pha thit k ca pht trin phn
mm v khi - c dng nh- th, n to ra li th ph r rt. Sau khi
thit k hon thnh mt l- ng ch yu ca m - c vit ra. iu
ny dn n vic pht trin mt s cng c Ada CASE to ra m
Ada.
8. 5. Cc tiu chun pht trin phn mmkhc.
Mt trong nhng tiu chun sm cho t- liu phn mm - c vn
phng ton quc Hoa k v tiu chun cng b nm 1976. Tiu
chun, gi l h- ng dn t- liu ch- ng trnh my tnh v cc h d
liu t ng - c cng b coi nh- n phm lin bang cc tiu chun
x l thng tin. Vic cng b tiu chun trng vi khi im khim
tn ca tin trin phn mm coi nh- b mn cng ngh. Vn phng
tiu chun cng b tip nm tiu chun phn mm na khong 1979
v 1984 bao gm:
H- ng dn v t- liu pha ban u.
H- ng dn nh gi cng c pht trin phn mm.
H- ng dn kim tra hp thc ho v th nghim phn mm.
H- ng dn bo tr phn mm.
Coi nh- c gng b- c u trong vic cung cp tiu chun ton
quc cc tiu chun ny l mt b- c tin ln ch yu. Nhng tiu
chun sm sa khc - c ANSI v nhiu ngnh qun s Hoa k cng
b trong nhng nm u 70.
O Chu u tiu chun Anh BS 6224 - c cng b vo nhng nm
1980 v t chc quc t chu u v tiu chun ho (ISO) cng
b cc tiu chun phn mm vo cui nhng nm 80.
Rt nhiu tiu chun tr thnh cng c hay k thut c hiu. Tiu
chun anh 6224 c k thut thit k l th - c coi l s cu trc
thit k (BS 6224 DSD) DSD l biu din bng th thit k phn
mm (ta nh- biu dng chy). Khuyn khch tip cn theo c cu
n qu trnh thit k k thut DSD c bn - c m t tuyt vi trong
chnh vn bn tiu chun
45
.
C nhiu k thut khng - c chp nhn rng ri. iu ny
dn n tnh trng phong ph v ph- ng php, tiu chun k thut.
Chng hn, vic bo m cht l- ng phn mm - c ch n
nhiu c trong tiu chun IEEE v DOD. Tiu chun chu u ISO
9000- 3 nm 1990 (ISO1990) gii thiu tip cn c bn c i cht
khc. Tiu chun chu u - a ra ngha rng hn v bo m cht
l- ng phn mm v bao gm:
45
k thut DSD - c Macro v Buxton cp nm 1987
QUAN L DU N PHN MM
168
+ Trch nhim qun l.
+ Kiu ton v r sot li.
+ Th nghim.
+ Kim tra cu hnh.
+ o to.
Nhiu nhng tiu chun khc - c to lp c chu u v Hoa
k. La chn tiu chun thch hp, bn thn n l mt hot ng d
n ch yu. Sai lm trong vic s dng tiu chun pht trin khng
phi phn mm hu ht l ln trong vic tm cch vn dng ton b
DOD Std. 2167A cho mt d n x l d liu nh.
Chc chn im quan trng hn c phi nh khi la chn tiu
chun thch hp cho d n phn mm l ch tiu chun pht trin l
ph- ng tin ch khng phi cu cnh tiu chun phi h tr thc hin
mc tiu: pht trin thnh cng phn mm c cht l- ng. Nu tiu
chun can thip vo vic hon thnh mc tiu ny th chc hn l
hoc tiu chun sai hoc - c vn dng khng ng.
Cc tiu chun thnh cng nht khi chy theo dng. Bt k khi no
c th - c tiu chun to ra khung hp thc cho cc qui trnh k
thut tt tn ti. Khi cc qui trnh hin c l v hiu, tiu chun
phi h tr thay th chng bng nhng qui trnh c th d dng - c
thc hin. Tht s kh m - a ra - c mt tiu chun p ng - c
s chng tr ca ng- i sn xut.
8. 6. Tmtt.
Tiu chun c th - c coi l tai ho cn thit v vic ng dng t
- c kt qu xng ng. N lm cho vic pht trin phn mm d
qun l hn. Mt trong nhng thch chnh cho ng- i qun l d
n l vic la chn b tiu chun ng ch- ng ny cung cp thng tin
c bn cn thit gip ng- i qun l d n tin hnh s la chn
.
Loi v bin thc ca cc tiu chun pht trin d n c nhiu v
khng c ch dn no l tiu chun tr thnh tiu chun c. Nhiu tiu
chun phn mm c - u th - c tho mn min l chng - c vn
dng ng. Cc tiu chun i hi k lut v k lut khng d p
t trong cng ngh phn mm.
Tiu chun 2167 ca B quc phng Hoa k gii quyt vn k
lut bng cch yu cu vn dng tiu chun nh- l b phn ca hp
ng d n nh nhm cho 2167 tr thnh tiu chun duy nht
chnh thc cho s pht trin phn mm h thng quc phng cho gii
qun s Hoa k.
B tiu chun t cng nhc hn 2167 do IEEE xy dng. Cc tiu
chun cng ngh phn mm ca IEEE l mt b tiu chun h- ng
QUAN L DU N PHN MM
169
dn v gp mm mng c th - c p dng cho nhiu loi ph- ng
php lun pht trin phn mm. Cc tiu chun t- ng i d s dng,
k c nhiu th d chng minh con - ng m cc tiu chun c nh
s dng.
C tiu chun IEEE v DOD - c cn c gii thch t- ng t v
cng ngh phn mm ch thut ng t- ng t v tip cn chung
t- ng t nh- th no. Tuy nhin mc tiu li khng t- ng t, Tiu
chun DOD 2167 - c xy dng h tr quyn li ca khch hng
(DOD) trong khi cc tiu chun IEEE - c xy dng v quyn li c
ca khch hng v ng- i sn xut nh- nhau trong tm tr.
Cc tiu chun - c xy dng phong ph Chu u v Hoa
k lm cho vic la chn tiu chun thch hp t thn l mt hot
ng ch yu ca d n.
Tiu chun pht trin phn mm l ph- ng tin ch khng phi cu
cnh. Tiu chun phi h tr thc hin mc tiu c bn: Pht trin
thnh cng phn mm c cht l- ng. Nu tiu chun can thip vo
vic thc hin mc tiu ny th hu nh- chc chn l hoc tiu chun
sai hoc khng - c vn dng ng.
Bi tp
1. DOD us hp ng vi bn pht trin phn mm cho tin ch ti
- u ho kim k. H thng s nhn - c thng tin v cc hng mc
qun s c tr- ng, s l- ng yu cu cho mi hng mc a im
kh nng tiu th d kin ca mi hng mc. Sau h thng s
- a ra cc mc kim k gp cho mi hng mc vi a im cho
mi kho kim k
H thng - c xy dng ph hp vi tiu chun 2167. Hy chun
b mt danh mc 10 yu cu ly kch th- c ch yu cho tiu chun
v gii thch l do cho mi yu cu.
2. So snh cc tiu chun IEEE v DOD lin quan vi d n m t
trong bi tp 1. Gii thch xem tiu chun no bn ngh l thch
hp hn c,
3. Chun b phc v c im yu cu cho d n m t trong bi
tp 1. Da trn h- ng dn IEEE cho c im yu cu tt, hy cho
th d v mi mt trong 7 c im m t.
4. M t khung lin quan n cc tiu chun DOD cho cng ngh
phn mm. S dng bng xc nh trong tiu chun 1002 phn loi
cng ngh phn mm IEEE. Cng xc nh khun kh cho cc
tiu chun IEEE.
5. Hy xt tnh thch nghi ca ngn ng lp trnh mc cao m bn
quen thuc s dng lm ngn ng thit k ch- ng trnh. Hy
bnh lun li v bt li ca ngn ng bn chn lm PDL
QUAN L DU N PHN MM
170
Ch- ng chn
lp lch d n
QUAN L DU N PHN MM
171
Lp lch: Vn .
Mi d n u c th hon thnh nu c - c mt l- ng thi gian v
ngun vn v hn cho. V thc tin m xt l- ng thi gian c pht
trin d n bao gi cng c hn. Trn thc t trong hu ht tr- ng hp n
li t hn l- ng thi gian m ng- i qun l d n coi l . C t d n
hon thnh tr- c thi hn: nhiu d n v- t qu lch ca chng.
Nhng chuyn c tht sau y nhn mnh n mt vi yu t chnh
trong vic lp lch.
Vo cui nhng nm 70, mt ch thu quc phng ln tr s
California pht trin mt h thng phng khng phc hp cho qun s.
Sau khi giai on hp nht bt u v trong thi k sau khong su thng,
mi c gng - c h- ng v th nghim h thng ch yu u tin, th
nghim ny - c tin hnh cng vi ch thu ph b bin pha ng.
Ch thu ph - c giao nhim v - a mt my bay ln khng trung,
my bay ny c th lin lc vi h thng my tnh ca ch thu ph b
bin pha ng v ri h thng ny li lin lc vi a im th nghim
chnh ca ch thu California. Nhiu chc nng h thng c bn
- c th thch trong th nghim h thng tn km . Khi thi gian th
nghim n gn, mi thnh vin ca i lm vic cc k cng thng
nhm m bo cho th nghim thnh cng. Vo ngy ch nh cho th
nghim, tt c nhn vin ca d n n ni lm vic t sng sm m
bo cho thit b hot ng ng n. Ch thu ph b bin ng cho
my bay ln v h thng hot ng
Th nghim tht bi. Khng ch th nghim tht bi m trn thc t
n chng bao gi bt u c. L do l khng ai nh n vic ra lnh cho
cc tuyn lin lc t cng ty in thoi gia im ch thu ph b bin
ng v im th nghim California. Mi c gng trong qun l cng
ty in thoi lp - ng dy trong ngy th nghim tht bi ch v
khng thi gian. Tht bi tng cng ca th nghim h thng tn
km nng cho ch thu v chc hn mt s nh qun l mt vic ca
mnh.
Giai thoi ny nhn mnh n nhiu vn ca vic qui hoch v lp
lch d n. Nhng sai lm c th xy ra v sc p thi gian do lp lch
khng thc tin hay v b tr nhn vin km trong phm vi t chc d n.
D n hu ht lun - c pht trin d- i sc p thi gian. C sc p
t - c tc v sc p cho chng ta xem ci g ang xy ra v tt
nhin c sc p i thu - c h thng lp t xong v chy. Chc hn sc
p ph bin nht lin quan n thi gian l get that darned bug fixed
quickly!. Loi sc p ny c th gy s tn ph k hoch pht trin d
n, lm cho ng- i qun l d n tm cch xoay s nhanh v bn thu hay
ct gim thng thng. V ct gim thng thng dn n loi sai lm tn
km - c m t trn.
Ch- ng ny bn n cc ph- ng php chun b lp lch thc tin m
cc nh qun l d n v i ng pht trin c th tham gia tho ng.
QUAN L DU N PHN MM
172
Tt nhin chun b ln lch thc tin khng ch l mc tiu, m quan
trng l c lch - c chp thun. Vic c - c s chp thun lch tho
dng v cc vn lin quan cng - c bn n.
D lch d n - c chun b tt th no chng na, th lch cng l v
dng nu n khng - c thc hin. y l trch nhim ca ng- i qun l
d n chng chi - c sc p v m bo l d n - c pht trin mt
cch trt t theo ng lch. Bt k khi no tnh hung thay i, khi
lch ca d n phi theo - c cp nht phn nh - c tnh hnh
mi. Lch ca d n l mt trong nhng phn quan trng nht ca k
hoch pht trin d n. K hoch khng ch bao gm ln lch cc hot
ng pht trin m cn ln lch cc ngun vn ca d n, c bit l con
ng- i. Ch- ng ny bn n vic ln lch trong bi cnh k hoch pht
trin d n.
9.1 K hoch pht trin d n.
K hoch pht trin d n l mt trong nhng vn bn hp thc u
tin do d n to ra. Trong vn bn ny, ng- i qun l d n m t chi
tit:
D n s - c pht trin th no
Cn c nhng ngun vn no
Nhng ngun vn - c s dng ra sao
K hoch pht trin d n m bo rng vic pht trin ca d n - c
ln biu r rng tr- c khi cc hot ng pht trin chnh bt u. Ngoi
lch pht trin c bn, k hoch cn cp n nhng vn nh- :
+ Cung cp kp thi thit b v cng c cho ng- i sn xut khi cn
n.
+ C - c i ng thc hin cc nhim v pht trin ph hp vi lch.
+ Cung cp k hoch t xut phng tr- ng hp ri ro ca d n
thnh hin thc.
+ Ch nh nhim v trong i ng pht trin v giao nhng nhim v
cho cc thnh vin i.
Ni dung ca k hoch pht trin d n c th thch nghi vi qui m
d n, c th l vn bn ln hay ng ch vi trang. Bng 9.1 gii thiu
phc v mt s ch nm trong k hoch pht trin d n.
Khng phi tt c ch trong bng 9.1 l vn dng - c cho mi d
n. chng hn, nhiu d n khng th qun l ngn sch ca chnh mnh.
Mt s t chc li c quan chc ti chnh chu trch nhim qun l ngn
sch d n. Giao din vi cc ngun bn ngoi l mt lnh vc khc
khng p dng - c cho mi d n. Thut ng ngun bn ngoi bao gm
QUAN L DU N PHN MM
173
cc hot ng nh- giao din vi ch thu ph, ng- i bn v i din ca
khch hng.
Bng 9.1 Cc hng mc k hoch pht trin d n phn mm.
1) Tng quan h thng
2) Qun l pht trin phn mm: T chc d n v ngun lc d n,
Ph- ng tin pht trin, C cu t chc d n, Nhn s
3) Lch v ct mc: Hot ng theo lch, - ng mc v ct mc, S
mng hot ng, Ngun lc thnh t h thng, Qun l ngn sch:
thanh ton ct mc, chi phi ngn sch ch yu, th tc cho php chi ph.
4) Phn tch ri ro
5) An ton
6) Giao din vi cc ngun bn ngoi
7) Th tc r sot li hp thc
8) Qu trnh tin hnh hiu chnh
9) Bo co v cc thay i/vn .
10) Cng ngh phn mm: Tiu chun v th tc, Ph- ng php lun
pht trin, Ngun lc pht trin, Nhn s - trnh v chc trch
11) Th tc th nghim
12) Qun l cu hnh phn mm
13) Bo m cht l- ng phn mm.
Nhiu tiu chun - c to ra cho k hoch pht trin d n. C cu
hp thc ca nm bi k hoch pht trin d n khc nhau tu thuc
tiu chun t- liu hin s dng chng hn tiu chun 2167 US DOD cung
cp kh nng tu chn m t cc k hoch th nghimu qun l cu hnh
v m bo cht l- ng trong ba vn bn ring bit. Vi nhng d n ln,
kh nng tu chn c th tr thnh yu cu.
Tiu chun IEEE 1058.1 m t ci - c coi l k hoch qun l d n
phn mm ch yu cng y nh- k hoch pht trin d n. Tiu chun ny
t- ng hp nhiu vi k hoch pht trin d n 2167 mc d r rng t chi
tit hn. Tiu chun ny li cng cung cp kh nng tu chn gp cc k
hoch qun l cu hnh v bo m cht l- ng hay m t chng trong cc
vn bn ring bit.
K hoch pht trin d n - c chun b l vn bn t lp theo ngha
n c - c v hiu - c khng cn phi xt ti cc vn bn khc. Do
y tng quan chung ca d n thng th- ng - c gp trong phn u
ca vn bn. Tt nhin tham chiu v chi tit ph phi lun c, k c ch
dn v nhng vn bn nh- hp ng d n, vn bn quan nim hay phn
tch, nghin cu th tr- ng.
Phn qun l pht trin phn mm, m n m t t chc v ngun lc
s - c s dng pht trin sn phm phi lun - c tnh vo phn
QUAN L DU N PHN MM
174
qun l xt. Xem cc ph- ng tin s - c t chc th no h tr c
gng pht trin. y l mt trong nhng phn cung cp nhiu chi tit cn
chun b ct li ca k hoch pht trin c th lch pht trin. Lch
cung cp cu tr li cho hai cu hi qui hoch c bn ci g v khi
no, trong khi phn ln nhng phn cn li cp nh- th no.
Vic bn lun trong cc phn nh- th no cung cp thng tin v d
n s - c t chc th no, nhng ri ro s - c x l th no, nhng r
sot li - c tin hnh th no, nhng tiu chun s - c vn dng th
no, nhng ph- ng php lun pht trin no s - c s dng v sn
phm no s - c th nghim.
Thng th- ng tt nht l cho vic hon thnh phn lch (phn 3
trong bng 9.1) cui cng lch tu thuc hu ht cc phn khc l phn
nhy cm nht ca k hoch pht trin. Sau khi bn d tho u ca k
hoch pht trin sn sng lch pht trin ban u c th theo - c
chun b. Nh- chng ta s thy lch theo - c tinh ch thm khi k
hoch pht trin i qua nhng ln lp li tun t
9.2 Hot ng v ct mc theo lch
Lch pht trin ca d n l danh mc cc hot ng v thi gian tin
liu thc hin c nhiu cch biu th lch: danh mc hot ng biu ,
th v.v. Ph- ng php thng th- ng nht ca biu din lch l biu
mng PERT biu GANTT v danh mc ct mc (nhng ph- ng php
ny s - c m t sau). Mi ph- ng php biu din lch ch yu phi
cung cp cng thng tin c bn hot ng v thi gian thc hin. Do y
b- c u trong vic chun b lch l xc nh cc hot ng d n. Nh-
chng ta thy lch pht trin d n l mt trong nhng yu t quan
trng nht ca k hoch pht trin d n v k hoch ny - c yu cu
nhng giai on u ca d n. Bt hnh l danh mc y cc hot
ng li th- ng khng c - c cho n khi hon ton vo giai on thit
k. Do y phin bn ban u ca lch d n th- ng bt u vi danh
sch hot ng mc cao v lch ban u ny tip tc - c tinh ch li khi
c - c nhiu thng tin hn. Ch- ng 6 c bn n y cc ph- ng
php tinh ch danh mc hot ng cng vic.
Lch ban u c th ch c nhng pha c bn ca chu k pht trin
phn mm (c t yu cu, thit k, thc hin.v.v.) phin bn u ny ca
lch th- ng - c to ra cng vi d ton ban u ca d n tr- c khi d
n chnh thc - c tung ra. Khi c nhiu chi tit hn, d ton tt hn
- c xy dng v lch - c tinh ch hn v tin cy hn. D ton d n
- c bn chi tit ch- ng 10.
9.2.1 Danh mc hot ng theo lch
Nh- chng ta thy, vic ln lch tr- c ht di hi danh mc cc hot
ng ca d n. Danh mc ny th- ng - c xy dng kt hp vi bng
c cu phn tch cng vic (WBS ). WBS phn tch mi hot ng ca d
QUAN L DU N PHN MM
175
n ti mc thp hn na gi l cc tc nhim. WBS cung cp mt ph- ng
php kim tra cng vic hin nay - c i pht trin thc hin v c
bit c ch trong vic giao nhim v cng vic mc thp cho nhn vin d
n. WBS - c bn thm ch- ng 6.
Bng 9.2 Danh mc hot ng th d
S
hiu
hot
ng
Tn
hot
ng
M t Ngy
bt u
Ngy
kt thc
Ph
thuc
Giao
trch
nhim
5. Tch
hp
Tch hp phn
cng v phn
mm h thng
10/01 31/01 5.3 Sn
5.1 Thit b Mua sm thit b
tch hp
20/01 20/04 5.1 Bo
5.2 Lp t Cung cp ni lp
t
1/01 28/01 Bnh
5.3 Qui
hoch
tch hp
Chun b qui
hoch tch hp
22/02 20/03 5.1 Kin
5.4 Pha 1 Pha tch hp S/W
nguyn thu
21/03 22/03 5.4 Kin
5.5 Demo 1 Ct mc tch hp
nguyn thu
15/03 30/4 5.4 Kin
5.6 Pha 2 Pha tch hp
S/W-H/W
1/05 /05 5.6,5.
5
Kin,
Bc
5.7 Demo 2 Ct mc tch hp
S/W-H/W
20/04 31/05 5.6 Kin,
Bc
5.8 Pha 3 Tch hp h
thng trn vn
1/06 2/06 5.8,5.
7
Kin
5.9 Demo3 Ct mc tch hp
h thng trn vn
6. Th
nghim
Th nghim h
thng anpha,bta,
chp thun
6.1 i th Thnh lp i th
nghim
15/04 30/04 Bnh
6.2 Cc ca
th
Chun b cc th
tc th v cc ca
th
1/05 31/05 6.1 Bnh,
Kim
6.3 Thit b
Anpha
Mua sm thit b
v tr anpha
1/04 30/04 Sn
QUAN L DU N PHN MM
176
6.4 Lp t
anpha
Lp t h thng
v tr anpha
1/05 15/05 6.3
6.5 Th
nghim
anpha
Th nghim h
thng chc nng
y tI v tr
anpha
15/05 30/06 6.2,
6.4
Kim,
Kin
6.6 Lp t
bta
Lp t thit b
anpha ti v tr
bta
1/07 5/07 6.5 Bo
6.7 Th
nghim
bta
Chy h thng
sng tI v tr
bta
6/07 31/07 6.6 Kim,
Kin
6.8 TRR R sot s sn
sng th nghim
cho vic th
nghim chp
thun
30/07 31/07 6.7 Kin
6.9 ATP Chp thun th
tc th nghim
hon tt vic
pht trin h
thng
1/08 4/08 6.8 Bnh,
Kin
6.10 Bo co
th
nghim
Chun b cc bo
co th nghim
ATP
5/08 8/08 6.9 Yn
Bng 9.2 c th d v mt phn ca danh mc hot ng theo lch ca
d n. Bng nu hai hot ng ch yu hp nht v th nghim danh mc
hot ng theo lch c cha thng tin sau:
1. Hot ng ID. y l s nhn bit thp phn c ngha t- ng t
hot ng ph hp vi cc phn loi b tr cng vic (coi Hnh 6.6)
2. Tn hot ng. iu ny t- ng t vi nhn bit bng s v - c s
dng tham chiu hot ng thch hp
3. M t. y l m t vn tt hot ng
4. Ngy khi cng. iu ny lin quan n ngy hot ng bt u
theo lch
5. Ngy hon thnh. iu ny lin quan n ngy hot ng phi
hon tt theo kch
6. Cc ph thuc. iu ny lin quan n cc hot ng khc m hot
ng ny tu thuc. Hot ng ny khng th hon thnh cho n khi cc
hot ng ph thuc hon tt.
7. U nhim/trch nhim. iu ny nhn bit ng- i - c giao trch
nhim cho hot ng .
QUAN L DU N PHN MM
177
Mi hot ng d n theo lch ln danh sch cng vi thng tin trn
giai on ny iu quan trng l m bo cc hot ng khng chng
cho, iu ny c nghi khng c 2 hot ng mc cao bao gm cng
hot ng mc thp. Cui cng danh mc theo lch - c phn loi ph
hp vi ngy khi ng cc hot ng. Th tc ny cung cp hnh thc
c bn nht ca lch d n.
9.2.2. Cc ct mc v - ng mc ch yu
R rng khng phi mi hot ng c tm quan trng nh- nhau. Mt
s hot ng c ngha l nhng s kin ln trong chu k pht trin ca
d n. vic hon thnh c t yu cu l ct mc ch yu khi hon thnh
c t thit k. Cc s kin ch yu khc c th bao gm vic hon thnh
mt nguyn mu hay thit lp h thng th nghim bta u tin. Tt
nhin s kin d n quan trng nht l kt thc d n, th- ng th hin
vic hon thnh cc th nghim nghim thu. Nhng s kin quan trng
xng ng - c ch c bit v - c ghi li trong danh sch ring
cc ct mc d n ch yu.
Nhng ct mc d n ch yu th- ng c thm - c tm quan trng do
n lin quan n cc s kin khc, nh- thanh ton ngn sch pht trin.
Cc thanh ton d n gi c nh th- ng lin quan vi khch hng v
vic hon tt thnh cng mt s ct mc tho thun. iu ny to ra
sc p ng k vi ng- i qun l d n trong vic hon thnh ct mc
ng hn (khng nghi ng g n chnh l nh ca khch hng). D sao,
cc lch ging nh- bt c b phn no khc ca k hoch pht trin, cn
- c cp nht nh k v th- ng l v li ch tt nht ca d n trong vic
sa i ngy hon tt ct mc. Khng phi lun - c khch cng nhn
mt s tht rng i khi khch hng phi hnh ng tri vi li ch tt
nht ca chnh mnh.
Nhng vn ny khng cn thit - c ng dng cho cc d n ni b
khi khch hng v i pht trin l mt phn ca cng mt t chc.
Trong nhng tr- ng hp khch hng c th l b phn tip th hay mt
nhm ng- i dng trong cng ty. Nhng thay i v ngn sch theo
- c cp qun l chung cho php c th nhy bn vi nhu cu ca c
khch hng ni b v i pht trin d n. Cc ct mc - c dng khng
ch lm im thanh ton nh- ng cng o tin d n v xc nh
- ng mc.
Nu ct mc - c m t nh- l s kin d n ch yu th cc - ng
mc c th - c m t l ct mc ch yu. nh ngha ca IEEE v t
- ng mc c cu c t tho thun chnh thc dng lm c s cho pht
trin sau ny. Cc - ng mc c ngha quan trng trong tiu chun
2167 US DOD cho s pht trin phn mm, c ni n cc im ti
hn trong qu trnh pht trin phn mm lc m cc quyt nh ch yu
QUAN L DU N PHN MM
178
- c hon thin. Chng hn, 2167 dnh c gng ng k cho cc th tc
chp thun i vi cc - ng mc d n phn mm.
Th- ng - ng mc d n u tin l vn bn c t yu cu ca h
thng - c chp thun, gi l - ng mc chc nng. Vn bn ny l c
s cho mi thit k v thi cng v c bit l c s cho th nghim v
chp nhn h thng. Do y n th- ng - c coi l - ng mc d n quan
trng nht. Cc th d v cc - ng mc d n khc l thit k h thng
v th- ng l nguyn mu h thng nu d- c chp nhn l c s cho pht
trin d n sau ny (coi phn 8.2.2).
9.3. Cc biu GANTT
Tr- c khi c my tnh ra i rt nhiu, Henry L. Gantt ly tn mnh
t cho biu din th n gin v rt c ch v lch pht trin d n.
Biu GANTT cho thy hu ht thng tin c trong danh sch hot ng
ca lch theo mt cch d hiu hn. Thng tin theo lch d nm bt v
hiu v cc hot ng c th so snh d dng. Biu GANTT gip
chng ta nhn thy vo bt c lc no c nhng hot ng g hn l ang
din ra trong d n .
Hnh 9.1 l mt th d in hnh v biu GANTT. Cc k hiu - c
dng trong biu l - c chp nhn mt cch rng ri, mc d l chng
ch- a - c chun ho. Chng hn, mi tam gic ng- c th- ng - c dng
chmt s kin ng k, v nh- l cc ct mc.
Biu GANTT trong hnh 9.1 thuyt minh cI s d dng nhanh
chng nhn thc - c cc thng tin lp lch quan trng. Chng ta c th
trc tip thy ngay rng khng k n pha bo tr, mi pha khc u gi
u nhau, v rng trong khong thi gian t thng 11 n gia thng 12
nm 1992 th ba hot ng mc cao l gi u nhau.
Mt biu chi tit hn cng c th bao gm cc tn ca cc k s-
gn vi tng hot ng v bao gm cc thit b cn thit cho tng hot
ng mt. Thng tin ny c th - c thm vo bn cnh - ng thi gian
hot ng trong th , hoc l mt bng tham kho lng vo bn
(t- ng t nh- danh sch cc ct mc ch yu trong hnh 9.1). Mt vi
bin th ca biu GANTT thc cng bao gm kiu thng tin
trn biu , nh- ng chng c th gy ra s ln xn, m nh- th li mu
thun vi chnh mc tiu ca biu : lm cho ng- i ta c th d dng
nhanh chng nm bt cc thng tin lch trnh.
Phn tch yu cu
Thit k mc cao
Thit k chi tit
SRR
V
PDR
V
CDR
V
QUAN L DU N PHN MM
179
Hnh 9.1
Biu GANTT mc cao - lch pht trin d n
Mt iu quan trng l cng phi hiu biu GANTT khng cung
cp nhng th g. Trong mt biu GANTT, rt kh cung cp cc thng
tin v khi l- ng cc ngun lc cn n hon thnh tng hot ng.
Mt s nhm ln th- ng thy l ng- i ta kt lun rng nu 5 k s- - c
trao trch nhim tch hp v hot ng tch hp - c bt u khong gia
thng 9 nm 1992 v kt thc vo gia thng 1 nm 1993 (bn thng), th
vic tch hp i hi 20 thng cng. S tht l, vic tch hp c th ch
bt u vi 1 k s- , thng th hai thm mt k s- na, cn ba ng- i cn
li - c thm vo thng th ba. i tch hp sau li c th gim cn
ba k s- trong thng th t- .
Hnh 9.1 ch c by hot ng khi c - c thm chi tit nhiu hot
ng mc thp hn c th - c - a vo biu khi biu c nhiu
hot ng hn mc hp l c th - c (quyt nh ch quan) th cc biu
b sung c th - c thm vo chng hn hot ng thit k c th
trnh by trn biu GANTT ring bit (coi Hnh 9.2).
Hnh 9.2 gii thiu c hot ng mc cao v mc thp. Chng hn,
tch hp m hnh pha I c ba hot ng mc thp: tch hp phn tin
hnh, tch hp h iu hnh v tch hp giao din ng- i dng. iu
ny to ra mt s ni kt lin tc gia biu GANTT chi tit (Hnh
9.2) v biu mc cao (Hnh 9.1).
Thi cng
Tch hp
Th nghim
Bo tr
1 2 3 4 5 6 7 8 9 10 11 12 1 2 3 4 5 6
1992 1993
Cc ct mc ch yu
SRR = R sot yu cu phn mm
PDR = R sot thit k s khi
CDR = R sot thit k ti hn
TRR = R sot th nghim
ATP = Th tc th nghim chp thun
TRR
V
ATP
V
Lp ni tch hp
QUAN L DU N PHN MM
180
Hnh 9.2
Biu GANTT chi tit - lch tch hp
Nn nh l mi thi k mt thng trong Hnh 9.2 - c phn thnh 4
tun. Mc du khng hon ton chnh xc y l xp x thng th- ng
cng s dng trong d ton (coi ch- ng 10) v ngoi tnh thun tin n
cng to ra mt s khe h cho cc iu chnh nh theo lch.
Cc biu GANTT chi tit t- ng t c th - c chun b cho mi
mt giai on pht trin d n ch yu. Cc hot ng khng pht trin
cng xut hin trn biu GANTT nh- cung cp cng c pht trin hay
nghin cu th tr- ng. iu ny c bit c ch khi mt s hot ng pht
trin no tu thuc cc hot ng khng pht trin khc nh- cung
cp cng c pht trin (th d b bin dch cn thit phi - c hon thnh
tr- c khi cc hot ng thi cng c th bt u. Tr- ng hp m nhng
quan h tu thuc nh- th c th b qua, chng s th- ng xut din
vic r sot li ca biu GANTT. Loi tu thuc ny gia cc hot
ng - c trnh din tt nht trong loi biu khc, gi l biu
mng quan h tr- c-sau hay biu PERT.
9.4. Cc biu PERT v - ng ti hn.
TRR
V
Tch hp gi/din ph/cng
Tch hp truyn thng
T/hp h thng y
Tch hp my ch
Th th tc sn sng
15/9 10 11 12 1
1992 1993
Tch hp m hnh pha II
Tch hp giao din ng: s/d
Tch hp h iu
hnh
Tch hp phn tin
hnh
Tch hp m hnh pha I
QUAN L DU N PHN MM
181
Giai thoi phng khng lin quan u ch- ng ny cung cp th d
tuyt vi v s l thuc gia cc hot ng theo lch. S tu thuc din ra
khi mt hot ng ny khng th thc hin nu khng c mt hot ng
khc - c thc hin. K thut th - c gi l biu quan h
tr- c-sau c th cung cp cu tr li cho hai trong cc vn lin quan
trong giai thoi ny: nhu cu nhn ra cc tu thuc v nhu cu m bo
trch nhim ny i vi mi nhim v - c giao.
K thut r sot li v nh gi ch- ng trnh (PERT)
46
s dng mng
quan h tr- c-sau lp k hoch cc hot ng d n v khng ch s
thc hin chng mt cch hiu qu. Ging nh- biu GANTT c nhiu
loi bin th ca biu PERT. K thut PERT qui - c c bn m t
mt mng vi cc node (mu) l cc s kin v cc cung ni l cc hot
ng.
Mi hot ng lin kt vi hai s kin lin quan u v cui. Node kt
thc ca mt hot ng c th trng vi node u hot ng th hai, khi
m s thc hin ca hot ng th hai tu thuc s thc hin ca hon
thnh ca hot ng th nht. iu ny c ngha mt hot ng ch c
th - c thc hin khi mi hot ng khc kt thc node khi im ca
chng - c hon thnh.
Hnh 9.3 cho thy mt th d ca biu mng PERT biu th dng
hot ng ca d n t node khi im ti node kt thc. Mi s kin
- c biu th bng mt vng trn c nh s mng bt u bng s kin
khi im, gi l node ngun v kt thc bng s kin kt thc gi l
node chm. Mi - ng lin kt biu th mt hot ng d n. Hot ng
A
i,j
m t hot ng bt u vi s kin i v kt thc s kin j. Thuc s
D
i,j
biu th l- ng thi gian tri theo lch, gia khi u v kt thc ca
hot ng A
i,j
.
Mt kha cnh quan trng ca ph- ng php biu PERT l quan
nim cc hot ng song hnh. Mi node s kin bt nhnh vo mt s
hot ng c th thc hin song hnh. Trong hnh 9.3 cc hot ng A
1,2
,
A
1,3
v A
1,4
c th thc hin song hnh. D sao mi mt hot ng A
S,1
v
A
10,E
khng th thc hin song song vi bt c hot ng no khc.
Chng ta cng c th thy biu l hot ng A
3,6
c th thc hin
song song vi hot ng A
5,8
hay hot ng A
8,10
nh- ng khng vi c hai.
T- ng t, mt thi im cho no hot ng A
3,6
c th thc hin
song song vi ch mt trong ba hot ng A
1,4
, A
4,7
v A
7,9
.
46
Mt m t chi tit v chnh xc ca k thut PERT xut hin trong Gillett (1976)
2 5
A
S,1
D
S,1
A
S,1
D
S,1
QUAN L DU N PHN MM
182
Hnh 9.3
Mt biu PERT in hnh
Hnh 9.4
Mt biu PERT c ch ra - ng ti hn
Hnh 9.4 l mt th d ca biu PERT bao gm nhiu thuc tnh
thi hn bng s chng ta c th thy biu l hot ng A
5,1
- c lp
lch tip tc trong 5 dn v thi gian (c th l tun) khi lch cn - c
rt ngn cc gi tr thi hn c th rt c ch cho ng- i qun l d n
A
S,1
D
S,1
A
S,1
D
S,1
A
S,1
D
S,1
A
S,1
D
S,1
S
8
4 7
E
9
6 3 1
A
S,1
D
S,1
A
S,1
D
S,1
A
S,1
D
S,1
A
S,1
D
S,1
A
S,1
D
S,1
A
S,1
D
S,1
A
S,1
D
S,1
A
S,1
D
S,1
A
i,j
= Hot ng bt u ti node i v kt thc ti node j.
D
i,j
= Thi gian ca hot ng A
i,j
A
S,1
3
A
S,1
5
A
S,1
4
A
S,1
12
A
S,1
6
A
S,1
2
A
S,1
12
A
S,1
8
A
S,1
5
A
S,1
3
A
S,1
10
A
S,1
6
A
S,1
6
A
S,1
11
S
2 5
8
4 7
E
1
0
9
6 3 1
A
i,j
= Hot ng bt u ti node I v kt thc ti node j.
D
i,j
= Thi gian ca hot ng A
i,j
QUAN L DU N PHN MM
183
gi tr thi hn c th gip nh v lnh vc m c gng b sung - c tt
nht.
9.4.1.Con - ng ti hn.
Khng hong ca cc lch pht trin d n l ph bin m nhng nh
qun l d n phi tin liu.Nhng d n ln c nhiu phn lin quan
trong vic ln lch nh- qun l tp on, khch hng ch thu ph, ng- i
bn, ng- i dng, b phn tip thv.v. Mt trong nhng khng hong
th- ng gp nht l v nhu cu rt ngn lch. Mt sai lm chung v pha
ng- i qun l d n l cho rng bt c mt c gng ph cng s rt ngn
- c lch. D sao trong mt s tr- ng hp nhng hot ng rt ngn s
tuyt i khng tc ng g n thi hn tng th ca lch.
Nhm xem xt hin t- ng ny tr- c ht chng ta phi hiu l trong
mi mng khng tm th- ng c nhiu - ng chuyn tin node ngun ti
node chm. chng hn trong hnh 9.4 con - ng kh d chy t node S ti
1 ti 3 ti 6 ti 9 ti 10 ti E. Mt con - ng kh d khc chy t nt S
ti 1 ti 2 ti 5 ti 8 ti 10 ti E. Mi con - ng c th - c c tr- ng
bng con s biu th tng thi hn ca con - ng,- c tnh bng cch ly
tng s cc thi hn cho mi hot ng dc con - ng.
Bng 9.3 c danh mc mi con - ng kh d t nt S ti nt E trong
biu PERT hnh 9.4 cng vi di ca mi con - ng. Con - ng
2 l di nht, 52 tun. Con - ng di nht - c coi l con - ng ti hn
xc nh thi hn ca d n.
Khi rt ngn thi hn hot ng ca mt hot ng dc con - ng ti
hn, th- ng chng ta c th rt ngn - c thi hn ca ton d n. c t
tr- ng hp thi cc m iu ny s khng din ra ch yu khi c hai con
- ng ti hn. D sao c mt iu l chc chn rt ngn hot ng khng
c trn con - ng ti hn s khng rt ngn - c chiu di ca ton b
d n.
Bng 9.3 Mi con - ng kh d t S ti E (cn c Hnh 9.4)
Con - ng di Con - ng ti hn
1. S.1.2.5.8.10.E 41
2. S. 1.3.5.8.10.E 52
3. S. 2.3.6.9.10.E 30
4. S. 1.4.7.9.10.E 34
9.4.2. Khi ch- ng trnh v nng cao PERT.
Mt s phin bn nng cao ca biu PERT h tr cc hot ng lp
k hoch ph nh- b tr nhn lc, phn b ngun lc v phn tch chi ph.
Khi biu ny c th thu ht ch vo nhng tnh hung m nhn
lc - c b tr nhiu trch nhim hn l nhng th h c th tin hnh
hoc ni m cc yu cu c tranh chp ngun lc.
QUAN L DU N PHN MM
184
Mt ng dng l th ca PERT - c gi l biu din th dng chy
m Riggs v Jones (1990) xy dng s dng mng quan h tr- c-sau
hon thin phn tch chi ph chu k vng i ca d n. K thut biu
dng chy phn tch chi ph d n da trn quan h gia l- ng, gi n
v, bin s thi gian, chi ph nhn lc v hc tp v.v. tt c u - c biu
th trn biu loi PERT.
K thut biu din bng biu dng chy t mt l- ng thng tin
ng k vo biu mng. Thng tin ny ng nh- thng tin c bn
PERT phi - c th- ng xuyn cp nht. Mt thay i nh cho biu
PERT rng ln c th i hi v loi ton b biu v tnh ton li con
- ng ti hn. S t nht xy ra khng thc y thm ho hng trong
vic gi cho biu cp nht. V l do nhiu tin ch PERT my tnh
ho hnh thnh.
Khi ch- ng trnh phn mm PERT c tc dng trong nhiu nm
nay ri, nh- ng ch trong vi nm qua th khi ch- ng trnh PERT chuyn
nghip tt mi dng - c trn my tnh c nhn v cc my tnh nh
khc. Nhng khi ch- ng trnh lm cho vic chun b cc biu
PERT bt t nht nhiu v cng p ng - c cc c im ph nh- cc
b phn tch k hoch trong vic b tr hot ng, kch bn what if v
phn b ngun lc. Nhng tin ch my tnh - c ra i hon thin
phn tch, biu din bng biu dng chy to ra nhng chi ph theo
lch cho cc hot ng d n ch yu
47
. Cc tin ch ny t ra v gi
cho cc nh qun l d n v gii phng nh qun l khi cng vic bn
gy cm ci, to cho h c nhiu thi gian hn qun l tch cc d n.
9.5. Nhn vin lp lch
Phn ny dnh cho kha cnh lp lch b my v qun l nhn lc.
ng c v qun l con ng- i - c bn n ch- ng 5.
V c bn, i pht trin l ngun lc ng y nh- thit b pht trin l
ngun lc. D sao lp lch nhn lc khng nh- lp lch thit b, ngun lc
d n quan trng nht ca ng- i qun l d n l i pht trin v do
cn quan tm c bit ti vic lp lch cc hot ng ca cc thnh vin
trong i. Khi s l- ng cc hot ng ca d n thay i th qui m ca
i pht trin cng thay i trong sut vng i pht trin ca d n. C
cu t chc ca i tr nn quan trng hn khi qui m ca i tng.
9.5.1. Qui m i pht trin.
Qui m i pht trin chu nh h- ng khng ch s l- ng m cn
c- ng hpt ng. Mt s hot ng l khn tr- ng lc khi u d n
v suy thoi vo lc kt thc v ng- c li. Chng hn vic quy hoch i
47
Riggs v Jones (1990) trong tham lun ca mnh v phn tch chi ph d n m t
ch- ng trnh vi tnh v k thut phn tch chi phi kinh t bng th (GECAT) cc khi
ch- ng trinh my tnh khc - c bn n trong Tahvanainen v Smolander (1990).
QUAN L DU N PHN MM
185
hi ngun nhn lc nhiu hn lc khi u d n v t hn lc kt thc
trong khi kim tra cu hnh i hi t hn lc u v nhiu hn lc
cui.
Lp k hoch
Kim tra
Yu cu Thit k Thc thi Tch hp Th nghim
Hnh 9.5
Chng minh mi quan h gia quy hoch v kim tra
Hnh 9.5 chng minh mi quan h gia qui hoch v kim tra khi
c- ng quy hoch gim s cn t ng- i hn cho hot ng ny. T- ng
t khi c- ng kim tra tng s cn nhiu ng- i hn cho nhng hot
ng nh- th nghim, bo m cht l- ng v qun l cu hnh.
Qui m i th- ng bin thin ng theo vi phn b chun hnh
chung. iu ny - c ch ra hnh 9.6 (a) m t i pht trin nh lc
khi u d n, i pht trin ln trong nhng giai on gia d n v ri
li i nh vo cui d n.
O hai u ca chu k pht trin khi qui m i l nh, nhiu nhng
chc nng t chc l khng cn thit. Trong nhiu tr- ng hp cc c cu
i ch tr nn cn thit vo cui giai on yu cu.Khi d n gn hon
thnh cc i c th - c gii th v mt hay hai thnh vin i c th
m nhn trch nhim cho cng vic pht trin ca ton i.
Trong vi tr- ng hp hnh 9.6 (a) c th khng biu th lp lch ca i
pht trin chnh xc. Hnh 9.6 (b) cho thy - ng cong lch khng
cn xng t- ng t phn b th- ng m t t sut nhn s chm hn lc
khi u d n v qui m nhn s suy gim nhanh vo lc cui. iu
ny thng th- ng in hnh ca cc d n phc tp khi cc giai on hp
nht v th nghim i hi c gng ln. Trn thc t, - ng biu din
lch ni chung tiu biu cho lp lch i ng hn - ng biu din chun
mc d mc nghing ca - ng biu din thay i.
M
QUAN L DU N PHN MM
186
T
0
t
m
t
e
(a)
Trc tung l qui m i pht trin, M l qui m ti a, T
0
l khi u d n,
t
m
l im nhn s ti a, t
e
l kt thc d n.
M
T
0
t
m
t
e
(b)
Hnh 9.6
Phn b qui m i pht trin (a) bnh th- ng (b) lch
M
QUAN L DU N PHN MM
187
T
0
t
m
t
e
Bo tr
Trc tung l qui m i pht trin, M l qui m ti a, T
0
l khi u d n,
t
m
l im nhn s ti a, t
e
l kt thc d n.
Hnh 9.7
Phn b qui m i, k c bo tr
Cch m giai on bo tr - c xt n tc ng n - ng cong nhn
s. - ng cong nhn s s trng khc i nu giai on bo tr - c coi
nh- b phn ca chu k pht trin. - ng biu din hnh thnh nh- trong
hnh 9.7 c gc hi i xung tip tc sut giai on bo tr.
Hnh 9.8 m t chc nng phn b qui m i ng kh d trong mt d
n c va vi qui m i ng ti a l 18. Lc u vi qui m i l ba,
kim tra cu hnh v m bo cht l- ng s do ng- i qun l d n tin
hnh, Khi i tng ln tm, nhng trch nhim s - c giao cho mt
thnh vin i cng c th hon thnh cc nhim v khc
48
. Khi i tng
ln 12 s cn mt k s- khng ch cu hnh v mt k s- m bo cht
l- ng t ra na thi gian khi i tng n qui m cao im hai k s ny
s phi yu cu lm trn thi gian.
Qui m i - c xc nh bi tng s ng- i - c u thc vo d n.
D sao khi lp lch qui m i. Vic b tr 2 ng- i mi ng- i cho na
phn u thc trn ngy khng nht thit bng vic u thc mt ng- i trn
ngy. Trong nhng tr- ng hp nh- bo m cht l- ng hay kim tra
cu hnh, chi ph cho d n v u thc nhng hot ng c th - c
gim khi chia nhng chc nng vi cc d n khc. iu ny c bit
ng vi cc d n nh.Th nghim l mt th d khc ca ngun nhn
lc chia x gia cc d n, Nhiu t chc c mt i th nghim c lp
khng trc tip l mt b phn ca i d n, i th nghim c lp tr
thnh tham gia vo d n ch yu vo cui chu k pht trin (mc d mt
s hot ng th nghim tin hnh sm hn nhiu). Nhng i nh- th
48
Khng phi bao gi cng c th giao cc hot ng kim tra cu hnh v bo m cht
l- ng cho cc thnh vin i pht trin khc v nhng hot ng ny i hi k nng v kin
thc c tr- ng chng hn tiu chun ANSI/IEEE 730 khuyn c ch- ng trnh bo m cht
l- ng thc s cho cc d n phn mm. T- ng t tiu chun 820 ANSI/IEEE xc nh
ch- ng trnh qun l cu hnh ng k.
QUAN L DU N PHN MM
188
c th chuyn t d n ny sang d n khc min l cc hot ng th
nghim - c lp lch thch hp.
18 Tch hp
16 Thi cng
14
Thit k
12 Th nghim
10
Yu cu
8
6
4 Thit lp d n
Kt thc d n
2
Hnh 9.8
Th d v qui m i pht trin
9.5.2. K nng v kinh nghim
Vic lp lch nhn lc khng chnh ch l tr- ng hp b tr s ng- i
vo mi giai on ca d n. Nhng k nng c tr- ng - c yu cu cho
mi hot ng v nhng ng- i chuyn mn c kinh nghim thch hp
phi - c u thc nhim v cho nhng hot ng . Bng 9.4 cho danh
mc mt s cc hng nhn lc cn cho mi giai on pht trin ca d
n. Khng phi mi d n yu cu mi hng nhn lc nh- trong trung
hp nhng d n nh khi hai hay nhiu c- ng v ca d n c th do mt
ng- i m nhn.
Bng 9.4 phn loi cc c- ng v d n phn mm
Phn loi C- ng v d n
Nh qun l Qun l d n, i tr- ng, K s- h thng
Qun tr vin Th- k, Tr l hnh chnh
Khng ch cu hnh Nh qun l cu hnh, Ng- i khng ch cu hnh
Bo m cht l- ng Qun l cht l- ng, K s- qun l cht l- ng
Phn tch v thit k Phn tch h thng, K s- thit k
Ng- i lp m Lp trnh vin
Ng- i vit ti liu
k thut
Ng- i vit t- liu, Ng- i bin tp
Ng- i th nghim i th nghim c lp, Ng- i pht trin cc ca
th nghim.
QUAN L DU N PHN MM
189
Mc d bng 9.4 xt n cc c- ng v ngh nghip khc nhau i hi
chuyn mn khc nhau lun c li th khi cc trnh chuyn mn ca
thnh vin i - c mm mng. Nh- th c th b tr li cc thnh vin
i trong d n, do tit kim - c chi ph con - ng biu din hc tp
th- ng cn thit lp lch cho nhng thnh vin i mi.
Hot ng ca - ng biu din hc tp th- ng b xao lng. Vic lm
quen d n i vi cc thnh vin mi khng ch l trung hp duy nht
khi c chi ph ca - ng biu din hc tp. o to cng l yu t quan
trng phi - c lp lch. D sao khng phi mi nhu cu o to c
th bit ngay - c trong nhng giai on u ca d n. Nhng yu cu
o to tr nn r rt khi c cc qui nh lin quan n mi tr- ng pht
trin (nh- ngn ng lp trnh, cc my tnh pht trin v.v.) v khi cc
thnh vin i - c la chn v k nng cng kinh nghim ca h - c
bit.
C nhiu vn lin quan n vic lp lch nhn lc. Kinh nghim v
k nng ca thnh vin khng l nhng o tin cy v s hon thin
trng i. Vic lp lch cc k s- cn c nhng c tnh gi nh o
- c t lu l mt ti tranh ci. Sackman v cng s vo u nhng
nm 1968 c bo co v cc tnh a dng bn cht trong vic hon thnh
cng vic ca ng- i lp trnh phn mm v - c trao i thm ch- ng
5 v ch- ng 10.
9.5.3. Thng cng
Ngun gc sai lm ph bin khc khi lp lch nhn lc l s khc nhau
trong cch m t man month (hay nh- by gi - c gi l workmonth)
- c dng. Nu ng- i qun l d n tnh ton rng mt hot ng
- c lp lch i hi su thng lm vic hon thnh, th phi chng
iu c ngha l mt k s- thch hp - c b tr lm vic trong 6
thng th hot ng c th hon thnh trong su thng?
Phi! cu tr li . . . l c th. Trong nhiu tr- ng hp, hot ng
khng th hon thnh trong 6 thng v mt ng- i t khi c - c 6 thng
lm vic trong thi hn lch 6 thng. Con ng- i phi ngh h, h t chc
ngh l v i khi h m. Ni chung con ng- i c khong 8 v 10 thng
lm vic trong mt thi k 12 thng. iu ny phi xt n khi chun b
lch trnh.
Khi nhng ng- i qun l d n bo co cp trn l d n s i hi
u t- trong 6 nm h phi bit r v loi nm no h ni. iu l 6
nm lch c ngha d n ch c th hon thnh 6 nm k t ngy khi
cng? Hay l 6 nm lm vic c ngha c 72 thng lm vic hon
thnh c th - c chia cho s nhn lc? hay thc ra l 48 thng cng
vi ph thm ngh h, ngh l, ngh m ?
Cng vy c mt s nhn lc ti a c th - c b tr cho d n 365
k s- khng th hon thnh d n mt nm trong 1 ngy v cng 9 ph
n khng th c hi nhi trong 1 thng. Mt s d n phi mt mt l- ng
QUAN L DU N PHN MM
190
thi gian bt k c bao nhiu ng- i - c b tr. Do ngha ca thi
hn d n cng nh- thi hn ca mi hot ng ch yu phi - c hiu
r rng v trnh by nh- b phn ca lp lch. Lp lch phi tnh n c
tnh trng v trch nhim, chi ph hnh chnh (tho lun hp hnh v c
ni chuyn khng u) v phi gii thch nhng hot ng no khng th
rt ngn v hot ng no c th.
Gim thi hn ca hot ng c ci gi ca n. Thm nhiu ng- i vo
d n ny sinh thm tng ph. Nu nm ng- i c th pht trin d n
trong 2 nm iu khng phi l 10 ng- i c th pht trin d n trong
1 nm. iu ny l do lin lc ph gia cc thnh vin cn nhiu cuc
hp hn, nhiu phi hp hn, nhiu qun l v qun tr hn. V tt nhin,
ng- i ta khng th tip tc gim thi hn hot ng bng cch b tr
thm nhiu nhiu ng- i vo d n. Lut phn hi gim dn c hiu hc
khi b tr con ng- i vo mt d n v im no con ng- i bt u i
theo con - ng ca nhau. ng gp s khng v thm ch c s m ng
ra c th t n nhanh hn khi m mt d n i vo pht trin tt v
- ng biu din hc tp tr nn di v tn km.
9.6. lp lch cc Ngun lc
Phn tr- c y bn n vic lp lch mt ngun lc ti quan trng
ca d n i pht trin. Nh- ng khng c nhng cng c cn thit i
pht trin khng th hi vng lm - c vic tt. Nu my tnh mc tiu
ca d n khng c ngay khi pha tch hp bt u, th pha tch hp rt c
th khng bt u tt - c. Do chng ta thy l mt dng ca nguyn
tc - ng ti hn cng vn dng - c cho tnh hiu lc ca cc ngun
lc pht trin. iu ny c ngha l nu tnh hiu lc ca mt ngun lc
ti hn theo lch b chm li n s lm chm vic hon thnh d n.
C nhng cch xem xt cc vn tnh hiu lc cho cc ngun lc
ti hn, nh- phn tch ri ro.Mc tiu l cung cp nhng k hoch phng
nga tnh hung m nhng ngun lc ti hn khng c - c theo nh- lp
lch. Phn tch ri ro - c bn thm ch- ng 2
9.6.1. Lp lch khng gian lm vic.
O ca ra ca d n, khng gian lm vic c bit khng gian vn
phng, th- ng l mt trong nhng ngun lc ch yu u tin yu cu.
Khu vc d n phi - c b tr tr- c v xc nh r. khi d n tin
trin,khu vc lm vic yu cu s tng.
Nhu cu v khng gian lm vic th- ng b xao lng khi lp lch
nhng ph- ng tin pht trin. Khu vc lm vic ring bit v xc nh r
l nhn t quan trng trong vic hnh thnh i pht trin khi cc thnh
vin d n ri rc khp khu vc trng ln v ln vi cc i d n khc
nhiu vn ny sinh lin lc gia cc thnh vin kh khn hn, qun l
tr nn phc tp hn v tinh thn i khng pht trin.
QUAN L DU N PHN MM
191
Khng gian lm vic v vn phng lp lch l mt trong nhng b- c
u tin trong vic lp lch ngun lc. Yu cu khng gian l hm s
nhng yu cu nhn lc d ton, yu cu thit b v lch nhn s ca d
n.
Bng 9.5 c danh mc kim tra mt s hng mc phi xem xt khi lp
lch khng gian lm vic. Tt nhin, khng phi mi chuyn u vn
dng - c cho mi d n. L d nhin, yu cu khng gian hin nay tu
thuc qui m v th loi mi d n. Nhiu d n quc phng v an ninh
i hi nhng khu vc tip cn hn ch c bit trong bng kim tra - c
coi l khu vc an ton. Hng mc khc phng l- u kho v kim k, ch
yu cu cho cc d n bao gm khi l- ng ln phn cng v thit b.
Bng 9.5 Hng mc khng gian lm vic danh mc kim
tra
1. Khng gian vn phng qun l
2. Khng gian vn phng hnh chnh th- k
3. Phng hp
4. Phng i pht trin v khng gian bn giy
5. Phng my tnh
6. Phng th nghim
7. Khu vc hp nht v th nghim
8. Khu vc n tr- a v gii tr
9. Phng kho v kim k
10.Khu vc an ton.
9.6.2. Thit b lp lch
Vi cng c thch hp cng vic bao gi cng - c tin hnh tt hn.
Nh- ng nh- chng ta thy c - c cng c thch hp khng cng c
phi - c s dng ng lc. Mc tiu ca vic lp lch thit b l m
bo cng c pht trin thch hp c - c vi s l- ng khi cn n.
Trong nhng nm u ca pht trin phn mm, cng c c bn bao
gm ng- i lp trnh, b bin son v my tnh lp m. Cng c pht
trin hin i ngy nay bao gm nhiu hn b bin son v my tnh. Cc
tin ch phn mm t cc cng c thit k hp nht ti cng c g ri v
th nghim tinh vi gi y c. Trn thc t my tnh - c tn dng
nh- l tr th cho ng- i sn xut h tr trong cc hot ng pht trin
c tr- ng, to nn t cng ngh phn mm h tr bng my tnh hay
CASE.
R rng, khng phi tt c d n my tnh thun nht l d n phn
mm. Nhiu h thng my tnh i hi pht trin ca c phn mm v
phn cng. iu ny bao gm nhng d n nh- h thng lin lc thng
QUAN L DU N PHN MM
192
tin, h thng qun s, rbt v cc h thng cng nghip. Trong nhng
tr- ng hp , thit b c mc ch c bit - c yu cu cho cc giai
on th nghim v hp nht. Vn phng d n trung tm sau phi
phi hp qui hoch v lp lch pht trin phn cng v phn mm m
bo tnh hiu lc kp thi ca thit b pht trin.
m bo tnh hiu lc ca thit b pht trin thch hp l b phn quan
trng ca qui hoch tt. Nh- ng lp lch km c th dn n nhng tnh
hung theo thnh vin ca i pht trin b ngi khng hay phn
no ngi khng ch i thit b bn giao. Ngay nu cc thnh vin c th
- c b tr li tm thi th hiu qa v tnh hiu lc ca h l ng- i sn
xut s b st gim ng k.
9.6.3. Ng- i bn hng v ch thu ph
Khng phi mi hot ng lp lch u ph thuc trc tip ng- i
qun l d n v i pht trin. Thng th- ng, cc b phn ngoi cuc li
cng tham gia vo vic pht trin d n nh- chng ta thy, vic giao
tht b kp thi l ct t cho lch pht trin v iu ny th- ng i hi s
cung cp ca cc bn ngoi cuc.
Khng phi khng bnh th- ng, c bit trong nhng d n ln hay
phc tp, vic c hp ng ph mt s phn ca d n cho cc cng ti
c chuyn mn c tr- ng nhng lnh vc lin quan. iu ny c ngha
l vic kim tra pht trin tc tip c th giao cho ch thu ph.
Kh c th lp lch cc ngun lc v hot ng m ng- i qun l d n
khng hon ton kim tra - c. Trong nhng tnh hung ng- i qun
l d n c hai ph- ng n.
1. Giao vic lp lch cho ch thu ph hay ng- i bn hng
2. Gi quyn kim tra ch thu ph hay ng- i bn hng.
Trong tr- ng hp u khi vic lp lch - c giao cho bn ngoi cuc,
ng- i qun l d n ph mc cho bn m ng khng kim tra - c. Nu
bn kia lch lch giao hng iu ny c th gy ra tnh trng tr- t lch
cho ton d n. iu ny - c x l tt nht bng cch:
1. ng vin bn ngoi cuc giao ng hn (th d pht bn v giao
chm)
2. Nhn bit vic giao chm l mt ri ro ca d n v chun b k
hoch t xut x l tnh hung nu xy ra.
Trong tr- ng hp th hai khi ng- i qun l d n nm quyn kim tra
bn ngoi cuc, nhiu nhng li ch cho thu ph b mt vi nhng d n
ln phi to c- ng v cho gim nh vin ca cc ch thu ph v ng- i
bn hng. y l trch nhim ca gim nh vin lun c thc v cng
vic m bn ngoi cuc phi hon thnh thng qua.
QUAN L DU N PHN MM
193
Cc thm ving vi pha ch thu ph v ng- i bn
R sot li v nh gi ct mc
Bo co nh k ca ch thu ph v ng- i bn
Thm na, iu quan trng cho gim nh vin l c kh nng ng
vin bn ngoi cuc bng cch gn thanh ton vi cc ct mc hon
thnh thng li v bng p t hnh pht v giao chm.
9.7 Kimtra v cp nht lch trnh.
Lch trnh khng phi l vn bn tnh v chu s thay i th- ng
xuyn. Lch trnh li thi t c (nu cn c) gi tr lch trnh l b phn
ca k hoch pht trin d n, phi - c cp nht nh k. Nhm cho
ng- i qun l d n c kh nng duy tr - c lch trnh cp nht thng tin
l- u hnh phi chy u n t i pht trin n. iu ny thc hin
- c qua cc bo co nh k, r sot li v cc hot ng kim tra khc.
9.7.1. Cc bo co nh k
Bo co nh k l mt trong nhng ph- ng php hp thc m bo
dng chy thng tin th- ng xuyn t i pht trin n ng- i qun l d
n. Thc hnh nn lm trong vic chun b v trnh bo co - c m t
phn 5.3.1
Cc thnh vin i phi trnh bo co nh k ca mnh cho lnh o
i ng ny sau tm tt cc bo co v trnh tm tt cng bn sao
cc bo co c nhn cho ng- i qun l d n. Sau ng- i qun l d n
tm tt cc bo co ca i tr- ng cng vi bo co ca nhn lc d n
khc nh- ng- i ph trch nhm th nghim. Vn bn c - c bao gm
c bo co ca ng- i qun l d n, hnh thnh bo co tin d n v
l vn bn d n chnh thc - c trinh qun l cp cao. Danh mc
phn b cho bo co tin d n cng c th bao gm cc thnh vin c
nhn ca i pht trin khch hng v cc ch thu ph ca d n.
Cc bo co nh k l nhng knh thng tin c bn - c s dng
nh gi v cp nht k hoch pht trin ca d n v c bit lch trnh
d n. D sao cc bo co nh k phi khng bo gi ch l ngun thng
tin duy nht cho cc hot ng . y l trch nhim ca ng- i qun l
d n phi th tnh hon ho v chnh xc ca thng tin - c bo co.
Tn bo co l vn m ng- i qun l d n quyt nh th- ng
mt bo co na thng mt l thch hp cho nhu cu d n ni b v bo
co hng thng l thch hp cho nhu cu d n bn ngoi. D sao trong
giai on ti hn ca d n, li c th cn c nhiu bo co th- ng xuyn
hn.
9.7.2. Cc hot ng kim tra lch trnh khc
QUAN L DU N PHN MM
194
Mt trong nhng yu t ca qun l tt l thit lp tip xc c nhn
gia qun l v nhn vin. Nhng cuc tip xc c nhn h tr nhiu mc
ch qun l, m mt trong l kim tra cc bo co tin .
Nhng vn chnh vi bo co th- ng k l vn khch quan, l
gii v chnh xc. Lch trnh d n khng phi bao gi cng - c l gii
hay nhn thc cng cch thc trong qun l nh- vi cc nh sn xut.
Hi chng ni ting 90/50 bn n ch- ng 5 l minh ho cho trnh
hung ny. iu ny, chng ta c th nhc li khng nh l mt 50%
thi gian hon thnh 90% cng vic v 50% ph v thi gian hon
thnh phn cn li 10% ca cng vic.
iu ny c ngha l khi cc thnh vin i bt u bo co l h
hu nh- hon thnh mt nhim v, chc chn c th cn mt l- ng thi
gian ng k thc s hon thnh nhim v. iu ny l do th- ng
t- ng i d c - c ci g xui xo nh- ng cng vic nng nhc v n
iu cn c gi ghm nhim v i hi l- ng cng vic ng k. V
tt nhin iu ny li thm vo tnh lc quan t nhin ca ng- i sn xut
k vng l khng c g s sai c.
C nhiu ph- ng php kim tra tin lin quan n tip xc c
nhn gia ng- i qun l d n v i pht trin. Nhng cuc hp i v
d n hng tun l nhng c hi tt tho lun tin b v nhng r sot
li khng chnh thc cc hot ng c tr- ng gip ng- i qun l d n
c th nhn v nh gi cng vic hin nay lm - c.
Khi mt lch trnh khng - c thc hin, i khi l du hiu rng
thnh vin i b tr vo mt hot ng c tr- ng khng h tr lch
trnh. Nhng tnh hung nhn mnh n tm quan trng l c - c
i pht trin tham gia vo vic chun b lch trnh. iu ny th- ng d
hn khi c ng- i sn xut tham gia vo vic chun b n
9.7.3. Cp nht lch trnh
Nh- chng ta thy, k hoch pht trin d n phi - c xem li
nh k. Lch trnh phi - c cp nht bt k khi no vic r sot li nh
k xc minh hay bt k khi no c mt s kin ng k din ra. Chng
hn, nu vic r sot li cho thy nhiu hot ng chm hn (hay nhanh
hn) lch hay c v vn hot ng mi cn b sung cho danh mc hot
ng th lch trnh mi cn - c xy dng. Cng vy nu s pht trin
ca b phn d n - c thay th khi mua mt thnh t t- ng i khi gi
th lch trnh phi - c sa i phn nh c gng pht trin nh nhoi.
Bng 9.6 c danh mc kim tra cc hng mc theo lch phi - c
duyt li (v c gng cp nht) nh k hay bt c khi no mt hin
t- ng ca d n ng k din ra.
Hng mc u trong bng 9.6 lin quan n vic cp nht danh mc
hot ng. Nhim v ny trn thc t l xut pht t cc hng mc khc
trong danh mc kim tra, nh- duyt li cc yu cu - c chp thun v
thay i thit k hay cp nht danh mc ri ro ca d n. Sau khi mi
QUAN L DU N PHN MM
195
hng mc - c duyt li v nu cn thit - c cp nht th mi tr- ng
biu th lch trmh cng phi cp nht (biu GANTT, biu
PERTv.v.) Nh- trong vic chun b lch trnh ban u lun c mt thi
quen tt l c nhng thnh vin ca i pht trin duyt li lch trnh
mi tr- c khi cng b. Nhng sai lm, s xut v tranh chp v lch trnh
theo c th nhn bit v hiu chnh tr- c khi lch trnh - c pht i.
Bng 9.6 Danh mc kim tra cp nht lch trnh
1. Danh mc hot ng
2. B tr nhn lc
3. Danh mc ri ro v phn tch ri ro
4. Phn b ngun lc
5. Qui ch bn th ba (ch thu ph, bn hng, cung ng)
6. Biu lch trnh (GANTT)
7. Mng - u tin (PERT)
8. Yu cu - c chp thun v thay i thit k.
9.8. Mt s h- ng dn chung v lp lch trnh v
qui hoch.
Qui hoch bt u vi vic khi cng d n v thm ch tr- c na
mt s tr- ng hp. Nh- chng ta thy phn 9.1, mi hot ng d
n phi - c qui hoch. Vic thiu qui hoch th- ng l nguyn nhn
chnh ca tht bi. B- c u tin tt p trong qui hoch l chun b phc
tho k hoch pht trin d n, nh- m t bng 9.1 v tun t bt u
in vo cc phn.
9.8.1. Tinh ch danh mc hot ng ban u
Nh- chng ta thy, danh mc hot ng ban u, cng vi ngy thng
c d n hon thnh hot ng to ra lch trnh ban u. Vic tinh ch
danh mc hot ng l mt qu trnh lp li c th to ra lch trnh pht
trin chi tit ca d n.
Khi lch trnh tin trin v tr nn chi tit hn, danh mc hot ng s
c cc hot ng mc thp - c giao cho cc thnh vin. Do iu
quan trng nht cho ng- i qun l d n l bao gm cc thnh vin trong
giai on ny. Tt hn l nn k s- xut lch trnh trong lnh vc
trch nhim ca h ch khng nn ch nh lch trnh cho h. Ni chung,
cc thnh vin cm thy tham gia nhiu hn vo lch trnh m h
chun b hn l lch trnh - c chun b cho h.
K thut chung l giao danh mc hot ng chi tit cho cc thnh vin
ca i pht trin v - c h trnh xut v ngy hon thnh. Sau
ng- i qun l d n phi triu tp cuc hp vi cc nhm pht trin
nhm gii quyt mi bt ng v vn . Sau y qa trnh ny - c lp
QUAN L DU N PHN MM
196
li n khi c - c lch trnh chp nhn - c v nht tr. Ch khi khng
th c - c nht tr, ng- i qun l d n mi dng quyn ca anh hay
ch v - a cc phn ca lch trnh cn bt ng vo.
Bng 9.7 Tm tt mt s h- ng dn c bn trong vic xy dng v duy
tr lch pht trin chi tit.
9.8.2. Ginh - c chp thun lch trnh
Chun b mt lch trnh thc tin khng phi l mc tiu duy nht ca
ng- i qun l d n: C - c lch trnh chp thun li mi l quan trng.
Rt th- ng khi, lch trnh thc tin - c ng- i qun l d n mt nhiu
cng sc chun b v - c trmh qun l tp on ch - c bc b
v l do kinh doanh. iu ny nhn mnh tm quan trng ca nhng
ng- i qun l d n c thc - c hnh nh rng hn ca tp on v
khng ch t hn ch mnh trong vin cnh k thut hn hp.
Khi chun b mt k hoch pht trin d n tng th, tt nhin ng- i
qun l d n phi d kin sc p trong hai lnh vc c bn (1)ngy hon
thnh v (2)chi ph pht trin. Cc sc p khc cng c th chu ng
- c nh- ng hai lnh vc c bn l ph bin.
Bng 9.7 H- ng dn lch trnh
1. Xc tin tham gia ca i
2. Lp li mc cao vi lch trnh chi tit
3. C thc v nhu cu ca khch hng, qun l, ng- i d v tip th.
4. Khng ch ln lch cc hot ng m c ngun lc v nhn lc na
5. Chng chi sc p htam gia lch trnh khng tho ng
6. S dng cng c ln lch my tnh ho
7. Ln lch k hoch t xut phng nga ri ro
8. Cp nht nh k lch trnh hay sau nhng s kin ch yu ca d
n.
Vi ng- i qun l d n, con - ng tt nht i ph vi sc p nh-
th l tm cch nhn nhn d n t vin cnh khc phi k thut.
Nu sc p phi chu l khch hng, ng- i qun l d n phi tm
cch hiu - c mi quan tm ca khch hng v tm cch - a nhng
quan tm trong phm vi lch trnh thc tin. Liu khch hng c chp
nhn vic sm giao mt phn h thng? liu c gii php sn c no tho
mn - c tm thi cho n khi ton b h thng hon thnh?
Nu sc p bt ngun t qun l cp cao, ng- i qun l d n phi tm
ra l do c sc p. Liu d n c tr nn qu ln cho ngn sch c - c ?
Nu nh- th, liu d n c th - c thc hin tng giai on vi nhiu
nhng iu tinh xo tr hon cho n khi c - c ti tr?
QUAN L DU N PHN MM
197
u t- v o t- ng t thn th- ng l chnh sch ti t nht. Tt nht l
trung thnh vi mt lch trnh thc hin - c tho ng. Cch tip cn tt
nht cho ng- i qun l d n l trung thc. Khng bao gi ha hn ci g
bn khng d liu c kh nng giao - c.
Mt cch tip cn hiu qa - c minh chng bao gi cng l trnh
by vn cng vi gii php. iu ny c ngha l khi lch trnh khng
th h tr mong i ca khch hng hay ca qun l cp cao th vn
phi - c trnh by v gii thch v lch trnh xen k phi - c gi cng
vi nhng mc tiu s i. Th d sau y s chng minh tip cn .
ACO, mt cng ty vng vng, tham gia vo dch v h tr bn l
trong nhiu nm. Mi y h quyt nh pht trin mt h thng mi
my tnh ho - c ho mng c th giao din vi cc h thng ng k
tin mt hin c nhm cung cp mt lot a dng dch v cho cc ca
hng v khch hng ca h. C hai iu sng t u ra: th nht l, c
nhu cu xc nh cho nhng dch v v th hai l, cng ty khng
phi l duy nht nhn ra yu cu .
BCO, cng ty - c cho hp ng pht trin, thy l h thng
khng phi khng quan trng. Bit - c l ACO mun ginh th tr- ng
ny, BCO trnh lch trnh thc tin thi gian ngn nht m h thy
c th tham gia. Tuy nhin lch trnh b ACO bc b.
Sau khi BCO iu tra thm, r rng l ACO chp nhn thi gian
giao hng ca h thng cho mt s khch hng ca mnh. ACO cng
cm thy c c hi d tiu ma trong th tr- ng v h c th mt khch
nu h khng th giao ng hn.
BCO xut mt h thng trung gian c th khng - c ho mng
gia cc ca hng v hn l thu gim chc nng. H thng trung gian ny
c th hot ng trn phn cng nh- th v mi chc nng c th t- ng
thch vi h thng chc nng trn vn cui cng. H thng trung gian ny
c th - c giao cho khch hng ca ACO sm hn v sau ny - c thay
th bng h thng trn vn. iu ny l chp nhn - c cho ACO.
BCO chng chi li ha hn ngy giao hng bt kh nhm m
bo l h c - c hp ng. H gii thch vn vi ACO v gi mt
gii php cho nhng vn ACO nhm ti vi khch hng ca h. Chn
cch lm nh- vy, BCO cng ginh - c tn nhim ca ACO th hin
sn sng gip sut c d n.
Khng mt ai, d l khch hng hay qun l, c th d liu c lp lun
iu bt kh. Do y, c - c chp thun cho lch trnh pht trin d
n, tin trnh hnh ng nn l:
1. Khng tnh by lch trnh trong chn khng. Lch trnh phi l b
phn ca k hoch pht trin tng th ca d n.
2. Hc bit vin cnh ca c to ca bn k c khch hng, qun l
cp cao v bn hng v tip th. Nghin cu cc quan im ca h v hiu
quan tm ca h.
QUAN L DU N PHN MM
198
3. m bo l lch trnh tho ng v - c chun b k. Sn sng xc
minh mi ct mc.
4. Tm hiu h tr ca cc chuyn gia khc v cc ngun tham kho
nghip v nhm bin minh mi vn bn trnh by.
5. Lun trnh by vn cng vi gii php.
6. T tin l mnh ng. Nu bn nghi ng nhng khng nh ca chnh
mnh th bn khng sn sng trnh by lch trnh - c lch trnh thc
tin - c qun l (hay khch hng) chp thun l b- c ch yu n hon
thnh thng li d n. Khi lch trnh l khng thc tin n th- ng b
ngu trang bi nhng t nh- cht ch, hung hng hay thch . D sao,
nhng lch trnh cht ch, hng hi hay thch him khi l hp dn cho
cc d n thnh cng
9.8.3. Quan h gia lch trnh, ngun lc cht l- ng v
chc nng.
Nh- chng ta thy, k hoch pht trin ca d n lp biu - ng i
t tnh hnh hin ti ti mc tiu ca d n. K hoch m t cc ngun
lc cn thit thc hin mc tiu trong phm vi mt lch trnh nh
ngha. Ngun lc yu cu v lch trnh c th - c tnh (hay d ton) c
hai cn c mc tiu ra ca d n.
Cc mc tiu ca d n, (l chc nng m t trong c t yu cu)
khng cn thit l nhng yu cu duy nht ca d n. Lch trnh danh
ngha cng c th - c yu cu (th d pht trin d n trong phm vi
mt nm). Nu lch trnh yu cu qa ngn th yu cu ph c th l yu
cu bt kh thi. D sao nu lch trnh yu cu cng khng qu ngn th
cng vi tnh chc nng, n s xc nh cc ngun lc yu cu.
Cc ngun lc cng c th l yu cu (qui m ti a ca i pht trin
hay my tnh pht trin c dng). Nu ngun lc l khng thch hp th
yu cu ph ny c th l yu cu bt kh thi. Khi ngun lc yu cu l
thch hp th cng vi tnh chc nng, chng s xc nh lch trnh cu
hi tn ti l ci g xy ra nu c ngun lc v lch trnh - c yu cu
th- ng iu ny c ngha l mc tnh chc nng s - c lch trnh v
ngun lc xc nh. iu ny c ngha l trong phm vi mt lch trnh
cho v vi ngun lc cho, l- ng tnh chc nng l hn ch. Mt cch
khc nhn vo tam gic cc thuc tnh n theo ca d n (tnh chc
nng lch trnh v ngun lc) l - a vo thuc tnh th t- cht l- ng.
iu ny c ngha l n c ba thuc tnh tr- c y - c nh tr- c th
cht l- ng ca sn phm phn mm cng - c xc nh. D sao ng- i
qun l d n i vo lnh a nguy him khi c bn thuc tnh - c xc
nh tr- c ( coi Hnh 9.9).
Vic xc nh t nht mt (tt hn hai) thuc tnh phi cho ng- i
qun l d n lm. Hon ton c hiu lc cho khch hng hay qun l
cp cao khi hi tnh chc nng no c th c - c vi ngn sch cho
v ngun lc cho v mc cht l- ng cho.
QUAN L DU N PHN MM
199
T- ng t. ng- i qun l d n c th - c hi cn ngun lc no
nhm pht trin tnh chc nng cho trong phm vi 18 thng mc cht
l- ng cho.
Hnh 9.9
Bn thuctnh ca d n pht trin (c ba xc nh th t- )
9.9. Tmtt
Lch trnh d n l mt trong nhng phn quan trng nht ca k
hoch pht trin d n. K hoch ny th- ng l vn bn chnh thc u
tin pht sinh trong phm vi d n v bao gm khng ch ln lch cc
hot ng pht trin m c ln lch cc ngun lc d n, c bit con
ng- i. K hoch pht trin d n m t chi tit ng- i qun l d n lp
k hoch pht trin d n ra sao, cn nhng ngun lc no v nhng
ngun lc - c s dng th no.
Lch trnh l danh mc cc hot ng v thi gian thc hin d kin c
nhiu cch biu th lch trnh; danh mc hot ng, s , th v.v.
ph- ng php ph thng nht biu th lch trnh l biu mng trnh t
tr- c-sau (nh- PERT) cc biu GANTT v danh mc ct mc.
Mt sai lm ph bin khi cho rng bt c khi no c c gng ph th
lch trnh s - c rt ngn. Vic rt ngn cc hot ng tuyt i s
khng c tc ng g n thi hn tng th ca lch trnh nu nhng hot
ng khng thuc v con - ng ti hn ca d n. - ng ti hn l
- ng di nht xuyn qua biu trnh t tr- c-sau ca mng t nt khi
im n nt tn cng.
Lp lch cc ngun lc cng quan trng nh- vic lp lch cc hot
ng. Nhng ngun lc pht trin bao gm ph- ng tin, khng gian lm
vic, thit b v ngun nhn lc.
Ngun lc d n quan trng nht ca ng- i qun l d n chnh l i
pht trin. V s l- ng cc hot ng d n thay i nn qui m ca i
pht trin cng thay i xuyn sut vng i pht trin d n. C cu t
chc ca i cng tr nn quan trng hn khi qui m ca i tng ln.
Mt lch trnh qu hn l t c gi tr. Mt lch trnh, vi vai tr l b
phn ca k hoch pht trin tng th ca d n, phi - c cp nhp nh
k. Nhm cho ng- i qun l d n c th duy tr - c lch trnh cp nht,
thng tin hin thi phi - c chy u n t i pht trin. iu ny
Cc ngun lc
Cht l- ng Chc nng
Lch
QUAN L DU N PHN MM
200
- c thc hin qua cc bo co nh k, r sot li nh k v cc hot
ng kim tra khc.
Mt lch trnh thc tin - c qun l (hay khch hng) chp thun l
mt b- c ch yu d n pht trin thnh cng. Khi mt lch trnh l
khng thc tin, n th- ng - c ngu trang bng nhng t nh- cht ch,
nng ng hay thch thc. D sao, cc lch trnh cht ch, nng ng v
thch thc him khi l hp dn cho cc d n thnh cng
Bi tp
1. Bn - c ch nh lm qun l d n cho mt h thng chuyn
qu v vn chuyn ca cng ty ln gio xe ti. Mi xe ti - c trang b
thit b lin lc bng s lin lc vi my tnh trung tm.
D n ca bn s pht trin phn mm lin lc vi xe ti v gi i
theo thut ton hnh trnh ti - u. H thng cng duy tr c s d liu chi
tit bao gm thng tin lin quan n xe ti ca cng ty, v tr hin thi
ca chng, ng- i li v l trnh giao nhn. H thng cng cung cp cu
hi trc tuyn v kh nng cp nht cng nh- cc b pht bo co.
Hy chun b mt danh nc hot ng cho d n . Nhn bit nhng
ct mc ch yu v xc nh cc - ng gc ca d n.
2. Hy chun b biu GANTT mc cao cho d n m t bi tp 1.
Chun b biu GANTT chi tit cho hai trong nhng hot ng mc
cao gii thch mi chng cho gia cc hot ng ti hn c th thay i
th no.
3. Chun b biu PERT mc cao cho d n m t bi tp 1.
Hy bao gm mi hot ng pht trin v khng pht trin. nh v
tr cc con - ng qua mng v nhn bit con - ng ti hn. Chng minh
- ng ti hn c th thay i th no khi ch mt thuc tnh thi hn
thay i. Gii thch s tu thuc nh- - c biu th trn biu .
4. Hy chun b lch trnh nhn s cho d n m t bi tp 1. M t
cn bao nhiu thnh vin i mi giai on, k nng ca h, s b tr
h trong phm vi d n ra sao ?
5. Hy chun b lch trnh ngun lc cho d n m t bi tp 1. M
t mi ngun lc pht trin v gii thch ti sao v khi no th s cn
Tho lun cc tc ng cho c gng pht trin d n ch khng c
kh nng c - c mi ngun lc.
6. Xem xt nhng nhn t no ca d n trong bi tp 1 c th tu
thuc cc bn ngoi cuc. tho lun xem nhng hot ng pht trin
no c th - c xem xt thu ph v nhng thnh t v c kh nng
mua.
7. Xem xt nhng vn c th d kin trong giai on hp nht ca
d n m t bi tp 1
8. Chun b mt bo co tin d n mu - c trnh hai tun sau
khi hp nht bt u.
QUAN L DU N PHN MM
201
Ch- ng m- i
Chun b d ton:
ph- ng php v k thut.
d ton: Mt s vn
D ton - c quan tm vic d on bt trc. N v vang hn l bi
ton mc d khng phi bao gi cng chnh xc. iu ny c th - c
minh ho r nht giai thoi sau.
Vo u nhng nm 80, mt ch thu quc phng ln ng- i M - c
giao mt hp ng pht trin phn mm ca B quc phng Hoa k.
Cng ty - c phn chia thnh nhng trung tm li nhun nh v
mi trung tm nh- th - c yu cu minh chng tn ti ca mnh lm sao
sinh li. Khi - c tin v u thc hp ng, cc k s- t trung tm li
nhun, sau khi khai tr- ng ban u, ra vic qui hoch tin hnh d
n. Ch khi h mi pht hin l chi ph pht trin d kin.... s dng
trong ngh ca h - c cn c d ton khi thu l 80 nm cng
trong khi nhng tnh ton mi ca h - a ra d ton 120 nm cng. iu
ny c kh nng gy ra tht thu ngn sch n 50 phn trm.
40 nm cng ph hn tn cho cng ty khong 4 triu la v trong
khi s tin ny khng lm v mt tp on ln nh- ng li l ng k cho
trung tm li nhun v hn lm cho mt s nh qun l hng u mt
vic.
Qun l trung tm li nhun gi th- cho B quc phng khng nh
l h tnh sai chi ph pht trin d n v yu cu l h - c php
trnh li ngh hiu chnh. Nu yu cu ca h b bc b, h khng
nh, h sn sng rt lui v cho php hp ng - c u thc cho cng ty
no trnh - c ngh khc tt nht. Cu tr li m h nhn - c
khng nh l khng ch h khng - c php trnh li ngh ca h
m nu h li tm cch rt lui. B quc phng s c hnh ng hp php
chng li cng ty r rng tn km cho h hn l tht thu d kin ca h.
n mc ny, nhng ng- i qun l ca trung tm li nhun chc hn
phi ang chun b thu dn bn lm vic ca mnh th mt trong cc k
s- - a ra gi . ng gi l mt siu i cc nh sn xut - c thnh
lp bao gm nhng k s- gii nht trong trung tm li nhun. Nhng k
s- hn phi - c - a ra khi nhng d n khc chc ang tin hnh v s
ng gp ca h hn khng phi khng ch cht. Ri siu i - c
u thc d n mi vi hon thnh d n c gng bi chi so vi d
ton khi thu cng t cng tt.
K hoch - c ph chun v i pht trin tt nht c - c - c hnh
thnh. Cc thnh vin ca siu i - c yu cu ht sc c gng hon
thnh d n t hn d ton 120 nm cng vi mc tiu gim bi chi ca
d ton 80 nm cng n mc c th - c.
QUAN L DU N PHN MM
202
Th l d n - c i tt nht v trung tm li nhun c th tp hp
- c pht trin trong thi hn ba nm v kt qu l d n - c pht
trin trong 60 nm cng. Giai thoi ngn ny nu bt nhng cu hi c
bn trong vic chun b d ton.
Nhng d ton chuyn mn - c chun b th no? u l d ton
ng trong tr- ng hp ny: 80, 120 hay 60 nm cng ?.
D ton theo nm cng c tt khng khi n l thuc nng vo nhn
thn ca ng- i lm cng vic?
Liu ng c ca nh sn xut c phi l yu t trong vic d ton
ngun cn thit cho vic pht trin d n?
V cui cng, ng- i qun l c th lp ngn sch mt d n ra sao nu
d ton chuyn mn c th b loi ra n 100% ?
Ch- ng ny s cp n nhng cu hi ny hay khc lin quan v
cung cp cng c gip cho nhng quan tm - c tnh n trong vic
chun b d ton pht trin d n.
10.1 D ton ca d n
Mi l- ng ch- a bit u c th d ton trong khi l- ng bit khi
cn d ton. Vi ng- i qun l d n phn mm, c nhiu l- ng ch- a
bit phi d ton. Nhng l- ng ny lin quan n cc lnh vc nh- :
1. Chi ph pht trin d n.
2. Lch trnh pht trin d n
3. Qui m i pht trin d n
4. Khi l- ng phn mm phi pht trin
5. Ngun phn cng yu cu
D ton nhng l- ng nh- th no khng ch l vn cn - c
cp: Cc n v s dng o nhng l- ng cng phi cn - c xem
xt.
Chi ph pht trin d n - c o l- ng tt nht bng n v tin t, nh-
ng ola M hay bng (Anh). D sao vi d ton ban u chp nhn
- c vic chun b bng mt o l- ng trung gian sau - c chuyn i
sang dn v tin t. n v trung gian ph thng cho chi ph pht trin d
n l thng cng (hay thng nhn cngi hay thng k s- v.v.). iu ny
sau c th chuyn i sang gi tr tin t bng cch d ton chi ph ca
mt thng cng n (phn 10.4 ni nhiu hn n vn ny).
Lch trnh pht trin d n r rng - c d ton bng n v thi gian
nh- ngy, tun, thng hay nm. Lch trnh ging nh- phn ln cc k
hoch, phi c khi u v kt thc. Th- ng khi mt d n - c quy
hoch, khi u chnh thc khng r, v do kt thc cng khng r
nt. Trong nhng tr- ng hp , tn tt ARO (sau khi nhn n t)
thng th- ng - c dng v lch trnh - c xy dng s dng nhng ch
QUAN L DU N PHN MM
203
nh nh- : Kt thc thit k mc cao - 4 thng ARO. Ch- ng 9 c trao i
chi tit hn v lch trnh pht trin d n phn mm.
D ton nhn s pht trin c yu cu cho d n r rng theo n v
ng- i. D sao, nhn s pht trin phi - c nhm theo mt s phn loi
ph thng nh- k s- phn mm, ng- i lp trnh, nhn lc h tr v.v. v
mi nhm phi - c d ton ring r. Thi gian v d n ca mi ng- i
cng phi - c d ton bng cch nhn ra vic h tham gia vo d n v
rt ra khi d n. iu ny thng th- ng s to ra - ng biu din phn
b chun
49
vi mt i ng pht trin nh lc ban u v lc kt thc d
n, cn pha gia th c c trung bnh
50
(xem Hnh 9.6.a).
Khi l- ng phn mm - c pht trin th- ng - c d ton bng c hai
cch o: bng dng m hay kil Bai (Kbyte) ca b nh. C hai ph- ng
php c im li v bt li. S dng m l mc o tiu biu nhiu hn v
mc c gng pht trin nh- ng cng l ngn ng tu thuc ch hng
trm dng m ng mc cao khng i hi cng mc c gng nh- hng
trm dng m trnh thnh ng kilobyte b nh l tiu biu v l- ng
phn mm v c th t- ng i tu thuc ngn ng lp trnh gc nh- ng
h- ng dn READ mc cao thi c th to ra nhiu m my hn l thut
ton ton hc phc tp.
L- ng ngun phn mm - c o bng nhiu n v ph thuc ngun
c bit - c xt n. iu ny - c tho lun chi tit hn phn 10.6.
10.2 D ton tng b- c
Ph- ng cch Phn chia v Khut phc th- ng - c p dng trong
nhiu lnh vc cng ngh phn mm (coi ch- ng 6). Ph- ng php ny
(phn chia vn ln thnh v vn vn nh hn) cng - c s dng
trong hu ht k thut d ton. Tip cn c bn l phn gii d n thnh
nhng hp phn d- c xc nh r rng v ri lp li tng b- c n khi
ch cn nhng n v nh sau c th - c d ton d dng hn.
B- c u tin trong phn gii d n l phn chia d n thnh bn loi
sau (coi h.10.1)
1. Hp phn c sn sng.
2. Hp phn y kinh nghim.
3. Hp phn kinh nghim 1 phn.
4. Pht trin mi.
Mi loi biu th mt lp hot ng pht trin c th - c d ton
ring r s dng cc ph- ng php d ton khc nhau. Nh- chng ta s
thy, mi loi cng gn vi mc ri ro lin quan trong pht trin phn
mm.
V mc ch ca tho lun ny, chng ta s ch xem xt cc hp phn
ca pht trin phn mm hin nay v khng phi cc hot ng lin quan
49
- ng biu din ny - c bn thm phn 10.5
50
phn b kch c i pht trin d n s - c tho lun sau trong on 9.5.1.
QUAN L DU N PHN MM
204
ca chng, chng hn nh- qun l, khng ch cu hnh hay bo m cht
l- ng, cc phn s - c bn n sau.
Hnh 10.1
Phn loi pht trin d n theo mc ri ro
10.2.1 Hp phn c sn sng.
Hp phn c sn sng l thnh phn d n tr- c y - c pht
trin l b phn ca cc d n khc. Nhng th d v cc thnh phn thi
th- ng l nhng th- vin ton hc ch- ng trnh con, cc hp th nghim
phn mm (b th nghim) b kch thch ngoi vi phn cng, cc thut
ton v thm ch c nhng hp phn d n ch yu c th - c gn vo
d n hin ti c t hay khng sa i.
Ri ro gy ra trong s dng hp phn c sn sng l ti thiu. Gn
loi hp phn vo d n c kh nng h thp chi ph pht trin ca d
n. Thc t ny - c khuyn khch nhiu trong tiu chun DOD 2167
(coi ch- ng 8). Theo - c coi l phn mm s dng li - c.
Ri ro ti thiu vn cn khc vi khng ri ro v ci gi phi tr cho
vic s dng hp phn c sn sng. Ci gi th- ng l phi tin hnh
vi t chc nng hay t thch ng hn so vi khi ti pht trin cc hp
phn chuyn bit cho d n. iu ny t- ng t b qun o lm sn khng
tn km so snh vi b may o tn km hn nhiu. i khi b lm sn
li c th thch ng hon ho nh- ng th- ng phi c i cht sa sang li.
Hp phn c sn sng th- ng i hi, mc no , thch ng d
n vi hp phn ch khng phi thch ng hp phn vi d n.
10.2.2 Hp phn c y kinh nghim
Hp phn c y kinh nghim l nng ct ca ci m cng ty thc
s tr thnh. Cc cng ty th- ng chuyn mn ho, c ngha Boeing lm
my bay ch khng my thu hnh v IBM lm my tnh ch khng my
bay. Nh- ng c hai u pht trin phn mm.
D n pht trin
phn mm
Cn cht ri ro Ri ro cao
Pht trin mi
hon ton
Ri ro va phi Ri ro ti thiu
C mt phn
kinh nghim
C y kinh
nghim
sn sng
hon ton
QUAN L DU N PHN MM
205
Hnh 10.2
Phn gii chi tit hn v phn loi kinh nghim mt phn
Bao gi cng c xu h- ng pht trin tinh thng trong nhng lnh vc
c th nh- ng s tin b ca cng ngh hn phi chm hn nu cc nhm
pht trin khng m rng nhng lnh vc thinh thng tng thi k.
iu ny cng l ng ca pht trin phn mm. Phn ng cc d n
cha nhng hp phn t- ng t cc hp phn trong cc d n pht trin
tr- c y. Th d v cc hp phn kinh nghim trn vn m cc cng ty
chuyn mn ho hn l s pht trin ca b bin dch. B ch- ng trnh
chuyn mn (h thng k ton hay h thng kim k) v c nhng h ph
ch yu nh- cc mng thng tin lin lc.
Hp phn kinh nghim trn vn l bt c b phn no ca d n phn
mm m chng ta c th ni: chng ti lm - c ci g t- ng t
trong thi gian qua. D sao quan trng hn c l nhn mnh rng t
ND
D
FE
ND : Pht trin mi
FE : C y kinh nghim
C mt phn kinh
nghim
D n pht trin
phn mm
C y kinh
nghim
sn sng hon
ton
Pht trin mi
hon ton
nh gi da trn
kinh nghim qu kh
Hon ton xc
nh
ND
D
ND
D
ND
D
FE FE
QUAN L DU N PHN MM
206
kinh nghim trn vn c hiu lc ch khi kinh nghim - c l- u li.
iu ny c ngha l mt s nhng ng- i sn xut tham gia vo cc d
n t- ng t tr- c y vn cn hin din (h khng ri b cng ty) v
thng tin lin quan n cc d n tr- c y tn ti v - c th nghim.
Vn mt cng ty tr- c y pht trin b bin dch Pascal t thn n
khng l do phn loi pht trin b bin dch C l hot ng kinh
nghim trn vn,c bit nu thnh vin ca i pht trin b bin dch
rt lui khi cng ty v khng li t- liu pht trin no.
C ri ro t- ng i nh trong pht trin cc hp phn kinh nghim trn
vn. C tha nhn c bn l nu chng ta tng lm iu g tr- c
khi chng ta c th lm li - c iu . iu ny khng phi bao gi
cng th. Ri ro chnh l trong sai lm phn loi. iu ny lin quan n
cc tr- ng hp cc hp phn trng c v t- ng t nh- ng trn thc t hon
ton khc. Mt th d hn l s phn loi pht trin h thng vin thng
c kinh nghim trn vn da trn pht trin mng khu vc a ph- ng
tr- c y (LAN). C hai tham gia pht trin h thng vin thng v bao
gm nhng nhim v nh- giao din bin bn v chuyn giao d liu
nh- ng vin thng tham gia cc vn rt khc vi nhng vn lin
quan n mng khu vc a ph- ng. Tip cn ng n hn l phn loi
hot ng trong tr- ng hp ny l kinh nghim mt phn - c bn n
trong phn sau.
Kinh nghim trn vn i hi kh nng nhc li cc hot ng thnh
cng tr- c y.
10.2.3. Cc hp phn kinh nghim mt phn
Kinh nghim mt phn lin quan n cc hp phn phn no t- ng t
vi cc hp phn pht trin trong cc d n tr- c. Th d v kinh nghim
mt phn hn l pht trin b bin dch m mt i tr- c y pht
trin b dch hp ng. i hn tch lu kinh nghim trong phn tch c
php v x l sai nh- ng khng trong ti - u ho hay phn tch c php
phc hp. Mt th d khc (m t cui phn tr- c) l pht trin h
thng vin thng da trn pht trin tr- c y ca mng khu vc a
ph- ng (LAN).
Hp phn kinh nghim mt phn l b phn ca d n phn mm m
chng ta c th ni chng ta quen thuc mc d chng ta hin c khng
pht trin ci g t- ng t trong thi gian qua.
C mc ri ro va phi trong pht trin hp phn kinh nghim mt
phn v y l s t nguyn trong vic gi nh ri ro dn cc cng ty
n m rng chuyn ngnh ca h dn d trong cc pha tin ho ca
cng ty. D sao, trong tr- ng hp ny na cng vn c nguy c phn loi
sai. Phn no quen thuc vi mt nhim v l - c nh ch quan. C vi
nhim v m trong chng ta khng th tm - c ci g chng ta quen
thuc. Nn vic phn loi hp phn l kinh nghim mt phn tu thuc
ng- i phn loi. Gii php c th - c l i hi tho thun ca nhiu
QUAN L DU N PHN MM
207
ng- i d ton v vic phn loi hp phn, nh- ng nh- chng ta s thy
sau, ph- ng cch ny l nn dng nhiu k thut trong d ton.
Yu cu chnh trong phn loi hp phn l kinh nghim mt phn l
kh nng nhn ra c nhng nhn t quen thuc v mi trong hp phn
vy.
10.2.4 Pht trin mi.
Pht trin mi lin quan n vic pht trin cc hp phn khi khng c
kinh nghim lin quan tn ti trong phm vi i pht trin. nh ngha
ny c th phn no du i cn c s vic l cc cng ty my tnh t khi
lp t v cc cng ty t t khi lp my tnh.Nn pht trin mi c th
phi rt ra mt s kinh nghim tr- c y v chng ta gi nh nhng k
nng c bn cn thit hon thin cc nhim v yu cu l c - c.
Khng phi mi nghin cu v pht trin u c bn cht tin ho. Th
d hon ho l ch- ng trnh khng gian Hoa k - c gi qua li tuyn
b c tnh cht tin on ca tng thng Kennedy rng vo cui thp k
(nhng nm 1960) Hoa k s b ng- i ln mt trng. Tr- c khng
c g t- ng t tng - c tin hnh - chc hn l mt pht trin
mi.
Pht trin mi r rng l lp hp phn kh d ton nht v chc mc
ri ro cao nht. Do nhng tin b nhanh chng trong khoa hc my tnh
vi thp k qua, cc d n my tnh cng c kh nng nhiu hn cha cc
hp phn pht trin mi hn l cc ngnh cng ngh khc. iu ny ny
c bit ng vi cc d n phn mm v l mt trong nhng l do chnh
v sao d n my tnh li chng minh ghi nhn - ng i ngho nn trong
d ton ca d n.
Pht trin mi th- ng thiu c s tin cy cho d ton v i hi vn
dng nhng ph- ng php c th c th c - c d ton y . iu
ny s - c bn n chi tit trong phn sau
10.2.5 Phn gii d n theo mc ri ro.
Nh- chng ta thy, phn gii ban u ca d n phn cng nhn ra
bn loi ch yu vi mc ri ro pht trin khc nhau gn theo. Sau b- c
u phn gii d n, chng ta kt thc vi cc hp phn d n hoc (1)
chng ta hin c (thi th- ng) (2) chng ta bit pht trin th no (kinh
nghim trn vn) (3) t ra phn no chng ta quen thuc (kinh nghim
mt phn)hay (4) hon ton mi vi chng ta (pht trin mi). Mc tiu
ca b- c sau trong phn gii d n l tip tc nhn ra nhim v quen
thuc v khng quen thuc. iu ny dn chng ta n xem xt li loi
kinh nghim ring phn (coi hnh 10.2)
Loi kinh nghim ring phn - c m t l cha nhng hp phn
d n phn no t- ng t vi cc hp phn tng - c pht trin trong
nhng d n tr- c y. Do chng ta hi vng c th phn gii tip
nhng hp phn thnh nhng hp phn pht trin mi v kinh nghim
QUAN L DU N PHN MM
208
trn vn nh hn. Trong mt s tr- ng hp nhng hp phn c sn
sng cng c th - c nhn ra trong vic phn gii tip loi kinh nghim
ring phn.
Khi p dng tinh lc tng b- c, chng ta c th lp li v nhn ra mi
hp phn mi ca d n. Lm nh- th, chng ta cng nhn ra mi hp
phn d n m chng ta c th vn dng t- ng i nhng ph- ng php
d ton tin cy da trn kinh nghim tr- c y ca chng ta. iu ny
lm tng thm tin cy ca cc d ton d n tng th ca chng ta.
10.3 D ton ph- ng php mi.
Khi chng ta khng c li ch v kinh nghim rt ra trong vic
chun b d ton chng ta phi tm kim thng tin b sung nhm thc y
t d ton ba pha ti mt ci g ng tin cy hn. Hai tip cn ph
thng l:
1. Ph- ng php nguyn mu.
2. Ph- ng php thng k.
C hai ph- ng php lin quan n mt s hot ng chung nh- l nhu
cu xy dng ci g lm cn c cho cc d n ca chng ta. D sao nh-
chng ta s bit, ph- ng php nguyn mu h- ng vo cng ngh hn
trong khi ph- ng php thng k khoa hc hn. Vi nhng ai ng h cng
ngh hn khoa hc, ch cn ni l nhiu nguyn mu khng bao gi lm
cho n thnh ca hng sn xut - c.
10.3.1 Ph- ng php nguyn mu.
Cc nguyn mu phn mm khc vi cc nguyn mu trong cc ngnh
cng nghip khc. Mt my bay nguyn mu tin tin c th rt t- ng t
my bay sn xut cui cng v c th - c s dng lm m hnh cho vic
xy dng dy chuyn sn xut. D sao,phn mm,ch - c xy dng mt
ln (mc d c th - c sa i sau ny). Mt khi phn mm - c pht
trin, nhng sn phm k tip c th - c to ra ngay bng cch sao chp
phn mm sn xut ra. y l c im duy nht ca phn mm (mc d
vn bn vit i cht t- ng t do kh nng sao chp - c).
Quan nim nguyn mu gi nh ngha hi khc khi p dng cho
phn mm. Cc nguyn mu phn mm l nhng phin bn chc nng rt
gim cu ton b sn phm.
Thm vo tnh chc nng rt gim ca nguyn mu phn mm, tiu
chun pht trin c th t cng nhc nn ph- ng php v tiu chun pht
trin chnh thc t cn n. Cng vy, tnh chc nng - c gi li trong
nguyn mu c th khng - c tinh lc nhiu hn trong ton b sn
phm: chng hn giao din ca ng- i dng c th t thch nghi cho ng- i
dng. Tt c nhng iu to cho chu k pht trin nhanh sn xut
nhanh, mt phin bn ban u ca ton b sn phm phn mm. Th
php ny th- ng - c gi l sao chp nhanh.
QUAN L DU N PHN MM
209
Th d sau chng minh vic s dng ly nguyn mu trong chun b d
ton pht trin phn mm.
Tp on nhng nh sn xut my tnh (CDI) tip th thnh cng
mt h thng chuyn vin cho cc lut gia tip cn lin - c thng tin v
cc tr- ng hp v - u tin php l, lut v th tc php l v danh mc
cc chuyn vin php l trong nhiu lnh vc lut php h thng tc ng
nh- mt h tr php l tin cy v rt nhanh v trong mt s tr- ng hp
thm ch c kh nng gi gii php cho cc vn php l.
Cn c thnh cng ca mnh trong lnh vc php l, qun l ca CDI
xem xt m rng th phn h thng chuyn vin v quyt nh nh
gi ri ro trong vic pht trin mt h thng chuyn vin cho bc s.
cho h nh gi ri ro c hiu qu, qun l ca CDI cn d ton v
qui m v chi ph pht trin h thng chuyn vin y t.
R rng h thng chuyn vin y t c hm cha mi loi ri ro ca d
n m t trong phn tr- c:
+ Cc hp phn c sn sng t h thng php l (nh- khi phc v
nhn ra d liu ca cc chuyn gia).
+ Cc hp phn kinh nghim trn vn (nh- c im bo tr d liu v
lch s tr- ng hp).
+ Cc hp phn kinh nghim ring phn (nh- giao din ng- i dng v
logic gp ).
+ Pht trin mi (nh- kim tra chun on y t v t- ng tc ca
thuc).
Tnh phc tp ca thnh phn chn on y t lm cho kh d ton. Do
, vic xy dng mt tr l chn on y t nguyn mu my tnh ho
hn s hu hiu trong vic cung cp thng tin v khi l- ng cng vic
cn thit pht trin mt phin bn trn vn ca h thng .
Nguyn mu chn on y t hn loI ra nhiu giao din thn thin vi
ng- i dng, cn thit cho vic cho mi thnh cng cc bc s - nhng
ng- i s dng tim n ca h ca h thng. N cng loi tr nhiu d
liu v thut ton y t c yu cu v hn l mt ch- ng trnh c lp
khng cn hp nht vi nhng c im khc ca mt h chuyn gia.
Tnh chc nng hin nay ca nguyn mu tu thuc thi gian v
ngn sch c - c pht trin. R rng ngn sch cng ln v thi gian
pht trin - c php cng di th nguyn mu cng c ch ri tr thnh
cng c d ton ton b h thng. O thi cc, nguyn mu hn cha
mi tnh chc nng ca ton b h thng v theo th hn cung cp d
ton chnh xc ca cc ngun pht trin yu cu.
iu ny v c bn, cng ging nh- pht trin tr- c v d ton sau
khi hon thnh. D ton ny l chnh xc nh- ng v dng. Trong nhiu
tr- ng hp, nguyn mu khng phi l vt vt b v sau ny - c tinh
lc v gn vo ton b h thng. Trong nhng tr- ng hp y c ri ro ca
vic s dng tiu chun ph hay ca cht l- ng phn mm ti trong sn
phm cui cng, nh- l t- liu c th thiu v lp m, thit k c th km
QUAN L DU N PHN MM
210
ci. iu ny c th trnh - c nu quyt nh - c - a ra tr- c khi pht
trin nguyn mu duy tr cc tiu chun pht trin nhm c th lm
cho nguyn mu s dng - c sau ny nh- b phn ca phn mm sn
xut.
tm tt, mt nguyn mu ca cc hp phn pht trin mi l cng
c tuyt ho c th cung cp thng tin ph cn thit chun b cc d
ton tin cy - c. D sao, mt nguyn mu c th l tn km v trn thc
t n cng c th h tr - c nhiu cho vic d ton tin cy th vic pht
trin n cng tn km by nhiu.
10.3.2 Ph- ng php thng k.
Cc ph- ng php thng k s dng k thut ly mu trong d ton cc
thnh phn ca d n. O mc phn gii d n thp nht, nhng n v hay
m un tiu biu - c la chn. Sau nhng n v ny th- ng - c
pht trin v theo th cung cp thng tin v c gng trng i c yu cu
cho s pht trin ca cc hp phn m chng i din. Ph- ng php ny
bao gm nhng pha c bn sau y:
1. Nhn ra mi hp phn phn mm pht trin mi
2. Chun b thit k ban u ca mi thnh phn bng cch nhn ra
nhng m un phn mm mun thc hin hp phn.
3. Phn chia m un thnh cc loi t- ng t theo:
+ phc tp (mc kh)
+ Chc nng (nh- lin lc thng tin, c s d liu, giao din ca
ng- i v.v.)
+ Loi (h thng iu hnh, tin ch th- vin, nhim v dch v v.v.).
4. O mi loi, chn mt m un i din cho cc m un khc.
5. Thc hin cc m un chn.
6. Cn c thng tin suy t thc hin cc m un chn, d ton
ngun yu cu cho mi loi.
7. Phi hp d ton cho mi loi theo th c - c d ton cho mi
phn mm pht trin mi.
Vi nhng d n ln, vic la chn hn mt m un cho mi loi s
to ra d ton chnh xc hn. Th kh x y t- ng t vi ly nguyn
mu. Cng nhiu m un la chn mi loi, d ton cng chnh xc. D
sao, trong tr- ng hp cc oan ta c th d ton mt hp phn da trn
s pht trin trn vn ca hp phn . Mt ln na, iu ny hn to ra
d ton chnh xc nh- ng v dng.
Vic la chn cc phm tr m un v b tr cc m un vo nhng
phm tr i hi kinh nghim rng trong pht trin phn mm. Vic
la chn cc m un i din cng i hi kinh nghim t- ng ng.
Nhng hot ng - c tin hnh tt nht khi ba hay bn ng- i chuyn
QUAN L DU N PHN MM
211
mn so snh cc ghi ch v tm cch t - c tho thun v vic phn
loi v la chn cc quyt nh.
Thit k ban u ca cc hp phn pha 2 l hot ng gay cn. Thit
k l pha ch cht trong pht trin phn mm, mc d s phn gii cc
hp phn thnh m un phi m bo pha ny. Ngay iu cng
khng phi l nhim v d dng, v vi cc d n ln c th i hi c
gng ng k. Thit k ban u cng vi vic thc hin cc m un
la chn i khi c th ti s dng - c trong ton b pht trin ca h
thng. D sao iu ny khng m bo cho ngn sch pht trin nguyn
mu ring r phi - c dnh cho nhng hot ng ny.
10.4 M hnh chi ph xy dng (COCOMO).
Giai thoi lin quan u ch- ng ny minh ho nhiu yu t tc ng
n vic tng qui m ngun lc i hi cho vic pht trin d n phn
mm. R rng, i pht trin tt s hon thnh d n sm hn v vi chi
ph pht trin thp hn i pht trin trung bnh. Nhiu nhng yu t
khc c th - c xem xt khi d ton chi ph pht trin phn mm.
Nhng yu t ph thng hn l:
1. Trnh nhn lc.
2. Mc phc tp.
3. Qui m d n.
4. Mi tr- ng pht trin .
5. Mc tin cy.
Cc yu t t- ng t lc u - c Boehm (1981) trnh by v c nm
trong m hnh chi ph xy dng ca ng, sau ny - c gi l ph- ng
php COCOMO chun b d ton chi ph phn mm. Cc m hnh
t- ng t, nh- m hnh trnh by y, t pht trin tip v - c
sa i, thot u trong vic la chn cng thc c gng s dng cho m
hnh (coi Jeffery and Low (1990) Anderson (1990), Balda v Gustafson
(1990)). M hnh COCOMO l mt thut ton ch n nm yu t trn
(v c th c nhng yu t khc) to lp d ton phi hp vi nh l- ng
ha ri ro. Nhiu ch- ng trnh my tnh c - c thc hin phn ln
cng vic nhm chn m m hnh yu cu (m t thm phn 10.5).
M hnh COCOMO bt u t d ton hin c v s l- ng cc dng
m phn mm phi pht trin. Sau m hnh cung cp cng c cho vic
d ton chi ph, lch trnh v qui m nhn lc pht trin. S dng m c
th - c d ton s dng phn gii d n v cc ph- ng php thng k
m t u ch- ng ny.
M hnh COCOMO cn c mt b cng thc, mi cng thc cho mt
yu t xem xt. Cc th d v cc cng thc nh- sau:
10.4.1 Trnh nhn lc
QUAN L DU N PHN MM
212
Vic o kh nng ca mt k s- phn mm phi ch n mt phm
vi hon thin rng. Sackman v cng s (1968) thuyt minh mt bin
ng 16 trn 1 trong nhng th nghim tin hnh v hon thin ca ng- i
lp trnh.
T khi Sackman tin hnh th nghim ca mnh nm 1968, nhim v
ca ng- i lp trnh tin trin thnh nhiu ngh mi nh- lp m, phn
tch, lp trnh h thng, k s- phn mm v khc na. T k s- phn
mm nhc n ng- i tt nghip i hc hay cao ng c th m nhn
trch nhim c thit k v lp m (lp trnh). Cng vy, t 1968, vai tr
ca lp m tht s gim i trong chu k pht trin tng th ca h
thng phn mm phc tp. Nh- chng ta bit ch- ng 4, cc vic
thit k, tch hp v th nghim chim hu ht cc ngun lc trong
ph- ng cch hin i cho vic pht trin d n. Nu thit k lm tt th
lp m c th i hi khng qu 10% ngun pht trin.
Do chng ta trng i l, qui m rng, bin ng rng ln trong
nng sut ng- i lp trnh m Sackman thy c th l do thiu k thut
pht trin phn mm c ph- ng php vo lc th nghim - c tin hnh.
Bt hnh l bin ng tho ng trng i v s hon thin ca k s-
phn mm vn cn o theo hng trm phn trm. Cho rng ph- ng php
lun pht trin phn mm c trt t - c s dng, vn c th trng i
bin ng ti 400%. iu ny c ngha l ly bnh qun, chng ta c th
trng i mt k s- phn mm c kinh nghim, thng minh v c ch
ch to ra gp 4 ln ng- i k s- phn mm mi bt u (hng xong).
D n cng ln, hon thin c nhn s cng t nh h- ng, yu t ny c
xu h- ng bnh qun da trn t- ng tc nhn lc v cc l thuyt bnh
qun thng k i c- ng.
Bng 10.1 Gii thiu mt th d hm theo bng.
PL = f(EEP,N,KSLOC)
trong trnh nhn lc PL s dng KSLOC l o qui m d n
51
(hng nghn dng ngun ca m) v EP l s hon thin trng i nm
gi- 1 v 4 cho mi k s- , ly tng vi N (l tng s k s- trong d n).
Bng 10.1 Nhn t chi ph cho trnh nhn lc.
(EEP)/N
PL = f(EEP,N,KSLOC)
KSLOC<25 25<KSLOC<300 300<KSLOC
51
Boehm (1981) s dng KDSI (ngn ch th ngun giao) trong nhiu cng thc ng dng.
D sao chi ph pht trin d n khng cn c s dng - c giao m s dng thc - c pht
trin. Hai th th- ng khng nh- nhau v cc dng - c giao c l l bt ngun t cc hp phn d
c (coi phn 10.2) v m pht trin c th khng - c giao v nhiu l do khc nhau (th d m
th nghim). Cng coi Ratchiff v Rollo (1990) tho lun v tnh m h do KDSI.
QUAN L DU N PHN MM
213
4 0.33 0.5. 0.75
3.5 0.45 0.65 0.81
3 0.66 0.85 0.90
2.5 1.00 1.00 1.00
2 2.20 1.80 1.50
1.5 3.50 2.50 2.20
1 4.00 4.00 4.00
Hm ny to ra mt tr s gia 0 v 4 s dng lm yu t - c vn
dng vo chi ph ca c gng pht trin. Tr s PL d- i 1 cho thy i
pht trin tt trong khi tr s PL trn 1 cho thy i pht trin yu tr s
PL bng 1 cho thy i pht trin trung bnh v s khng lm cho d ton
chi ph thay i.
Nh l khi qui m d n tng, tc ng ca trnh nhn lc ti bin
ng chi ph d n li gim. D sao, trnh nhn lc trung bnh l 1 to
nn mt nhn t xu nht cho mi qui m d n, m bng trn - c
biu th l tha s 4.
Cc hm khc t- ng t c th khin ch nhiu n ng gp ca
trnh nhn lc bng cch to ra mt s tri rng cc tr s. O thi cc,
chng ta c th gi nh l trong mt s tr- ng hp i pht trin vi
ton b k s- trnh 1 c th khng bao gi hon thnh mt d n ln.
Nhng con s c thc trong bng trn th hin s ci tin trong mt d
ton kinh ph khng phn tch chi tit. D sao nhng con s phi - c
coi l th d, v vic tinh lc bng phi cn c kinh nghim c thc
- c tch lu trong t chc m d n - c pht trin.
Vic ci tin b sung bng 10.1 nh- vy c th - c thc hin khi tinh
lc cc b- c qui m d n. Bng 10.1 s dng ba b- c qui m d n
KSLOC<25, 25<KSLOC<300 v 300<KSLOC. B qui m b- c chi tit
hn s dng 25, 100, 300 v 500.
Chng ta s xem xt ba th d cn c ba qui m d n vi ba nhm
k s- . Hy xt cc d n sau:
+ D n A vi 10.000 SLOC d ton.
+ D n B vi 100.000 SLOC d ton.
+ D n C vi 500.000 SLOC d ton.
Chng ta s gn cc tr s
1 Cho k s- khi im.
2 Cho k s- trung bnh.
3 Cho k s- trnh cao.
4 Cho k s- ngoi bng.
QUAN L DU N PHN MM
214
C l trng i c hu ht k s- d n c trnh 2 hay 3. Nhiu d
n cho php 1 hay 2 ng- i khi im v nhiu d n s c 1 hay 2 lo
t- ng. Nn nh l trong tr- ng hp lin quan u ch- ng ny, chng ta
c th gi nh i - c tp hp bao gm k s- trnh 3 v 4.
Vi d n A chng ta gi nh 3 k s- phn mm (bao gm thit k h
thng, lp trnh v.v.) khng ng- i no trnh 1, mt trnh 2, mt
trnh 3 v mt trnh 4.
Vi d n B, chng ta gi nh 35 k s- phn mm: chn trnh 1,
m- i tm trnh 2, su trnh 3 v hai trnh 4.
Vi d n C, chng ta gi nh 190 k s- phn mm, 20 trnh 1,
100 trnh 2, 65 trnh 3 v 5 trnh 4.
Bng 10.2 tm tt tnh ton tha s chi ph d n PL cn c tr s
bng 10.1
Tr s PL cho d n C - c tnh trn st vi tr s bng 10.1. Ngoi
suy tuyn tnh gia cc tr s (EEP)/N cng c th - c s dng xc
nh tr s PL cho d n C.
Kt qa nu bng 10.2 cho thy d n A - c h tr bi i pht
trin cp cao gim chi ph pht trin d n d tan i mt phn ba. Mt
khc, d n B c i pht trin yu hu nh- gp i chi ph pht trin d
n. D n C c i pht trin trung bnh.
Bng 10.2 Tha s chi ph PL tnh cho ba d n.
A B C
KSLOC 10 100 500
N 3 35 190
EEP
9 71 435
(EEP)/N
3.0 2.0 2.3
PL 0.66 1.80 1.00
Bi tp 4 cui ch- ng ny cp n cc hm ci tin ln h s
trnh nhn lc vo tnh ton d ton chi ph.
10.4.2 Mc phc tp.
Mc phc tp ca phn mm l yu t c ngha trong vic chun
b d ton d n. R rng l mt s lp phn mm kh pht trin hn
nhng lp khc nhiu. Ni chung, cc h thng iu hnh phc tp hn
h x l d liu in hnh nn s - ng ca m khng l ph- ng tin
so snh c hiu lc cho s pht trin ca hai h thng nh- th. Do y
tho ng l phn chia hp phn phn mm thnh lp theo mc phc
tp ca chng v c nhng o phc tp khc nhau cho mi lp.
Boehm (1981) chn ra ba mc phc tp ca ch- ng trnh, hu c
tch mt na v gn lin. Chng ta hy xem bn lp phc tp sau:
QUAN L DU N PHN MM
215
1. Phn mm ca h thng: Lp phn mm ny bao gm bt c phn
mm no gn vi phn cng nh- h vn hnh v h thng tin lin lc.
2. Phn mm thut ton: lp ny bao gm bt c phn mm ph thuc
nhiu logic v thut ton phc tp nh- ch- ng trnh khoa hc, tin ch
loi v phn mm dung th li.
3. Phn mm dch v: Bao gm tin ch c bn nh- bin tp, x l
lnh v ch- ng trnh ho.
4. Phn mm x l d liu: bao gm ng dng c bn d liu chung
nh- ch- ng trnh kim k, b pht bo co v phiu mu.
Bn lp phn mm i din cho nhng mc chnh ca tnh phc
tp phn mm. Trong vi tr- ng hp s khng hin nhin ngay l mt s
ch- ng trnh thuc v lp no. Trong nhng tr- ng hp , chng ta phi
xp ch- ng trnh vo lp gn gi nht v phc tp. Hn l c th xem
th d v s pht trin b bin dch. Tnh phc tp ca b bin dch ch
yu do nhng thut ton phc tp tham gia vo cc lnh vc nh- phn tch
c php, phn tch ng php, ti - u ho v.v. Do y b bin dch phi xp
hng l phn mm thut ton.
Cng thc sau c i cht t- ng t vi cng thc m Boehm ngh
dng bn lp phc tp thay cho ba (khng k l dng KSLOC thay cho
KDSI). Mi cng thc to ra mt s d ton cc thng k s- phn mm
(SEM) da trn s dng m v lp phc tp phn mm.
Phn mm h thng SEM = 3,6 (KSLOC)
1,20
Phn mm thut ton SEM = 3,2 (KSLOC)
1,5
Phn mm dch v SEM = 2,8 (KSLOC)
1,10
Phn mm x l d liu SEM = 2,4 (KSLOC)
1,05
Cc hm t- ng t - c Faily (1985) lp th v cc th chng
minh s phn nhnh ca c gng d ton (thng k s- phn mm) khi qui
m ca d n tnh bng s dng m l tng ln. iu c ngha l trong
vic chun b cc d ton, vic phn loi phn mm tr nn quan trng
hn khi qui m ca d n tng. Trn 100.000 dng m iu tr nn c
ngha v trn 300.000 dng m th chnh lch gia 2 thi cc (h
thng v x l d liu) c th trn 200 phn trm.
Nhiu d n cha cc hp phn phn mm thuc cc lp phc tp khc
nhau. Cch hiu qu nht vn dng cng thc trn l phn gii phn
mm thnh cc hp phn, xp mi hp phn vo mt lp phc tp ca n
v ri d ton cho mi hp phn ring r. B cacs d ton c - c s
- ck kt hp li - a ra mt d ton duy nht cho ton b d n.
Trong cc tho lun trn, thng cng k s- phn mm (SEM) - c
nhc ti cho mi kiu ng- i chuyn nghip phn mm tham gia trong
vic pht trin phn mm. iu quan trng cn nh l thng cng k s-
khng ging nh- thng lch. C th xy ra l mt 6 tun hon tt cng
QUAN L DU N PHN MM
216
vic 1 thng cng k s- phn mm (cng y nh- i vi cc nhn vin
khc), do v au m, v ngh php, cc ngh khc. Cc ch ny - c
tho lun k hn trong ch- ng 9.
10.4.3 Yu t o tin cy
Mc tin cy i hi trong d n phn mm c th c tc ng ch
yu n chi ph pht trin. tin cy, cng nh- nhn t phc tp
bn n tr- c y, cng c th lm tng chi ph pht trin ln hn 200%.
tin l phn cht l- ng tn km trong cc h thng my tnh v c
th - c thc hin qua phn cng, qua phn mm hay qua phi hp ca
c hai. tin cng kh thc hin v phi - c tnh h s trong d ton
chi ph pht trin. Cc h thng dung sai khuyt tt i hi cc pha th
nghim v hp nht tn km. iu ny l do dung sai khuyt tt v cc
mc tin cy khc kh th nghim.
Tr- c khi xem xt cc nhn t tin cy cho vic pht trin phn
mm, phi c quyt nh v con s cc mc tin cy s - c dng.
Boehm (1981) s dng nm mc tin cy cn c tc ng li ca h
thng.
Khng thun li nh.
D phc hi tht thot.
Kh khn va trong phc hi tht thot.
Tht thot ti chnh cao.
Ri ro cho tnh mng ng- i.
Thun tin l thy tht thot v tnh mng con ng- i - c xp hng
cao hn tht thot v ti chnh.
Ph- ng cch c bn l phn tin cy ca h thng thnh cc mc t
tin cy ti thiu (khng c cng sc c th dnh cho tin cy) ti
mc cao nht (dung th li ti a).
Nhm - a tha s tin cy vo d ton chi ph cho vic pht trin d n
phn mm, cn c pha phn gii ban u, t- ng t ph- ng php m t
trong phn 10.2. D sao, mc tiu lc ny l phn gii h thng thnh lp
hp phn theo mc tin cy.
Sau phn gii ban u, bng cc nhn t cng sc tin cy - c vn
dng cho mi hp phn v - c tnh cho mi mc, cung cp nhn t
cng sc tin cy (REM). Tr s trong bng tin cy - c cn c
kinh nghim m cng ty hay t chc chu trch nhim pht trin d n
phn mm tch lu - c. Hm cng sc tin cy t- ng t hm m t
d- i y c th - c dng to ra bng tha s tin cy.
REM = 0,75 + (L-1) 1,25/(2 N-L-1)
Trong hm ny, N l o mc tin cy v L l mc nhn t phi
tnh. Cc nhn t tin cy c - c hm ny cho 5 mc - c trnh by
bng 10.3.
QUAN L DU N PHN MM
217
Bng 10.3 nhn t tin cy cho 5 mc tin cy
Mc tin cy Nhn t tin cy
1. Khng cn cng sc 0,75
2. Thp 0,93
3. S ton vn d liu - c bo ton 1,17
4. Yu cu tin cy cao 1,50
5. Dung th li y 2,00
Trong th d trn, nhn t cng sc tin cy tng 0.18 t mc 1 ti
mc 2 v tng 0.5 t mc 4 ti 5. iu ny ch ra khi tin cy tr nn
ti hn i vi d n, th cng sc t- ng ng tng ln phi tuyn tnh. S
tht l vi cc h dung th li hon ton (mc 5) th i hi phn ch yu
ca cng sc pht trin l u t- vo vic m bo tin cy.
Cng ng ghi nhn l hm trn - a vo mt mc tng chi ph ln ti
200 phn trm chi ph trong vic thc hin cc mc tin cy khc nhau
(0.75 ti 2.00). iu ny c ngha l pht trin mt h thng vi tin
cy cao c th tn km gp ba ln pht trin mt h thng khng i hi
cng sc u t- vo tin cy. Trong mt s tr- ng hp, nhn t thm
ch c th cao hn,
Sau khi xc nh nhn t cng sc tin cy, nhng tr s ny - c
vn dng vo d ton SEM (coi tho lun tr- c y v tnh phc tp ca
d n) cho mi hp phn trong mi lp theo th - a h s tin cy vo
d ton. D ton cho mi hp phn - c phi hp to nn d ton chi
ph tng th cho d n.
10.4.4 Mi tr- ng pht trin.
Bt c ai c khi no tm cch sa cha nh nh hay xe u nhn thy
nhim v d thc hin hn khi c - c cng c ng. Phi mt bao lu
xn bi c? , iu ph thuc kch c bi c. D sao, iu
cng tu thuc vic bn s dng my xn c y hay my xn c chy
dng c (chng ta thy l iu cng ph thuc con ng- i ng
sau my xn c). Khng ngc nhin g, nhn xt ny cng l ng nh-
pht trin phn mm.
Mt trong nhng ng gp ph bin nht cho yu t mi tr- ng pht
trin l s dng ngn ng bc cao. Khi m ngn ng lp trnh tr ln hiu
lc hn, ng- i ta t mun pht trin phn mm trong cc hp ng, tr
trong nhng tr- ng hp him hoi. C nng sut v tin cy ca m
nhiu ln cao hn khi s dng ngn ng bc cao so snh vi hp ng.
Nhn xt n gin tnh nhiu n ph pht trin phn mm, nn n
th- ng - c - a lm yu t trong chi ph pht trin mt cch ring r.
QUAN L DU N PHN MM
218
Ngn ng lp trnh thc ra ch l mt th d n gin v tc ng m
mi tr- ng pht trin c th c i vi nng sut. Mi tr- ng phn cng
cng l quan trng nh- cng c phn mm sn c trong phn cng pht
trin. Nu phn cng mc ch c bit - c pht trin cho d n th
nhng b tm sai tt v cc cng c phn tch phn mm khc tr nn
chnh yu cho vic th nghim v tch hp c hiu qu. Goldberg (1938)
cp n s ng gp ng k ca mi tr- ng cng ngh phn mm
- c h tr bng my tnh tt (mI tr- ng CASE) n nng sut pht
trin.
Bng 10.4 Nhn t mi tr- ng pht trin
Mc mi tr- ng pht trin Nhn t chi ph d n
1 (ngho) 1,5
2 (va phi) 1,0
3 (tuyt) 0,5
Khc vi cc yu t tr- c y cp, tc ng ca mi tr- ng pht
trin ti nng sut khng nht thit chu nh h- ng bi qui m d n. Cc
mi tr- ng pht trin tt l cn thit cho c cc d n ln v c cc d n
nh. Cn c gi nh , mt bng duy nht cc nhn t cho mi mc
mi tr- ng pht trin - c s dng vn dng yu t ny vo chi ph
pht trin d n.
Chng ta hy xem ba mc mi tr- ng pht trin:
1 Mt mi tr- ng pht trin ngho c t cng c pht trin v ph- ng
tin phn cng pht trin khng thch hp.
2 Mt mi tr- ng pht trin va phi c cng c pht trin tt v nn
tng pht trin phn cng tt.
3 Mt mi tr- ng pht trin tuyt vi vi cng c CASE v ph- ng
tin pht trin phn cng ring bit cho mi k s- .
Chng ta s gi nh l mi tr- ng pht trin tt c th tng hiu qa
pht trin ln 50% v mi tr- ng pht trin ti c th gim hiu qu i
50%. B nhn t c - c - c gii thiu bng 10.4.
Bng ny c th - c tinh ch bng cch - a vo nm mc nhn t,
mt gia mc 1 v 2 (cho cc mi tr- ng c mt s cng c c bn
nh- ng vn ch- a thch hp) v mt gia mc 2 v 3 (cho cc mi tr- ng
c b cng c tt nh- ng khng tuyt vi). Mt cch tinh ch ph ca
bng hn l - a vo qui m d n trong yu t. iu ny hn i hi tng
tc ng ca mc pht trin khi d n tng v qui m. Trong cc d n
m phn cng mi - c pht trin, th tr- ng hp ny rt c th xy ra
l cc ph- ng tin khng thch hp c th lm cho pha hp nht cc d
n ln hu nh- kh c th hon thnh.
QUAN L DU N PHN MM
219
10.4.5 Cc h ph
Buc phn gii u tin cho cc h thng phn mm ln th- ng thu hi
nhng hp phn h thng c lp mc cao, th- ng - c gi l h thng
ph (iu ny - c ni k hn ch- ng 6). Cc h thng ph l hp
phn ca mt h thng, m n c th cng - c coi l bn thn n l mt
h thng. Chng ta c th c li tnh c lp t- ng i ca cc h thng
ph trong ci tin d ton kinh ph cho ton h thng.
Cc h thng ph c th biu th bng yu cu phc tp, tin cy
v cc thuc tnh khc, cng theo cch t- ng t vi cch m cc hp
phn mc thp - c phn loi. Mt h thng th qu ngn hng t ng
c th c h thng ph lin lc trung tm h thng ph th qu v h
thng ph giao din c s d liu ca ngn hng hai trong cc h thng
ph tu thuc nng vo lin lc v mt th ph thuc nng vo x l
d liu v c s d liu. Mt h thng th qu ngn hng t ng c th
- c cu thnh t h con truyn thng trung tm, h con th qu v h
con giao din c s d liu ngn hng.
Bng 10.5 Nhn t tin cy h thng ph cho nm mc tin
cy
Mc tin cy ca h thng ph Nhn t tin cy h thng ph
1 Khng cn cng sc g 0.80
2 Thp 0.95
3 D phng ton vn d liu 1.10
4 Yu cu tin cy cao 1.25
5 Dung sai khuyt tt ton b 1.40
Thuc tnh ca cc h thng ph c th - c - a thnh tha s vo d
ton chi ph h thng s dng cc ph- ng php t- ng t nh- m t cho
h thng hon chnh. H thng ph ny khng - c phn gii tip nh- ng
li - c coi l mt n nguyn duy nht tp trung vo cc hot ng mc
ca h thng ph nh- hp nht v th nghim.
Trong th d th qu ngn hng t ng trn, chng ta vn dng bng
nhn t tin cy t- ng t nhu bng 10.3 nh- ng nhng nhn t ch
- c vn dng cho h thng ph nh- l mt n nguyn duy nht (coi
bng 10.5).
H thng ph c s d liu trong th d tr- c c th - c t cho mc
tin cy 3 m bo tnh ton vn ca d liu v chng ta c th hi kh
chu nu ngn hng ca chng ta khng th cung cp ngay cho chng ta
quyt ton ti khong ca chng ta nh- ng hn chng ta bt bnh nht nu
nht thi ngn hng gi cho chng ta nhng thng bo ngn hng sai ln.
H thng ph lin lc trung tm c - c xp loi mc 5 v tht bi trong
h thng c th gy ra tht bi trong mi my mc th qu t ng. H
QUAN L DU N PHN MM
220
thng ph th qu c th phn loi mc 4 ngay d khng c c s d liu
th- ng trc nm trong my th qu v ngn hng hn ht sc chc chn
trnh sai lm trong vic pht tin mt cho khch hng ca mnh. Nhn t
thch hp sau c th vn dng cho mi h thng ph nhm - a thnh
tha s mc tin cy vo d ton chi ph. Nh l sai lch tr s cho cc
nhn t ca h thng ph r rng t hn vi cc hp phn mc thp (so
snh vi bng 10.3). iu ny - c lm b tr cho tha s tin
cy - c - a vo mc thp hn.
Tnh phc tp ca h thng ph c th - c - a thnh tha s vo chi
ph pht trin theo cch t- ng t. Nhng nhn t phc tp c th - c b
tr cho mi h thng ph. iu ny n gin hn ph- ng php m t
tr- c y cho cc hp phn phn mm mc thp hn v qui m ch- ng
trnh khng - c tnh n mc h thng ph.
Cc thuc tnh khng tu thuc h thng ph phi khng - c p
dng mc h thng ph. Trong phn ln tr- ng hp, trnh nhn lc
phi khng - c - a thnh tha s vo d ton chi ph pht trin v tha
s no bnh th- ng phi khng bin bin trn mc h thng ph. Khi
nhn lc hon ton khc - c b tr cho mi h thng ph, thng th- ng
chng ta coi mi h thng ph l h thng ring r v mc ch chi ph
pht trin d ton.
10.4.6 Thut ton d ton chi ph
Thut ton sau suy t m thc gc COCOMO ca Boehm (Boehm
1981) v gm 10 b- c c bn cho vic hnh thnh d ton chi ph pht
trin d n. Cc b- c ny bao gm vic phn gii d n ra hp phn, vn
dng cng thc cng sc cho mi hp phn v phi hp mi d liu to
thnh vo d ton chi ph duy nht ca d n. Thut ton c bn bao gm
cc b- c sau:
1. Phn gii h thng phn mm s dng tinh lc tng b- c, thnh cc
h thng ph v ri phn gii mi h thng ph thnh cc m un phn
mm mc thp.
2. S dng ph- ng php d ton qui m (nh- m t trong phn 10.2)
d ton qui m ca mi m un. Sau phi hp cc d ton cho mi
m un theo th to lp d ton cho qui m ca mi h thng ph v cho
ton b h thng.
3. Xc nh cc nhn t cng sc cho mi m un, s dng cc
ph- ng php t- ng t nh- m t u phn ny. Cc nhn t cng sc s
dng t ra phi bao gm cng thc v:
+ Trnh nhn lc.
+ Qui m d n.
+ tin.
+ Mi tr- ng pht trin.
+ Tnh phc tp ca m un.
QUAN L DU N PHN MM
221
4. Vn dng cc nhn t cng sc vo mi m un s dng ph- ng
php nh- m t u phn ny theo th to lp d ton cho mi m un
5. Xc nh cc nhn t cng sc ca h thng ph cho mi h thng
ph
6. Phi hp cc d ton cho m un trong mi h thng ph vi nhn
t cng sc h thng ph theo th to lp d ton cho mi h thng ph.
7. Phi hp d ton cho mi h thng ph, theo th to lp d ton
cho ton b h thng.
8. Duyt li cc yu t - c xem xt trn m un v mc h thng
ph. Tm t- ng tc gia cc yu t, cc m un v gi- cc h thng ph
v to lp t- ng tc
9. Tm chi ph ph b qun d ton h thng nh- phn tch, th
tr- ng, tng ph v.v.. V phi hp vi d ton h thng to lp.
10. Chun b d ton c lp th hai (v nu c th th ba). So snh cc
d ton m mi nhm to lp v kim tra bt c khc bit c bn no.
Gii quyt bt k khc bit no v to lp d ton chi ph d n duy nht
- c chp nhn
Thut ton ny to lp d ton chi ph pht trin d n ch mi yu
t ch yu. B- c 10 cng cp sai lm c nhn trong d ton bng cch
yu cu gii thch v gii quyt nhng khc bit ch yu.
B- c 1 lin quan n t m un phn mm ng ngha vi n v phn
gii phn mm mc thp nht.
B- c 8 nhm xc nh cc yu t sao chp phn no hay trn vn
trong d ton. Th d v sao chp trn vn l b tr yu t phc tp cho
hp phn phn mm do nhng yu cu v tin cy cao v vic to lp
tin cy th- ng i hi logic phc tp. iu ny c th dn n trong d
ton cho hp phn - c gia tng hai ln, mi ln v cng l do.
Nhng yu t ph c th gp trong cc b- c 3 v 5 cn c c im
ca d n hin nay - c d ton. iu ny c th bao gm nhng yu t
nh- tnh phc tp ca ngn ng lp trnh (nu, chng hn Ada - c s
dng ln u) hay tnh quen thuc vi phn cng ch ch (nu phn
cng c bit - c pht trin v do y pha hp nht hn kh khn hn).
Vic thc hin y thut ton ny phi bao gm mi cng thc cng
sc trn.
iu quan trng l phi nh rng s phn loi cc hp phn phn mm
l khc nhau cho mi yu t. iu ny c ngha l khi vn dng nhng
nhn t tin cy vo cc lp hp phn, nhng hp phn chc hn
phi khc nht vi hp phn to lp khi cc nhn t phc tp - c vn
dng. Do y mi b nhn t - c vn dng ring cho mi hp phn
phn gii. C iu ny ni no m my tnh c th c ch nht v nh-
ni tr- c y, mt s cc khi ch- ng trnh my tnh COCOMO phi
c - c hon thin nhng nhim v nhm chn ny
Thut ton COCOMO tu thuc nng cc quyt nh ch quan ca
ng- i d ton. iu ny dn n vic pht trin nhiu bin thc ca
QUAN L DU N PHN MM
222
COCOMO v cng thc cng sc (coi Jeffery v Low (1990), Anderson
(1990) v Balda v Gustafson (1990)). Vn sai lch ca cc d ton
ch quan ca nhng ng- i d ton khc nhau - c nu trong phn 10.6.
10.5 Phn tch l- ng chc nng.
C mu thun ln khi xem xt tr s dng m ngun l o qui m
d n (coi Ratcliff v Rollo (1990) Jeffery v Low (1990)). Khng c
nh ngha chung no cho o SLOC: c th hay khng th bao gm
m th nghim hay m dng li hay trong mt s tr- ng hp c m th-
vin. Cng vy liu SLOC c thc s c ngha nh- vy cho m ch- ng
trnh hp ng v m Ada khng ? hay m Cobol v m C ? liu mt yu
t duy nht c th thc s - c vn dng (coi cc yu t COCOMO)
lm cho mi ngn ng lp trnh so snh - c?
Cc cu hi ny c th trnh - c khi h- ng qu trnh d ton vo qui
m vn d n ch khng phi qui m m d n. Phn tch l- ng chc
nng (FPA) l ph- ng php to lp d ton d n cn c qui m vn .
Qui m vn l o suy t cc pha d n ban u c bit pha yu
cu. S l- ng cc chc nng trong d n xc nh qui m vn , n - c
biu th bng s (tr s FPA).
Tr s FPA ca mt d n c th - c dng :
+ So snh tnh phc tp ca cc d n
+ So snh cng sc t- ng i c yu cu hon thnh d n
+ Pht sinh cc o d n khc (nh- SLOC
52
)
C nhiu bin thc ca kthut FPA. Nhiu nhng bin thc
nhm thch ng k thut vo cc loi d n c tr- ng hay tng - c
chnh xc bng cch thm thuc tnh d n vo qu trnh FPA.
10.5.1 Cc b- c FPA c bn.
Qa trnh FPA c bn bao gm tm b- c. Hai trong nhng b- c c
th - c chun b c lp vi d n - c d ton v chng tham gia vo
cc danh sch nh cc loi hm s v cc thuc tnh phc tp - c s
dng phn loi c tnh ca d n. Tm b- c FPA c bn l:
1. Xc nh danh sch cc loi chc nng ph thuc vo v ra. iu
ny c th bao gm
53
.
Chc nng yu cu/nhp vo t ngoi
+ u vo d liu hay kim tra ca ng- i dng.
52
Jeffery v Low (1990) m t ch- ng trnh gi l CLAIR - c dng i l- ng chc nng
(FP) sang SLOC.
53
cc phm tr hm s ny t- ng t vi nhng phm tr m Albrecht v Gaffney (1983) gi .
QUAN L DU N PHN MM
223
+ Cu hi ca ng- i dng i tr li.
Chc nng ra ngoi
+ Chc nng ra tn hiu hay d liu d thy
Chc nng file trung gian lgic
+ Thng tin iu khin hay d liu
Chc nng file giao din ngoi
+ Thng tin kim tra v d liu v cc file dng chung.
2. S cc chc nng phn mm c bn ca mi loi - c xc nh.
Mt chc nng phi - c tnh nu c d kin yu cu x l c bit.
3. Mi chc nng - c m trong b- c 2 - c xp loi l:
n gin: tip cn file ti thiu, t kiu d liu khc bit v tham
gia ti thiu ca ng- i dng.
Trung bnh: Loi ny - c dnh cho nhng chc nng gia n
gin v phc tp. Trung bnh c th - c phn chia ph thnh nhiu
hn mt phn loi trung gian.
Phc tp: Nhiu tip cn file, nhiu kiu d liu khc nhau v tham
gia rng ri ca ng- i dng.
4. Tr s trng s - c gn cho mi b phn loi trong b- c 3 (th d
n gin = 6, trung bnh = 8, phc tp = 10 hay trung bnh c th m
rng thnh 7, 8 v 9). Mi loi chc nng c th c mt b trng s khc
nhau.
Cc gi tr ca mi chc nng gn trng s u - c thm vo,
cung cp tr s FPA ch- a - c phn xt (UFP).
5. Cc thuc tnh ca phc tp x l - c xc nh. N c th
bao gm
54
.
Chc nng truyn thng d liu
+ D liu v kim tra - c chuyn khu vc v xa xi
Chc nng phn tn
+ Chc nng d liu phn tn
+ Chc nng x l phn tn
Hon thnh
+ Mc tiu hon thin nh- nh h- ng ca nng sut v
nhy ti cc hot ng pht trin
S dng cu hnh
+ Mc s dng phn cng: - ng truyn thng.
T sut giao dch
+ Mc t sut giao dch nh h- ng ti pht trin
Nhp d liu th- ng trc
+ Mc cc chc nng d liu th- ng trc - c h thng x
l
Hiu qu ca ng- i dng cui
54
Albrecht v Gaffney (1983) gi 14 nhn t phc tp v Symons (1988) gi ph thm 6.
QUAN L DU N PHN MM
224
+ Hiu qu yu cu ca x l chc nng d liu th- ng trc
- c ng- i dng cui thc hin
Cp nht th- ng trc
+ Mc cp nht yu cu cho cc file trung gian logic
X l phc tp
+ Mc nh h- ng ca x l phc tp ti pht trin. iu ny
bao gm b x l ngt, m tI-nhp, thut ton phc tp, I/O v.v.
Ti s dng
+ Mc m phi - c pht trin coi nh- ti s dng cho cc
h khc.
D lp t.
+ Mc vic d lp t nh h- ng n pht trin
D vn hnh
+ Mc yu cu v vic d dng trong nhng chc nng nh-
sao l- u, phc hi, giao din ca ng- i v.v.
Cc im a bi
+ Mc m h thng - c pht trin cho cc im khc nhau
v cc loi ng- i dng khc nhau
Gip thay i d dng
+ Mc m phn mm phi - c pht trin h tr thay i
chc nng d dng.
6. Mi mc x l nh h- ng ti mi yu t phc tp - c ch nh
cn c 1 trong nhng tr s sau:
0: Khng tn ti
1: Khng ng k
2: Va phi
3: Trung bnh
4: ng k
5: Mnh
v tng s cc tr s yu t phc tp - c tnh cung cp tng mc nh
h- ng (TGI)
7. Tng mc tr s nh h- ng - c chuyn i sang nhn t iu
chnh phc tp (CAP). Hm chuyn i n gin c th l:
8. o chc nng iu chnh (APF) cho d n - c tnh l:
AFP = CAF x UFP
D
CAF
D
5 x (s cc nhn t phc tp)
TGI
D
QUAN L DU N PHN MM
225
Bng 10.6 Cc tr FPA cho cc kiu d n khc nhau
D n HFP C FP A FP
H thi gian v bo
d- ng
1200 0.42 504
H kim tra trung nhp 680 0.87 592
Bng 10.6 c th d v cc tr s c - c phn tch chc nng. Cc tr
s trong bng chng minh tm quan trng ca vic iu chnh tr s chc
nng. H thng x l d liu th- ng mi ln cung cp UFP ln hn h
thng thi gian thc nhiu, nh- ng s iu chnh phc tp ny to nn tr
s iu chnh cao hn cho h thng thi gian thc. Kt lun l ngay d h
thi gian v bo d- ng c s chc nng hu nh- gp i h thng kim
tra truy nhp, tnh phc tp ca h kim tra truy nhp cho thy n s i
hi nhiu cng sc hn pht trin.
10.5.2 ng dng ca FPA.
C nhiu t- ng ng gia d ton COCOMO v phn tch im hm.
C hai ph- ng php s dng k thut cht ti t- ng t iu chnh d
ton ban u. D sao, FPA ch ph thuc tnh chc nng ca h thng
- c d ton v khng bt c d ton no tnh tr- c y (th d
dng m, qui m m un).
Nh- chng ta thy, o FPA l c ch so snh cng sc ca cc
d n. N khng trc tip cung cp ph- ng php d ton chi ph ca mt
d n. Cn c gi nh l c t- ng quan cao gia chi ph v cng sc,
hm c th vn dng cho tr s FPA c - c d ton chi ph. Hm n
gin c th da trn kinh nghim qua chng hn d n tr- c y vi tr
s AFP l 1.000 ht $1.500.000 pht trin v d n khc tr- c y vi
tr s AFP 850 ht $900.000. Nu ta gi nh t- ng quan tuyn tnh, th
nu d n hin hnh - c d ton c tr s AFP l 920 chng ta s d
ton chi ph ca n l $1.800.000.
Nhng ph- ng php tnh vi hn tnh cng sc (gi lao ng) v qui
m m (KS LOC) - c Albrecht v Gaffney tnh by (1983).
Ph- ng php COCOMO c th - c vn dng cng vi phn tch im
hm thc hin 2 mc ch quan trng:
1. m bo l d ton hp l (khng chnh lch kt qu ng k)
2. To ra b d ton d hiu hn(tr s so snh v tr s chi ph).
Vic s dng COCOMO kt hp vi phn tch im hm - c Ratcliff
v Rollo (1990) nghin cu i n kt lun l mc tiu di hn c hiu
qu hn hn l s pht trin m hnh d ton chung mi cho h dc vn
hnh c th cho rng to nn mt b d ton d hiu hn. Nh- ng
QUAN L DU N PHN MM
226
COCOMO v FPA - c phi hp thnh cng trong tin ch d ton
my tnh ho d hiu
55
s dng c hai k thut v d ton chi ph v qui
m ca d n phn mm. Thuc tnh ca hai k thut trong tin ch ny
c th - c iu chnh da trn kinh nghim c - c trong khi s dng
v nhng m hnh xen k (thuc tnh ca mi m hnh) c th - c l- u
gi ring r v cht ti trong tin ch khi cn.
10.6 D ton l mt lnh vc.
Nu - c hi mt bao lu pht trin mt m un phn mm duy
nht, th mt nh lp trnh c kinh nghim hn chc chn tr li iu
cn tu. Nh- chng ta thy, iu tu thuc vo ngn ng lp trnh,
vo tnh phc tp logic, vo c nhn ng- i lp trnh v c th vo nhng
yu t khc. Nu b p tr li r, th cng nh lp trnh hn s tr li l
c th mt u trong khong t 2 ngy v n 2 tun. y l cu tr
l c ga tr. D ton th- ng - c trnh by l mt phm tr. Phm tr l
mt d ton c ch trong lp k hoch pht trin ca mt d n. Qun l
th- ng s sn sng chp nhn mt d n ban u cho bit d n s khng
d- i $400.000 v khng trn $750.000. Nhng loi d ton nh- th
th- ng - c dng trong cc pha qui hoch ca mt d n. Cng tr nn
c - c nhiu thng tin, phm vi cng tr nn - c thu hp hn, thm ch
c khi tr thnh mt con s duy nht.
L thuyt thng k ng sau tip cn cp n quan nim khong
tin cy. Nu mt bin x c xc sut p gia hai tr s a v b th chng ta
ni khong cch (a,b) l khong cch p-tin-cy ca x.
Ly th d, nu (2,14) l khong cch tin cy 95% c s ngy cn
pht trin mt m un th chng ta c 95% chc chn l pht trin m
un s mt hn 2 v km 14 ngy pht trin
56
.
Thm mt li v s chc chn 95%. Nu chng ta c 95% chc chn v
ci g, th trong 100 tr- ng hp chng ta trng i ng 95% ln v 5 ln
sai.
Phm tr xt n y s l phm tr d tnh cho mt thuc tnh d n
cho, nh- SLOC, nh- chi ph pht trin, nh- qui m i pht trin, nh-
thi gian pht trin v.v.
Chng ta hy gi nh mt phn b chnh tc d ton. iu c
ngha nu 20 ng- i chuyn mn phn mm - c yu cu d ton s ngy
cn pht trin mt m un phn mm c tr- ng, chng ta trng i kt
qu t- ng t trong phn b vi nh- sau:
1 ng- i s d ton 2 ngy
55
Tin ch my tnh tham kho, Before you leap (BYL) - c nhm Gordon Group, California cng
b
56
cht ch m ni nh ngha ton hc ca xc sut lin quan n hm to ra tr s gia 0 v 1.
D sao trong th gii thc c bit trong th gii kinh doanh xc sut th- ng - c biu th theo
phn trm trn thc t trong th nghim ny ca bn thn tc gi phn trm - c chung trong
trnh by d ton cho qun l v chng d nm bt hn.
QUAN L DU N PHN MM
227
2 s d ton 5 ngy
4 s d ton 7 ngy
6 s d ton 9 ngy
4 s d ton 11 ngy
2 s d ton 13 ngy
1 s d ton 16 ngy
H10.3
Phn b chnh tc d ton.
Hnh 10.3 gii thiu th ca cc con s trn. - ng biu din c
- c, trong thng k - c coi l th hm phn b chnh tc
57
. Phn b
chnh tc biu th bng th hnh dng u n c phn b u cc din
bin quanh s trung bnh. Trong th d trn, d ton bnh qun l 9 ngy
vi cng nhiu d ton d- i trung bnh nh- trn v quan trng hn c, c
tn v c ly cc d ton d- i trung bnh l t- ng t nh- nhng ci trn
trung bnh. Nhiu s kin th- ng xy ra v bn cht din ra l phn b
chnh tc; chng hn chiu cao ca sinh vin nam (hay n) trong mt lp
hc, hay s nhng ngy m- a thng 4 ngoi tr n- c Anh, bao gi cng
l 30.
Gi chng ta hy xem mt b tr li th hai c th c ca mt nhm
cc nh chuyn mn phn mm vi cng cu hi:
1 ng- i s d ton 7 ngy
4 s d ton 9 ngy
10 s d ton 121 ngy
57
vi ng- i c ngh ton hc, cht ch m ni, phn b chnh tc l phn b lin tc. n gin,
chng ta cng bao gm c php tnh gn ng ring bit ca phn b chun
Ngy
D t
QUAN L DU N PHN MM
228
4 s d ton 13 ngy
1 s d ton 15 ngy
H10.4
Phn b chnh tc cc d ton c sai d t.
Hnh 10.4 gii thiu th ca nhng kt qu mi . - ng biu
din, biu th cc kt qu , khng nghi ng g cng biu th phn b
chnh tc. D sao, - ng biu din hnh chung mng li v cao hn
trong th d u. Trong tr- ng hp ny. Tnh trng tr li sai trch c t
hn trong th d u. Trung bnh cng li chuyn dch by gi l 11
ngy.
Mi phn b chnh tc - c biu th bi hai thng s, s trung bnh
(k vng) v mc sai lch (ph- ng sai). S trung bnh - c coi l tr
s mong i, , v mc sai lch - c coi l sai lch chun o.
Chng ta s s dng cc php tnh gn ng
58
tnh cc tham s phn
b chnh tc ca cc d ton d n phn mm khc nhau.
B- c u l tnh d ton tr- ng hp ti nht v d ton tr- ng hp tt
nht. Nhng s c th - c d ton bi mt cc nhn hoc bi mt
nhm ng- i. Nu chng ta d ton rng vic pht trin ca mt b driver
truyn thng ng b hn nhanh nht l 4 tun, chm nht l 12 tun, khi
d ton tr- ng hp tt nht l 4 tun cn tr- ng hp xu nht l 12
tun ( khng phi l mt khong tin cy, v rng chng ta khng bit
xc sut ca s ng n trong tnh ton ca chng ta). Hai gi tr ny,
58
th d php tnh gn ng ca Nienbung (1989), bit - c y v cc ch ny n ca
thng k, hy coi Fraser (1976).
D t
Ngy
D t
QUAN L DU N PHN MM
229
tr- ng hp tt nht v tr- ng hp xu nht, l gn ng vi hai thI cc
ca - ng cong phn b chun.
By gi chng ta cn phi tnh ton cc gi tr trung gian gia hai gi
tr thi cc . Cng nhiu d ton chng ta c th cung cp gia hai cc
- ng biu din chnh tc cng chnh xc hn. D sao, chng ta s tnh
gn ng vi tr s trung gian duy nht, tr- ng hp rt c th nht. Trong
th d trn nu bn tun l d ton tr- ng hp tt nht v 12 tun l d
ton tr- ng hp ti nht, chng ta c th kt lun l su tun l tho ng
v thi gian cho s pht trin ca ng- i iu khin lin lc ng b,
thnh th 7 tun l tr- ng hp rt c th nht (Xin nh l tr- ng hp rt
c th nht l d ton c lp v khng phi l s trung bnh suy t cc
d ton tr- ng hp tt nht v tr- ng hp ti nht). Khi s dng ba d
ton, tr- ng hp tt nht, rt c th v ti nht chng ta s qui xc sut
cho mi tr- ng hp nh- sau
59
:

= (tr- ng hp ti nht + 4 rt c th + tr- ng hp tt nht)
P (tr- ng hp tt nht) = 0,2
P (tr- ng hp rt c th) = 0,6
P (tr- ng hp ti nht)= 0,2
nh ngha ring bit ca tr s trng i l = E (x) = ExP(x)
Bi vy, gn ng ca tr s trng i trong th d tr- c hn l:

=E(x) = 0,24 + 0,67 + 0,212 = 7,4
Gi tr gn ng n gin cho sai d chun hn l hiu s ca cc cc
tr nhn vi xc sut ca chng:

o = (tr- ng hp ti nht) P(tr- ng hp ti nht) -
(tr- ng hp tt nht P(tr- ng hp tt nht)
Trong th d tr- c s cho: o = 0,212 0,24 = 16
Chng ta c th s dng cc bng phn b chnh tc pht hin l:
(o, +o) cho 68% khong tin cy
(2o, +2o) cho 95% khong tin cy
(3o, +3o) cho 99% khong tin cy
59
C Nienburg (1989) v Sodhi (1990) u dng cng thc tim cn tung t sau:
= (tr- ng hp xu nht + 4 tr- ng hp hay xy ra nht + tr- ng hp tt nht)/6
QUAN L DU N PHN MM
230
iu ny c ngha l trong th d trn, chng ta c th d ton thi
gian pht trin ng- i iu khin lin lc ng b l gia 4,2 ngy v
106 ngy v chng ta tin cy 95% ca d ton ca chng ta. Chng ta
cng c th d ton thi gian pht trin l gia 5,8 ngy v 9 ngy, nh- ng
ri chng ta ch c th l 68% tin cy ca d ton ca chng ta.
Mt trong nhng tnh cht ca thng k l kt qu ch c th tt nh- d
liu m chng da vo mt dng ca rc vo th rc ra. Do y quan
trng l phi dnh thi gian v cng sc cn thit cho s pht trin ca 3
d ton c hiu qu. Mt ph- ng cch vng vng, mc d cng phu l
i hi mt s cc k s- phn mm (cho l 6) chun b cc d ton c
nhn cho cc tr- ng hp ti nht, tt nht v rt c th. D ton tr- ng
hp ti nht v d ton tr- ng hp tt nht hn l hai cc c - c trong
khi d ton tr- ng hp rt c th hn hoc l trung bnh, bnh qun hay
th- ng xuyn nht (ph- ng thc)
10.7 d ton cc ngun lc phn cng.
Phn mm phi - c thit k khp mt cch thoi mi vo mi tr- ng
ch ca n. Mi tr- ng ch bao gm phn cng mc tiu v cc thuc
tnh ca n. Thit k phn mm ti c th dn n qu ti ca cng sut
CPU hay c th v- t qu cng sut b nh c - c hay l- u gi hng lot.
i khi iu ny c th, nh- ng khng phi lun lun - c b cu bng
cch trin khai (m rng) phn cng mc tiu, dn n tng chi ph d
n.
Cc ngun phn cng - c o bng n v ca ngun c bit xt n:
vi truyn thng th bit/giy (hay byte/giy), vi kho nh l kilobyte,
megabyte v thm ch l gigabyte, vi CPU c hiu th phn trm ca ti
CPU. Ngun ch yu, tu thuc nng n vo phn cng, mc d trn thc
t khng phi bn thn phn cng, m l tc . D ton tc m t k
vng ca chng ta v mt thc hin ca h thng phi pht trin v
th- ng - c yu cu trong pha quy hoch sm ca d n.
Chng ta s xem xt cc ph- ng php d ton ba ngun chnh sau:
1 ti CPU
2 l- u gi d liu
3 tc
Tc mc d l thuc tnh ch khng l ngun, vn s - c xem l
ngun cho mc ch tho lun ny.
10.7.1 Ti CPU
D ton ti CPU c th hon ton phc tp, c bit trong mi tr- ng
x l nhiu h m vo mt thi gian cho phi hn mt x l c th
phi cnh tranh cho CPU. iu ny t- ng t nh- xp hng dch v trong
mt s nhng ng- i yu cu dch v ch i dch v, - c mt hay
QUAN L DU N PHN MM
231
nhiu ngui cung cp dch v cung cp. Trong bi cnh ca chng ta,
nhng ng- i yu cu dch v l b x l v ng- i cung cp dch v l
CPU. Vn xp hng dch v l vn ph thng trong vn tr hc v
- c gii quyt c s h tr ca hng k. tho lun thm v l thuyt
xp hng, hy coi Gillett (1976)
Chng ta s xem xt ti CPU trong mi tr- ng xc nh hn mt cht.
Chng ta gi s rng vo thi gian cho, chng ta c th xc nh c
nhng yu cu no i vi cc ngun x l CPU. Khng c gi nh ,
d ton ti CPU khng th tnh ton - c v ch c th - c suy ra khi
m phng mi tr- ng thc ti v quan st kt qa.
Ti CPU - c d ton thnh phn trm. Chng ta ni mt h thng c
75% ti CPU, c ngha l 25% ca nng sut x l c - c cho nhng
nhim v ph.
Ti CPU lun - c o trong tnh hung tr- ng hp ti nht. Mt ti
CPU 75%, c ngha l vo mt thi gian cho, khng qu 75% CPU
- c s dng. D sao, iu ny l nghch l, v trn thc t chng ta bit
l vo mi lc 100% CPU - c s dng. Do chng ta phi nh
ngha l ci m ta mun ni v s dng CPU cho mc ch d ton.
Th nht ti CPU lun - c o trong phm vi mt ca s thi gian c
bit. iu ny c ngha trong mt khong thi gian c bit (chng hn
c th l 10 ly giy) chng ta o l- ng thi gian m CPU s dng. Mt
trong nhng nhim v chnh l la chn ca s thi gian thch hp.
Nhm xc nh s dng CPU trong phm vi ca s thi gian, chng ta
ch xem xt nhng nhim v khng th x l - c vo bt c thi gian
no khc. Trong mt h thng m chn on - c thc hin trn c s
bt c khi no CPU khng c nhim v no khc x l, chng ta hn
khng xem xt nhng nhim v chn on trong tnh ton ti CPU.
Mt h thng c 60% ti CPU trong mt ca s 10 ly giy c th chp
nhn mt nhim v ph 4 ly giy nu nhim v c th dung th chm
tr ti a 10 ly giy.
Ly th d, chng ta s xem xt u vo d liu mt cng bin lc.
Nu cng c sut chuyn giao d liu 1200 bit mi giy th mt byte c
th c - c cng u vo xp x mi 7 ly giy v chng ta phi sn
sng rt mt byte khi cng tr- c khi byte sau tr nn c - c. Do y
b iu khin cng lin lc c th chp nhn dung sai ti a 6 ly
giychm tr (gi nh 1 ly gy thi gian x l).
D ton ti CPU hu ht th- ng - c yu cu trong cc h thng thi
gian thc. Chng hiu khi - c yu cu trong h thng x l d liu
th- ng mi. Cc h thng thi gian thc th- ng - c biu th bng vng
lp h thng c bn, th- ng - c coi l vng lp chnh hay iu hnh.
y l nhim v mc thp, vng lp v hn, thc hin mt b cc
nhim v ng b iu khin h thng. Trong nhiu tr- ng hp, vnh lp
mc thp hin nay l b phn ca h thng iu hnh, n c th - c cu
hnh da trn thi gian chu k yu cu cho vng lp. Vng lp mc thp
ny c th - c la chn coi nh- ca s thi gian cho vic tnh ton ti
QUAN L DU N PHN MM
232
CPU. Nu vng lp chnh thm ch bao gm nhiu vng lp nhanh mc
thp th vng lp nhanh nh nht c th - c chn lm ca s thi gian
(coi hnh 10.5).
H10.5
Thi hnh thi gian thc vi cc vng lp nhanh v chm
B cc b- c sau m t ph- ng php tnh ti CPU. Ph- ng php ny i
hi mt phn gii ban u h thng thnh nhng nhim v phn mm ch
yu v thi gian thc hin d kin cho mi nhim v.
1. Xc nh vng lp h thng mc thp nht, v suy t iu l ca s
thi gian - c s dng xc nh ti CPU.
2. Hon thin nh thi gian cho phn tch h thng v nhn ra mi
nhim v c th - c yu cu x l trong phm vi ca s thi gian .
3. Phi hp cc thi gian thc hin d ton cho mi nhim v - c
nhn ra b- c 2.
4. Phn chia kt qu ca b- c 3 theo qui m ca ca s thi gian.
Xin nh l b- c 2 i hi nhn ra bt c nhim v no c th yu cu
(ch ch- a - c yu cu) thi gian CPU trong phm vi ca s thi gian.
iu ny bao gm nhng nhim v ca h thng iu hnh.
Ta hy xt th d sau. Mt h thng iu khin quan tm tch cc - c
my tnh ho bao gm nhng hp phn chnh sau.
1. u vo d liu t thit b iu khin.
2. Bo ng.
3. Hp phn giao din ng- i dng.
4. Vng lp iu hnh c thit b iu khin.
Vng lp iu hnh bao gm:
Vng lp nhanh
Vng lp nhanh
Vng lp nhanh
Vng lp nhanh
Vng lp chm
QUAN L DU N PHN MM
233
i. Mt vng lp chnh c hai u vo phc tp cng i hi mt cht
x l logic c mi 400 mili-giy mt ln. Nhim v ny - c d ton l
60 mili-giy mt.
ii. Mt vng lp nhanh c hai u vo mi 40 mili-giy. Nhng
nhim v ny - c d ton l 10 mili-giy mt.
C 10 vng lp nhanh trong phm vi vng lp chnh. Chng ta s la
chn ca s thi gian l 40 mili-giy. Khi tnh tr- ng hp ti nht, trong
phm vi ca s thi gian, chng ta s xem xt nhng nhim v vng lp
nhanh tr- c . Nhng nhim v s dng ngay lp tc ti 20 mili-giy
trong 40 mili-giy c - c. iu c ngha l trong vng lp chnh 400
mili-giy ch c 200 mili-giy - c dnh cho nhng nhim v khc.
Trong , 120 mili-giy - c yu cu cho hai nhim v vng lp chnh
c th - c iu tit d dng. Nu chng ta chia u nhng nhng nhim
v ca vng lp chnh, gia cc vng lp nhanh, th 12 mili-giy ph s
phi c - c ca s thi gian, c - c tng s 32 mili-giy.
Hnh 10.6
Thi hnh thc vi cc vng lp chm v nhanh
(a) Cm th phn tch thi gian
(b) nh thi thi hnh thc ti
Spare & Background
Vng lp chm
Vng lp nhanh
D ton da trn
D ton da trn
Spare & Background
Vng lp chm
Vng lp nhanh
40 80 120 160 200 240 280 320 360 400 t(ms)
40 80 120 160 200 240 280 320 360 400 t(ms)
(a)
(b)
QUAN L DU N PHN MM
234
Chng ta c th gi nh l mt khi nhim v bo ng - c thc hin,
mi nhim v khc b thui cht, khin chng ta khng cn thit xem xt
thi gian thc hin cho cc nhim v ny na. Hp phn giao din ca
ng- i dng khng phi l hot ng thi gian thc (so snh vi cc
nhim v u vo iu khin thit b) v chng ta c th cho l giao din
ca ng- i dng s - c x l bng phn d tr ca vng lp chnh 20
mili-giy trn nn. Cc vng lp iu hnh t- ng i n gin v - c d
kin yu cu t hn 1 mili-giy v do cng - c loi ra khi vic tnh
ton ny.
Ti CPU cho h thng iu khin - c tng c- ng l
32/40 = 0,80
c ngha l ti CPU bng 80% trong ca s thi gian 40 mili-giy. Th
d ny - c minh ho trong hnh 10.6.
Mt tnh hung l th xut hin khi vic tnh ton ti CPU cho kt qu
ln hn 1.00. iu ny c ngha ti CPU ln hn 100%! Kt lun hin
nhin l b x l bt lc trong vic thc hin cc nhim v yu cu.
Trong nhng tr- ng hp , c hai bin php sa cha c th - c: hoc
yu cu ca h thng phi gim i hoc b x l nhanh hn phi - c s
dng.
Nu nh- mt ti CPU 100% c gy ra vn , th thng th- ng mt ti
CPU 90% cng gy ra vn khng km. iu ny l do vic d ton ti
CPU t khi - c chnh xc. Cng vy, vic thay i yu cu trong pht
trin d n th- ng lm tng ti CPU. Mt nhn nh khc l nhu cu d
tr ngun CPU cho s m rng sau ny ca h thng. Bao gi y cng l
thc tin tt thit k mt h phn mm, sao cho khi thu khng cn
qu 60% ti CPU.
10.7.2 L- u gi d liu.
Chng ta s s dng t l- u gi d liu cho c l- u gi phn cng v l- u
gi d liu. Vic s dng l- u gi d liu cn bn l mt vn thit k v
thng th- ng c lin quan (tuy hn ch) v yu cu cho h thng. Cc
yu cu c th xem xt l- u gi d liu kha cnh chi ph, do vic thit
k l- u gi d liu khng hiu qa c th i hi qu mc cc ph- ng tin
l- u gi, nh- b iu khin a v b nh. Nhng ph- ng tin khng nht
thit bin thnh nhng khon chi ph b xung.
Cc yu cu cng c th ni n l- u gi d liu trong khung cnh
khng gian. Yu cu l h thng phi cung cp 30% khng gian a mm
v b nh d tr, c th c tc ng to ln n thit k l- u gi d liu,
c bit trong mt h thng m vic tiu dng b nh trong vn hnh l
st ti gii hn.
Xt ton cc, ng- i dng khng nhn thy - c vic s dng l- u gi
d liu, v do vic ch yu - c xc nh trong pha thit k. D
sao, d ton s b b- c u - c yu cu ngay nhng pha sm nht
QUAN L DU N PHN MM
235
ca d n, m bo l cu hnh phn cng mc tiu c kh nng tr
gip cho h thng phn mm - c pht trin.
o khi l- ng phn mm - c pht trin nh- bn n tr- c y
phn 10.1 c th l cc dng m (KSLOC) hay Kilobyte b nh. Vic l- u
gi b nh ch - c d ton theo n v Kbyte. B- c u lin quan n
phn gii h thng, t- ng t ph- ng php d ton tng b- c m t trong
phn 10.2. Ln ny mc tiu l m t khi l- ng b nh m mi m un
yu cu lc thc hin.
Cc hp phn phn mm mc thp - c phi hp thnh nhng nhm
m un c- tr cnh tranh b nh. Ri cc yu t khc cng - c xem xt
nh- phn b ng b nh, yu cu ca h thng iu hnh v cc buffer
c- tr trong b nh v cc bng.
Ph- ng php sau cung cp d ton b nh da trn s dng b nh
tr- ng hp ti nht trong qu trnh thc hin h thng.
1. Phn gii h thng thnh cc m un mc thp v d ton cc yu
cu b nh cho mi m un vo lc thc hin.
2. Nhn ra cc yu cu b nh gin tip ca mi m un cn c vo
Cc vng hot ng ng v tnh ca b nh.
Cc bng c- tr b nh
Cc buffer (m)
3. Nhn ra b cc m un b nh vi yu cu b nh phi hp ln
nht m s l c- tr ca b nh bt c lc no.
4. Duyt li t- ng quan gia m un b nh v loi b yu cu b nh
trng lp.
5. Lp li cc b- c 3 v 4 cho n khi vic s dng b nh li nht
- c trng i nhn ra.
6. Nhn ra cc yu cu b nh mc h thng nh- :
Bng c- tr ca b nh
Buffer file (b m ca file)
Kch c stack.
7. Tnh ton ton b s dng b nh ca h thng iu hnh.
8. Phi hp cc kt qu ca cc b- c 5,6, v 7 to nn ton b d ton
l- u tr b nh.
B- c 7 phi loi tr cc yu cu b nh - c xt n cc b- c 5
v 6 (th d cng cc buffer ca file phi khng xt n b- c 2 v b- c
6).
Vic s dng overlay c th gim yu cu b nh, nh- ng iu ny s
t - c vi s thc hin tn km cho cc ngun ph, nh- a I/O, ti
CPU v tc thc hin. Tnh trng bt li (trade-off) gia b nh v tc
l mt yu t thng th- ng trong hu ht cc h thng phn mm.
D ton cc yu cu l- u gi hng lot t gay cn hn d ton yu cu
ca b nh. B nh ca my tnh hn ch nhiu hn l- u gi a. L- u gi
a th- ng ch b hn ch chi ph thit b l- u gi a.
QUAN L DU N PHN MM
236
D ton s dng l- u gi a phi tnh n cc tiu dng l- u gi a sau
y:
1. H thng iu hnh c nh v tin ch dch v.
2. Cc yu cu h thng iu hnh bin i (overlay v vng swap,
cc file h thng v.v..)
3. Phn mm d n.
4. Cc file d liu.
V mc ch tho lun ny, chng ta s bao gm c khi ch- ng trnh
dch v, nh- c s d liu v ch- ng trnh truyn thng, coi nh- b phn
ca h thng iu hnh. Thng tin lin quan n s dng a ca h
thng iu hnh th- ng do ng- i bn hng ca h thng iu hnh cung
cp. D sao, nu h thng iu hnh - c pht trin nh- l b phn ca
d n th n phi - c d ton hoc coi nh- h thng ring bit hoc coi
nh- b phn ca phn mm d n (coi nh- h thng ph).
Nhng yu cu ca h thng iu hnh thay i ph thuc cu hnh
ca h thng iu hnh c tr- ng v phn mm ca d n. Ly th d
khng gian ca a cho overlay va l chc nng ca h thng iu hnh
va l chc nng ca thit k phn mm thc ti. Nhng h thng iu
hnh tin tin th- ng cung cp tin ch cho tng ph a d ton (c v
mt l- u gi v truy nhp) khi s dng overlay. Vng swap cho cc ng
dng a ng- i dng th- ng cng c th ly kch c da trn cc tin ch
ca h thng iu hnh tiu chun. Cc tin ch th- ng l phn chung
iu chnh hay cng c cu hnh ca cc h iu hnh.
D ton cho cc yu cu v a ca phn mm d n l thng thng v
- c lp bi vic phn gii h thng. Tng cc d ton cho mi m un
to ra cc yu cu v a cho ton b phn mm d n - c pht trin.
Vic chun b d ton cho mi kch c file d liu l mt nhim v
bun t v - c cn c kch c ca cc bn ghi ring r v s ti a cc
bn ghi cho mi file. N tr nn phc tp hn khi cc bn ghi c chin
di thay i. Trong nhng tr- ng hp nh- th, cc c file bn ghi cc i
hay trung bnh nn - c s dng. Mi file d liu u c phn u bao
gm ch s v - ng dn th- mc. Nhng th ny cng phi - c - a
thnh nhn t d ton v tu thuc nng n vo c s d liu hay h
thng file ang - c s dng.
Trong nhiu tr- ng hp d ton d liu l- u gi hng lot khng cn
cn c ch- ng trnh kch bn ti nht. iu ny thot tin do tnh d
dng t- ng i trong vic b sung thm b nh l- u gi hng lot vo cu
hnh. Do y, trong nhiu tr- ng hp, yu cu l- u gi hng lot trung
bnh l c th . R rng rng c mt s tr- ng hp phi s dng kch
bn ti nht, nh- khi m vic s dng kho nh hng lot tng qu nhanh.
D ton thit k cho yu cu l- u gi d liu cng phi chiu c n
phn d tr. iu ny c bit ng vi cc d ton b nh, lc thit
k phi dnh khong 33% cho d phng. B nh d phng ny phi c
QUAN L DU N PHN MM
237
sn bao trm cc sai khi d ton, cc thay i yu cu v s pht trin
trong t- ng lai.
10.7.3 Tc
Tc ca mt h thng - c o theo thi gian p ng. iu ny
th- ng cn c b yu cu rt ngt ngho xc nh p ng ca h thng
vi cc s c c th.
Mt th d l c th l yu cu rng giao din ca ng- i dng phi p
ng u vo ca ng- i dng khng qu ba giy. Tht l cc k tht vng
phi ch i tr- c my th qi ngn hng t ng ch tr li mt yu
cu. Nhiu nhng my th qi nh- th gim nh ni tht vng phi
ch i bng cch to ra ting ng nn vang ln ta nh- tin ang - c
m bng my c kh hay t ra nh- c ci g ang din ra. Mt gii php
n gin hn l pht ra tn hiu nhp nhy ni rng l giao dch ang
- c thc hin.
Hnh 10.7
Thi gian p ng nhn - c khi lin h ng- c
Hatley v Pirbhai (1988) phn bit nh thi gian ngoi ti v nh thi
gian ni ti. nh gi ni ti l mt kt qu ca thit k trong khi nh
thi gian ngoi ti l kt qu yu cu. Theo th nh thi gian ngoi ti c
th c l thi gian p ng. Hatley v Pirbhai gi mt bin (m cc
ng gi l bc tranh yu cu nh thi gian) tham gia vo vic phn tch
cc rng buc lin quan ti nh thi gian h thng.
Trong phn tch nh thi gian ni ti ca cc ng, Hatley v Pirbhai
bao gm cc hot ng khng lin quan trc tip ti thi gian p ng.
Chng ta s ch xt y cc kt qu nh thi gian trc tip lin quan
vi thi gian p ng nh- - c nhng ng- i dng h thng nhn nhn
bn ngoi.
Thi gian p ng c th - c cm nhn l phn hi h thng pht
sinh bi mt s c bn ngoI: iu ny - c minh ho hnh 10.7. Do
y tho ng l chia thi gian p ng ra ba hp phn c bn.
1. Thi gian t cui s xut hin ca s c cho n khi s c - c h
thng nhn ra.
S kin t bn ngoi Lin h ng- c
Tin trnh
Ng- i dng
Tin trnh
QUAN L DU N PHN MM
238
2. Thi gian x l s c.
3. Thi gian t khi kt thc x l s c cho ti khi khi u p ng
ngoi ti
Hnh 10.8
Khong thi gian p ng
Nh l thi gian p ng khng bao gm thi gian thc hin s c
c hai u: n ch bao gm thi khong gia hai s c (coi hnh 10.8).
Khi d ton thi gian p ng, th vin cnh l mt trong ba th u
vo, x l v u ra. Hp phn u vo lin quan n thi gian h
thng nhn - c thng tin v cui ca mt s c v nhn ra n.
Bng 10.7 th d v bng phn b thi gian p ng
Thi gian p ng (tnh theo giy) T l phn trm
0-3 20%
3-6 60%
6-9 39%
9-15 1%
Hp phn u ra lin quan n thi gian cn c h thng c - c
kt qu x l v truyn thng n ti b pht p ng bn ngoi.
Hp phn x l s c bao gm mi hot ng trong phm vi h thng
cn c pht ra p ng. iu ny cng bao gm chm tr do nhng
hot ng - u tin cao hn v do nhng thit b bn ngoi nh- b iu
khin a phi - c tuy cp c - c thng tin lin quan.
Thi gian p ng thng th- ng - c cung cp nh- p ng tr- ng
hp ti nht v p ng trung bnh.
Trong th d tr- c th qu t ng tr li u vo bn phm ca ng- i
dng. Ln n phm cui cng ca mt giao dch (th- ng l enter) bt u
thi khong p ng. Hin th mn hnh hay phn phi tin mt kt thc
thi khong ny ngay khi bt u vn hnh. Nh- chng ta thy ng- i
thit k h thng hn la bp bng cch tr li bng thng bo xin i
mt cht tr- c khi c sn sng cu tr li thc. iu ny l mt thc t
c hiu lc hon m v lm cho h thng tr nn thn thin vi ng- i
Tin trnh S kin
p ng
Thi gian p ng
D to
QUAN L DU N PHN MM
239
dng hn. D sao, thng bo nhp nhy bo rng xin i, by gi ang
x l yu cu cng c th tr thnh gy tc bc nu hin hnh qu lu.
p ng trung bnh ca th qu t ng tr- c yu cu quyt ton c
th i hi thi gian p ng l 5 giy. Trong mt s tr- ng hp khi h
thng th qu ang vo lc s dng cao im v thi gian p ng
tr- ng hp hn l 15 giy. Mc d vic chm tr li 15 giy c th gy
tc bc cho ng- i dng th n li c th chp nhn - c cho ngn hng
nu iu ny khng xy ra qu th- ng xuyn. Do y cn thit khi trnh
by d ton thi gian p ng phi bao gm c d ton phn trm thi
gian m tr- ng hp ti nht c th d kin xy ra. Chng hn chng ta c
th khng nh:
1. Thi gian p ng trung bnh d kin: 5 giy
2. Thi gian p ng tr- ng hp ti nht d kin: 15 giy trong 1%
giao dch.
Vic m t chi tit hn thi gian p ng d kin th- ng - c yu cu.
Trong th d trn 10% chm tr p ng c th chim 14,5 giy l khng
th chp nhn - c, nh- ng li c th khp vi cc con s trn. Trong
nhng tr- ng hp , th bng phn phi (coi bng 10.7) - c - a thch
hn.
10.8 tng ph khng pht trin.
Nhng d ton tt - c cn c hu ht d liu mi nht v d hiu
hin c. Do y, iu quan trng l phi cp nht nh k mi d ton
ca d n. Ch t ra th iu ny phi - c lm cc ct mc d n ch
yu, nh- tng duyt yu cu phn mm, tng duyt thit k s b v tng
duyt thit k ti hn. Cc d ton - c cp nht phi l b phn ca
nhng th giao - c c yu cu nhng tng duyt .
Bng 10.8 Tng ph ca cc hot ng khng pht trin
Tng ph
Qun l 0.1
Khng ch cu hnh 0,02
Bo m cht l- ng phn mm 0,03
Sau khi kt thc pha thit k, cc d ton phi - c xem li tr- c khi
tch hp bt u v ri li xem li tr- c khi th nghim bt u. Cng
vy, mi s c khng d kin xy ra trong chu k pht trin c th i
hi phi tnh ton li d ton. Mt th d ca s c nh- th c th l vic
thay th hp phn li thi bng mt hp phn pht trin mi hay mt s
chm tr d n ch- a lp lch ch yu.
Cng vy mi thay i n c t yu cu ca d n bao gi cng phi
km theo phn tch tc ng ca thay i n lch d n. Mi thay i
QUAN L DU N PHN MM
240
vi s rt t ngoi l u phi thay i chi ph. Ngay vic rt b mt yu
cu cng gy ra thay i chi ph d n d ton.
Mt khi l- ng thay i no n c t yu cu th- ng - c tnh
n trong vic chun b d ton ca d n. iu ny bao gm trong d
ton d- i hnh thc yu t an ton. Yu t an ton cng c th - c dng
b tr cho cc tr- ng hp khc nh- sai lm trong d ton v chm tr
khng d kin.
Cc yu t an ton th- ng s dng t 20% n 40% tu theo mc
tin cy trong d ton, d on cc thay i yu cu v cc chm tr c
th xy ra.
Cc yu t t- ng t cng i khi - c dng - a vo trong d ton
nhng hot ng khng pht trin nh- qun l d n, kim tra cu hnh
v bo m cht l- ng. D sao, chng mc ca nhng nhim v ny cng
ph thuc cc c tnh khc ca d n. Mt d n rt nh c th khng
yu cu bt c qun l trc tip no trong khi d n ln hn b kt ti nu
khng c n.
Bng 10.8 trnh by cc yu t tng ph cho cc hot ng khng pht
trin ca d n phn mm.
Cc con s trong bng 10.8 cho thy l d n phn mm vi i ng
100 k s- hn yu cu xp x 10 nh qun l (thi gian lm vic 100%).
iu ny bao gm nhng nhim v nh- qun l d n, ph qun l d n,
lnh o i (c th dnh mt phn thi gian vo cng vic pht trin)
v.v. Trong mt d n nh ba ng- i, mt trong nhng thnh vin hn - c
d kin ginh khong 1/3 thi gian ca anh hay ch cho vic qun l d
n.
T- ng t, trong mt d n phn mm do mt i ng 100 k s- pht
trin, chng ta c th d kin b tr 2 k s- kim tra cu hnh v 3 k s-
m bo cht l- ng phn mm. Cng b tng qut hn c th l trong
mt d n 100 nm cng, 2 nm cng hn - c ginh cho kim tra cu
hnh v 3 nm cng cho bo m cht l- ng tng hp.
Cc con s trong bn 10.8 c th hi b nh h- ng bi qui m d n v
bi cc yu t bn n phn 10.4. Mt d n phc tp s i hi phn
chung nhiu hn mt d n t- ng i n gin.
Ton b tng ph d n cho nhng hot ng tt c khong 15%.
Quan trng l phi nh rng iu ny ch lin quan n tng ph d n
trc tip v khng tnh n thi gian u t- ca qun l cp cao, th- k
vn phng v.v..
10.9 Tmtt
Ch- ng ny minh ho d ton - c vn dng th no trong vic tin
liu bt trc. Bt c l- ng khng bit no u c th - c d ton trong
khi cc l- ng bit khng cn d ton. Vi ng- i qun l d n phn
mm c nhiu l- ng khng bit phi - c d ton. Nhng l- ng ny lin
kt vi cc lnh vc nh- :
QUAN L DU N PHN MM
241
1. Chi ph pht trin d n
2. Lch pht trin d n
3. Qui m i pht trin d n
4. Khi l- ng phn mm phi pht trin
5. Ngun lc phn cng i hi.
D ton tng b- c th- ng - c coi l ph- ng cch phn chia v
khut phc, phn vn ln ra v vn vn nh v - c dng trong
hu ht k thut d ton. Ph- ng cch c bn l phn gii d n thnh
nhng hp phn xc nh r v ri lp li tng b- c n khi ch cn li
nhng n v nh c th d d ton hn.
Phn gii ban u ca d n phn mm nhn ra 4 phm tr ch yu vi
nhng mc ri ro pht trin khc nhau lin kt vi chng. B- c u
tin ca phn gii d n to ra nhng hp phn d n hoc (1) chng ta
c (2) chng ta bit pht trin nh- th no (kinh nghim trn vn) (3)
t ra chng ta phn no lm quen vi chng (kinh nghim mt phn)
hay (4) hon ton mi vi chng ta (pht trin mi). Cc k thut d ton
c th c th - c vn dng cho mi loi hp phn d n khc nhau.
Mt ph- ng php khc, gi l thut ton chi ph kin thit (COCOMO)
bao gm 10 b- c c bn. Nhng b- c bao gm phn gii d n ra cc
hp phn, vn dng cng thc cng sc vo mi hp phn v phi hp
mi d liu to ra vo mt d ton chi ph duy nht ca d n.
Phn tch chc nng (FPA) to ra d ton ca d n cn c qui m
vn . S l- ng cc chc nng trong d n xc nh qui m vn , - c
biu th bng gi tr s (tr s FPA). Tr s FPA ca d n c th - c
dng :
+ So snh phc tp ca cc d n.
+ So snh cng sc t- ng i i hi hon thnh d n.
+ Pht sinh cc o d n khc (nh- cc SLOC)
D ton cng th- ng - c trnh by nh- l mt phm vi. Mt phm vi
l mt d ton c ch trong qui hoch pht trin ca d n. L thuyt
thng k ng sau ph- ng cch ny cp quan nim khong tin cy, n
cung cp xc sut l chi ph pht trin s nm trong mt phm vi cho.
Cc ph- ng php d ton ti CPU, l- u gi d liu v tc - c cn
c phn gii phn mm thnh nhng m un d ton - c. Ri nhng
d ton ny - c phi hp vi yu t an ton. Cc yu t an ton th- ng
s dng t 20% n 40% tu theo mc tin cy trong d ton v thi
hn d kin thay i yu cu.
Cc d ton tt - c cn c- hu ht d liu mi nht, d hiu hin c
- c. Do , quan trng l phi cp nht nh k ton b d ton ca d
n. D sao, bt k ph- ng php d ton no - c s dng, quan trng l
QUAN L DU N PHN MM
242
bao gi cng phi nh rng d ton ch c th tt - c nh- cc d liu
- c dng lm cn c.
Bi tp.
1. Phn tch mt h thng kim warehouse do mt cng ty pht trin
vn tr- c y pht trin h thng kim ca hng bch ho. Hy s
dng d ton tng b- c, phn gii h thng thnh 4 phm tr hp phn.
Sau phn gii hp phn kinh nghim tng phn thnh hp phn kinh
nghim y v hp phn pht trin mi. c t mi gi nh lm.
2. Pht trin tip vn bi tp 1 v chun b mt k hochd ton
cc hp phn pht trin mi bng cch s dng ly mu thng k. Xc
nh nhm phm tr v b tr mi hp phn pht trin mi vo loi thch
hp ca n. Chn nhng m un tiu biu t mi phm tr thc hin.
Gii thch iu hp l ca vic gn cc loi v la chn lm.
3. Cn c kinh nghim ca bn, hy b tr nhng con s KSLOC hp
l vo mi hp phn bi tp 2. Gi nh mi ng- i pht trin cho 5
KSLOC. Gi nh l 10% nhn lc l mc 1, 30% mc 2, 45% mc 3 v
15% mc 4. Hy tnh nhn t PL cho d n pht trin. Bnh lun tc ng
ca tr s PL ti chi ph pht trin d n.
4. Duyt li bng 10.1, c lin quan n vic tnh ton tha s PL.
Cho mt gi mt bng tr s khc, cn c cc mc KSLOC ph thm
v mt phm tr rng cc nhn t. Bnh lun l do chn nhng con s
trong bng ca bn. Tnh li tr s trong bng 10.2 cn c bng bn
ngh.
5. Phn gii h thng kim k warehouse m t bi tp 1 thnh cc
hp phn theo mc phc tp. Cn c cc con s KSLOC b tr bi tp
3, hy tnh SEM cho mi lp hp phn v phi hp kt qu c - c
con s d ton ca SEM cho ton b d n.
6. Phn gii h thng kim k kho m t bi tp 1 ra nhng hp
phn theo mc tin cy. S dng nm mc tin cy - a thnh tha s
tin cy vo d ton SEM tnh bi tp 5.
7.
(a) Gi bng cc nhn t 5 mc cho mc mi tr- ng pht trin, cn
c ba mc qui m d n.
KSLOC < 25,
25 < KSLOC < 300,
3000 < KSLOC
(b) Gi bng nhn t - a thnh tha s tnh phc tp ca h thng
ph. Tho lun v c ca d n phi xt.
8. Tnh d ton chi ph cho h thng kim warehouse m t bi tp 1
bng cch s dng cc kt qu ca cc bi tp 3, 5, 6 v 7.
9. Xem xt vic vn dng thut ton phn tch l- ng chc nng vo h
thng kim warehouse m t bi tp 1. Li v bt li trong vic s dng
ph- ng php FPA ? So snh FPA vi COCOMO cho d n c bit ny.
QUAN L DU N PHN MM
243
10. Xp loi u thc: cn c cc kt qu c nhn khc nhau cho bi
tp 8, hy tnh khong cch tin cy 68% v 95% cho phm vi chi ph pht
trin i vi h thng kim warehouse ny.
11. (a) Duyt li th d ti CPU phn 10.6. Gi nh rng u vo ca
thit b - c iu khin bi cc ngt. Mi ngt c tng ph 0,5 mili-giy.
Vy ti CPU l bao?
(b) Ti CPU phi l g nu vng lp nhanh c ba u vo ch khng
phi 2 ? Bnh lun tc ng ca kt qu.

You might also like